Sie sind auf Seite 1von 222

OBG-Gyn

A patient complains that her otherwise healthy 66-year-old husband takes longer to achieve an erection that he did at age
40. You advise that:
a) At his age, sexual response is naturally slower
b) He makes an appointment with a sex therapist
c) He takes 20 mg of methyltestosterone daily
d) She uses psychogenic stimulation
e) He makes an appointment with a urologist
The correct answer is A
Explanation
Part of the normal aging process changes in the human sexual response. Although it is natural for a 66 year-old man to take
longer to achieve an erection, it is possible that other factors may be involved. This includes the use of other medications
and chronic medical conditions. Should this be the case, then appropriate referral is indicated.
B. See above explanation.
C. See above explanation.
D. See above explanation.
E. See above explanation.
A 24-year-old gravida 1 para 1 who is 2 weeks post partum complains of double vision, shortness of breath, and almost
dropping her baby while trying to hold her. She says her symptoms worsen as the day progresses. She has no family history
of neurologic or muscular illness. A physical examination is normal except for unilateral ptosis and 4/5 proximal weakness of
both arms. Breath sounds are generally decreased. Routine blood tests, including TSH and creatine kinase levels, are normal.
A chest radiograph and an MRI of the brain and cervical spine are also normal. Of the following, this presentation is most
consistent with

a)
b)
c)
d)
e)

Fibromyalgia syndrome
Sheehans syndrome (postpartum hypopituitarism)
Polymyositis
Myasthenia gravis
Stroke

The correct answer is D


Explanation
Common neurologic disorders in young women include multiple sclerosis, Guillain-Barr syndrome, and myasthenia gravis.
Myasthenia gravis is part of the differential diagnosis for sudden neurologic weakness, and Guillain-Barr syndrome must
also be considered in this patient. Multiple sclerosis would not result in respiratory compromise.
Myasthenia gravis is an autoimmune neuromuscular disease characterized by varying degrees of skeletal muscle weakness.
Symptoms, which vary in type and severity, may include ptosis of one or both eyelids; blurred vision; diplopia; unstable gait;
weakness in the arms, hands, fingers, legs, and neck; difficulty swallowing; shortness of breath; and impaired speech
(dysarthria). In most cases, the first noticeable symptom is weakness of the eye muscles. Muscles that control respiration
and neck and limb movements may also be affected. Symptoms typically worsen through the day or as the muscles are
repetitively used, and improve with rest.
Fibromyalgia does not produce objective neurologic findings, and Sheehans syndrome would not cause a localized
neurologic deficit. In addition, the TSH level would be low or zero, and the MRI of the brain would be abnormal. An MRI of
the brain would also be abnormal if stroke symptoms had been present for 2 weeks.
The patient is unlikely to have unilateral symptoms with polymyositis, and creatine kinase would be elevated.

Which one of the following is indicated for treatment of chlamydial urethritis during pregnancy?

a)
b)
c)
d)

Ciprofloxacin (Cipro)
Doxycycline
Erythromycin base
Ofloxacin (Floxin)

The correct answer is C


Explanation
Erythromycin is safe and effective for treatment of chlaymdial infection during pregnancy. Ciprofloxacin is not consistently
effective and is contraindicated during pregnancy. Ofloxacin and doxycycline are effective, but are contraindicated in
pregnancy.

What is the diagnostic test used to confirm a diagnosis of down syndrome in the fetus of a pregnant woman at 12 weeks
gestation?

a)
b)
c)
d)

Chorionic sampling villous


Amniocentesis
Ultrasound
Triple screen

The correct answer is A


Explanation
The triple screen test is a maternal blood screening test that looks for three specific substances: AFP, hCG, and Estriol. The
triple screen test is performed between the 15th and 20th week of pregnancy although results obtained in the 16th -18th
week are said to be the most accurate.
Low values for AFP and Estriol and high values for hCG are seen in Down's Syndrome. Low levels of all three seen in trisomy
18. High levels for AFP are seen in neural tube defects like spina bifida
Ultrasound Screening is used to confirm the gestational age of the fetus (it's more accurate than dating from the mother's
last menstrual cycle). Another benefit of the ultrasound can also pick up problems of a serious medical nature, such as
blockage of the small intestine or heart defects. Knowing these defects exist as early as possible will benefit the treatment of
the child after birth.
Chorionic Villus Sampling (CVS) is a procedure where a small amount of tissue is taken from the young placenta (also called
the chorionic layer). These cells contain the fetal chromosomes that can be tested for Down syndrome. CVS is usually carried
out between the 10th and 12th weeks of pregnancy.
Amniocentesis is a procedure used to collect amniotic fluid, the liquid that is in the womb. A needle is inserted through the
mother's abdominal wall into the uterus, using ultrasound to guide the needle. This fluid contains fetal cells that can be
examined for chromosome tests. Amniocentesis is usually carried out between the 14th and 18th week of pregnancy; some
doctors may do them as early as the 13th week.
During a routine urinalysis in pregnancy, which of the following is most likely to be a normal finding?

a) Glucosuria
b) Hematuria
c) Pyuria
d) Bacteriuria
e) Proteinuria
The correct answer is A
Explanation
Due to the increase in glomerular filtration rate by 50%, the transport maximum for glucose can be reached at a much lower
serum concentration. Occasional spillage of glucose in the urine is a normal finding in pregnancy.
B. Routine urinalysis should not have the presence of blood.
C. Routine urinalysis should not show evidence of infection.
D. Routine urinalysis should not show evidence of infection.
E. True proteinuria is abnormal in pregnancy and may either represent preeclampsia or an underlying renal abnormality.
A 23 year old white primigravida comes in for her regular prenatal visit. Her past medical history is unremarkable. Physical
examination is notable only for a gravid uterus consistent with a 36 week gestation. Screening and appropriate management
of which one of the following genitourinary infections at this visit has been shown to reduce the patients risk for preterm
delivery?

a) Group B streptococcal bacteriuria


b) Bacterial vaginosis
c) Trichomonas vaginitis
d) Ureaplasma urealyticum
e) Chlamydia trachomatis
The correct answer is A
Explanation

Offer all women screening for group B streptococcal disease at 35 to 37 weeks' gestation (with culture done from one swab
first to the vagina then to the rectal area), since it appears that identifying and treating asymptomatic baceteriuria decreases
the risk of preterm delivery (/Society of Obstetricians and Gynaecologists of Canada)/. It has been specifically demonstrated
that treatment of group B streptococcal bacteriuria decreases the rate of preterm delivery. Although studies have shown an
association between preterm delivery and bacterial vaginosis and Trichomonas vaginitis, studies have not shown that
treatment reduces the risk. It remains uncertain whether or not chlamydial infection is associated with preterm delivery.
The definition of vaginismus is
a) Vaginal tightness causing pain and inability to have intercourse
b) Painful during sexual intercourse
c) Pain during menstruation
d) When no hymenal opening is present
The correct answer is A
Explanation
Vaginismus is vaginal tightness causing discomfort, burning, pain, penetration problems, or complete inability to have
intercourse.
The other choices b), c) and d) are definitions for dyspareunia, dysmennorhea and imperforate hymen respectively.

At the time of her annual examination, you find an 11-week-sized irregular uterus on an asymptomatic 40-year-old woman.
Her last exam 1 year prior was normal. Your next step in the management of this patient is:
a) Hysterectomy
b) Endometrial biopsy
c) Reexamination in 6 months
d) Fractional dilation and curettage
e) Gonadotropin releasing hormone agonist therapy
The correct answer is C
Explanation
Management of an asymptomatic 45-year-old with leiomyomata: reexamine in 6 months. Leiomyomas are a frequent finding
in a reproductive age woman. If they are asympatomic (absence pf pain, menorrhagia, urinary symptoms, gastrointestinal
symptoms), and if they are small and not rapidly changing in size, then they can be followed. Since her last exam 1 year ago
was reportedly normal, reexamination in less than 1 year would be appropriate.
A. Indicated for symptomatic fibroid uterus in a woman who does not desire fertility.
B. Necessary only if the woman is having abnormal uterine bleeding.
D. See answer to B.
E. Can be used for symptomatic leiomyomas in a reproductive age woman, but no more than 6 months of continuous
therapy. This woman is without symptoms.
You see a 17-year-old female for a routine visit. She tells you she has been sexually active for 3 years and that her partners
have used condoms, but not consistently. She has never had a Papanicolaou (Pap) test. You provide counseling regarding
contraception and sexually transmitted diseases, and perform a gynecologic examination, including a Pap test. The results of
the test are reported as atypical squamous cells of undetermined significance (ASC-US). According to the guidelines for
Colposcopy and Cervical Pathology, which one of the following would be most appropriate with regard to the abnormal
smear?
a) A repeat Pap test in 6 months
b) A repeat Pap test in 12 months
c) HPV DNA testing
d) Colposcopy
e) Loop electrical excisional procedure (LEEP)
The correct answer is A
Explanation
The risk associated with abnormal results on cytologic testing vary with the age of the patient. Adolescents have a high
prevalence of human papillomavirus (HPV) infection, but a very low risk for invasive cervical cancer. The vast majority of HPV
infections will clear within 2 years after the initial infection, and have little clinical significance.

ascus diagram diagnos check shavad

A patient at 40 weeks gestation has had a fundal height 3-4 cm greater than expected relative to dates for the last several
visits. Ultrasonography 2 days ago showed a fetus in the vertex position with an estimated fetal weight of 4200 g (9 lb 4 oz).
On examination today the patients cervix is closed, long, posterior, and firm, with the vertex at ?2 station. Her pregnancy
has been otherwise uncomplicated. Appropriate management at this point would be

a)
b)
c)
d)

Cesarean section
Induction of labor with oxytocin (Pitocin)
Cervical ripening with prostaglandins
Scheduling a routine prenatal visit in 1 week
The correct answer is D

Explanation
Fetal macrosomia at term is defined by various authorities as birth weight above 4000 ? 4500 g. Ultrasonography,
unfortunately, does not provide a particularly accurate estimate of fetal weight for large fetuses. The risk of difficult vaginal
delivery and shoulder dystocia does increase with birth weight above 4000 ? 4500 g. This has led to attempts to prevent
shoulder dystocia and possible birth injury by either performing an elective cesarean section or inducing labor when the
fetus is estimated to be macrosomic. However, no studies have shown a benefit to either intervention in otherwise
uncomplicated pregnancies. Suspected macrosomnia on its own is not longer considered an indication for induction or
cesarean section. However, should this patient not spontaneously go into labor she will soon need to be managed as a postdates pregnancy and thus a return visit should be scheduled in a week.
A 24 year old female presents with abdominal pain. Beta-HCG is negative. Pelvic ultrasound shows a 5 cm right ovarian cyst.
You would

a) Perform immediate laporotomy


b) Perform immediate laproscopy
c) Aspirate the cyst under ultrasonographic guidance
d) Order a CBC and a CA125
e) Expectant management with repeat ultrasound in 8 weeks
The correct answer is E
Explanation
The management of ovarian cysts depends on a number of factors, including age of the woman, size of the cyst, type of cyst
as determined by ultrasound (simple or complex), level of CA-125 and the presence or not of symptoms.
If ultrasound identifies that the cyst is simple, a wait-and-see plan ('expectant management') may be appropriate, because
many simple ovarian cysts resolve spontaneously. In fact one study of 278 women aged 14 to 81 years with simple cysts
found that 44% of cysts resolved with no treatment.
With expectant management, the woman has a repeat ultrasound 6-8 weeks after the simple cyst was first diagnosed. In the
past, combined oral contraceptives were often prescribed to pre-menopausal patients during this time, but it is now
accepted that these agents only prevent the development of functional cysts and do not suppress them. If the cyst has
persisted after the observation period, then the patient is usually referred for surgical evaluation.
A 28-year-old female at 20 weeks gestation is hospitalized with
pyelonephritis and treated with appropriate intravenous antibiotics. A

urine culture is positive for a pan-sensitive organism and you plan to


discharge her with a prescription for a course of oral antibiotics.
Which one of the following would be CONTRAINDICATED for her outpatient
treatment?
a) Amoxicillin
b) Cephalexin (Keflex)
c) Cefixime (Suprax)
d) Levofloxacin (Levaquin)
e) Amoxicillin/clavulanate (Augmentin)
The correct answer is D
Explanation
Most antibiotics cross the placenta, making it necessary to avoid those
that may be harmful to the fetus. Penicillins, cephalosporins, and
nitrofurantoin are commonly used in pregnancy, and there is no evidence
for adverse fetal outcomes. Fluoroquinolones are contraindicated in
pregnancy because they have been found to cause an irreversible
arthropathy in animal studies. Nitrofurantoin is safe and effective for
the treatment of cystitis, but does not attain adequate renal
parenchymal penetration for treatment of pyelonephritis.
The two most common indicators for transcervical aminoinfusion in labor are

a)
b)
c)
d)
e)

Oligohydramnios and intrauterine growth restriction


Variable decelerations and meconium-stained fluid
Umbilical cord prolapse and amniotic fluid embolism
Late decelerations and placental abruption
Hypertonic labor and low fetal scalp pH

Explanation
Transcervical amnioinfusion is an effective treatment for severe
variable decelerations in labor. Controlled prospective trials have
established that it relieves variable decelerations in a majority of
cases and allows the parturient to continue in labor. Its use for
preventing meconium aspiration is less well established, but a Cochrane
review recently favored its use for the patient in labor with a thick
meconium.
The procedure is not indicated for oligohydramnios or fetal growth
restriction, although pregnancies with these complications have a higher
risk of developing indications for amnioinfusion. Umbilical cord
prolapse and amniotic fluid embolism may be complications of
amnioinfusion, but this is not fully established. Late decelerations,
placental abruption, hypertonic labor, and low fetal scalp pH are all
contraindications to the use of amnioinfusion.
A 24 year old female has a positive pregnancy test, she presents with 2
day history vaginal bleeding. An ultrasound shows a 3 cm mass in the
left adnexa and an empty uterus. Her pregnancy symptoms and signs have
disappeared and her cervix is closed. What is the most likely diagnosis

a) Threatened abortion
b) Inevitable abortion
c) Complete abortion
d) Incomplete abortion
The correct answer is C
Explanation The following table explains the different types of
abortions that may occur during a pregnancy:
Type of abortion
Threatened
Inevitable
Incomplete
Complete

Vaginal bleeding
y
y
y
y

Cervical dilation
n
y
y
y or n

The above patient most likely has had a complete abortion.

Passage of products of conception


n
n
y
y

A 26-year-old gravida 1 para 0 at 28 weeks gestation has a 1-hour plasma


glucose level of 145 mg/dL on a 50 g/hr glucose challenge test. A 3-hour
glucose tolerance test confirms gestational diabetes. You initially
recommend home glucose monitoring and treatment with diet and exercise.
You would recommend insulin therapy if her 2-hour postprandial blood
glucose levels are not consistently below a target level of

a) 80 mg/dL
b) 100 mg/dL
c) 120 mg/dL
d) 140 mg/dL
e) 160 mg/dL
The correct answer is C
Explanation
Patients diagnosed with gestational diabetes should receive nutrition
counseling, monitor their blood glucose levels, and exercise to help
maintain normoglycemia. The commonly accepted treatment goal is to
maintain a fasting capillary blood glucose level <95?105 mg/dL. The goal
for the postprandial capillary blood glucose level should be <140 mg/dL
at 1 hour and <120 mg/dL at 2 hours.
A patient at 28 weeks EGA was hospitalized recently with preterm labor.
Contractions ceased after parenteral tocolysis and she was placed on 2.5
mg of oral terbutaline every 4 hours. When she is seen on rounds the
following morning, she is asymptomatic, her lungs are clear to
auscultation, and she denies uterine contractions. Pulse rate is 110 bpm
and regular. A II/VI systolic murmur is noted along the left sternal
border. The next appropriate step in the management of this patient is to:
a) Decrease the dose of terbutaline
b) Discontinue the tocolytic agent
c) Continue present management
d) Obtain an EKG
e) Lengthen the interval between doses
The correct answer is C
Explanation
The use of oral beta adrenergic tocolytics is controversial. In order to
reach a therapeutic level, significant maternal side effects are usually
seen, the most common of which are cardiovascular and metabolic. Resting
tachycardia and a murmur of increased flow are usually present. The
development of arrhythmias or pulmonary edema usually requires stopping
therapy and switching to another class of tocolytic agent. Oral doses
are given every 2-4 hours and continued until 34-36 weeks gestation.
In order to reach a therapeutic dose of an oral tocolytic,
cardiovascular side effect are commonly present as a result.
The patient does not have an arrhythmia, EKG not needed.
Intervals longer than every 4 hours are usually not effective with oral
terbutaline as a tocolytic.
A 32 year old sexually active female has a Pap smear that shows moderate
to severe dysplasia. What is the next step?
a) Colposcopic examination
b) Cone biopsy
c) Laser ablation of the cervix
d) Repeat Pap in 4-6 months
The correct answer is A
Explanation This algorithm demonstrates how to proceed with pap smear
results:

All women who have a pap smear that shows moderate to severe dysplasia
should have a colposcopy with biopsy done.
Which of the following is the easiest and most reliable way of detecting
a retained succenturiate placental lobe?

a)
b)
c)
d)
e)

Palpation of the uterus


Inspection of the maternal side of the placenta
Visualization of the cervix on speculum exam
Inspection of the fetal side of the placenta
Pelvic ultrasound

The correct answer is D


Explanation
The maternal surface of the placenta should be inspected to be certain
that all cotyledons are present. Then the fetal membranes should be
inspected past the edges of the placenta. Large vessels beyond these
edges indicate the possibility that an entire placental lobe (e.g.,
succenturiate or accessory lobe) may have been retained. This can be
detected by inspection of the fetal side of the placenta.
An 18 year old pregnant woman presents to the clinic for a routine checkup.
She is at the 5th week of gestation. Except for morning nausea, she
denies any problems with her pregnancy so far. The patient is allergic
to penicillin. Physical exam is unremarkable and appropriate for
gestational age.
Routine screening lab test were ordered. VDRL screening returned
positive and was confirmed by the FTA-ABS test.
Which of the following is considered the best management of this patient?

a) Ceftriaxone
b) Doxycycline
c) Erythromycin
d) Penicillin desensitization
e) Tetracycline
The correct answer is D
Explanation

The drug of choice for treatment of a pregnant woman with syphilis and
allergic to penicillin is penicillin desensitization. Syphilis in
pregnancy is associated with mental retardation, stillbirth and sudden
infant death syndrome; therefore it should be treated promptly.
Erythromycin does not cross placenta. Tetracyclin and doxycycline are
contraindicated in pregnancy and ceftriaxone is much less effective than
penicillin.
This is a hot topic for the Licensing exams!

A term baby, delivered by spontaneous vaginal delivery, is noted to have


a large cephalohematoma. His birth weight is 4900 grams. Which one of
the following maternal factors is most likely to have contributed to the
baby's clinical condition?

a)
b)
c)
d)
e)

Preeclampsia
Diabetes mellitus
Alcohol abuse
Familial obesity
Graves disease
The correct answer is B

Explanation
During the birth of a normal infant, there are several maternal and
infant factors as well as obstetric interventions that may contribute to
birth trauma.
Risk factors for birth trauma include the use of instrumentation (i.e.
forceps, vacuum), large for gestational age babies (>4500 grams),
macrocephaly, vaginal breech delivery, prolonged labor, prima gravidity,
and excessive force or traction during delivery. Maternal diabetes is a
known cause of babies that are born large for gestational age.
Cephalohematoma is a subperiosteal hemorrhage. Because the bleeding is
subperiosteal, the hematoma will be confined by the suture lines where
the periosteum is bound tightly to the bone. Cephalohematomas are
gradually resorbed and may take from two weeks to three months to
resolve completely.
23 year old woman regularly has episodes of pain associated with
menstrual periods. Following investigations, you diagnose endometriosis.
Which one of the following is the commonest site of this disease outside
of the pelvis

a) Gastrointestinal tract
b) Lung
c) Pleura
d) Kidney
e) Spleen
The correct answer is A
Explanation
In endometriosis, small or large patches of endometrial tissue, which
are usually located only in the lining of the uterus (endometrium),
appear in other parts of the body. How and why the tissue appears in
other locations is unclear. When this happens it causes dysmenorrhea,
dysparuenia and infertility.
The most common locations include the ovaries, fallopian tubes, and
ligaments supporting the uterus. But the misplaced tissue may also
appear in other locations in the pelvis and abdomen.
Common locations of misplaced endometrial tissue (called implants)
include the ovaries, the ligaments that support the uterus, the space
between the rectum and vagina or cervix, and the fallopian tubes. Less
common locations include the outer surface of the small and large
intestines, the ureters (tubes leading from the kidneys to the bladder),

the bladder, and the vagina.

Which of the following is true regarding mastitis


a) Gram negative infection
b) Incision and drainage is the treatment
c) Associated with lactation
d) Antibiotics not used
The correct answer is C
Explanation
Mastitis is painful inflammation of the breast, usually accompanied by
infection.
Fever later in the puerperium is frequently due to mastitis. Symptoms
include high fever, erythema, induration, tenderness, pain, swelling,
and warmth to the touch. Treatment includes encouragement of fluid
intake and antibiotics aimed at Staphylococcus aureus, the most common
causative pathogen. Examples are dicloxacillin and, for women allergic
to penicillin, erythromycin. Breastfeeding should be continued during
treatment because treatment includes emptying the affected breast.
All of the following are considered possible indications for cesarean
section, except
a) Placenta previa
b) Fetal distress
c) Genital herpes
d) Carcinoma in situ of the cervix
e) Prior urethropexy
The correct answer is D
Explanation
Abnormal placentation (as in the presence of a placenta previa),
nonreassuring fetal heart rate, genital herpes infections and a prior
urethropexy are all indications for a cesarean section. Carcinoma in
situ of the cervix is not.
The most common pathogen causing urinary tract infection (UTI) in
pregnancy is
a) Pseudomonas aeruginosa
b) Proteus mirabilis
c) Haemophilus influenzae
d) Escherichia coli
e) Klebsiella pneumoniae
Correct Answer:* d)
Explanation
Urinary tract infections are common during pregnancy, and the most
common causative organism is Escherichia coli. Asymptomatic bacteriuria
can lead to the development of cystitis or pyelonephritis. All pregnant
women should be screened for bacteriuria and subsequently treated with
antibiotics such as nitrofurantoin, sulfisoxazole or cephalexin.
Ampicillin should no longer be used in the treatment of asymptomatic
bacteriuria because of high rates of resistance.

Pyelonephritis can be a life-threatening illness, with increased risk of


perinatal and neonatal morbidity. Recurrent infections are common during
pregnancy and require prophylactic treatment. Pregnant women with
urinary group B streptococcal infection should be treated and should
receive intrapartum prophylactic therapy.
Which one of the following is true regarding the fetal biophysical profile?
a) Adequate amniotic fluid is defined as two or more pockets of fluid measuring > 2 cm in vertical diameter
b) The biophysical profile combines multiple observations from dynamic ultrasonography with a nonstress
test
c) A biophysical profile score of 8 out of 10 (8/10) indicates adequate fetal reserve for at least 7 days
d) The biophysical profile is useful for identifying intrauterine growth retardation
The correct answer is B
Explanation
The fetal biophysical profile is a means of assessing fetal well-being
by combining information from dynamic ultrasonography with fetal heart
rate response to fetal movements. The ultrasound components include
fetal breathing movements, gross body movements, fetal tone, and a
qualitative assessment of amniotic fluid volume. This test is commonly
done after 28 weeks gestation (after which fetal viability is likely)
and can be used to assess fetal well-being in patients with intrauterine
growth retardation (IUGR) or other high-risk pregnancies.
The test is performed at varying frequencies depending upon the
obstetric or fetal risk level. A normal score (10/10) is reliable
evidence of fetal well-being at the time of the test and that fetal
demise is unlikely in the following 7 days. Lower scores have variable
interpretations, depending on the clinical circumstances. For example, a
score of 8/10 with low amniotic fluid volume may indicate a risk for
acute decompensation. Adequate amniotic fluid is defined as one of more
pockets of fluid > 2 cm in vertical diameter.
A 23-year-old white female has a 10-year history of headaches. She
reports no preceding illness or injury, and no obvious precipitating
event. Her mother and older sister also have headaches. On average, she
has one to two attacks of right or left hemicranial throbbing headaches
every month. The pain intensifies with almost any physical activity, and
some attacks are so severe she has to stay home. A typical attack begins
gradually, reaches peak intensity in 2-3 hours, and lasts 1-2 days if
not adequately treated. There is accompanying sensitivity to light and
sound, and often nausea and vomiting. She says that nearly all of the
attacks occur within 2 days before or after the first day of
menstruation. Other identifiable triggers include stress, hunger, strong
smells, and foods high in monosodium glutamate.
Which one of the following would be the preferred medication for this
patients headaches
a) Hydrochlorothiazide
b) Alfa-Tocopherol (vitamin E)
c) Medroxyprogesterone (Provera)
d) Bromocriptine (Parlodel)
e) Ibuprofen
The correct answer is E
Explanation
Menstrual migraine begins at menarche in one-third of affected women. It
occurs mainly at the time of menses in many women and exclusively with
menses in some. Menstrual migraine is usually not associated with aura
and often lasts longer than nonmenstrual migraine. The headaches occur
just before or during menses as a result of estrogen withdrawal,
mediated by prostaglandins. NSAIDs are effective first-line treatment,
because they inhibit prostaglandin synthesis or block prostaglandin
receptors. If the initial NSAID is ineffective, other classes of NSAIDs
should be tried. When headaches cannot be controlled with NSAIDs, a 5-HT
agonist (triptan) may be used. Diuretics and vitamins are popular but
ineffective treatments. Estrogens, with or without progesterone, can be
used as preventive treatment, but progesterones alone will not prevent
migraines.
A woman presents with vaginal bleeding. Her pregnancy test is positive.

However an ultrasound shows an empty uterus and left adnexal mass. What
is the most likely diagnosis

a) Acute salpingitis
b) Ectopic pregnancy
c) UTI
d) Trichomonas
The correct answer is B
Explanation
In ectopic pregnancy, implantation occurs in a site other than the
endometrial lining of the uterine cavity?in the fallopian tube, uterine
interstitium, cervix, ovary, or abdominal or pelvic cavity. Ectopic
pregnancies cannot be carried to term and eventually rupture or involute.
Early symptoms and signs include pelvic pain, vaginal bleeding, and
cervical motion tenderness. Syncope or hemorrhagic shock can occur with
rupture. Diagnosis is by ?-human chorionic gonadotropin measurement and
ultrasonography. Absence of an intrauterine sac with a ?-hCG level >
2000 mIU/mL strongly suggests an ectopic pregnancy Treatment is with
laparoscopic or open surgical resection or with IM methotrexate.
A 15 year old patient request evaluation of masculinization and failure
to begin menstruation. She was taller than her peers during childhood.
Pubic hair growth began at 6 years, excessive facial hair growth began
at 10. She now shaves 3-4 times a week. She is 150 cm (63 inches) tall,
and her BP is 120/80. She has prominent musculature, and her breasts are
Tanner stage 2. Pelvic exam reveals an enlarged clitoris, moderate
posterior labial-scrotal fusion, and a cervix in the vaginal vault.
There are no pelvic masses on bimanual exam. This patients sex
chromosomes are most likely:
a)
b)
c)
d)
e)

XX
XXY
X/XY
XX/XY
XYY
The correct answer is A

Explanation
This patient is demonstrating signs of increased androgen production. As
her secondary sexual characteristics started appearing at age 6, she
has, by definition, had a heterosexual precocious puberty. Since a
cervix is present, the Mullerian system must have developed, meaning
that the gonad is not testes, and that there is no Y component to her
sex chromosomes. Her most likely sex chromosome pattern is XX.
B. The presence of a Y chromosome would lead the gonad to develop into
testes. Anti-Mullerian hormone would be produced, and no
cervix/uterus/fallopian tubes would develop.
C. See answer to B.
D. See answer to B.
E. See answer to B.
A 25-year-old multipara at 38 weeks presents in early labor. Leopold
maneuvers note a soft, ballotable structure at the symphysis, small
parts along the patients left side, fetal back along the patients
right side, and a hard mobile object in the right upper quadrant. The
most likely presentation is:
a) Vertex
b) Transverse
c) Breech
d) Compound
e) Face
The correct answer is C
Explanation
Leopolds can be used to assess the fetal lie. The gold standard is
ultrasound, though a vaginal exam can help confirm the findings on

abdominal exam. Only 3-4% of pregnancies at term present as nonvertex.


External cephalic version can help reduce the number of cesarean
sections done for this indication.

A. The cephalic pole should be palpable at the symphysis and the podalic
pole at one of the upper quadrants.
B. No palpable pole should be noted at the symphysis.
D. This can be difficult to determine via Leopold maneuvers and is often
detected only by pelvic exam.
E. This is usually detected by pelvic exam.
A 17 year old white female has had no menstrual period for the last 6
months. She has no significant previous medical history, but admits to a
history of irregular periods since menarche at age 14. A urine pregnancy
test is negative and a physical examination is unremarkable. The most
appropriate initial workup of this patient would include which one of
the following?

a) Reassurance and reevaluation in 1 month


b) Measurement of serum FSH and LH
c) Administration of conjugated estrogens
d) Single administration of medroxyprogesterone acetate
e) A CT scan of the brain
The correct answer is B
Explanation
Menses are often irregular in young adolescents; however, menstrual
cycles usually become regular within 2 years after menarche. Secondary
amenorrhea is present when a female who has been menstruating has not
had a period for more than three cycle intervals, or 6 months. Once
pregnancy has been ruled out, a disturbance in the
hypothalamic-pituitary-ovarian axis should be investigated.
When the history and physical examination fail to reveal any clues to
the diagnosis, the initial workup begins with a serum
thyroid-stimulating hormone (TSH) level, LH and FSH levels, a prolactin
level, and then a progestational challenge. The purpose of the
progestational challenge, without the addition of any exogenous estrogen
production and the uterus are normal. If the patient develops menstrual
bleeding 2-7 days after completing the progesterone regimen, the
diagnosis of anovulation can safely be made.
Evaluation for a pituitary adenoma is reserved for patients with

galactorrhea with or without an elevated prolactin level. Serum FSH and


LH levels are investigated when the patient fails to have withdrawal
bleeding after a progestational challenge, and to assess gonadotropin or
follicular activity.
A woman delivers her sixth child. Post delivery approximately 1 and half
hours later she develops profuse vaginal bleeding. What is the most
common reason for this

a) Uterine atony
b) Cervical laceration
c) Laceration in the vulva
d) Endometritis
The correct answer is A
Explanation
Postpartum hemorrhage is blood loss of > 500 mL during or immediately
after the 3rd stage of labor.
Risk factors for bleeding include uterine atony due to overdistention
(caused by multifetal pregnancy, polyhydramnios, or an abnormally large
fetus), prolonged or dysfunctional labor, grand multiparity (delivery of
? 5 viable fetuses), relaxant anesthetics, rapid labor,
chorioamnionitis, and retention of placental tissue (eg, due to placenta
accreta).
Treatment involves intravascular volume replenishment with IV fluids.
Blood transfusion is used if this volume of saline is inadequate.
Hemostasis is attempted by bimanual uterine massage and IV oxytocin.
A previously low-risk primigravida is found to have a uterine
measurement of 33 cm at her 37-week visit. On pelvic examination there
is no evidence of leaking fluid. A sonogram shows borderline
oligohydramnios with an amniotic fluid index (AFI) of 6.5. No anomalies
are evident on the sonogram and the estimated fetal weight is at the
50th percentile for gestational age. A nonstress test is reactive.
Which one of the following would be most appropriate at this point?
a) Cesarean delivery
b) Induction of labor
c) Amniocentesis and induction of labor if fetal lung maturity is confirmed
d) Repeat nonstress testing and AFI measurement in a few days
The correct answer is D
Explanation
Borderline oligohydramnios, defined as an amniotic fluid index (AFI) of
5.0-8.0, has not been associated with poor outcomes except when a fetal
anomaly or growth restriction is present. After testing for normal
growth and fetal well-being, regular follow-up is adequate. Even in the
presence of true isolated oligohydramnios (AFI <5), the patient can be
followed if the hydration raises the AFI above 5, as long as
intrauterine growth retardation and anomalies are not present. At 41
weeks gestation and beyond, oligohydramnios is an indication for
induction of labor.
A 36 year old female, who is not pregnant comes in with chief complaint
of galactorrhea. Which of the following would be an appropriate lab test
to order to investigate the cause?
a) Beta-HCG
b) Urinalysis
c) CT of head
d) TSH/FT4
e) Prolactin level
The correct answer is E
Explanation
Galactorrhea, or inappropriate lactation has a wide variety of causes
which include medicines, abnormal hormones levels, pregnancy and tumors
of the pituitary gland.

In this patient a good first step would be to obtain some basic labs
which should include a prolactin level. Prolactin is a hormone produced
the anterior pituitary. Use the mnemonic ?FLAT PiG? to remember the
hormones produced by the anterior pituitary (FSH, LH, ACTH, TSH,
Prolactin and Growth Hormone). Prolatin stimulates the mammary glands to
produce milk (lactation).
See below for a basic algorithm for the initial assessment of galatorrhea:

A woman who reports two previous miscarriages and family history of


neural tube defects (anencephaly) is planning a pregnancy within next 6
months. What is the best advice you would give?

a) Start folic acid now


b) Start multivitamins now
c) Start calcium now
d) Start vitamin D now
The correct answer is A
Explanation
Insufficient folic acid (folate) in the diet increases the chance that a
fetus will develop spina bifida (in which the spine does not completely
enclose the spinal cord) and anencephaly (in which a large part of the
brain and skull is missing) or other abnormalities of the brain or
spinal cord known as neural tube defects.
Prenatal diagnosis by amniocentesis and ultrasonography is recommended
for couples who have at least a 1% risk of having a baby with a neural
tube defect.
The risk of having a baby with a neural tube defect is increased by
having a family history (including the couple's own children) of such
defects. For couples who have had a baby with spina bifida or
anencephaly, the risk of having another baby with one of these defects
is 2 to 3%.
A 17 year old girl complains of severe bleeding during her menstrual
cycle. She uses 20 tampons per cycle, and it lasts for 7 days and comes
every 2 months. What is the most appropriate management?

a)
b)
c)
d)

NSAIDS
Iron
Use more tampons/pads
OCP

The correct answer is D


Explanation
Menorrhagia is a term used to define menstrual periods lasting longer
than seven days. Drug therapy for menorrhagia may include:
Iron supplements: If the condition is accompanied by anemia, your doctor
may recommend that you take iron supplements regularly. If your iron
levels are low but you're not yet anemic, you may be started on iron
supplements rather than waiting until you become anemic.
Nonsteroidal anti-inflammatory drugs (NSAIDs): NSAIDs such as ibuprofen
(Advil, Motrin, others) help reduce menstrual blood loss. NSAIDs have
the added benefit of relieving painful menstrual cramps (dysmenorrhea).
Oral contraceptives (OCPs): Aside from providing effective birth
control, oral contraceptives can help regulate ovulation and reduce
episodes of excessive or prolonged menstrual bleeding.
Which is true about congenital anomalies
a) Congenital anomalies rarely affect growth
b) If symmetrical IUGR is detected, look for congenital anomalies
c) Trisomy 13 and 18 will not cause growth retardation
d) Asymmetrical IUGR usually occurs early in the first trimester
The correct answer is B
Explanation
During routine prenatal care, if a fetal ultrasound shows Intra-uterine
Growth Retardation (IUGR) it is important to investigate further.
In Symmetrical IUGR there is inadequate growth of the head, body and
extremities. The growth problem is the result of a decrease in the rate
of cell reproduction, resulting in fewer cells. This usually has its
onset prior to 32 weeks of pregnancy and has a 25% risk for chromosomal
abnormalities (Down syndrome, trisomy 13, trisomy 18) which should
definitely be investigated.
Asymmetrical IUGR usually occurs early in the third trimester and
implies a fetus who is undernourished and is directing most of its
energy to maintaining growth of vital organs, such as the brain and
heart, at the expense of the liver, muscle and fat. This type of growth
restriction is usually the result of placental insufficiency.

If the woman is not breast-feeding, menstruation could return soon after


delivery. After what period of time, 70-90% of women menstruate?

a)
b)
c)
d)
e)

4 weeks
2 months
3 months
6 months
> 8 months

The correct answer is C


Explanation
If the woman is not breast-feeding, menstruation could return within 6
weeks of delivery. After 3 months, 70-90% of women menstruate; 6 months
later, almost all have resumed menstruation (average time for resumption
is 8 weeks). This timeline varies widely if the mother is
breast-feeding. Restoration of normal menses can be interfered with by
injectable contraceptives, stress/depression or sometimes even sleep
deprivation with a new baby.
A 23 year old woman who is 39 weeks pregnant with her second child is
having regular uterine contractions every three minutes. Which one of
the following is the best criterion for assessing if she has entered the
active phase of labour?
a) The cervix is effaced over 90%
b) The contraction duration is over 30 seconds
c) The presenting part is low in the pelvis
d) The cervical dilation is at least 4 cm
e) The membranes are ruptured
The correct answer is D
Explanation
Active labor usually occurs after the cervix dilates to ? 4 cm.
Normally, cervical dilation and descent of the head into the pelvis
proceed at a rate of at least 1 cm/h and more quickly in multiparous women.
The daily intake of vitamins and minerals recommended varies according
to sex, age, and condition. The recommended daily allowance of vitamin D
is greatest for which one of the following?

a) A 15-year-old nonpregnant female


b) A 25-year-old pregnant female
c) A 35-year-old lactating female
d) A 55-year-old female
e) A 75-year-old female
The correct answer is E
Explanation
The current Dietary Reference Intake (DRI) recommendation for vitamin D
is 200 IU/day for all women between the ages of 9 and 50 years;
pregnancy or lactation does not affect the recommendation. The DRI
doubles to 400 IU daily for women age 51?70 and triples to 600 IU daily
for women over the age of 70. The maximum daily oral intake of vitamin D
thought to be safe is 2000 IU/day for all females over the age of 12 months.
The main contraceptive action of the copper-based intrauterine device is:
a) Prevention of implantation of the fertilized ovum
b) Cessation of ovulation
c) Induced abortion
d) Production of a spermicidal environment
e) Elevation of serum copper level
The correct answer is D
Explanation
The main mechanism of the contraceptive action of *copper* bearing IUDs
in the human is as a spermicide. The presence of a device in the uterus
prompts the release of leukocytes and prostaglandins by the endometrium.
These substances are hostile to both sperm and eggs; the presence of
copper increases the spermicidal effect. Because of the spermicidal
action of the copper IUD, few, if any, sperm reach the oviduct, and the
ovum usually does not become fertilized.

A. Although there is a local inflammatory reaction, the main effect is


spermicidal.
B. No effect on ovulation occurs from the copper IUD since it is
nonhormonal.
C. See the answer to A.
E. No change in serum copper level occurs.

In a 27 year old white female with irregular menstrual cycles and


infertility, which one of the following would be more indicative of
Cushings syndrome rather than the more common polycystic ovarian syndrome?

a) Easy bruising
b) Acne
c) Hirsutism
d) Androgenic alopecia
e) Acanthosis nigricans
The correct answer is A
Explanation
Easy bruising, moon facies, buffalo hump, abdominal striae,
hypertension, and proximal myopathy suggest Cushings syndrome. Because
this syndrome is very rare compared to polycystic ovarian syndrome,
routine screening is not indicated in women with hypoangrogenic anovulation.
A pregnant woman comes to you for her first prenatal visit at 36 weeks
gestation. What is an appropriate test at this time?

a) Triple Marker (Triple screen)


b) Amniocentesis
c) Screen for gestational diabetes
d) Group B strep
The correct answer is D
Explanation
Routine prenatal care begins at the onset of pregnancy. By 36 weeks
gestation the majority of the tests that are usually done are no longer
able to be done since they fall well outside the recommended time frame
for the test results.
The following is a brief summary of the approximate time for some of the
prenatal tests:
Triple marker: 15-18 weeks gestation
Amniocentesis: if necessary, usually15-18 weeks
Screen for gestational diabetes: 24-28 weeks gestation
Group B strep: 35-37 weeks gestation
A 19 year old patient presents with acne on her face and increase hair
growth on her body. During physical exam you palpate an adnexal mass.
What is the most likely diagnoses
a) Ovarian tumour

b) Dermoid cyst
c) Idiopathic androgen secretion
d) PCOS
The correct answer is D
Explanation
Polycystic ovary syndrome (PCOS) is characterized by mild obesity,
irregular menses or amenorrhea, and signs of androgen excess (hirsutism,
acne). Typically, the ovaries contain multiple cysts. Typically, ovaries
contain many 2-6 mm follicular cysts and sometimes larger.
Testing includes pregnancy testing and measurement of serum estradiol,
follicle-stimulating hormone, prolactin, and thyroid-stimulating
hormone. Diagnosis is confirmed by ultrasonography showing > 10
follicles per ovary.
A 25 year old white female comes to your office for counseling regarding
birth control. She has had compliance problems with oral contraceptives
and asks about alternatives. You discuss various options including the
vaginal contraceptive ring (NuvaRing), and she asks for more
information. Which one of the following is true regarding the advantages
and disadvantages of this form of contraception?

a) It protects against sexually transmitted diseases


b) To assure maximum contraceptive protection, it should be replaced every 3 months
c) It is associated with a higher incidence of breakthrough bleeding than levonorgestrel/ethinyl estradiol
pills
d) Up to 50% of women have significant difficulty inserting the device correctly
e) If the device is expelled from the vagina for more than 3 hours, backup contraception should be used
temporarily
The correct answer is E
Explanation
The vaginal ring works by releasing etonogestrel and ethinyl estradiol
intravaginally (etonogestrel is a progestin and ethinyl estradiol is an
estrogen). Because it is not a barrier method of contraception, it does
not protect against STDs. It is currently recommended that the ring be
left in place for 3 weeks and then removed for 1 week so that withdrawal
bleeding occurs. A new ring is then inserted. The vaginal ring has a
lower incidence of breakthrough bleeding than levonorgestrel/ethinyl
estradoil oral contraceptives. In a 1-year study, the majority of women
who used the vaginal ring considered insertion and removal of the device
easy, and 90% used the device correctly. If for some reason the ring is
out of the vagina for more than 3 hours, back-up contraception should be
used until the device has been back in place for 7 days.
An obese woman presents with erythema on her vulva. The area is also
whitish, showing satellite lesions. You diagnose her with candida
vaginitis. What is the most likely predisposing disease this patient has

a) Diabetes mellitus
b) Vulvar carcinoma
c) Lichen sclerosis
d) Trichomonas
The correct answer is A
Explanation
Candidiasis is skin infection with Candida albicans. Infections can
occur anywhere and are most common in skinfolds and web spaces, on the
penis or vagina, and around fingernails. Symptoms and signs vary by
site. Diagnosis is by clinical appearance and potassium hydroxide wet
mount of skin scrapings. Treatment is with drying agents and antifungals.
Risk factors for candidiasis include hot weather, restrictive clothing,
poor hygiene, infrequent diaper or undergarment changes in children and
elderly patients, altered flora from antibiotic therapy, and
immunosuppression resulting from corticosteroid and immunosuppressive
drugs, pregnancy, diabetes, other endocrinopathies.
A 32 year old white female comes to see you because of moderately severe

pelvic pain that has been present for several years and is worse with
menses. She describes the pain as bilateral, deep in the pelvis, and
intermittently cramping and steady. She has never been pregnant,
although she has not been using any contraception during the 6 years she
has been married. She is not interested in fertility at this time. She
has no history of previous sexually transmitted disease, IUD use, or
abdominal or pelvic surgery. She currently uses ibuprofen, 600 mg 3-4
times a day as needed, with moderate pain relief. She is a nonsmoker.
Physical examination reveals a blood pressure of 120/70 mm Hg and normal
findings on examination of the heart, lungs, and abdomen. The vagina and
cervix are normal in appearance. Bimanual examination reveals a
normal-sized uterus and adnexa with no masses, but mild tenderness on
palpation of the posterior uterus and posterior cul-de-sac. Recent
screening laboratory work was normal, including a CBC, thyroid function
tests, lipid levels, and liver function tests. What is the most
appropriate management at this time?
a) Referral for hysterosalpinography
b) Prescribing a COX-2 inhibitor such as rofecoxib (Vioxx) or celecoxib (Celebrex) to be used instead of
ibuprofen
c) Starting her on an oral contraceptive containing both estrogen and progesterone
d) Starting her on a danazol (Danocrine), 600 mg/day
The correct answer is C
Explanation
This patient most likely has endometriosis with chronic, cyclical pelvic
pain. Since she is not interested in fertility, the next reasonable step
is to induce a hormonal pseudopregnancy using combination oral
contraceptives.
Pelvic inflammatory disease is characterized by all of the following EXCEPT:
a) Leukocystosis
b) Pelvic pain
c) Fever
d) Anemia
e) Cervical motion tenderness
The correct answer is D
Explanation
Pelvic inflammatory disease (PID) has a high association with gonorrhea
and Chlamydia. After several days of inflammation, the bacterial flora
is often polymicrobial. Pain, cervical motion tenderness, leucorrhea
from the cervical os, fever, and leukocytosis are all common signs found
when a patient presents with PID.
A. This is one of the criteria that is often used in making the
diagnosis of pelvic inflammatory disease.
B. See answer to A.
C. See answer to A.
E. See answer to A. This is due to the inflammation of the tubes and
peritoneum; moving the cervix from side to side will result in
significant pain from the stretching of the inflamed peritoneum.
Which of the following maternal serum concentrations is increased during
normal pregnancy?
a) Calcium
b) Albumin
c) Creatinine
d) Bicarbonate
e) Cholesterol
The correct answer is E
Explanation
Albumin will decrease due to dilutional effects. Bicarbonate decreases
due to increased renal excretion to correct for the respiratory
alkalosis (blowing off more CO2). Due to the 50% rise in GFR, the serum
creatinine falls. Due to the decline in serum albumin, the total serum
calcium concentration falls. Plasma levels of lipids increase during the

latter part of pregnancy, the most marked rises occurring with


triglycerides, cholesterol, and free fatty acids.
A. Serum albumin falls, and the total calcium concentration falls as well.
B. Serum albumin levels fall due to dilutional effects.
C. Creatinine falls due to a 50% increase in glomerular filtration rate.
This occurs as early as 12 weeks gestation.
D. Bicarbonate decreases as the kidney excretes more the compensate for
the drop in CO2 levels (due to increase in minute ventilation; renal
compensation for a respiratory alkalosis).
You are attending the delivery of a 34 year old gravida 2 para 1 with no
prenatal complications who entered spontaneous labor at full term
several hours ago. All fetal heart tones have been reassuring. The head
delivers in the occiput anterior position over a posterior midline
episiotomy without problems. However, the delivery stalls with the
infants chin pressing against the perineum. When there is a contraction
or the mother attempts to push, the head descends slightly and then
returns to the same position. After you call for additional assistance,
which one of the following should you do first to facilitate delivery?

a) Apply gradually increasing amounts of traction on the infants head


b) Ask an assistant to apply strong fundal pressure on the mothers abdomen
c) Deliberately fracture the infants anterior clavicle to free it from behind the mothers pubis
d) Place the mothers knees and hips into extreme flexion onto the abdomen (McRoberts maneuver)
e) Replace the infants head back inside the birth canal and prepare for immediate cesarean section
(Zavanelli maneuver)
The correct answer is D
Explanation
The scenario described represents a case of shoulder dystocia. This
complication cannot be reliably predicted prior to delivery, and all
physicians performing deliveries must be familiar with its presentation
and management. Overly vigorous traction of the infants head or neck in
this situation may cause serious damage to the infant. Having an
assistant apply moderate suprapubic pressure with gentle downward
traction of the fetal head is permissible. If this does not result in
delivery of the shoulders, the McRoberts maneuver has been universally
recognized as a safe and effective procedure for allowing the infants
anterior shoulder to be freed. Other maneuvers that are more invasive
and carry higher risks should be used only if the above maneuvers are
ineffective. Applying fundal pressure without other maneuvers has been
shown to cause a 77% complication rate and should be avoided.
The gold standard for the diagnoses of endometriosis is
a) Laparoscopy
b) Ultrasound
c) Vaginal exam
d) Pap smear
The correct answer is A
Explanation
Endometriosis is a noncancerous disorder in which functioning
endometrial tissue is implanted outside the uterine cavity. These
ectopic pieces of endometrial tissue can travel and implant in areas
such as the ovaries and uterosacral ligament, grow and cause severe pain.

Symptoms depend on location of the implants and may include


dysmenorrhea, dyspareunia, infertility, dysuria, and pain during
defecation.
Diagnosis is suspected based on typical symptoms but must be confirmed
by biopsy, usually via pelvic laparoscopy. These ectopic tissue must be
visualized.
Treatments include anti-inflammatory drugs, drugs to suppress ovarian
function and endometrial tissue growth, surgical ablation and excision
of endometriotic implants, and, if disease is severe and no childbearing
is planned, hysterectomy plus oophorectomy.
Mixing vaginal discharge with potassium hydroxide (KOH) creates an odor
that is helpful in diagnosing:
a) Bacterial vaginosis
b) Trichomoniasis
c) Moniliasis
d) Gonorrhea
e) Chlamydia
The correct answer is A
Explanation
Bacterial vaginosis is due to an overgrowth of anaerobic bacteria,
replacing the normal peroxide-producing lactobacillus species. The
discharge is described as thin and gray-white in color. It is mildly
adherent to the vaginal walls on speculum exam. In the presence of basic
environment (semen, KOH), the aromatic amines are released, giving rise
to the characteristic fishy odor.
B. In order to create an odor with a strong base, there must be aromatic
amines present. These are created by certain anaerobic bacteria.
C. See answer to B.
D. See answer to B.
E. See answer to B
The treatment of choice for thrombotic events in the antiphospholipid
antibody syndrome is
a) Intravenous steroids
b) High-dose oral steroids with a rapid taper
c) Penicillamine
d) Aspirin
e) Heparin
The correct answer is E
Explanation
Treatment of the initial thrombosis in patients with the
antiphospholipid antibody syndrome does not generally differ from
treatment of patients with the same disorder who do not have the
antiphospholipid antibody syndrome. Anticoagulation with heparin and

then subsequently with oral anticoagulation is initiated. The duration


of anticaogulation in patients without the antiphospholipid antibody
syndrome is generally 3-6 months. In patients with the antiphospholipid
antibody syndrome, the risk of recurrence is relatively high for both
arterial and venous thrombotic events. As a result, patients are
generally started on long-term (in some cases life-long) oral
anticoagulation.
The treatment of women who are pregnant and have the antiphospholipid
antibody syndrome can result in a much higher success rate for the
pregnancy. Several regimens have been studied including heparin.
Which one of the following is appropriate treatment for asymptomatic
chlamydial infection during the second trimester of pregnancy?

a) Azithromycin (Zithromax)
b) Doxycycline
c) Metronidazole (Flagyl)
d) Levofloxacin (Levaquin)
The correct answer is A
Explanation
Several clinical trials suggest that 7-day regimens of erythromycin or
amoxicillin, and single-dose regimens of azithromycin, are effective for
treating chlamydial infections during pregnancy. Doxycycline and
levofloxacin are contraindicated during pregnancy due to potential ill
effects on the fetus, and metronidazole is not effective for the
treatment of chlamydial infections.
Your patient is in the second stage of labor, and you determine that the
fetus is in face presentation, mentum anterior. Progress has been rapid
and fetal heart tones are normal. You would now

a)
b)
c)
d)

Perform an immediate cesarean delivery


Proceed with midforceps delivery
Anticipate vaginal delivery with close fetal monitoring
Manually convert to vertex presentation
The correct answer is C

Explanation
Most infants with face presentation, mentum anterior, can be delivered
vaginally, either spontaneously or with low forceps. Cesarean section is
indicated for fetal distress and failure to progress. Midforceps
delivery is not indicated. If fetal electrodes are attached, the chin is
the preferred location.
A 30 year old white primigravida asks you about the benefits and
drawbacks of corticosteroid therapy for premature labor at 30 weeks
gestation. Which one of the following statements is most accurate?
a) Therapy will decrease the risk of neonatal necrotizing enterocolitis
b) Weekly corticosteroid injections until 34 weeks gestation is the standard regimen to prevent respiratory
distress syndrome
c) Therapy is associated with a higher rate of neonatal intraventricular hemorrhage
d) Therapy is associated with a higher rate of persistent patent ductus arteriosus
e) Therapy decreases the risk of respiratory distress syndrome but not total neonatal mortality
The correct answer is A
Explanation
Treatment of premature labor with betamethasone or dexamethasone has
been associated with up to a 65% reduction in necrotizing enterocolitis
in the neonate. Weekly treatment is not generally accepted. Increasing
reports of adverse consequences with repeated courses of treatment has
caused most experts to recommend a single course of antenatal
corticosteroids for women at risk of preterm birth at 24-34 weeks gestation.
Reported benefits of antenatal steroid therapy for preterm labor include
reduced rates of intraventricular hemorrhage and persistent patent
ductus arteriosus. Both neonatal mortality and the incidence of

respiratory distress syndrome are also reduced.


A primigravida has been given meperidine during the course of her labor.
Delivery is uneventful, but the baby shows poor respiratory effort.
Immediate management includes all of the following, *except*

a) Drying all the exposed skin


b) Clearance of airway
c) Intravenous sodium bicarbonate
d) Administration of naloxone 0.01 mg/kg
e) Oxygen
The correct answer is C
Explanation
Meperidine has been studied extensively in relation to the high
incidence of neonatal respiratory depression in newborns whose mothers
received low doses of this drug. Years of study have revealed that
following intravenous maternal analgesia with meperidine there are fewer
neonatal effects if delivery occurs within 1 hour of a single 25 to 50
mg dose. The fetal dose of meperidine increases for about 3 hours
following maternal intramuscular administration.
The toxic fetal metabolite normeperidine increases for many hours and
contributes to neonatal depression. There is no longer any doubt that
fetal metabolism contributes to the peripartal formation of normeperidine.
Immediate management includes drying all the exposed skin, clearance of
airway and oxygen. Noloxone in large doses (200 micrograms) can have a
long-lasting (40 hour) competitive effect in the reversal of
meperidine-induced neonatal depression.
Which one of the following is associated with galactorrhea?

a) Hypothyroidism
b) Breast cancer
c) Fibrocystic breast disease
d) Adrenal insufficiency
e) Graves disease
The correct answer is A
Explanation
Galactorrhea, or inappropriate lactation, is a relatively common problem
with multiple causes. Systemic disease is one cause, the most common
being hypothyroidism. Low levels of thyroid hormone result in increased
levels of thyrotopin-releasing hormone, which increases prolactin
secretion. Galactorrhea and symptoms of hypothyroidism abate with
thyroid hormone replacement therapy. This condition is not associated
with breast cancer or fibrocystic disease of the breast. Cushings
disease, rather than adrenal insufficiency, is associated with galactorrhea.
A 29-year-old female presents with increasingly severe menstrual cramps.
Naproxen (Naprosyn) has provided only minimal relief. She has been
unable to take oral contraceptives because of regular migraines.
Of the following alternative approaches that have been studied, the one
with the best evidence of effectiveness for this problem is
a) Glyceryl trinitrate
b) Fish oil supplements
c) Spinal manipulation
d) Behavioral interactions
The correct answer is B
Explanation
NSAIDs are the initial therapy of choice for primary dysmenorrheal
(strength of recommendation A). Treatments supported by limited-quality,
patient-oriented evidence (strength of recommendation B) includes oral
contraceptives, depot medroxyprogesterone, acupuncture, fish oil
supplements, low-fat vegetarian diets, thiamine, toki-shakyaku-san (a
Japanese herbal product), transcutaneous electrical nerve stimulation

(TENS), and vitamin E. Measures of uncertain effectiveness (strength of


recommendation B) include behavioral interventions such as exercise,
surgical interruption of pelvic nerves, and the use of glyceryl
triniate, nifedipine, and terbutaline. Spinal manipulation has been
shown to be ineffective (strength of recommendation B).
At a routine physical examination, the Pap smear of a 27 year old woman
shows evidence of marked inflammation suggestive of moderate dysplasia
(HGSIL). Her last Pap smear 2 years ago was normal. Pelvic examination
today is normal. She has never been pregnant and her menstrual periods
are regular. She has been in a stable relationship with the same man for
3 years and she uses a diaphragm with spermicidal jelly for
contraception. The best next step is to
a) Advise the patient that her partner should use condoms for contraception and repeat the Pap smear in 3
months
b) Do colposcopic examination of the cervix after application of 5% acetic acid solution
c) Do conization of the cervix
d) Reassure the patient and repeat the Pap smear in 3 months
e) Treat the patient with metronidazole for 2 weeks and repeat the Pap smear in 3 months
The correct answer is B
Explanation
The Papanicolaou smear is a screening test for precancerous cells. Any
cervical dysplasia (formerly cervical intraepithelial neoplasia IIII,
now low-grade and high-grade squamous intraepithelial lesion)
necessitates colposcopy for definitive diagnosis. The diaphragm has not
been associated with abnormal Pap smears, so changing the method of
contraception would not be of any use. Conization of the cervix is
always preceded by a colposcopy, and is indicated for unsatisfactory
colposcopy, inconsistency of colposcopic findings with Pap smear
findings, an endocervical curettage with abnormal cells, or a lesion on
colposcopy extending into the endocervix. Reassurance and repeating the
Pap smear in three months can be used for ASCUS (atypical squamous cells
of undetermined significance); two serial ASCUS Pap smears necessitate
colposcopy.
A 35 year old woman presents with complaints of dysmenorrhea,
dyspareunia and also is unable to become pregnant with her husband for
13 months now. What is the most likely reason for her complaints
a) Pelvic inflammatory disease
b) Uterine fibroids
c) Ovarian cysts
d) Endometriosis
The correct answer is D
Explanation
Endometriosis is a noncancerous disorder in which functioning
endometrial tissue is implanted outside the uterine cavity. Symptoms
depend on location of the implants and may include dysmenorrhea,
dyspareunia, infertility, dysuria, and pain during defecation.
Pelvic pain, pelvic mass, alteration of menses, and infertility are
typical. Some women with extensive endometriosis are asymptomatic; some
with minimal disease have incapacitating pain. Dyspareunia and midline
pelvic pain before or during menses may develop. Such dysmenorrhea is an
important diagnostic clue, particularly if it begins after several years
of pain-free menses.
Diagnosis is suspected based on typical symptoms but must be confirmed
by biopsy, usually via pelvic laparoscopy.
Treatments include anti-inflammatory drugs, drugs to suppress ovarian
function and endometrial tissue growth, surgical ablation and excision
of endometriotic implants, and, if disease is severe and no childbearing
is planned, hysterectomy plus oophorectomy.
A 72 year old white female has had annual Papanicolaou smears with
normal findings for the past 30 years. She now finds it difficult and
uncomfortable to undergo pelvic examinations because of severe arthritis
in both hips, as well as vaginal atrophy. She has not been sexually
active since the death of her husband 10 years ago. What is the most

appropriate approach to further screening for cervical neoplasia in this


patient?
a) She should be screened every 3-5 years
b) She should undergo a test for the presence of human papillomavirus; if it is negative, screening can be
discontinued
c) She should undergo ultrasound examination of the pelvis every 3-5 years
d) Screening can be discontinued now
The correct answer is D
Explanation
Cervical neoplasia is thought to be the result of sexually acquired
infection with the human papillomavirus (HPV). In the patient described,
the likelihood of detecting an abnormality is minimal, and screening can
therefore be safely stopped.
The appropriate next step in a woman found to have 1 abnormal Pap smear
showing mild dysplasia (CIN1) is
a) Repeat in 1 year's time
b) Repeat in 4-6 months time with colposcopy if abnormal
c) Send patient directly for colposcopy and LEEP excision
d) Send for colposcopy
e) Repeat Pap immediately and treat for HPV infection
The correct answer is D
Explanation
Cervical cancer is usually a squamous cell carcinoma that is caused by
human papillomavirus infection or an adenocarcinoma. Early cancer is
asymptomatic; the 1st symptom of later cancer is usually postcoital
vaginal bleeding. Diagnosis is by screening cervical Papanicolaou (Pap)
test and biopsy.
CIN is graded as 1 (mild cervical dysplasia), 2 (moderate dysplasia), or
3 (severe dysplasia and carcinoma in situ).
Patients with CIN 1 findings (also known as LGSIL ? low grade squamous
intraepithelial lesion) should have a colposcopy done.
You are asked to see a young woman in the Emergency Department after an
alleged sexual assault that occurred today. She is an otherwise healthy
28 years old. A serum pregnancy test is negative. Her menstrual cycle is
regular, every 28 days, and her last period was 14 days ago. She is not
currently on contraception and desires to minimize her chance of
becoming pregnant from this episode. Of the following, the best option is:
a) Immediate placement of a copper bearing intrauterine device
b) Give Ovral 2 tablets followed by two more tablets 12 hours later
c) Start a daily low dose triphasic birth control pill
d) Start diethylstilbestrol (DES) 50 mg per day for 5 days
e) Immediate dilation and suction curettage
The correct answer is B
Explanation
Currently, postcoital birth control can be done either with an IUD or
with hormonal therapy. With OCPs, you need to give two doses, each of at
least 100 ?g of ethinyl estradiol (2 Ovral). Less than 2% of women will
become pregnant with this dose (prevents 75% of expected pregnancies),
and it can be given up to 72 hours after coitus. DES has a slightly
higher success rate, but due to the significant side effect rate,
compliance with this regimen is much less, making is less effective. An
IUD is an option if there is no risk for sexually transmitted diseases,
so it is not indicated after a sexual assault. Some countries also use
two doses of 0.75 mg levonorgestrel, which has a similar success rate to
the Ovral regimen. Note that the clinical pregnancy rate of unprotected
midcycle coitus is about 7%.
A. IUDs can be used for emergency postcoital contraception, but are not
indicated when the risk for a sexually transmitted disease is present.
C. A minimum of 100 ?g of ethinyl estradiol in two divided doses needs
to be given.
D. Significant side effects (nausea) make compliance with this regimen
much less, making it less effective.

E. At this point, the fertilized ovum is still within the fallopian tube.
Which one of the following vaccines is contraindicated in pregnancy?
a) Tetanus and diphtheria (Td)
b) Influenza
c) Rabies
d) Hepatitis B
e) MMR
The correct answer is E
Explanation
Although the risk of transmitting a virus to a developing fetus is
primarily theoretic, live-virus vaccines are generally contraindicated
in pregnancy. MMR is a live attenuated vaccine and thus should be
avoided in pregnancy. If it is inadvertently given, or a woman becomes
pregnant within 4 weeks of administration, the woman should be counseled
regarding potential adverse effects on the fetus, but should not be
advised that it is an indication for termination.
Tetanus and diphtheria vaccine (Td) should be routinely recommended for
pregnant women who have not received Td in the past 10 years, or who
have never been immunized. The influenza vaccine is a killed-virus
preparation and is recommended for all women who will be pregnant during
the influenza season.
Rabies vaccination has not been associated with any fetal abnormalities.
Due to the potential severe consequences of rabies exposure to the
mother and fetus, the guidelines for postexposure prophylaxis in
pregnancy are similar to those for the nonpregnant patient. Hepatitis B
vaccine is composed of a viral surface antigen made through recombinant
DNA technology and poses no risk to the mother or fetus. It should be
routinely recommended to women who have risk factors including multiple
sexual partners, a history of intravenous drug use or a partner with a
history of intravenous drug use, occupational exposure, or household
contact with a chronic carrier or acutely infected person.
A 28 year old pregnant woman develops sudden onset of dyspnea and
tachycardia. Which one of the following is the most likely explanation?
a) Acute mitral valve regurgitation
b) Pulmonary embolism
c) Myocardial infarction
d) Acute respiratory distress syndrome
e) Lobar pneumonia
The correct answer is B
Explanation
Thromboembolic disease is the leading cause of death in pregnant women.
In thromboembolic disease, blood clots form in blood vessels. They may
travel through the bloodstream and block an artery. The risk of
developing thromboembolic disease is increased for about 6 to 8 weeks
after delivery. Most complications due to blood clots result from
injuries that occur during delivery. The risk is much greater after a
cesarean section than after vaginal delivery.
Blood clots usually form in the superficial veins of the legs as
thrombophlebitis or in the deep veins as deep vein thrombosis. Symptoms
include swelling, pain in the calves, and tenderness. The severity of
the symptoms does not correlate with the severity of the disease. A clot
can move from the legs to the lungs, where it may block one or more
arteries in the lungs. This blockage, called pulmonary embolism, can be
life threatening.
If pulmonary embolism is suspected, a lung ventilation and perfusion
scan may be performed to confirm the diagnosis.
A 20 year old single white female who is a patient of yours was raped in
her apartment at 7:00 a.m. today. She is brought to your office at 9:00
a.m. for assessment and treatment. Despite having occasional intercourse
with her boyfriend, she has never used any type of contraceptive. Their
last intercourse was approximately 1 week ago, and the boyfriend has
been out of town on business since then. The patient has a history of
irregular periods, and her last normal period was approximately 2 1/2

weeks ago. You note live sperm on a wet mount. In addition to many other
issues that must be addressed at this visit, the patient asks about
emergency contraception. Which one of the following would be accurate
advice to the patient regarding this topic?
a) Emergency contraception does not interfere with an established, post-implantation pregnancy
b) The estrogen/progestin combination regimen appears to be more effective than the levonorgestrel-only
regimen
c) To be most effect, each dose of the 2-dose regimen should be administered at least 72 hours apart
d) Fetal malformations have been reported as a result of the unsuccessful use of the high-dose emergency
contraceptive regimen
The correct answer is A
Explanation
An FDA Advisory Committee has recommended over-the-counter marketing of
Plan B, an emergency contraceptive package that contains two 0.75-mg
tablets of levonorgestrel to be taken 12 hours apart. Plan B is one of
the two FDA-approved products for this indication. The Preven emergency
contraceptive kit includes four tablets, each containing 0.25 mg of
levonorgestrel and 50 ?g of ethinyl estradiol; these are taken two at a
time 12 hours apart. In a randomized, controlled trial comparing the
single versus combined estrogen/progestin, the single-drug regimen was
shown to be more effective. Pregnancy occurred in 11 of 976 women (1.1%)
given levonorgestrel alone, and in 31 of 979 (3.2%) given ethinyl
estradiol plus levonorgestrel. The proportion of pregnancies prevented,
compared to the expected number without treatment, was 85% with
levonorgestrel and 57% with the combination. In both regimens, the
interval between individual doses is 12 hours. In this case, emergency
contraception may be appropriate in the face of a possible pregnancy
from previous consensual intercourse. Emergency contraception has not
been found to interfere with an established post-implantation pregnancy.
Furthermore, no fetal malformations have been reported as a results of
the unsuccessful use of high-dose oral contraceptives for emergency
contraception.
A sexually active woman presents with dysuria and vaginal discharge. All
the following can cause this condition, except

a) Gonorrhea
b) Chlamydia
c) Trichomoniasis
d) Condyloma acuminata
The correct answer is D
Explanation
Gonorrhea, chlamydia and trichomoniasis are all sexually transmitted
diseases that can cause dysuria and vaginal discharge.
Genital warts (condylomata acuminata) are growths in or around the
vagina, penis, or rectum caused by sexually transmitted
papillomaviruses. Many people have no symptoms from the warts, but some
feel occasional burning pain.
The warts usually appear 1 to 6 months after infection with
papillomavirus, beginning as tiny, soft, moist, pink or red swellings.
They grow rapidly and appear as rough, irregular bumps, which sometimes
grow out from the skin on narrow stalks. Groups of warts often grow in
the same area, and their rough surfaces give them the appearance of a
small cauliflower.
In order to stop intractable uterine bleeding from postpartum
hemorrhage, ligation of the internal iliac arteries is performed. Blood
flow will be greatly diminished in all of the following arteries EXCEPT:
a) Obturator
b) Superior gluteal
c) Inferior gluteal
d) Superior vesical
e) Superior rectal
The correct answer is E
Explanation
The internal iliac branches into an anterior and a posterior division.

The posterior division gives rise to the superior gluteal, iliolumbar,


and lateral sacral. The anterior division gives off the obturator,
internal pudendal, uterine, superior and inferior vesicle, vaginal
branches, and the obliterated umbilical artery. The middle and inferior
rectal arteries arise off of the internal pudendal. The superior rectal
is the final branch of the inferior mesenteric. The external pudendal
arises off of the external iliac at the level of the inguinal ligament
(along with the deep inferior epigastric and circumflex iliac, and
sometimes the aberrant obturator).

In screening for cervical neoplasia, which one of the following


statements about the Papanicolaou (pap) test is correct?
a) It is not necessary following a complete hysterectomy
b) It has a high false negative rate
c) It should always include an endocervical component
d) It should not cause cervical bleeding
e) It is best done in the premenstrual phase of the cycle
The correct answer is A
Explanation
Women who have had a hysterectomy are no longer at risk for acquiring
cervical cancer. According to recent studies, in a vast majority of
women who receive complete hysterectomies, an annual Pap smear is
unnecessary.
A complete hysterectomy must include removal of the cervix. Women whose
cervix has been removed and who have had no cancerous or precancerous
lesions detected in the years preceding the procedure do not have to
undergo an annual Pap exam according to the study.
This does not imply however that women do not need an annual OB/GYN
appointment. On the contrary, all women should still submit to an annual
pelvic exam, as all women are still at risk for reproductive disorders
including ovarian disease.
An 18 year old white female presents with small, localized warts on the
vulva and lower vaginal mucosa. She wants to avoid injections and
surgical treatment if possible. Which one of the following is an
acceptable topical agent for treating these vaginal lesions

a) Trichloroacetic acid
b) Podofilox gel (Condylox)
c) Imiquimod cream (Aldara)
d) Interferon
e) Podophyllin 25% solution in alcohol (Podocon-25, Podofin)
The correct answer is A
Explanation
Tricholoroacetic acid is acceptable for use on vaginal mucosa. It is
also acceptable for use when pregnancy is a possibility. Professional
application is necessary. Podofilox and podophyllin in alcohol are not
safe for use on mucosa. Imiquimod cream is also not approved for mucosal
use. Interferon requires infection.
A 19 year old has intercourse with her boyfriend. Later she finds a tear
in her diaphragm contraceptive device. She comes in to see you the next
day worried about pregnancy that she does not want. What is an
appropriate action?

a) Do pregnancy test
b) Reassurance
c) Oral contraceptive pills, daily for 2 weeks
d) Levonorgestrel, one pill now and another in 12 hours
The correct answer is D
Explanation

Emergency contraception, the so-called morning-after pill, consists of


hormones or drugs that affect hormones. It is used within 72 hours after
one act of unprotected sexual intercourse or after one occasion when a
contraceptive method fails (for example, when a condom breaks).
Emergency contraception decreases the chance of pregnancy after one act
of unprotected intercourse, including when the act occurs near the time
the egg is released (ovulation)?when conception is most likely. Near
ovulation, the chance of pregnancy is about 8% without contraception.
The sooner emergency contraception is taken, the more likely it is to be
effective.
Levonorgestrel is a progestin often taken in lower doses for
contraception, is most commonly used. Usually, one dose is taken by
mouth, followed by another dose 12 hours later. If the first dose is
taken within 72 hours of intercourse, the chance of pregnancy decreases
by almost 90%. If the first dose is taken within 24 hours of
intercourse, the chance decreases by about 95%.
The most frequently reported symptom of vulvar cancer is which one of
the following?

a) Longstanding pruritis
b) Bleeding
c) Pain
d) Discharge
e) Dysuria
The correct answer is A
Explanation The most common symptom of vulvar cancer is longstanding
pruritis. The other symptoms mentioned occur less frequently.
A 24 year old G1P1 female has an uncomplicated delivery of a 10 pound
male infant. The patient is seen in the maternity ward 24 hours after
vaginal delivery and repair of a fourth degree perineal laceration. The
patient tells you that she is concerned about her insurance company
requirement that she stay in the hospital no longer than 48 hours post
partum.
Which of the following would be the most important indication for
extending her hospital stay beyond 48 hours post partum?

a) Abdominal distention and lack of appetite at 48 hours post partum


b) Lack of bowel movement post partum
c) Need for nursing assistance with breast-feeding
d) Palpation of the uterus above the pubic symphysis for more than 48
hours post partum
e) Persistence of lochia for more than 24 hours post partum
The correct answer is A
Explanation
A common postpartum complication is infection. The presentation of a
puerperal infection may be atypical because of the altered physiology of
the postpartum period. Abdominal distention (ileus) and lack of appetite
may be the first manifestation of abdominopelvic peritonitis. Careful
evaluation of the patient is required with respect to the genitourinary
tract. Risks for the development of postpartum infection include vaginal
trauma (which this patient had), anemia (this patients hemoglobin is
10.8), multiple pelvic examinations, internal fetal monitoring,
prolonged rupture of the membranes, and indigent status. Continued
surveillance is indicated so that the reason for distention and lack of
appetite can be identified and treated.
The duration of the patients labor is not given, but it is likely that
she has not eaten solid food in some time. There may not be significant
stool in her large intestine available for evacuation. Also, some
narcotic medications used for analgesia during labor may contribute to
decreased intestinal motility. The mild ileus that follows delivery,
together with perineal discomfort and postpartum fluid loss by other

routes, predisposes to sluggish bowel evacuation during the puerperium.


Strategies to improve postpartum bowel function include initiating a
low-residue diet, prescription of a short course of stool softeners, and
a mild laxative on the first postpartum night. She should be reassured
that normal bowel function can be anticipated.
A 20 year old woman who is 35 weeks pregnant with her first child is
admitted to the hospital because of persistent hypertension and 1+
protein on urinalysis obtained 36 hours ago. She is confined to bed
awaiting further diagnostic studies for preeclampsia. Her blood pressure
is now 160/100 mm Hg. She is complaining of headaches, blurred vision
and epigastric pain. At this time it is most appropriate to conclude
that this patient has which of the following?

a) Chronic hypertension and requires antihypertensive therapy


b) Chronic hypertension, but no antihypertensive therapy is necessary
c) Mild preeclampsia and should continue bed rest pending further
diagnostic studies
d) Severe preeclampsia and requires immediate medical management and
delivery
e) Severe preeclampsia and should continue bed rest pending further
diagnostic studies
The correct answer is D
Explanation
This patient has severe preeclampsia. Preeclampsia is diagnosed on the
basis of her elevated blood pressure and proteinuria occurring after 20
weeks gestation. She has severe preeclampsia on the basis of her
headaches, blurred vision, and epigastric pain. This patient requires
magnesium sulfate to prevent eclampsia (seizures occurring in the
setting of preeclampsia). Magnesium sulfate has been shown to be the
most effective agent to use for the prevention of seizures in women with
preeclampsia. She also requires delivery to arrest the disease process.
Delivery is the only ?cure? for preeclampsia. Bed rest pending further
diagnostic studies is inappropriate because delivery is necessary to
prevent a worsening of the disease process that can be fatal for mother
and fetus.
Premature rupture of membrane is associated with all, except

a) Maternal alcohol use


b) Sexually transmitted infections such as chlamydia and gonorrhea
c) Previous preterm birth
d) Cigarette smoking during pregnancy
The correct answer is A
Explanation
Premature rupture of the membranes (PROM) near the end of pregnancy may
be caused by a natural weakening of the membranes or from the force of
contractions.
Other factors that may be linked to PROM include the following: low
socioeconomic conditions (as women in lower socioeconomic conditions are
less likely to receive proper prenatal care), sexually transmitted
infections such as chlamydia and gonorrhea, previous preterm birth,
vaginal bleeding, cigarette smoking during pregnancy.
Which one of the following is an evidence-based clinical recommendation
for maternity care?

a) All pregnant women should be screened serologically for herpes


simplex virus
infection
b) All pregnant women should be screened for bacterial vaginosis
c) All pregnant women should be screened for asymptomatic bacteriuria
by urine culture
d) Only at-risk women should be tested for HIV infection
e) Only women 25 years of age or younger should be screened for
chlamydial infection
The correct answer is C

Explanation
All pregnant women should be screened for asymptomatic bacteriuria due
to the increased risk for pyelonephritis and preterm labor. All pregnant
women should also be screened for active hepatitis B, syphilis, and HIV
infection. Only at-risk women (those 25 years of age or younger and
those at risk of STDs) need to be screened for Chlamydia. Routine
screening for bacterial vaginosis is not recommended.
A G2P1 with history of previous Cesareans for cephalo-pelvic
disproportion presents with onset of labor. As the nurses are getting
ready to start helping the patient with pushing, a gush of blood is seen
coming out from vagina. She is in a tremendous amount of pain. What is
the likely cause?

a) Placenta previa
b) Breech presentation
c) Bloody show
d) Rupture of uterus
The correct answer is D
Explanation
Uterine rupture is a concern in women who have had prior cesarean
deliveries, and the risk of rupture does rise with the number of
previous cesarean deliveries. It is of particular concern if the woman
is in labor.
A bloody show would not be this dramatic. Placenta previa is
implantation of the placenta over or near the internal os of the cervix.
Typically, bright red painless vaginal bleeding occurs during late
pregnancy.
A 21-year-old nulliparous woman is in active labor. Her membranes have
been ruptured for the last 4 hours. She is having contractions every 3-5
minutes, each lasting about 60 seconds and of moderate intensity. In
your evaluation of her progress, you perform a cervical exam and note
that she is 5 cm dilated. The fetal vertex and skull sutures are readily
palpable. You determine that the current position of the fetal vertex
(see Figure D) is:

a) Occiput anterior
b) Right occiput anterior
c) Left occiput anterior
d) Right occiput posterior
e) Left occiput posterior
The correct answer is E
Explanation
The position of the fetal vertex is determined by the location of the
fetal occiput relative to the maternal pelvis. When the sagittal suture
is in the anterior-posterior axis of the pelvis, with the occiput
closest to the symphysis, the vertex is considered to be occiput
anterior (OA). When the sagittal suture is in the transverse axis of the
pelvis, the vertex is either right or left occiput transverse (ROT,
LOT). In this diagram , the occiput, as noted by the triangular shape of
the posterior fontanelle, is to the patients left side and posterior.
The anterior fontanelle is anterior and to the patients right. Since
the landmark is the occiput, the position is left occiput posterior.

A 25 year old healthy female presents with a 1 week history of a thin,


mildly odorous vaginal discharge accompanied by mild vaginal itching.
Physical examination is normal except for a thin, homogenous discharge,
with no significant cervical or vaginal inflammation. A wet-mount
preparation of the discharge reveals numerous epithelial cells coated
with small non-motile organisms. A KOH preparation is negative but has a
?fishy? odor. Which one of the following is correct regarding this
patients condition?

a) Failure to treat the male partner may result in urethritis or


prostatitis
b) Symptomatic relief of the infection frequently requires several
courses of treatment in patients taking oral contraceptives
c) The infection should be reported to local health authorities for
contact tracking
d) This infection can be treated with oral or vaginal clindamycin
(Cleocin)
The correct answer is D
Explanation
The patient described has a classic case of bacterial vaginosis. The
role of sexual transmission is unclear and there is no documented
benefit to treating the male partner(s). Treatment is fairly easily
accomplished with oral or vaginal clindamycin or metronidazole.
A 56-year-old female presents for a health maintenance examination. She
has a history of a total hysterectomy for benign disease 4 years ago.
You are able to document that the hysterectomy pathology was benign and
that she has had normal Papanicolaou (Pap) tests for 10 years.
The patient asks about regular Pap smears. Which one of the following
would be the most appropriate recommendation?

a) Routine Pap smears should be continued until age 70


b) A Pap smear should be done every 3 years
c) A Pap smear is not indicated

d) A Pap smear should be done yearly for 3 years and only if


indicated thereafter
The correct answer is C
Explanation
After a hysterectomy for documented benign disease, cytologic screening
may be discontinued. Papanicolaou (Pap) smears in this population are
low yield and may cause unnecessary testing due to false positives. Pap
smears may be continued if
the reason for the hysterectomy is uncertain. If there is a history of
invasive cervical cancer or DES exposure, screening should be continued,
although there is not a great deal of data to support this practice.
During a routine physical examination of a 35 year old Asian female, you
note a right adnexal fullness. She has had no symptoms of pain or
bloating and has been menstruating normally. Her menses occur
approximately every 30 days and her next period is expected to occur in
1 week. Pelvic ultrasonography reveals a thin-walled simple cyst 5 cm in
diameter. No other abnormalities are seen in the pelvic structures.
Which one of the following is the best course of management for this
condition?

a) Reassurance only
b) Checking for any increase in adnexal fullness at her next annual
physical examination
c) Repeat ultrasonography in 2-3 months to confirm resolution of the
cyst
d) Referral for ultrasound guided aspiration of the cyst
e) Referral for laparoscopic removal of the cyst
The correct answer is C
Explanation
Adnexal masses in women under 45 years of age are benign in 80%-85% of
cases. The specific findings of this case also strongly suggest a benign
etiology, namely a thin-walled, simple cyst, a lesion that is less than
8 cm in size, and a patient of relatively young age. No aggressive means
are indicated in these situations unless there are significant clinical
symptoms such as pain, abdominal pressure, urinary symptoms, or
gastrointestinal symptoms. Most experts currently recommend a
conservative approach with repeat ultrasonography in at least 2 months,
during which time the vast majority of benign cysts resolve spontaneously.
A 27 year old nulligravida presents in the office complaining of
bilateral painful breasts. Clinical examination reveals that both
breasts are tender and contain multiple tender, shotty nodules. The most
likely diagnosis is

a) Carcinoma of the breast


b) Galactocele
c) Fibrocystic disease of the breast
d) Intraductal papilloma
e) Fibroadenoma
The correct answer is C
Explanation
Fibrocystic changes refer to mastalgia, breast cysts, and nondescript
lumpiness, which may occur in isolation or together; breasts have a
nodular and dense texture and are frequently tender when palpated.
Fibrocystic changes cause the most commonly reported breast symptoms and
have many causes.
Most causes are not associated with increased risk of cancer; they
include adenosis, ductal ectasia, simple fibroadenoma, fibrosis,
mastitis, mild hyperplasia, cysts, and apocrine or squamous metaplasia.
Other causes, particularly if fibrocystic changes require biopsy, may
slightly increase risk of breast cancer. Fibrocystic changes are more
common among women who had early menarche, who had their 1st live birth
at age > 30, or who are nulliparous.

Oligohydramnios is associated with which of the following fetal conditions

a) Tracheo-esophageal fistula
b) Talipes equinovarus (club foot)
c) Anencephaly
d) Fetal erythoblastosis
e) Down syndrome
The correct answer is B
Explanation
Potter syndrome is a term used to describe the typical physical
appearances of a fetus or neonate due to a dramatically decreased
amniotic fluid volume oligohydramnios, or absent amniotic fluid
anhydramnios, secondary to renal diseases such as bilateral renal
agenesis. Other causes of Potter syndrome can be obstruction of the
urinary tract, polycystic or multicystic kidney diseases, renal
hypoplasia and rupture of the amniotic sac.
The decreased volume of amniotic fluid causes the growing fetus to
become compressed by the mother's uterus. This compression can cause
many physical deformities of the fetus, most common of which is Potter
facies. Lower extremity anomalies are frequent in these cases, which
often presents with clubbed feet and/or bowing of the legs.
Which one of the following fetal ultrasound measurements gives the most
accurate estimate of gestational age in the first trimester (up to 14
weeks)?

a) Femur length
b) Biparietal diameter
c) Abdominal circumference
d) Crown-rump length
e) Scapulo-sacral length
The correct answer is D
Explanation
Because the growth pattern of the fetus varies throughout pregnancy, the
accuracy of measurements and their usefulness in determining gestational
age and growth vary with each trimester. Crown-rump length is the
distance from the top of the head to the bottom of the fetal spine. It
is most accurate as a measure of gestational age at 7?14 weeks. After
that, other measurements are more reliable. In the second trimester,
biparietal diameter and femur length are used. During the third
trimester, biparietal diameter, abdominal circumference, and femur
length are best for estimating gestational age.
A 16 year old girl returns to the health center for a family planning
follow-up visit. She gave birth to a healthy baby girl 8 months ago and
does not want to become pregnant again. She is monogamous with the
father of her baby but worries that he is not monogamous with her. They
live together and are not married. She is taking an oral contraceptive
and "sometimes" uses condoms. She says that she has great trouble
remembering to take her pills and wants to discuss other contraceptive
options. In addressing this issue, which of the following is the most
appropriate next step?

a) Advise her to continue taking an oral contraceptive because it is


one of the most effective methods of birth control
b) Advise her that an intrauterine device would be a good
contraceptive choice for her
c) Discuss implantable or injection long-term progestational
contraceptive agents
d) Discuss the option of tubal ligation
e) Fit the patient with a diaphragm and instruct her on proper use
The correct answer is C
Explanation

The basic issue in this question is that of patient compliance. In an


ideal world, all patients would take the appropriate therapy at the
appropriate times. However, in actual practice, patients are not
entirely compliant and therefore, management of their medical issues
needs to be tailored to that fact. For this patient, it is clearly
stated that she sometimes uses condoms and has ?great trouble?
remembering to take her oral contraceptive pills. Therefore, a
long-acting birth control agent that does not require compliance is ideal.
The most appropriate place to obtain a smear for the diagnosis of
gonorrhea in a woman who presents with vaginal discharge is

a) Vulva
b) Vagina
c) Cervix
d) Anus
The correct answer is C
Explanation
Gonorrhea is caused by the bacterium neisseria gonorrhoeae. It typically
infects epithelia of the urethra, cervix, rectum, pharynx, or eyes,
causing irritation and purulent discharge. Dissemination to skin and
joints occurs infrequently. Diagnosis is by culture or genetic methods.
Diagnosis is by Gram stain and culture. Endocervical swabs should be
inserted at least 2 cm and rotated for 10 sec to obtain a proper specimen.
A woman with urinary incontinence, loses urine when she hears the
hissing sound of tap water and during straining while laughing or
sneezing. What type of incontinence does she have?

a) Stress incontinence
b) Detrusor instability
c) Urge incontinence
d) Overflow incontinence
The correct answer is A
Explanation
There are three main types of incontinence in women, although they can
occur in men also.
Stress incontinence is loss of urine when you exert pressure or stress
on your bladder by coughing, sneezing, laughing, exercising or lifting
something heavy. It has nothing to do with psychological stress. Stress
incontinence occurs when the sphincter muscle at the bladder is
weakened. The problem is especially noticeable when you let your bladder
get too full. Stress incontinence is one of the most common types of
incontinence, often affecting women. Physical changes resulting from
pregnancy, childbirth and menopause can cause stress incontinence. In
men, removal of the prostate gland can lead to this type of incontinence.
Urge incontinence is a sudden, intense urge to urinate, followed by an
involuntary loss of urine. Your bladder muscle contracts and may give
you a warning of only a few seconds to a minute to reach a toilet. With
urge incontinence, you may also need to urinate often. The need to
urinate may even wake you up several times a night. Simply going from
sitting to standing may even cause you to leak urine. In urge
incontinence, the bladder is said to be "overactive" it's contracting
even when your bladder isn't full.
Overflow incontinence is when one has frequent or constant dribble
urine. This is an inability to empty your bladder, leading to overflow.
With overflow incontinence, sometimes you may feel as if you never
completely empty your bladder. When you try to urinate, you may produce
only a weak stream of urine. This type of incontinence is common in
people with a damaged bladder or blocked urethra and in men with
prostate gland problems.
Which one of the following intravenous antibiotic regimens is most

appropriate for the treatment of postpartum endometritis

a) Ciprofloxacin (Cipro) plus nafcillin (Unipen)


b) Ciprofloxacin plus doxycycline (Vibramycin)
c) Gentamicin (Garamycin) plus methicillin
d) Gentamicin plus clindamycin (Cleocin)
e) Clindamycin plus metronidazole (Flagyl)
The correct answer is D
Explanation
The usual recommendation is to treat postpartum endometritis with
clindamycin and gentamicin. This combination cover anaerobes, group B
Streptococcus, and gram-negative organisms. Extended-spectrum
cephalosporins or imipenem-cliastatin or ampiciln-sulbactam are
frequently used; however, the clindamycin/gentamyicin regimen remains
the gold standard when endometritis is suspected.
In a differential diagnosis of third trimester bleed, which of the
following should not be included

a)
b)
c)
d)
e)

Placenta previa
Vasa previa
Abruptio placentae
Threatened abortion
Marginal sinus hemorrhage
The correct answer is D

Explanation
Causes of bleeding during the third trimester include placenta previa
(20%), placental abruption (30%), ruptured vasa previa, uterine scar
disruption and bloody show.
Threatened abortion is vaginal bleeding occurring before the 20th week
of pregnancy and indicating that spontaneous abortion may occur.
A 65 year old woman who is postmenopausal and taking bisphosphonate is
now complaining of vaginal bleeding. What is the most common cause?

a) Uterine fibroids
b) UTI
c) Endometrial cancer
d) Atrophic vaginitis
The correct answer is D
Explanation
Up to 40% of postmenopausal women have symptoms of atrophic vaginitis.
Because the condition is attributable to estrogen deficiency. Those
symptoms are dryness, itching, burning and dyspareunia and vaginal bleeding.
About 10% of postmenopausal women with unexpected vaginal bleeding will
have endometrial cancer. However, the most common cause of bleeding in
these women is atrophy of the vaginal mucosa.
Postmenopausal vaginal bleeding is cancer until proven otherwise. All
postmenopausal women who report vaginal bleeding must have an
endometrial biopsy done. Hysterectomy is the treatment, if indeed the
patient has endometrial cancer.
Note: don't get confused...this presentation can be followed by
different questions. (example: "what's the most common cause" or "what
do you have to exclude first" or "what's the initial test you will
order" etc)
A woman who is positive for hepatitis B surface antigen (HBsAg), but
negative for hepatitis B antigen (HBeAg), delivers at term. What would
be the best management for this woman's infant?

a) Administer gamma globulin intramuscularly immediately and at 1


month of age
b) Administer hepatitis B (HB) vaccine immediately and at 1 month and
6 months of age
c) Administer hepatitis B immune globulin (HBIG) if cord blood is
positive for HbsAg
d) Administer HBIG and HB vaccine immediately and HB vaccine again at
1 month and 6 months of age
e) Advise mother that breastfeeding is contraindicated
The correct answer is D
Explanation
The recommendation for infants with HBsAg-negative mother is to give the
hepatitis B vaccine between 0 to 2 months, then again between 1 to 4
months, and finally a third time between 6 to 18 months.
The recommendation for infants with HBsAg-positive mother is to give the
Hepatitis B immune globulin and vaccination within 12 hours of birth,
then the vaccine between 1 to 2 months and again at 6 months.
The commonest cause of disseminated intravascular coagulation during
pregnancy is

a) Amniotic fluid embolism


b) Abruptio placentae
c) Placenta previa
d) Missed abortion
e) Chorioamnionitis
The correct answer is B
Explanation
Disseminated intravascular coagulation (DIC) involves abnormal,
excessive generation of thrombin and fibrin in the circulating blood.
During the process, increased platelet aggregation and coagulation
factor consumption occur. DIC that evolves slowly (over weeks or months)
causes primarily venous thrombotic and embolic manifestations; DIC that
evolves rapidly (over hours or days) causes primarily bleeding.
DIC occurs in the following clinical circumstances:
Complications of obstetricseg, abruptio placentae (most common),
saline-induced therapeutic abortion, retained dead fetus or products of
conception, or amniotic fluid embolism. Placental tissue with tissue
factor activity enters or is exposed to the maternal circulation.
Infection, particularly with gram-negative organisms. Gram-negative
endotoxin causes generation of tissue factor activity in phagocytic,
endothelial, and tissue cells.
Malignancy, particularly mucin-secreting adenocarcinomas of the pancreas
and prostate and acute promyelocytic leukemia, in which tumor cells
expose or release tissue factor activity.
A 24 year old woman appears at 8 weeks of pregnancy and reveals a
history of pulmonary embolism 7 years ago during her first pregnancy.
She was treated with intravenous heparin followed by several months of
oral warfarin and has had no further evidence of thromboembolic disease
for over 6 years. Which of the following statements about her current
condition is true?

a) Having no evidence of disease for over 5 years means that her risk
of thromboembolism is not greater than normal
b) Impedance plethysmography (IPG) is not a useful study to evaluate
her for deep venous thrombosis in pregnancy
c) Doppler ultrasonography is not a useful technique to evaluate her
for deep venous thrombosis in pregnancy
d) The patient should be placed on low molecular weight heparin
therapy throughout her pregnancy and the puerperium
e) She is at highest risk for recurrent thromboembolism during the

second trimester of pregnancy


The correct answer is D
Explanation
As with deep venous thrombosis (DVT), pulmonary embolism (PE) requires
objective diagnostic testing to confidently confirm or exclude the
diagnosis. This is particularly true in pregnancies because the
diagnosis of DVT or PE requires (1) prolonged therapy (<9 months of
heparin during pregnancy), (2) prophylaxis during future pregnancies,
and (3) avoidance of oral contraceptive pills.
The first objective diagnostic test should be compression
ultrasonography; if it is not available, IPG is adequate. If the
findings from noninvasive leg studies are negative, then proceed to
ventilation-perfusion lung scanning. Perfusion scanning alone is
recommended initially, and the ventilation scan is added when perfusion
defects are noted.
Although only a relatively modest amount of data have been gathered, low
molecular weight heparin, which does not cross the placenta, can be
given once a day and does not require monitoring. Low molecular weight
heparin has not been shown to increase the risk of bleeding with
surgical procedures, including cesarean delivery, in a small number of
patients.
A healthy 30-year-old primigravid woman at 16 weeks gestation presents
for prenatal care. She tells you that her biological brother has cystic
fibrosis, and that their parents have no evidence of the disease. The
father of the baby has no family history of cystic fibrosis. If the
carrier rate in the general population is 1/22, what is the risk that
this fetus will have cystic fibrosis

a) 1/176
b) 1/132
c) 1/44
d) 1/66
e) 1/88
The correct answer is B
Explanation
The risk that she is a carrier is 2/3. Her parents must both be
carriers. Since she is healthy, she is either a carrier (2/4) or
homozygous normal (1/4), giving a 2/(1+2) or 2/3 risk of being a
carrier. The carrier risk of the father is 1/22 (normal population
risk). The risk of two carriers having an affected offspring is 1/4.
Thus the final risk is 2/3 times 1/22 times 1/4, which equals 1/132.
A. See explanation above.
C. See explanation above.
D. See explanation above.
E. See explanation above.
During normal pregnancy, thyroid function is associated with an increase in:

a) Free triiodothyronine (T3)


b) Basal metabolic rate of 50%
c) Thyroid stimulating hormone (TSH) transfer to the fetus
d) Protein binding of thyroid hormones
e) Elevated T3 resin uptake
The correct answer is D
Explanation
TSH does not cross the placenta in significant amounts, but T4 transfer
in early gestation may have a protective role in the early neural
development of the fetus. Thyroid-binding globulin levels are increased,
leading to an increase in the amount of bound thyroid hormone. Free
thyroid hormone stays in the normal range, and total thyroid hormone
levels are elevated. The T3 resin uptake is inversely proportional to
the hormone binding capacity, so it decreases in normal pregnancy.
A. Stays essentially unchanged.

B. Basal metabolic rate remains essentially unchanged.


C. TSH does not cross the placenta in significant amounts.
E. The T3 resin uptake decreases since the percent of thyroid hormone
that is bound has increased.
In a woman with mild persistent asthma, which one of the following
agents is thought to be the best choice for maintenance therapy during
pregnancy?

a) Inhaled corticosteroids
b) Inhaled cromolyn (Intal)
c) A long-acting -agonist
d) A leukotriene receptor antagonist
e) A leukotriene synthesis inhibitor
The correct answer is A
Explanation
Inhaled corticosteroids are currently the recommended maintenance agent
for pregnant patients with mild persistent asthma. They have proven
efficacy with a relatively low risk profile. Cromolyn has a good safety
record but somewhat disappointing results. Its effectiveness is limited
compared to that of inhaled corticosteroids. Long-acting -agonists with
inhaled corticosteroids are more often used in more severe persistent
asthma, as they decrease the frequency of attacks. There is some
controversy about their effect on mortality.
Leukotriene receptor antagonists are probably a reasonable add-on
therapy for asthma in pregnancy. They are not as effective as inhaled
corticosteroids, however, and do not have as long a track record of
safety. Leukotriene synthesis inhibitors are not recommended for use in
pregnancy.
When taking a patient history, which of the following questions will
most accurately ascertain the length of the patients menstrual cycle?

a) How often do you menstruate?


b) Do you menstruate every month?
c) How many days are there between your periods
d) How many days are there from the beginning of one period to the
beginning of the next?
e) How many days are there from the end of one period to the
beginning of the next?
The correct answer is D
Explanation
The first day of the cycle is the first day of the menses. The length of
the cycle is from the first day of one menses to the first day of the
next menses. Many patients think that the length of the cycle is from
the end of the menses to the beginning of the next. Thus, a number of
women think that they have 21- to 23-day cycles.
A. Many patients count the days between periods as their cycle length.
This will falsely shorten the length of their true cycle.
B. Even if the answer to this question is yes, there is no way to
ascertain the actual length of their cycle.
C. See answer to A.
E. The last few days of a menses can be variable in length and degree of
bleeding/spotting. It is less accurate to count this way than to count
from the beginning of one cycle to the beginning of the next.
An 18 year old gravida 1 para 0 at 40 5/7 weeks gestation presents
stating that she ?thinks her water broke 2 days ago?. A sterile speculum
examination confirms rupture of the membranes. There are no signs of
active labor. The patient is afebrile, her uterus is nontender, and
fetal heart tones are reassuring. The remainder of the prenatal history
is unremarkable. A group B Streptococcus culture obtained 4 weeks ago
was negative. The patient has no known drug allergies.
In addition to induction of labor, which one of the following is the
most appropriate management for this patient?

a) No antibiotic prophylaxis
b) Ampicillin, 2 g intravenously initially, then 1 g intravenously
every 4 hours until delivery
c) Cefazolin (Ancef), 2 g intravenously initially, then 1 g
intravenously every 8 hours until delivery
d) Clindamycin (Cleocin), 900 mg intravenously every 8 hours until
delivery
e) Vancomycin (Vancocin), 1 g intravenously every 12 hours until
delivery
The correct answer is A
Explanation
According to current guidelines, women with negative vaginal and rectal
group B /Streptococcus/ screening within 5 weeks of delivery do not
require intrapartum antimicrobial prophylaxis. This holds even if
certain obstetric risk factors develop (delivery at <37 weeks gestation,
duration of membrane rupture >18 hours, or temperature >100.4?F or
38?C). In patients requiring intrapartum antibiotic prophylaxis,
penicillin is the first-line agent, with ampicillin as an acceptable
alternative. For women who have a known penicillin allergy, cefazolin
should be used if the patient is not at high risk for anaphylaxis. If
the patient is at high risk for anaphylaxis, clindamycin is an
acceptable alternative if prior cultures have shown susceptibility. If
the susceptibility is unknown, vancomycin should be used.
In the postmenopausal woman with hot flushes who cannot take estrogen,
which one of the following may help?

a) Diazepam (Valium)
b) Beta-Blockers
c) Clonidine (Catapres)
d) Meclizine (Antivert)
e) Vitamin C
The correct answer is C
Explanation
When patients with hot flushes cannot take estrogen, transdermal
clonidine using the 100 micrograms dose is recommended. Side effects are
minimal, and a modest impact can be expected.
In endometriosis, the most common location for disease is

a) Broad ligament
b) Cul-de-sac
c) Ovaries
d) Appendix
e) Uterosacral ligament
The correct answer is C
Explanation
Endometriosis is a condition in which bits of tissue from the lining of
the uterus (endometrium) grow outside the uterus. The endometrial
tissue, called an endometrial implant, usually adheres to the pelvic
organs, which include the ovaries (most common), uterus, fallopian
tubes, the cavity behind the uterus, and the ligaments that support the
uterus. Endometrial implants may also adhere to the tubes leading from
the kidneys to the bladder, the bladder, the vagina, the outer surface
of the small and large intestine, or the lining of the chest cavity.
These locations, however, are not as common.
An emergency cesarean section is being performed because of fetal
distress. At delivery, the baby is covered with thick meconium and is
apneic. The most appropriate management is to

a) Give oxygen
b) Intubate the trachea and ventilate the baby
c) Ventilate with bag and mask

d) Aspirate the baby's gastric contents


e) Suction trachea under direct vision
The correct answer is E
Explanation
Meconium is a thick, green, tar like substance that lines the baby's
intestines during pregnancy. Typically this substance is not released in
the baby's bowel movements until after birth. However, occasionally you
will find that the baby will have a bowel movement prior to birth,
excreting the meconium into the amniotic fluid.
One of the concerns when there is meconium present in the amniotic fluid
is that the baby will aspirate the meconium during the labor or birth.
This aspiration of meconium is dealt with by vigorous suctioning
immediately upon the birth of the baby's head, even before the body is
born. This can lessen the amount of meconium available for the baby to
aspirate.
Which event is matched with the correct timing?

a) Maternal serum screening test: 20-22 weeks


b) Post-partum visit: 6 weeks
c) Chorionic villus sampling: 4-6 weeks
d) Oral glucose challenge test: 14-16 weeks
e) Rhogam to Rh negative women: 12-14 weeks
The correct answer is B
Explanation
Maternal serum triple marker screening is done between 15-18 weeks. Post
partum visit happens 6 weeks after delivery. CVS can be done between
10-12 weeks. GTT is done between 24-28 weeks. Rhogam is given at 28
weeks to RH negative women.
At a routine prenatal visit at 16 weeks gestation a 38 year old gravida
3 para 2 has a pulse rate of 110 beats/min and has lost 2 kg (4 lb)
since her previous visit. At age 26, she was treated for Graves disease
with radioactive iodine and has been euthyroid on no medication for over
10 years. A CBC shows a mild anemia. Her hematocrit is 34% (N 35-45) and
her TSH level is 0.00 U/mL (N 0.5-5.0). Which one of the following would
be most appropriate at this time?

a) Propylthiouracil
b) Propylthiouracil plus levothyroxine (Synthroid)
c) Methimazole (Tapazole)
d) Radioactive iodine therapy
e) Immediate surgery
The correct answer is A
Explanation
There is a 5%-10% recurrence rate for Graves disease after treatment
with radioactive iodine. Years may pass before recurrence. Radioactive
iodine therapy is contraindicated in pregnancy, and immediate surgery
might present hazards to both the mother and the fetus. Propranolol
would control the patients heart rate, but would do nothing about the
underlying hyperthyroidism.
Propylthiorucail has been used extensively in pregnancy and has never
been shown to have any teratogenic effect. The combination of
propylthiouracil and levothyroxine is frequently used for
hyperthyroidism in nonpregnant patients, but transplacental passage of
the levothyroxine would be harmful to the developing fetus. Methimazole
crosses the placenta more readily than propylthiouracil and is
associated with aplasia cutis.
A 35-year-old woman presents to your office. She and her 32-year-old
husband have been unsuccessful in their attempts to get pregnant for the
last 6 years. He has fathered two children in a prior marriage and has a
normal semen analysis. Her basal body temperature chart is biphasic. Her
past history notes multiple episodes of chlamydia and gonorrhea. A
hysterosalpingogram demonstrates blocked fallopian tubes bilaterally,

and a laparoscope notes dense and profuse peritubal and pelvic


adhesions, along with bilateral clubbed tubes. The most appropriate
fertility treatment would be:

a) Intrauterine insemination with husbands sperm (IUI)


b) Intracytoplasmic sperm injection with husbands sperm (ICSI)
c) Gonadotropin induction of ovulation
d) In vitro fertilization (IVF)
e) Gamete intrafallopian transfer (GIFT)
The correct answer is D
Explanation
With extensive tubal disease on both the HSG and laparoscopy, operative
assistance will be needed in order for an egg to reach the uterine
cavity. Due to the tubal disease, GIFT is not possible. ICSI is the
treatment of choice for azoospermia and severe oligospermia. The patient
is ovulatory based on her basal body temperature chart, so ovulation
induction alone is not necessary. IVF with transcervical transfer of the
embryo is the optimal treatment for this couple. With blastocyst
transfer, the current success rates are above 50%.
A. The two tests of tubal function both demonstrate that it is highly
unlikely for the egg to successfully transport down the tube. Thus, IUI
will be of no benefit, since the sperm and egg will not meet.
B. ICSI is used for oligospermic and even some azospermic males to
achieve fertilization.
C. Again, ovulation induction alone will not be successful if the tubes
are blocked bilaterally.
E. This technique can only be used if there is tubal patency. The egg
and sperm mixture is placed in the distal fallopian tube via
laparoscopy. The tubes here are blocked.
A woman who delivers after a prolonged labor presents to you after 2
days. All of the following suggest endometritis, except

a) Tender uterus
b) Foul smelling
c) Uterus 9 cm below umbilicus
d) Pyrexia
The correct answer is C
Explanation
Puerperal endometritis is uterine infection, typically caused by
bacteria ascending from the lower genital or GI tract. Symptoms are
abdominal tenderness and pain, fever, malaise, and sometimes discharge.
Diagnosis is clinical, rarely aided by culture. Treatment is with
broad-spectrum antibiotics (eg, clindamycin plus gentamycin).
Typically, the 1st symptoms are lower abdominal pain and uterine
tenderness, followed by fever?most commonly within the 1st 24 to 72
hours postpartum. Chills, headache, malaise, and anorexia are common.
Sometimes the only symptom is a low-grade fever.
Pallor, tachycardia, and leukocytosis usually occur, and the uterus is
soft, large, and tender. Lochia may be decreased or profuse and
malodorous. When parametria are affected, pain and pyrexia are severe;
the large, tender uterus is indurated at the base of the broad
ligaments, extending to the pelvic walls or posterior cul-de-sac. Pelvic
abscess may present as a palpable mass separate from and adjacent to the
uterus.
A 31 year old woman has a dilation and curettage done (D&C). Some months
later she comes back complaining that she has amenorrhea but pain
monthly around the time she usually has her periods. No menstrual flow
is seen with estrogen and progesterone challenge. What is the most
likely diagnosis

a) Asherman syndrome

b) Uterine fibroids
c) Ectopic pregnancy
d) Bicornuate uterus
The correct answer is A
Explanation
Asherman's Syndrome is an acquired disease which is characterized by the
formation of adhesions (scar tissue) in the women's uterus. Asherman's
syndrome is the presence of intrauterine adhesions that typically occur
as a result of scar formation after uterine surgery, especially after a
dilatation and curettage ( D&C ). The adhesions may cause amenorrhea
and/or infertility.
Ashermans syndrome's patients have scanty or absent periods (amenorrhea)
but some have normal periods. Some patients have no periods but feel
pain at the time each month that their period would normally arrive.
This pain may indicate that menstruation is occurring but the blood
cannot exit the uterus because the cervix is blocked by adhesions.
Recurrent miscarriage and infertility could also be considered as
symptoms. Symptoms may be related to several conditions and are more
likely to indicate Asherman's syndrome if they occur suddenly after a
D&C or other uterine surgery.
Asherman's syndrome should be treated if it is causing infertility or
amenorrhea. Surgical treatment includes cutting and removing adhesions
or scar tissue within the uterine cavity.
A 25 year old multiparous patient at 28 weeks gestation has condyloma
lata. Six hours after receiving her first intramuscular dose of
penicillin G bezathine, 2.4 mIU, she experiences fever, chills, malaise,
headache, and myalgia. She also states that her lesions have become
acutely painful. Her temperature is 38.3?C (100.9?F), pulse is 110 bpm,
blood pressure in 90/60 mm Hg, and respirations are 24/min. The most
likely diagnosis is:

a) Waterhouse-Friderichsen syndrome
b) Allergic reaction to penicillin
c) Jarisch-Herxheimer reaction
d) Secondary bacteremia
e) Disseminated HPV viremia
The correct answer is C
Explanation
A. This is seen in meningococcemia.
B. Allergic reactions do not appear 6 hours after administration of the
medication.
D. Secondary bacteremia will not make the lesions painful. Chills,
malaise, and fever can be seen with bacteremia.
E. Condyloma lata is one of the lesions of secondary syphilis, not human
papillomavirus.
A 39 year old black multigravida at 36 weeks gestation presents with a
temperature of 40.0C (104.0F), chills, backache and vomiting. On
physical examination, the uterus is noted to be nontender. There is
slight bilateral costovertebral angle tenderness. A urinalysis reveals
many leukocytes, some in clumps, as well as numerous bacteria. Of the
following, the most appropriate therapy at this time would be

a) Oral trimethoprim/sulfamethoxazole (Bactrim, Septra)


b) Oral nitrofurantoin (Macrodantin)
c) Oral levofloxacin
d) Intravenous doxycycline (Vibramycin)
e) Intravenous ceftriaxone (Rocephin)
The correct answer is E
Explanation
Pyelonephritis is the most common medical complication of pregnancy. The
diagnosis is usually straightforward, as in this case. Since the patient
is quite ill, treatment is best undertaken in the hospital, at least
until the patient is stabilized and cultures are available. Ampicillin

is widely used as an agent of first choice, but because of variable drug


resistance some studies suggest adding an aminogycoside for a woman who
is seriously ill. Alternatively, an extended-spectrum penicillin or a
third-generation cephalosporin may be used.
Sulfonamides are contraindicated late in pregnancy because they may
increase the incidence of kernicterus. Tetracyclines are contraindicated
because administration late in pregnancy may lead to discoloration of
the childs deciduous teeth. Nitrofurantion may induce hemolysis in
women who are deficient in G-6-PD, which includes approximately 2% of
black women. The safety of levofloxacin in pregnancy has not been
established, and it should not be used unless the potential benefit
outweighs the risk.
A 20 year old single black female complains of 2 days of dysuria and
frequency. She has no history of previous similar symptoms. She is not
sexually active, has no vaginal discharge, and her menses occurred as
expected about 3 weeks ago. She has not had any fever or rigors. A
clean-catch, mid-stream urine specimen shows numerous white blood cells
and rod-like bacteria per high-power field. She developed a generalized,
pruritic rash when treated with trimethoprim/sulfamethoxazole (Bactrim,
Septra) for a bout of otitis media as a child. In addition, your
hospital laboratory has been reporting a high incidence (>20%) of
Escherichia coli resistant to trimethoprim/sulfamethoxazole. In this
clinical setting, which one of the following regimens is the best treatment?

a) Azithromycin (Zithromax) in a single dose


b) Metronidazole (Flagyl) in a single dose
c) Amoxicillin/clavulanate (Augmentin) for 5 days
d) Sustained-release nitrofurantoin (Macrobid) for 5 days
e) Cefpodoxime (Vantin) for 10 days
The correct answer is D
Explanation
Acute uncomplicated cystitis in women can be effectively and
inexpensively treated with a 3-day course of
trimethoprim/sulfamethoxazole before the infecting organism is known.
Generally, either trimethoprim/sulfamethoxazole, a fluoroquinolone, or
nitrofurantoin is a suitable choice for uncomplicated cystitis. Because
of a probable drug allergy and a high prevalence of resistant
Escherichia coli in the community, this patient should not be given
trimethoprim/sulfamethoxazole. A fluroquinolone is not listed among the
choices. Thus, in a patient with acute, uncomplicated cystitis, the most
appropriate regimen from the choices listed is nitrofurantoin for 5-7 days.
Urinary tract infections that recur after the use of antimicrobial
agents, or that are acquired in the hospital or nursing home, are more
likely to be due to antibiotic-resistant gram-negative bacilli. A
fluoroquinolone, oral amoxicillin/clavulanate, or an oral
third-generation cephalosporin such as cefpodoxime, cefdinir, or
ceftibuten can be useful in treating such infections in the outpatient
setting. However, the patient described here does not fall into these
categories. Neither azithromycin nor metronidazole alone is indicated as
the drug of choice for a simple bacterial urinary tract infection. They
are commonly used for the treatment of chlamydial cervicits and
trichomoniasis of the urogenital tract, respectively.
A 35 year old G0 woman presents with her husband to the infertility
clinic for a follow-up visit.
The couple has been trying to get pregnant for the past 2 years but has
not had any success. A spermogram and a hysterosalpingogram as well as
estrogen, progesterone, FSH and LH blood levels were all normal. Her
menarche was at the age of 13 years and her cycles have always been
regular, occurring every 30 days.
The womans past medical history is significant for dysmenorrhea of 5
years duration and dyschezia for the last few months. Her last menstrual
period was 1 week ago. She is taking no medications except for her daily
multivitamins.

Vitals are within normal limits. Physical exam reveals a non specific
pelvic tenderness, a left adnexal mass and multiple tender nodular
masses along the thickened uterosacral ligaments. Heart, lung, and
abdominal examinations are unremarkable.
Which of the following is considered the gold standard test for
diagnosis of the patients condition?

a) Endometrial biopsy
b) Laparoscopy
c) MRI of the abdomen and pelvis
d) Pelvic ultrasound
e) Serum prolactin level
The correct answer is B
Explanation
Endometriosis is the presence of endometrial-like tissue outside the
uterine cavity, which induces a chronic inflammatory reaction. It can
occur in various pelvic sites such as on the ovaries, fallopian tubes,
vagina, cervix, or uterosacral ligaments or in the rectovaginal septum.
This condition is often associated with pelvic pain and infertility, but
it is most often asymptomatic. The classic presentation is the
Dys-syndrome: Dysmenorrhea, Dyspareunia, Dyschezia and Dysuria. Although
not always done, laparoscopy is the gold standard test to visualize and
confirm the diagnosis of endometriosis.
Endometrial biopsy is the gold standard test for post menopausal
bleeding diagnosis.
Serum prolactin would the best initial test for suspected prolactinoma.
Pelvic US is an excellent test that is usually done for endometriosis
and may show the adnexal cyst (chocolate cyst) or endometrial seedings
in the Douglas pouch, but it is not the gold standard test.
MRI gives detailed image of the pelvic area and would be useful but
again, not the gold standard.
A 23 year old woman presents to your office complaining of several
nontender, asymptomatic, slightly umbilicated, 3-mm nodules on her lower
abdomen. Hypodermic probe of a nodule reveals a cheesy substance. Which
of the following is the most likely diagnosis

a) Sebaceous cysts
b) Condylomata lata
c) Lichen planus
d) Psoriasis
e) Molluscum contagiosum
The correct answer is E
Explanation
Molluscum is caused by a pox virus and is spread by direct contact. It
is mildly contagious. The classic lesion is a small nodule, or domed
papule with an umbilicated center. These lesions range from 1-5 mm in
diameter and have a caseous material filling them. Treatment is excision
with a dermal curette followed by chemical treatment of the base with

Monsels or trichloroacetic acid.


A. Sebaceous cysts are not umbilicated and are below the dermis.
B. This is a flat fleshy lesion of secondary syphilis, usually found on
mucous membranes.
C. This is a flat popular dermatologic lesion. It is not umbilicated.
D. This is an exfoliative popular dermatologic lesion. It is not
umbilicated.
The definition of a post-term pregnancy is a pregnancy that has reached

a) 40 weeks gestation
b) 41 weeks gestation
c) 42 weeks gestation
d) 39 weeks gestation
The correct answer is C
Explanation
Postdate and post-term pregnancy are terms that are used
interchangeably. The postdate pregnancy is defined as a pregnancy that
has reached 42 weeks of amenorrhea. This is important because perinatal
mortality doubles at 42 weeks gestational age. The diagnosis of postdate
pregnancy depends heavily on accurate dating methods.
An 18-year-old primigravida at 38 weeks gestation complains of a
headache. Her blood pressure is 130/92 mm Hg. The fetal heart rate is
140 beats/min. A urine dipstick shows 2 + protein.
Laboratory Findings
Hemoglobin----------------------10.8 g/dL (N 12.0 - 16.0)
Hematocrit----------------------32.4% (N 36.0 - 46.0)
Platelets----------------------110,000/mm3 (N 150,000 - 400,000)
WBCs----------------------14,900/mm3 (N 4000 - 10,000)
Creatinine----------------------..0.5 mg/dL (N 0.8 - 1.3)
AST (SGOT) ----------------------31 U/L (N 0 - 37)
ALT (SGPT) ----------------------60 U/L (N 0 - 65)
LDH----------------------240 U/L (N 100 - 190)
Bilirubin----------------------1.9 mg/dL (N 0.0 - 1.0)
A nonstress test is reactive and the amniotic fluid index is 9.4 (N 8.0
- 20.0). The patient is admitted for further testing. After 24 hours
repeat testing shows the following:
Hemoglobin----------------------9.8 g/dL
Hematocrit----------------------30.2%
Platelets----------------------92,000/mm3
WBCs----------------------15,200/mm3
Creatinine----------------------.0.6 mg/dL
AST (SGOT) ----------------------72 U/L
ALT (SGPT) ----------------------98 U/L
LDH----------------------620 U/L
Bilirubin----------------------2.4 mg/dL
24-hour urine protein----------------------2400 mg
Which one of the following would be the most appropriate course of
action at this point?

a) Continued monitoring, repeating the 24-hour urine collection, and


repeating the laboratory studies tomorrow
b) Immediate delivery by cesarean section
c) Discharge to home on bed rest, with close follow-up
d) Induction of labor with oxytocin (Pitocin) if the cervix is favorable
The correct answer is D
Explanation
This patient has hemolysis, elevated liver enzymes, and low platelets
(HELLP) syndrome and needs to be delivered. There is no reason to delay
delivery in a term pregnancy. HELLP syndrome is a form of severe
preeclampsia. If the patient has a favorable cervical examination, labor

induction with oxytocin is appropriate. If the cervix is unfavorable,


cesarean delivery should be considered to expedite delivery.
A 5 cm right ovarian cyst is found incidentally during the first
prenatal examination of an otherwise healthy 22 year old primigravida at
12 weeks gestation. The most likely diagnosis is

a) A dermoid cyst
b) A mucinous cystadenoma
c) An endometrioma
d) A follicular cyst
e) A corpus luteum cyst
The correct answer is E
Explanation
An ovarian cyst is an enlargement of the ovary that appears to be filled
with fluid. The vast majority of ovarian cysts diagnosed in early
pregnancy represent a physiological cyst known as the corpus luteum
(cystic or hemorrhagic). This is a normal finding in early pregnancy.
The corpus luteum supports the lining of the womb and in turn the
pregnancy itself. The corpus luteum does this by producing the pregnancy
hormone progesterone which helps nurture the pregnancy. Once the
pregnancy gets beyond the 1st trimester, the corpus luteum is no longer
needed and therefore this resolves spontaneously, not causing any harm
whatsoever to the mother or baby.
Most ovarian cysts diagnosed in early pregnancy do not cause any
symptoms. They tend to be an incidental finding and women are unaware of
their presence. However, if an ovarian cyst ruptures, twists or if there
is bleeding into the middle of the cyst, then lower abdominal pain on
the side of the ovarian cyst occurs. Most ovarian cysts diagnosed in
early pregnancy do not represent ovarian cancer; in fact the risk of
ovarian cancer in pregnancy is extremely rare indeed (1 in 15,000 to 1
in 32,000 pregnancies).
All of the following will be diagnosed by ultrasound at 16 weeks, except

a)
b)
c)
d)

Anencephaly
Neural tube defect
Trisomy 21
Gender of fetus
The correct answer is D

Explanation
Obstetric ultrasound done at 16 weeks can detect abnormalities such as a
trisomy (eg down syndrome), neural tube defects (eg spina bifida,
anencephaly).
The sex of the baby can usually be determined by ultrasound at any time
after 16 weeks, often at the dating scan around 20 weeks into the
pregnancy depending upon the quality of the sonographic machine and
skill of the operator. This is also the best time to have an ultrasound
done as most infants are the same size at this stage of development.
A 36 year old woman develops fever, chills and flank pain. She presents
to the ER and is diagnosed with pyelonephritis. What is the most likely
causative organism?

a) E. coli
b) Chlamydia
c) S. pneumonia
d) S. aureus
The correct answer is A
Explanation
Pyelonephritis is a bacterial infection of one or both kidneys.

Infection can spread up the urinary tract to the kidneys, or the kidneys
may become infected through bacteria in the bloodstream. Chills, fever,
back pain, nausea, and vomiting can occur.
Urine and sometimes blood tests are done to diagnose pyelonephritis.
Escherichia coli, a type of bacteria normally in the large intestine,
causes about 90% of cases of pyelonephritis. Antibiotics are given to
treat the infection.
A 30-year-old black female presents with a vaginal discharge. On
examination the discharge is homogeneous with a pH of 5.5, a positive
whiff test, and many clue cells.
Which one of the following findings in this patient is most specific for
the diagnosis of bacterial vaginosis

a) The pH of the discharge


b) The presence of clue cells
c) The character of the discharge
d) The whiff test
e) Age of patient
The correct answer is B
Explanation
Patients must have 3 of 4 Amsel criteria to be diagnosed with bacterial
vaginosis. These include a pH > 4.5 (most sensitive), clue cells > 20%
(most specific), a homogeneous discharge, and a postive whiff test
(amine odor with addition of KOH).
In a normal pregnancy at 16 weeks gestation, which of the following has
the highest alpha fetoprotein concentration?

a)
b)
c)
d)
e)

Amniotic fluid
Fetal cerebrospinal fluid
Maternal serum
Fetal serum
Fetal urine
The correct answer is D

Explanation
Produced primarily in the fetal liver, alpha fetoprotein is the major
oncotic protein in the fetus. It reaches a peak value in fetal serum at
12-14 weeks, at a level of about 3 mg/ml. The peak value in fetal
amniotic fluid is around 40 ?m/ml and occurs at or just after that in
fetal serum. The maternal serum reaches its peak of 200 ng/ml at the end
of the second trimester and begins to gradually decrease after 30 weeks.
Thus, the concentration of AFP in the fetal serum is 10,000-fold greater
than that in maternal serum.
A. The concentration of AFP in the amniotic fluid is less than that in
fetal blood.
B. The concentration of AFP in the cerebrospinal fluid is less than that
in fetal blood.
C. The concentration of AFP in the maternal serum is 10,000-fold less
than that in fetal blood.
E. The concentration of AFP in the fetal urine is less than that of
fetal blood. In cases of severe fetal nephritic syndrome, fetal urine
will have a high AFP concentration, and the MSAFP will often be
significantly elevated (sometimes well beyond that found with a neural
tube defect).
A 22 year old G0 female presents to the office complaining of irregular
cycles.
She rarely had regular cycles since her menarche (11). She is not
sexually active. Her past medical history is non contributory. Her
family history is significant for diabetes in both parents.
Her Temp is 37.0 C, BP is 140/87 mm Hg. She is 170 cm tall and weighs 80
Kg. Black hair is noted on her chin and upper lip. Her physical exam

including speculum and bimanual examination is normal, without any


tenderness or palpable masses.
What is the most likely diagnosis of this patient?

a) Hypothyroidism
b) Polycystic ovarian syndrome (PCOS)
c) Prolactinoma
d) Sheehans syndrome
e) Turner syndrome
The correct answer is B
Explanation
According to Rotterdam criteria, any two of the following diagnose
polycystic ovarian syndrome (PCOS):
1-Oligo and/or Amenorrhea
2-Clinical and/or Biochemical signs of an increased androgens status
(acne, hirsutism)
3-Ultrasound detection of polycystic ovaries
Our patient meets the first two criteria for diagnosis; however
ultrasound of the ovaries, hormonal assessment (LH/FSH ratio,
androgens) and metabolic evaluation (FBS, lipid profile) would be
warranted.
Turners syndrome presents with primary amenorrhea, short stature and
other congenital abnormalities.
Hypothyroidism usually presents with cold intolerance, fatigue,
sleepiness, irregular heavy cycles, weight gain?
The usual presentation of prolactinoma includes amenorrhea,
galactorrhea, decreased libido and visual fields defects (Bitemporal
hemianopia or Tube Vision).
Sheehans syndrome is the postpartum pituitary necrosis caused by a
severe postpartum hemorrhage.
A 34 year old white primigravida in her first trimester had established
moderate hypertension before becoming pregnant. She currently has a
blood pressure of 168/108 mm Hg. You are considering how to best manage
her hypertension during the pregnancy. Which one of the following is
associated with the greatest risk of fetal growth retardation if used
for hypertension throughout pregnancy?

a) Atenolol (Tenormin)
b) Nimodipine (Nimotop)
c) Methyldopa (Aldomet)
d) Hydralazine (Apresoline)
e) Nifedipine (Procardia, Adalat)
The correct answer is A
Explanation
Atenolol and propranolol are associated with intrauterine growth
retardation when used for prolonged periods during pregnancy. They are
class D agents during pregnancy. Other beta-blockers may not share this
risk.
Methyldopa, hydralazine, and calcium channel blockers have not been
associated with intrauterine growth retardation. They are generally
acceptable agents to use for established, significant hypertension
during pregnancy.
A 28 year old gravida 2 para 2 notes bilateral milky discharge from her
breasts. She delivered her last child 2 years ago, and breastfed
exclusively for 8 months and at night for a few more months. She totally
stopped breastfeeding several months ago, but she can still express milk
from both breasts daily. She takes no medications, and uses a diaphragm

for contraception. The physical examination is unremarkable except that


a milky discharge is easily expressible from both nipples. The most
likely diagnosis is

a) Intraductal papillomatosis
b) Mammary duct ectasia
c) Empty sella syndrome
d) Illicit drug ingestion
e) Physiologic galactorrhea
The correct answer is E
Explanation
The causes of galactorrhea are multiple, including intraductal
papillomatosis, mammary duct ectasia, empty sella syndrome,
hyperprolactinemia, hypothyroidism, and illicit drug ingestion. However,
bilateral galactorrhea, or milk production, can be physiologic for up to
2 years after breastfeeding an infant. It is also more likely if there
continues to be breast stimulation, such as this womans daily
expression of milk.
A recently married 29-year-old nulliparous black female presents with
uncomplicated cystitis. She is otherwise healthy. She reports that she
is currently using barrier brith control methods but plans to
discontinue this soon, as she would like to become pregnant. Which one
of the following supplemental vitamins or minerals would you advise this
patient to begin taking?

a) Thiamine
b) Iron
c) Calcium
d) Folate
e) Iodine
The correct answer is D
Explanation
A significant number of neural tube defects can be prevented with a
daily intake of supplementary folate through the preconceptional and
prenatal period. The current health guidelines recommend a daily
supplement of 0.4-0.8 mg of folic acid, beginning at least 1 month prior
to conception and continuing through the first trimester in otherwise
healthy women. Women with a prior history of a pregnancy affected by a
neural tube defect should take 4 mg/day in the preconceptional period.
The other options listed do not require supplementation in the
preconceptional period.
The selective estrogen receptor modulator Raloxifene has all of the
following benefits, except

a) No agonistic effect on breasts


b) Provides estrogenic benefit on bone
c) Mimics estrogenic effect on cardiovascular status
d) No agonistic effect on uterine tissue
e) Provides relief of hot flushes associated with menopause
The correct answer is E
Explanation
Raloxifene is a selective estrogen receptor modulator that produces both
estrogen-agonistic effects on bone and lipid metabolism and
estrogen-antagonistic effects on uterine endometrium and breast tissue.
Because of its tissue selectivity, raloxifene may have fewer side
effects than are typically observed with estrogen therapy. The most
common adverse effects of raloxifene are hot flushes and leg cramps. The
drug is also associated with an increased risk of thromboembolic events.
The beneficial estrogenic activities of raloxifene include a lowering of
total and low-density lipoprotein cholesterol levels and an augmentation
of bone mineral density. Raloxifene has been labeled for the prevention
of osteoporosis.

A 24 year old primiparous female delivered a 4200-g (9 lb 4 oz) male


infant after a prolonged 18-hour labor requiring oxytocin (Pitocin)
augmentation. A vacuum-assisted delivery was performed because of
maternal exhaustion, and the mother required repair of a third degree
perineal laceration. Thirty minutes after you leave the delivery room,
the delivery nurse calls you urgently to report that the mothers blood
pressure is 80 mm Hg systolic and that the bed is soaked with blood. The
most likely cause of this problem is

a) Occult cervical lacerations


b) Uterine atony
c) Uterine rupture
d) Disseminated intravascular coagulopathy
The correct answer is B
Explanation
Ninety percent of early and immediate postpartum hemorrhage is due to
failure of the uterus to contract satisfactorily (uterine atony). Other
less frequent causes are lacerations of the cervix, vagina, or perineum;
hematomas, usually located near lacerations or episiotomy repairs; and
uterine rupture, either spontaneous or iatrogenic. All of these occur in
the immediate postpartum period. Delayed hemorrhage, occurring beyond
the first 24 hours after delivery, is usually caused by retained
placental fragments. Interestingly, placenta accreta is among the most
common causes of postpartum hemorrhage necessitating hysterectomy.
Which of the following is safe to give to a pregnant women with deep
venous thrombosis (DVT)?

a) Warfarin
b) Heparin
c) Antistreptokinase
d) Alteplase
The correct answer is B
Explanation
A deep vein thrombosis (DVT) is a blood clot that forms in a deep vein.
Veins are blood vessels that take blood towards the heart. Deep veins in
the leg run through the muscles of the calf and thighs. The most common
sites for a DVT during pregnancy and after birth are in a vein in the
leg (especially the calf or thigh) or in the pelvis (lower part of
abdomen).
Treatment is with an injection of heparin. This medication is said to
thin the blood (an anti-coagulant). There are different types of
heparin. The type that is commonly chosen in pregnancy is low molecular
weight heparin (LMWH). Heparin is safe to give during pregnancy because
it does not cross the placenta.
The recommended time to screen asymptomatic pregnant women without risk
factors for gestational diabetes is

a) In the first trimester


b) At 16-20 weeks gestation
c) At 24-28 weeks gestation
d) At 35-37 weeks gestation
The correct answer is C
Explanation
The recommended time to screen for gestational diabetes is 24-28 weeks
gestation. The patient may be given a 50-g oral glucose load followed by
a glucose determination 1 hour later.
A 70 year old woman presents to your office with a lump in her breast.
Which one of the following is the greatest risk factor for cancer?

a) Nulliparity
b) Positive family history
c) Use of hormone replacement therapy
d) Early menarche
e) Advanced age
The correct answer is E
Explanation
Strong risk factors for breast cancer:
Increasing age: the primary risk factor for breast cancer in most women
is older age. Overall, 85 percent of cases occur in women 50 years of
age and older, while only 5 percent of breast cancers develop in women
younger than age 40.
Family history: women who have a family history of breast or ovarian
cancer are at a higher risk for breast cancer than those who lack such a
history.
Other risk factors:
Early menarche: during a woman's reproductive years, estrogen stimulates
cells of the breast's glandular tissue to divide. The longer a woman is
exposed to estrogen, the greater her risk for breast cancer. Estrogen
exposure is increased if a woman began menstruating at or before 11
years of age, or if she experiences menopause at age 55 years or older.
Nulliparity: women who have never given birth are more likely to develop
breast cancer after menopause than women who have given birth multiple
times.
Hormone replacement therapy (HRT): Studies have shown that long-term use
of combined estrogen-progestin (approximately five years) in women ages
50 to 79 increases a woman's risk of breast cancer.
A 6 month old baby is found to be HIV positive by ELISA and Western
blot. What is the most likely scenario?

a) Baby is negative and the mothers status is unknown


b) Baby is positive mother is unknown
c) Baby is positive and mother is positive
d) More testing on the baby is needed to determine HIV status
The correct answer is C
Explanation
The HIV virus can be transmitted by unprotected sexual contact (vaginal,
anal, or oral sex), sharing needles, transfused blood products, mother
to newborn (30% risk), and occupational needle stick exposures.
A relatively simple, accurate blood test that detects antibodies to HIV
(ELISA test) is used to screen people for HIV infection. If the ELISA
result is positive, it is confirmed with a more accurate test, usually
the Western Blot. Both tests often are not positive in the first month
or two after HIV infection because it takes the body that long to
produce antibodies against the virus.
This baby is definitely HIV positive, who most likely obtained the virus
from her mother. Since the age is only 6 months it is unlikely that the
child obtained the HIV through the other types of transmission (eg
sexual contact, sharing needles). Since 1985 all blood products are
tested for HIV before being given to others for transfusion.
A 26 year old primigravida presents at 40 weeks in active labour with
contractions every 2 minutes. She is diagnosed as having a transverse
lie with the back up. Which of the following would be the most
appropriate next step?

a) Start isoxuprine (Vasodilan)

b) Perform an external version


c) Prepare for an immediate C-section
d) Rupture membranes and then perform an internal version
The correct answer is C
Explanation
Transverse lie is when the baby is laying sideways in the uterus.
Sometimes the baby can change positions before labor starts. Sometimes
an external version is being done where the baby is manually scooted
into a better position.
Fetal position is transverse, with the fetal long axis oblique or
perpendicular rather than parallel to the maternal long axis.
Shoulder-first presentation requires cesarean section unless the fetus
is a 2nd twin. Should a transverse lie complicate the labour then it can
be spotted early and a Caesarean delivery carried out.

Which one of the following is contraindicated for the treatment of


hypertension in pregnancy?

a)
b)
c)
d)

Methyldopa (Aldomet)
Lisinopril (Prinivil, Zestril)
Labetalol (Normodyne)
Nifedipine (Procardia)
The correct answer is B

Explanation
ACE inhibitors such as lisinopril can be severely damaging to the fetus,
with deformity, renal failure, and death possible. Placental blood flow
may be markedly reduced and extreme fetal distress and in utero death
may occur. The drugs are not recommended for use, even short-term use,
during pregnancy. Methyldopa, nifedipine, labetalol, and atenolol all
are used for the treatment of hypertension in pregnancy.

The most common cause of intrauterine growth retardation is

a) Chronic hypertension
b) Alcohol intake
c) Preeclampsia
d) Herpesvirus infection
e) Diabetes mellitus
The correct answer is A
Explanation
The most common cause of intrauterine growth retardation (IUGR) is

chronic hypertension. Moreover, infants of hypertensive mothers have a


threefold increase in perinatal mortality compared to infants with IUGR
who are born to normotensive mothers.
Which of the following drugs is contraindicated in gestational diabetes
and diabetes mellitus type 2?

a) Rosiglitazone
b) Glyburide
c) Chlorpropamide
d) Metformin
The correct answer is C
Explanation
Ordinarily, the hypoglycemia of infants of diabetic mothers is brief and
asymptomatic. It is considered to result from fetal hyperinsulinism
secondary to prenatal hyperglycemia. However reports of prolonged
symptomatic hypoglycemia associated with maternal chlorpropamide
(Diabinese) therapy has been reported. Therefore this drug is not
recommended in gestational diabetes.
The other choices are commonly uses medicines in the treatment of diabetes.
The commonest indication for amniocentesis is

a) Chromosome anomaly of the mother


b) Teenage pregnancy
c) Advanced maternal age
d) Family history of Down syndrome
e) Family history of cystic fibrosis
The correct answer is C
Explanation
The indications for amniocentesis include: Women who are 35 or older
when their baby is due (most common indication). Women who have had an
abnormal first or second trimester screen. Any couple who has had a
previous child with Down syndrome or other chromosome abnormality. Any
couple who has had a previous child with spina bifida or anencephaly.
Any couple for whom one parent has a known chromosome rearrangement.
Women at risk for a child with a genetic condition such as hemophilia,
muscular dystrophy, Tay-Sachs, cystic fibrosis, or a hemoglobinopathy.
Women who take certain medications to control seizures.
A sexually active 24 year old woman presents with frothy vaginal
discharge. You perform a wet mount and it shows Trichomonas vaginalis.
What is the best treatment?

a) Metronidazole
b) Ceftriaxone
c) Doxycycline
d) Topical antifungal
The correct answer is A
Explanation
This is a classic wet mount showing the motile trichomonads and their
flagella. Treatment of choice for Trichomonas vaginals is metronidazole
for both the patient and her sexual partner.

Ceftriaxone is used to treat gonorrhea, doxycycline for chlamydia and


antifungals for yeast infections such as candida.
A 24-year-old female presents with a complaint of mild fullness in the
neck. A review of systems is negative, except for some poor sleep
related to the care of her 4-month-old infant, and mild palpitations at
times when she is tired. Her pregnancy was uneventful, and breastfeeding
is going well. Findings on examination are normal except for enlargement
of the thyroid. Her TSH level is 0.1 ?U/mL (N 0.3-5.0).
Which one of the following would be most appropriate at this point?

a) Reassure the patient that this is a common event that requires no


treatment unless significant symptoms develop
b) Prescribe propranolol (Inderal) and have the patient stop
breastfeeding
c) Advise the patient to stop breastfeeding, start low-dose
propylthiouracil, and adjust on a biweekly basis until the TSH has
returned to normal
d) Arrange for a radioactive iodine uptake scan, and if low start
levothyroxine (Synthroid), 0.05 mg/day, and recheck the TSH level in 6
weeks
The correct answer is A
Explanation
Postpartum thyroiditis is a common condition, occurring after 3%-16% of
pregnancies. It is thought to be a variation of Hashimotos disease and
can present in several ways. It can cause hyperthyroidism, beginning 1-4
months after delivery and lasting for 2-8 weeks. Thyroid function then
either returns to normal or the patient develops transient or permanent
hypothyroidism. Another possible manifestation is hypothyroidism
beginning 2-6 months after delivery, which again can either be transient
or become permanent. A third possibility is that the patient can develop
a euthyroid goiter.
As in this case, the symptoms are usually mild, and can be confused with
the typical feelings of a new mother. Since the symptoms are mild and
the hyperthyroid stage is brief, treatment is not necessary in the
majority of cases. The hyperthyroid symptoms should be explained to the
patient, and she should also be made aware of the symptoms of
hypothyroidism, since it is a common development after the hyperthyroid
stage has passed and may be permanent. Breastfeeding was going well in
this patient, and should be continued. Propranolol is not needed unless
the palpitations worsen. Propylthiouracil is used for Graves disease,
to counteract overproduction of thyroid hormone. With
postpartum thyroiditis, as with other types of thyroiditis, thyroid
hormone is released from the gland as a result of autoimmune injury, but
production of thyroid hormone is actually low. Propylthiouracil has no
place in the treatment of thyroiditis.

Radioactive thyroid scanning is not necessary unless symptoms are


significant and are not resolving, in which case Graves disease
masquerading as thyroiditis must be ruled out. Thyroiditis would cause
low uptake, but this has no bearing as to whether thyroid hormone needs
to be given. Thyroid hormone is used in postpartum thyroiditis if the
person is found to be hypothyroid (with high levels of TSH), with
symptoms significant enough to require treatment. Treatment would be
continued for 1-2 months and then stopped, and the TSH level rechecked 1
month later to see if the hypothyroid condition has resolved.
For the last 5 years, this obese 33-year-old nulligravid has been
unsuccessful in her attempts at getting pregnant. Her menses are
irregular and have been that way since menarche at age 12. She has never
used contraception. She frequently has to shave unsightly facial hair.
Her pelvic exam shows thin watery cervical mucus with somewhat enlarged
adnexa bilaterally. All of the following would be helpful in the workup
and/or management of this condition EXCEPT:

a) Serum fasting glucose to insulin ratio


b) Serum free testosterone level
c) Serum estradiol level
d) Endometrial biopsy
e) Ovulation induction with clomiphene citrate
The correct answer is C
Explanation
This is the classic picture of ?polycystic ovarian disease?. The name of
course is a misnomer, since the ovarian findings are simply a
manifestation of the disease process and not the cause. On exam, she
clearly has an abundance of estrogenic mucus, so a serum estradiol level
would not be helpful. Hirsutism, acanthosis, nigricans,
hyperandrogenism, and insulin resistance are classic features of this
syndrome.
A. A ratio of less than 4.5 is consistent with insulin resistance.
B. Testosterone levels should be measured in the workup of
hyperandrogenic states. It is possible that an ovarian or adrenal tumor
could be the source of elevated androgens.
D. With a long history of unopposed estrogen, the endometrium is at risk
for neoplasia.
E. Clomiphene citrate lowers the negative feedback of estrogen at the
hypothalamus. This leads to an increase in the levels of FSH and LH. It
is not effective in the hypoestrogenic patient.
A 37 year old gravida 3 para 2 at 33 weeks gestation reports the onset
of brisk vaginal bleeding. On examination the uterus is nontender and 32
cm above the symphysis. Pelvic examination reveals the presence of a
large amount of bright red vaginal blood. This presentation is most
consistent with

a) Threatened abortion
b) Hemorrhagic cystitis
c) Placenta previa
d) Chorioamnionitis
e) Abruptio placentae
The correct answer is C
Explanation
The classical clinical presentation of placenta previa is painless,
bright red vaginal bleeding. This diagnosis must be considered in all
patients beyond 24 weeks gestation who present with bleeding. Threatened
abortion is unlikely at this stage of pregnancy and hemorrhagic cystitis
is not accompanied by brisk bleeding. Abruption of the placenta is the
most common cause of intrapartum fetal death but is associated not only
with brisk vaginal bleeding, but also with uterine tenderness that may
be marked. Clinical signs of chorioamnionitis include purulent vaginal
discharge, fever, tachycardia, and uterine tenderness.
A 24 year old female at 36 weeks gestation plans to breastfeed her
infant. She has a history of bipolar disorder, but is currently doing

well without medication, and also has a history of frequent urinary


tract infections. She asks you about medications that she may need to
take after delivery, and how they may affect her newborn. Which one of
the following would be contraindicated if she breastfeeds her infant?

a) Amoxicillin
b) Macrodantin (Macrobid)
c) Valproic acid (Depakote)
d) Lithium
The correct answer is D
Explanation
Of the drugs listed, the only maternal medication that affects the
infant is lithium. Breastfed infants of women taking lithium can have
blood lithium concentrations that are 30% - 50% of therapeutic levels.
Breast engorgement in a woman eager to breastfeed is best managed by

a) Frequent nursing
b) Diuretics
c) Oxytocin nasal spray
d) Tight binder until condition relieved
e) Bromocriptine administration
The correct answer is A
Explanation
Breastmilk usually "comes in" sometime during the first week after
delivery. This means the milk changes from colostrum, or early milk, to
mature milk. The body may make more than a baby needs during this period
and it is easy to become overly full.
It is important to tell patients that to prevent engorgement: nurse
frequently, about 8-12 times per day. Make sure the baby latches-on well
so he/she will empty the breasts effectively. Do not skip feedings or
give formula feedings during the first several weeks.
A 28 year old woman has a 3 year history of primary infertility. She
presents with increasing symptoms of steady, aching lower abdominal pain
at the time of menses. The pain persists throughout menstruation and
often after, and radiates into the rectum. Tender nodules in the
uterosacral ligaments are noted on pelvic examination. Which one of the
following would be the most contributory investigation?

a) Postcoital test
b) Diagnostic laparoscopy
c) Hysterosalpingogram on day 9 of her cycle
d) Endometrial biopsy on day 26 of her cycle
e) Basal body temperature charting
The correct answer is B
Explanation
Endometriosis is a noncancerous disorder in which functioning
endometrial tissue is implanted outside the uterine cavity. Symptoms
depend on location of the implants and may include dysmenorrhea,
dyspareunia, infertility, dysuria, and pain during defecation.
Endometriosis is usually confined to the peritoneal or serosal surfaces
of pelvic organs, commonly the ovaries, broad ligaments, posterior
cul-de-sac, and uterosacral ligaments.
Pelvic pain, pelvic mass, alteration of menses, and infertility are
typical. Some women with extensive endometriosis are asymptomatic; some
with minimal disease have incapacitating pain. Dyspareunia and midline
pelvic pain before or during menses may develop. Such dysmenorrhea is an
important diagnostic clue, particularly if it begins after several years
of pain-free menses.
Diagnosis is suspected based on typical symptoms but must be confirmed

by biopsy, usually via pelvic laparoscopy.


A 42 year old white female has a 2-month history of vaginal itching.
Another physician prescribed three courses of antifungal therapy, but
the condition persists. The patient says there has been no discharge,
and there are no skin lesions anywhere else. The physical examination is
remarkable only for a lacy white rash on the labia minora. Which one of
the following is the most likely diagnosis

a) Atrophic vaginitis
b) Lichen planus
c) Contact dermatitis
d) Chronic candidiasis
e) Vaginal adenosis
The correct answer is B
Explanation
This lesion fits best with the diagnosis of vulvar lichen planus,
analogous to oral lichen planus. The genitals are a common site, and
when mucous membranes are involved, there will be a lacy white or light
gray reticulate pattern like that seen on the buccal mucosa with oral
lesions. The cause of the condition is unknown, and it typically
resolves spontaneously after 6-24 months. There is no treatment that
shortens the disease, but topical corticosteroids may relieve the
itching and improve cosmetic appearance. This woman is a little too
young for atrophic vaginiti and a little too old for vaginal adenosis.
Contact dermatitis is a nonspecific erythema without white patches.
Chronic candidiasis in an otherwise healthy woman would be expected to
respond to conventional therapy.
A 13-year-old patient has had regular menses for 1 year, with
debilitating pain beginning in the lower abdomen a few hours before
menses and lasting 24 hours. Physical examination is completely normal.
Optimal management at this time is:

a) Psychiatric referral
b) Diagnostic laparoscopy
c) Trial of oral contraceptives
d) Trial of prostaglandin synthetase inhibitors
e) Reassurance with follow-up evaluation in 6 months
The correct answer is D
Explanation
Dysmenorrhea is defined as a severe painful cramping sensation in the
lower abdomen, often accompanied by other biologic symptoms, including
sweating, tachycardia, headaches, nausea, vomiting, and diarrhea. All of
these occur during or just before menses. The term primary dysmenorrheal
is reserved for women with no obvious pathologic condition, and this is
due to the effects of endogenous prostaglandins.
A. Dysmenorrhea in a 13-year-old is usually due to the effects of
endogenous prostaglandins.
B. Usually, no visible peritoneal pathology can be found in primary
dysmenorrheal.
C. Although OCPs have been used for this condition, they are not as
effective as prostaglandin synthetase inhibitors.
E. This pain is debilitating to the patient. Reassurance with follow-up
evaluation most likely will not decrease her pain and discomfort.
At her first obstetric visit, a patient does not remember the date of
her last menstrual period. She has not felt fetal motion. The fetal
heartbeat is audible with the Doppler fetoscope but not with the DeLee
stethoscope. At her second visit one month later she has just felt fetal
quickening. The uterine fundus measure 17 cm. Which one of the following
would describe the state of gestation?

a) 10 to 12 weeks
b) 14 to 16 weeks

c) 16 to 20 weeks
d) 20 to 24 weeks
e) 24 to 28 weeks
The correct answer is C
Explanation
In pregnancy terms, the moment of quickening refers to the initial
motion of the fetus in the uterus as it is perceived or felt by the
pregnant woman. According to the Oxford English Dictionary, to "quicken"
means "to reach the stage of pregnancy at which the child shows signs of
life."
A woman pregnant for the first time (i.e. a primiparous woman) typically
feels fetal movements at about 20-21 weeks, whereas a woman who has
already given birth at least two times (i.e. a multiparous woman) will
typically feel movements around 18 weeks.
Since as a rule of thumb, the fundal height (in centimeters) should
roughly equal the number of weeks of pregnancy, this patient is probably
at 16-20 weeks gestation.
You see a pregnant woman at 12 weeks gestation. She has a history of
preterm labour in a previous pregnancy at 33 weeks. You perform a
vaginal swab and it is positive for bacterial vaginosis, but she is
asymptomatic. What is the appropriate management?

a) Give oral metronidazole


b) Give vaginal clindamycin
c) No treatment
d) Give doxycycline
e) IV Ceftriaxone
The correct answer is A
Explanation
There is significant evidence that links Bacterial Vaginosis with
preterm labor. There have been many recent studies that have been
conducted to verify this information and find a method of prevention.
Other studies have also shown a possible link to miscarriages, low birth
weight and premature rupture of membranes.
Treatment is highly recommended to avoid any chance of preterm labor.
There are various treatments which include:
Oral medications ? Clindamycin 300 mg or Metronidazole 500 mg twice
daily for 7 days
Topical medications ? Clindamycin 5 g or Metronidazole at bedtime for 5
days (this treatment may give symptomatic relief but is insufficient in
preventing pregnancy complications).
A healthy 36-year-old female presents with vaginal bleeding 3 weeks
after a missed menstrual period. A pelvic examination reveals a dilated
cervix with products of conception visible in the vaginal vault.
Which one of the following is the most likely cause of this condition?

a) Caffeine use
b) Advanced maternal age
c) Previous history of elective abortion
d) Recent sexual activity
e) Chromosomal abnormality
The correct answer is E
Explanation
Although heavy caffeine use, advanced maternal age, and a previous
history of multiple elective abortions are all considered risk factors
for spontaneous abortion, the most common cause, which accounts for
nearly 50% of spontaneous abortions, is chromosomal abnormalities. Most
chromosomal abnormalities are random events, such as maternal and

paternal gametogenesis errors, dispermy, and nondisjunction. Sexual


activity does not raise the risk of spontaneous abortion in women with
uncomplicated pregnancies.
A woman in labor with twins successfully delivers her first baby
vaginally. The second baby however is in breech position and still
intrauterine. Which of the following is the least appropriate management?

a) Cesarean section
b) Spontaneous breech birth
c) Partial breech extraction
d) Total breech extraction
The correct answer is A
Explanation
Under such unusual circumstances, when a second twin is breech or a
breech vaginal birth is progressing quickly, a cesarean is neither
recommended nor possible.
There are several different types of vaginal breech deliveries:
The delivery can occur without help from a health professional
(spontaneous breech birth). During a spontaneous breech birth, the fetus
comes out of the vagina without problems. The health professional just
supports the fetus's body as it emerges from the birth canal.
A health professional may need to help deliver the upper part of the
fetus's body (partial breech extraction). During a partial breech
extraction, a health professional pulls gently downward on the fetus and
rotates its body as needed to deliver the shoulders, arms, and head.
Occasionally, a health professional may need to help the fetus during
the entire delivery (total breech extraction). This is usually done only
when the fetus is having problems and needs to be delivered as quickly
as possible.
During a partial or total breech extraction, the health professional may
need to reach into the birth canal in order to move part of the fetus
into a better position for delivery. If the health professional is
having difficulty delivering the fetus's head, forceps may be used to
guide the head through the birth canal. Forceps may also be used to
speed delivery if the fetus is in danger.
You are asked to evaluate a 6-year-old girl who has fallen off her
Brothers bicycle and is complaining of severe vulvar pain. The girl
will not permit anyone to touch her vulva. However, on inspection, the
upper labia majus is blue and there is vaginal bleeding. What is the
next step in managing her injury?

a)
b)
c)
d)

Perform the examination under anesthesia


Have her mother restrain her during the examination
Have a medical assistant restrain her during the examination
Send her home to use ice packs and reschedule the examination for

the next day


e) Perform a laparotomy to evaluate for penetrating trauma
The correct answer is A
Explanation
In a young child, if the straddle injury is nonpenetrating and
associated with a nonexpanding small vulvar hematoma, then ice packs and
conservative therapy are preferred. In the presence of vaginal bleeding,
an examination under anesthesia is required. The depth of many
lacerations is greater than initially suspected and can involve
neighboring organs and structures. Although a fall is the usual cause of
a straddle injury in a young child, sexual abuse must always be
considered in the differential diagnosis.
B. Having anyone, including the patients mother, attempting to restrain
the girl while she is being examined will often result in an

unsatisfactory exam as well as emotional trauma.


C. See answer to B.
D. With the presence of vaginal bleeding, penetrating trauma must be
ruled out.
E. If peritoneal contents are seen spilling into the vagina during the
exam under anesthesia, then a laparotomy may be necessary.
What is the test used to diagnose cervical incompetence in a pregnant woman?

a) Abdominal ultrasound
b) Hysterosalpingogram
c) CT of pelvis
d) Transvaginal ultrasound
The correct answer is D
Explanation
Cervical incompetence is painless cervical dilation resulting in
delivery of a live fetus between 16 and 22 weeks.
In women with weak cervical tissue, the enlarging products of conception
cause the cervix to dilate prematurely. Overall risk of recurrence of
cervical incompetence is probably ? 30%. Risk is greatest for women with
? 3 prior 2nd-trimester fetal losses.
Cervical incompetence is diagnosed clinically. There is increased use of
routine second trimester transvaginal ultrasound to diagnose.
Cerclage (reinforcement of the cervical ring with suture material)
appears to prevent preterm delivery in patients with ? 3 prior
2nd-trimester fetal losses.
Which one of the following is true regarding routine prenatal screening
ultrasonography before 24 weeks gestation?

a) It has been shown to reduce maternal mortality


b) It has been shown to reduce perinatal mortality
c) It has been associated with increased hearing abnormalities in
infants
d) It has been associated with better Apgar scores in infants at 1
minute of life
e) It has not been proven to have any significant benefits
The correct answer is E
Explanation
Routine ultrasonography at around 18-22 weeks gestation has become the
standard of care in many communities. Acceptance is based on many
factors, including patient preference, medical-legal pressure, and the
perceived benefit by physicians. However, rigorous testing has found
little scientific benefit for, or harm from, routine screening
ultrasonography.
A 26 year old pregnant woman delivers a male infant at 40 weeks, with a
birth weight of 2.0 kg. His head circumference is 34 cm and there are no
dysmorphic features. Antenatal ultrasounds have been normal until 32
weeks, when intrauterine growth restriction (IUGR) has been diagnosed.
Which one of the following is the most likely cause of the low birth
weight?

a) Congenital intrauterine infection


b) Chromosomal or genetic abnormality
c) Placental insufficiency
d) Congenital hydrocephalus
e) Maternal age
The correct answer is C
Explanation
The size and nutrient transfer capacity of the placenta play central
roles in determining the prenatal growth trajectory of the fetus.
Abnormalities in placental structure and function are central to many
cases of IUGR.

Placental insufficiency results in fetal hypoxia and hypoglycemia during


late gestation.
Amniocentesis is used in the diagnosis of all of the following
conditions, except

a) Trisomy 21 (Down syndrome)


b) Metabolic errors
c) Neural tube defects
d) Genitourinary anomalies
e) Trisomy 18
The correct answer is D
Explanation
In amniocentesis, a needle is inserted transabdominally into the
amniotic sac to withdraw amniotic fluid and fetal cells for testing,
including measurement of chemical markers (eg, ?-fetoprotein,
acetylcholinesterase). The safest time for amniocentesis is after 14
weeks gestation. Immediately before amniocentesis, ultrasonography is
done to assess fetal cardiac motion and determine gestational age,
placental position, amniotic fluid location, and fetal number.
Amniocentesis has traditionally been offered to pregnant women > 35
because their risk of having an infant with chromosomal abnormalities
(eg Trisomy), neural tube defects and metabolic errors are increased.
Amniocentesis is not a method used to diagnose genitourinary anomalies.
Which of the following physiologic changes occurs during a normal pregnancy?

a) Hyperuricemia
b) Proteinuria
c) Hypertension
d) A 40% increase in the glomerular filtration rate (GFR)
e) Metabolic alkalosis
The correct answer is D
Explanation
Compared with pre-pregnancy values uric acid concentrations decreased
significantly by 8 weeks gestation and this reduced level was maintained
until about 24 weeks. Proteinuria changes little during pregnancy and if
more than 500mg/24h is lost, a disease process should be suspected.
Normally, the patient's blood pressure will not rise in pregnancy.
Glucosuria during pregnancy is not necessarily abnormal, may be
explained by the increase in GFR with impairment of tubular reabsortion
capacity for filtered glucose. Increased levels of urinary glucose also
contribute to increased susceptibility of pregnant women to urinary
tract infection. With increased minute ventilation, the gravida will
have increased alveolar ventilation, leading to respiratory alkalosis.
A 78-year-old white female notices a scant milky secretion from her
breast. Which one of the following medications can cause this symptom?

a) Famotidine (Pepcid)
b) Diazepam (Valium)
c) Nifedipine (Procardia)
d) Risperidone (Risperdal)
e) Zaleplon (Sonata)
The correct answer is D
Explanation
Many medications can cause galactorrhea, including the dopamine receptor
blocker risperidone. Additional drugs that cause this condition include
cimetidine, verapamil, and morphine. The other drugs listed do not cause
galactorrhea.
A 23 year old female is informed by one of her sexual partners that he
was recently treated for gonorrhea. They have had unprotected

intercourse on many occasions in the past few months. Her last menstrual
period was 6 weeks ago. Testing for chlamydial infection is negative,
but her gonococcal culture is positive and a pregnancy test is positive.
According to guidelines, the best treatment plan for the gonococcal
infection is

a) Ofloxacin (Floxin), 400 mg orally


b) Ceftriaxone (Rocephin), 125 mg intramuscularly in a single dose
c) Azithromycin (Zithromax), 1 g orally in a single dose
d) Doxycycline (Vibramycin), 100 mg orally twice a day for 7 days
The correct answer is B
Explanation
For patients with uncomplicated gonococcal infections of the cervix,
urethra, and rectum, it is recommended treatment with cefixime,
cefrtiaxone, ciprofloxacin, ofloxacin, or levofloxacin. If chlamydial
infection has not been ruled out, or if the likelihood of chlamydial
infection is high, azithromycin or doxycycline should also be given.
Because this patient is pregnant, she should not receive quinolones or
tetracyclines. She should receive a cephalosporin, and ceftriaxone would
be the best choice because it provides higher, more sustained levels of
bacterial activity. If chlamydial infection had not been ruled out,
erythromycin or amoxicillin would also be recommended.
A 32-year-old white female at 16 weeks gestation presents to your office
with right lower quadrant pain. Which one of the following imaging
studies would be most appropriate for initial evaluation of this patient?

a) CT of the abdomen
b) MRI of the abdomen
c) Ultrasonography of the abdomen
d) A small bowel series
e) Intravenous pyelography
The correct answer is C
Explanation
CT has demonstrated superiority over transabdominal ultrasonography for
identifying appendicitis, associated abscess, and alternative diagnoses.
However, ultrasonography is indicated for the evaluation of women who
are pregnant and women in whom there is a high degree of suspicion for
gynecologic disease.
A 24 year old female G1P0 is admitted to hospital at 34 weeks gestation.
The patient describes passing a large volume of clear fluid per vagina
for 36 hours. Examination reveals the following: temp 38.5C, no uterine
contractions, vague lower abdominal tenderness, fetal heart rate 185
bpm. Lab investigations reveal: Hb 120, WBC 19.0, + nitrazine test of
vaginal fluid, + ferning of vaginal fluid. Which of the following is the
most likely diagnosis

a) Premature rupture of membranes


b) Premature rupture of membranes and chorioamnionitis
c) Premature labour
d) Acute pyelonephritis
e) Acute vaginitis of pregnancy
The correct answer is B
Explanation
Rupture of the membranes before onset of labor is considered premature;
it sometimes results in infection. Diagnosis is clinical. If fetal lungs
are immature and infection is absent, treatment is bed rest plus delay
of delivery with Mg sulfate and other tocolytic drugs as needed. If
fetal lungs are mature or if fetal compromise or infection is present,
treatment is expedited delivery (eg, by inducing labor).

Chorioamnionitis is infection of the chorion and amnion, usually


occurring near term. Chorioamnionitis may result from an infection that
ascends through the genital tract. Risk factors include premature
rupture of membranes and prolonged labor. Consequences of
chorioamnionitis include premature rupture of membranes and premature
labor, and increased risk of neonatal pneumonia, bacteremia, meningitis,
and death.
Diagnosis is suggested by fever occurring late in pregnancy. Fetal heart
rate monitoring is required. Fetal heart rate increases during fever
but, in the absence of chorioamnionitis, returns to baseline as fever
resolves. Fetal tachycardia out of proportion to or in the absence of
fever suggests chorioamnionitis.
Which one of the following statements regarding nutrition during pregnancy is correct?
a) Maternal age is not related to nutritional status
b) In an uncomplicated pregnancy, iron supplementation is more important during the first trimester
c) In nonobese women, lack of weight gain is associated with an increased risk of fetal growth retardation
d) Vitamin B12 is the only vitamin supplementation required with the usual diet during pregnancy
e) Adding extra salt to foods must be avoided
The correct answer is C
Explanation
The greatest demand for iron is during the latter half of pregnancy. Only vegetarians and those with actual serum vitamin
B12 deficiency require vitamin B12 supplementation. Unless there are complications, e.g., hypertension or cardiovascular
disease, there is no reason the pregnant patient cannot salt her food to taste.
Obstetric risk factors for teenagers include poor nutrition, smoking, alcohol and drug abuse, and genital infections. In women
of average or low weight, lack of weight gain throughout pregnancy is often associated with fetal growth retardation.
Complications of cesarean section include each of the following, except

a)
b)
c)
d)
e)

Hemorrhage
Infection
Laceration of the fetus
Subsequent rupture of the uterine scar
Inversion of the uterus

The correct answer is E


Explanation
Cesarean complications, although rare, include infections, blood clots, and abdominal adhesions. While bleeding after csection is normal, uncontrollable bleeding is not, and needs immediate treatment. An infection after cesarean section occurs
in about 1 to 2 out of every 20 surgeries. Fetal injury during a c-section is rare but possible, such as nerve damage or
lacerations. Although rare, uterine rupture after cesarean section can occur, especially if a scar is already present.
An obese woman who has poorly controlled diabetes on glyburide tells you that she wants to get pregnant. What will you
advise her?

a) Her diabetes should be well controlled on glyburide before she becomes pregnant
b) Her diabetes should be maintained on insulin
c) Change glyburide to another medicine
d) Maintain blood glucose on diet and exercise only
The correct answer is A
Explanation
Glyburide is probably safe during pregnancy in women with type 2 diabetes. But those on other oral agents should be
switched to insulin before pregnancy or as soon as possible after conception.
Abundant data clearly show that uncontrolled maternal diabetes is teratogenic. It appears that in many cases, adverse fetal
outcomes that have been attributed to oral glucose-lowering agentsincluding various anomalies, stillbirths, macrosomia,
and neonatal hypoglycemia?were probably due to the diabetes itself.
In the best scenario, the woman should have optimization of glucose control and HbA1C before pregnancy. Diet and exercise
are standard therapy. Insulin should be prescribed if glucose levels continue to be elevated.

If she has been taking oral agents, the dose should be adjusted to achieve optimal diabetes control while on adequate
contraception, then switched to insulin once HbA1C is optimized and she's ready to become pregnant.
A 28-year-old previously healthy female presents with a 2-day history of a painful mass on her vulva. On examination vital
signs are normal. The patient is unable to sit normally because of her pain. She has an exquisitely tender red mass in the
posterior right labia majora. It is oval in shape, with the largest dimension being about 6 cm. The treatment of choice for this
condition would be

a) Marsupialization of the lesion


b) Surgical excision
c) Antibiotic therapy
d) Incision and packing with iodoform gauze
e) Insertion of a Word catheter
The correct answer is E
Explanation
The patient has a Bartholin's abscess. Marsupialization can be used for a Bartholins cyst, but is not appropriate for an
acutely infection lesion like the one presented. Surgical incision is used for recurrent, but not acutely infected, cysts. Incision
and drainage alone relieves the immediate pain, but is associated with a very high recurrence rate. As is the case with other
abscesses, antibiotic therapy is not effective for a Bartholins abscess. Incision of the lesion, followed by insertion of a Word
catheter, creates a long-lasting drainage tract and usually cures the problem permanently.

A 20-year-old female college tennis player presents with painful anterior lower leg lesions. You note several 2- to 3-cm deep,
tender, warm lesions over both shins. The patient denies specific trauma or increased exercise. The most significant etiology
to be considered in this case is

a) Papular urticaria
b) Early rheumatoid arthritis
c) Shin splints
d) Superficial thrombophlebitis
e) Oral contraceptive use
The correct answer is E
Explanation
This patient has typical erythema nodosum, likely related to oral contraceptive use. Careful evaluation of the skin lesions
would exclude early rheumatoid arthritis, shin splints, and superficial thrombophlebitis. Papular urticaria, a hypersensitivity
reaction to common insect bites such as fleas, mosquitos, bedbugs and other insects, is another possibility to consider,
although these lesions are generally smaller, and papular or papulovesicular in appearance.

Which one of the following is consistent with terminology used in the Bethesda System for reporting cervical cytology?
a) Atypical squamous cells ? cannot exclude HSIL (ASC-H)
b) Atypical squamous cells of unknown significance ? favor neoplastic (ASCUS ? favor neoplastic)
c) Atypical squamous cells of unknown significance ? favor reactive (ASCUS ? favor reactive)
d) Atypical glandular cells of unknown significance (AGUS)
The correct answer is A
Explanation
In the Bethesda System, atypical squamous cells of unknown significance (ASCUS) was replaced by atypical squamous cells
(ACS). ACS is divided into atypical squamous cells ? cannot exclude HSIL (ASC-H) and atypical squamous cells of unknown
signifiance (ASC-US). ASCUS-favor reactive has been downgraded to negative in the system. Atypical glandular cells of
unknown significance (AGUS) has been replaced by atypical glandular cells (AGC).
Amnioinfusion during labor is indicated for which one of the following conditions
a)
b)
c)
d)

Moderate or thick meconium-stained amniotic fluid


Prolonged rupture of membranes
Persistent late fetal heart rate decelerations
Fetal tachycardia

e) Uterine hyperstimulation
The correct answer is A
Explanation
Amnioinfusion has been shown to be helpful in improving perinatal outcomes in patients with moderate or thick meconiumstained amniotic fluid. It also is useful for suspected umbilical cord compression during labor. Amnioinfusion has been used
for preterm rupture of membranes and oligohydramnios, but there is not enough evidence to support its use. It also is not
considered a therapeutic option for prolonged rupture of membranes, persistent late fetal heart rate decelerations, fetal
tachycardia, or uterine hyperstimulation.
Which one of the following is appropriate and effective treatment for genitourinary gonorrhea in a 20-year-old male with a
purulent urethral discharge?

a) Amoxicillin, 3.5 g orally once


b) Ciprofloxacin (Cipro), 500 mg orally once
c) Ceftriaxone (Rocephin), 125 mg intramuscularly once
d) Doxycycline, 100 mg 2 times daily for 3 days
e) Erythromycin, 500 mg 4 times daily for 7 days
The correct answer is C
Explanation
Not only has the incidence of gonorrhea increased since 2002, but the rate of quinolone-resistant infection has also
increased. Ceftriaxone is therefore the currently recommended treatment, and amoxicillin, ciprofloxacin, and erythromycin
are no longer recommended because of resistance to these drugs. Doxycycline can be used but should be continued for 7
days.
A female patient develops a cystocele and procidentia. What is the most appropriate treatment?
a) Vaginal hysterectomy
b) Vaginal hysterectomy and repair
c) Abdominal hysterectomy
d) Laparoscopic hysterectomy
The correct answer is B
Explanation
A cystocele occurs when the wall between a womans bladder and her vagina weakens and allows the bladder to
droop into the vagina. A procidentia is illustrated below.
All these defects can be repaired and treated via a vaginal hysterectomy. During surgery, all pelvic support defects
should be corrected. Minimal defects that are not repaired are likely to worsen.
Three days after her menses started, this 21-year-old woman began having
sudden onset of nausea, vomiting, diarrhea, and a flu-like malaise. She
does not use tampons, but has had sexual relations in the last several
days and uses a cervical cap for contraception. On evaluation, you find
her blood pressure to be 75/35 mm Hg, pulse of 130 bpm, and an oral
temperature of 39.3?C (102.7?F). She has a diffuse macular rash over her
entire body. Of the following, which is correct?
a) Blood cultures will be positive for /Staphylococcus aureus/
b) Blood cultures will be positive for /Neisseria gonorrhoeae/
c) Most of the clinical signs and symptoms are due to a bacterial
endotoxin
d) Intravenous fluid resuscitation to correct hypotension is the
first priority in therapy
e) Beta lactamase resistant penicillin antibiotic therapy is the
first priority in therapy
The correct answer is D
Explanation
This is the classic picture for toxic shock syndrome. Although it is
more commonly associated with tampon usage, it can occur after use of a
contraceptive sponge, diaphragm, or cervical cap. It can also occur
postoperatively in a patient with gauze packing. Cultures are usually
negative, though Staphylococcus aureus is the most common pathogen. An
exotoxin is the causative agent for the systemic effects. Correction of
circulatory compromise is the most important initial therapy in treating

this condition. If a source exists for the bacteria, it must be removed


as well (i.e., tampon, etc).
A. In most cases of toxic shock syndrome, the causative agent will not
be found in the blood stream.
B. This is not the typical presentation of disseminated gonococcemia.
C. The causative agent for the systemic effects is an exotoxin produced
by the bacteria.
E. Correction of circulatory compromise and removal of the bacterial
source are the most important initial therapies.
A 21-year-old primigravida has just given birth. Examining her infant,
you note that it has what appears to be an enlarged protuberant clitoris
along with partially fused, rugated, and pigmented labia majora. No
palpable gonads are noted. Which of the following enzyme defects is the
infant most likely to have?
a) 11 beta hydroxylase
b) 3 beta hydroxysteroid dehydrogenase
c) 17 beta hydroxysteroid dehydrogenase
d) 21 alpha hydroxylase
e) 17,20 desmolase
The correct answer is D
Explanation
21-PH deficiency accounts for 95% of the cases of congenital adrenal
hyperplasia. It is also the most frequent endocrine cause of neonatal
death. With severe forms, salt wasting, shock, and significant
virilization occur. The genes for this enzyme are within the HLA complex
on the short arm of chromosome 6w.
A. This is one of the causes of congenital adrenal hyperplasia, but it
is much less common than 21 alpha hydroxylase deficiency.
B. This enzyme converts pregnenolone to progesterone as well as 17 OH
prenenolone to 17 OH progesterone, as well as DHEA to androstenedione.
C. This converts androstenedione to testosterone and estrone to estradiol.
E. This converts 17 OH pregnenolone to DHEA and 17 OH progesterone to
androstenedione.
A 17 year old girl consults in the outpatient clinic because of painful
menstrual periods. She missed 3 days of school per month. Pelvic exam is
normal. Which one of the following measures is most appropriate in the
initial management of this patient?

a) Diagnostic laparoscopy
b) Danazol
c) Oral prostaglandin inhibitor
d) Codeine and acetaminophen combination
e) Reassurance
The correct answer is C
Explanation
Dysmenorrhea is pelvic pain with menses. Primary dysmenorrhea begins
during adolescence and cannot be explained by structural gynecologic
disorders. Usually, secondary dysmenorrhea begins during adulthood and
is due to underlying pelvic abnormalities. Diagnosis is clinical and by
exclusion of structural disorders with pelvic ultrasonography and tests
directed at any other clinically suspected causes.
Underlying disorders are treated. Pain is treated with NSAIDs and
sometimes with low-dose estrogen-progestin contraceptives. Nonsteroidal
anti-inflammatory drugs (NSAIDs) block the Cox enzymes and reduce
prostaglandins throughout the body. As a consequence, ongoing
inflammation, pain, and fever are reduced.
A 70 year old woman presents to the clinic complaining of involuntary
loss of urine. Her bladder never feels empty.
The patients history is significant for a 15 year history of
uncontrolled diabetes mellitus. She is on daily insulin injections,
however her HbA1c has been always around 10%. She denies any history of

trauma or spine surgery.


Cystometric evaluation reveals increased residual volume. Which of the
following is the most appropriate next step of management?

a) Advise for Kegels exercises


b) Place a permanent urinary catheter
c) Prescribe Amitryptiline
d) Prescribe Bethanechol
e) Refer to surgery
The correct answer is D
Explanation
The patient has classic signs and symptoms of overflow incontinence most
likely due to autonomic neuropathy caused by the uncontrolled diabetes.
Detrusor hypotonia will result in the overflow incontinence.
Treatment of overflow incontinence includes cholinergic drugs such as
bethanechol which will promote detrusor muscle contraction.
Intermittent, not continuous, catheterization may also be helpful.
Kegels exercises and surgery (Urethropexy) are appropriate for stress
incontinence.
Amitryptiline has anticholinergic effects and would worsen the patients
condition. It is beneficial for patients with urge incontinence.

A 72 year old white female who is otherwise healthy complains of


occasional incontinence. She reports that this occur mainly at night
when she awakens with an intense desire to void, and by the time she is
able to get to the bathroom she has ?wet herself?. The most likely
diagnosis is
a)
b)
c)
d)

Sphincter incompetence
Detrusor instability
Detrusor hypotonia
Uninhibited neurogenic bladder

The correct answer is B


Explanation
Urinary incontinence is very common in the elderly female. Treatment
depends entirely on a careful history to ascertain the exact
circumstances when the patient wets herself. One of the most common
types of incontinence results from uninhibited contractions of the
detrustor muscle. This detrustor instability causes an intense urge to
void, which overcomes the patients voluntary attempt to hold the
sphincter closed; hence, the common term urge incontinence.
Other common causes of incontinence include a weak sphincter (sphincter
incompetence), which leads to leakage associated with ordinary activites
such as coughing or lifting (stress incontinence). Another common cause
is overflow of urine from an abnormally distended, hypotonic, poorly
contractile bladder (detrusor hypotonia). This is probably more common
in males with longstanding obstruction due to prostatic hypertrophy. A
rare type of incontinence is caused by spinal cord damage. This reflex
incontinence is due to the patient being unable to sense the need to void.

A 20 year old pregnant woman is brought to the emergency department


complaining of severe abdominal pain. She is at 37-weeks of gestation.
Her pregnancy was uneventful until now. She denies vaginal bleeding or
any history of trauma.
The patient is cold and diaphoretic. Her vitals show a BP of 85/45 mmHg,
pulse is 120/min, respiratory rate is 20/min and temp is 36.50C. Pelvic
exam shows a closed cervical os without vaginal bleeding.
Which of the following would most likely explain the patients presentation?

a) Chorioamnionitis
b) Placenta previa
c) Placental abruption
d) Preeclampsia
e) Thromboembolism
The correct answer is C
Explanation
Placental abruption is the most likely diagnosis of this patient. Please
be aware that the absence of vaginal bleeding does not rule out
placental abruption. This is a common trick on the Boards, don?t fall in
this trap. 20 % of abruptio placentae cases occur in the retroplacental
space and do not manifest by vaginal bleeding. Abdominal pain is always
present.
Placenta previa can be ruled out in the absence of vaginal bleeding.
Hypertension, proteinuria and edema are the classic triad of preeclampsia.
Thromboembolism usually manifests as unilateral limb pain, chest pain,
dyspnea, ischemia?
In the absence of fever and risk factors (UTI or membrane rupture)
chorioamnionitis is unlikely.
As a single measurement, which one of the following provides the most
accurate estimate of gestational age by ultrasound determination during
the second trimester?

a) Transabdominal diameter
b) Biparietal diameter
c) Femur length
d) Crown-rump length
The correct answer is B
Explanation
All of the options listed can be assessed by ultrasonography. Crown-rump
length is a very accurate parameter in the first trimester, but the
biparietal diameter is the most accurate parameter during the second
trimester. Both have a 95% confidence level of being within 5-10 days of
the actual gestational age when used at the proper time.
A 19-year-old primigravida at approximately 40 weeks gestation comes to
the hospital with painful contractions. She has received no prenatal
care. Examination reveals that her cervix is 4 cm dilated and 85%
effaced at ?1 station. Her blood pressure is 164/111 mm Hg and a urine
dipstick shows 3+ protein. She reports that she has had severe headaches
for 3 days and has noticed a lot of swelling in her legs and feet.
Moments after blood is drawn and intravenous access is obtained, she has
a generalized tonic-clonic seizure and fetal heart tones drop to 60
beats/min.
Which one of the following is the most appropriate immediate course of
action?
a) Emergency cesarean section
b) Lorazepam (Ativan), 2 mg intravenous push, repeated in 2 minutes
if necessary
c) Magnesium sulfate, 4g loading dose intravenously over 20mins,
followed by a drip at 2 g/hr
d) Attachment of a fetal scalp electrode
e) Terbutaline (Brethine), 0.25 mg subcutaneously
The correct answer is C
Explanation
This patient has eclampsia. When an eclamptic seizure occurs, the first
priority is to control the convulsions and prevent their recurrence with
a 4- to 6-g intravenous or intramuscular loading dose of magnesium

sulfate given over 15?20 minutes, followed by a drip at 2 g/hr. Although


the only cure for eclampsia is delivery, the patient should be
stabilized first.
-Fetal bradycardia is an expected occurrence during an eclamptic seizure
and does not necessitate an emergency cesarean section unless it fails
to resolve within a few minutes.
-Lorazepam is frequently used to control seizures in nonpregnant
patients, but magnesium sulfate is the agent of choice for an eclamptic
seizure. [note: benzodiazepine or phenytoin can be used for seizures
that are not responsive to magnesium sulfate]
-A fetal scalp electrode can be helpful to confirm fetal heart tones,
but should not take precedence over trying to control the seizure.
-Terbutaline is a tocolytic agent that does not have a role in the acute
management of an eclamptic seizure.
Uterine rupture is a potential complication of attempted vaginal birth
after cesarean (VBAC). The most reliable indication that uterine rupture
may have occurred is

a) Cessation of uterine contractions during active labor


b) Fetal bradycardia
c) Vaginal bleeding
d) Sudden lower abdominal pain
e) Maternal hypotension
The correct answer is B
Explanation
Uterine rupture occurs in 0.2%-1.0% of women in labor after one previous
low transverse cesarean section. Obviously, this can have devastating
consequences for the mother and baby, so vigilance during labor is
paramount. Uterine pain, cessation of contractions, vaginal bleeding,
failure of labor to progress, or fetal regression may occur, but none of
these are as consistent as fetal bradycardia in cases of uterine rupture
during labor for VBAC patients.
A 23-year-old sexually active woman with a prior history of pelvic
inflammatory disease presents with sudden onset of pelvic pain. On
initial workup and exam, you note the following: Beta HCG titer 5,400
mIU/ml; WBC 4,500 (units); differential: 63 PMNs, 0 Bands, 37
lymphocytes; temperature 37.3?C (99.1?F). An endovaginal ultrasound
shows nothing in the uterus, a 2-cm simple left ovarian cyst, and
moderate free fluid in the cul-de-sac. The most likely diagnosis is:
a)
b)
c)
d)
e)

Recurrent pelvic inflammatory disease


Ectopic pregnancy
Ruptured ovarian cyst
Endometriosis
Irritable bowel syndrome
The correct answer is B

Explanation
With her prior history of PID, her chances of tubal damage are
significantly elevated. Since she is pregnant with an HCG titer over
2000 mIU/ml, an intrauterine gestation sac should have been seen on the
endovaginal ultrasound. With the moderate amount of free fluid in the
cul-de-sac, along with the pelvic pain and normal white count and
temperature, the index of suspicion for an ectopic must be high.
A. The white count is normal and her temperature is normal as well. With
a positive HCG titer, an ectopic should be the first suspicion.
C. This can cause free fluid in the cul-de-sac as well as pelvic pain.
With her history of PID in the past, the presence of tubal damage is
high; so one should be much more suspicious of an ectopic. At an HCG
titer of 5,400, an IUP should have been seen.
D. Although a source of pelvic pain, with the HCG titer, absence of an
IUP on ultrasound, and free fluid in the cul-de-sac, ectopic pregnancy
should be the primary diagnosis.
E. Can be a source of pelvic pain. See answer to D.
A 19-year-old primigravid at 40 weeks gestation has been in labor for
the last 8 hours. Fetal heart tones have a baseline for 135/min with

normal variability, multiple accelerations and no decelerations. She has


been completely dilated for the last hour, and with pushing, has
descended from a +1 station to a +3 station at present. The vertex is
direct occiput anterior. Your next course of action is to recommend:
a)
b)
c)
d)
e)

Forcep-assisted vaginal delivery


Vacuum-assisted vaginal delivery
Continue to push
Pitocin augmentation
Cesarean section
The correct answer is C

Explanation
This patient is progressing in normal fashion. She is allowed up to 2
hours in the second stage and even longer if the heart tones are
reassuring. There is no need to intervene, and one would anticipate that
the patient will be having a normal spontaneous vaginal delivery within
the next hour.
A. The patient has made adequate descent in the last hour. She can push
for at least another hour and maybe more if the fetal condition remains
reassuring. No indication for instrumental vaginal delivery is present.
B. The patient has made adequate descent in the last hour. She can push
for at least another hour and maybe more if the fetal condition remains
reassuring. No indication for instrumental vaginal delivery is present.
D. The patient has made adequate descent in the last hour. She can push
for at least another hour and maybe more if the fetal condition remains
reassuring. Since progress has been made, there is no need to augment
the labor with pitocin.
E. The patient has made adequate descent in the last hour. She can push
for at least another hour and maybe more if the fetal condition remains
reassuring. No indication for cesarean delivery is present.
A young female enters your office wanting to start oral contraceptive
pills. All of the following are contraindications to starting OCPs, except

a)
b)
c)
d)
e)

Impaired liver function


Undiagnosed abnormal uterine bleeding
Congenital hyperlipidemia
Past history thrombophlebitis
Obesity
The correct answer is E

Explanation
Absolute contraindications to oral contraceptive pill (OCP) use are:
Thrombophlebitis, thromboembolic disorders, cerebrovascular disorders,
ischemic heart disease, coronary artery disease, known or suspected
cancer of the breast, known or suspected estrogendependent cancer, known
or suspected pregnancy, benign or malignant liver tumor, undiagnosed
abnormal genital bleeding.
6-year-old female presents with lower abdominal pain and vaginal
bleeding. Her last menstrual period was 7 weeks ago. A urine pregnancy
test is positive, and a quantitative beta-hCG level is 2500 mIU/mL.
Intravaginal ultrasonography shows no evidence of an intrauterine
gestational sac. Baseline laboratory tests, including a CBC, liver
function tests, and renal function tests, are all normal. She is treated
with a single dose of intramuscular methotrexate (Trexall) at 50 mg/m^2
of body surface. Four days later the patient presents for reevaluation,
and her quantitative beta-hCG level is found to be 2800 mIU/mL.
Which one of the following is the most appropriate next step?
a) A repeat dose of methotrexate, 50 mg/m^2 of body surface
b) Methotrexate, 1 mg/kg every other day, plus leucovorin, 0.1 mg/kg
on alternate days
c) Repeat transvaginal ultrasonography to evaluate for a viable
intrauterine pregnancy
d) Laparoscopy with salpingostomy

e) Expectant management
The correct answer is D
Explanation
Management of ectopic pregnancy with methotrexate is appropriate in
patients who have a beta-hCG level <15,000 mIU/mL; who are without liver
or renal disease, immune or platelet compromise, or significant
pulmonary disease; and who are reliable and able to follow up daily if
necessary.
Patients may be treated either with single-dose or multiple-dose
methotrexate regimens, which may be repeated if the beta-hCG level does
not decline. If the beta-hCG level increases, surgical intervention is
needed. Laparoscopy with salpingostomy is the preferred method.
Expectant management is appropriate only if a patient has a beta-hCG
level <1000 mIU/mL that is declining.
A 30 year old female presents to your office complaining of painful
sexual intercourse for the past few weeks. The pain was getting
progressively worse until it became unbearable.
Her past medical history is significant for infertility; she has been
trying to conceive for the past 3 years without success. She admits to
having cramping pain that usually begins a few days prior to and
resolves a few days after her period. She denies any sexually
transmitted infections or pelvic inflammatory disease.
Physical exam is significant for immobile uterus with nodularity along
the uterosacral ligaments and palpable tender right adnexal mass.
Ultrasound shows a heterogeneous content of the right ovary. Which of
the following is the most likely diagnosis

a)
b)
c)
d)
e)

Endometriosis
Pelvic congestion syndrome
Pelvic inflammatory disease
Polycystic ovarian syndrome
Vaginismus
The correct answer is A

Explanation
Endometriosis is a benign condition, the presence of endometrium-like
glands and stroma outside the uterus. It is most often found in the
ovaries but can also be found in other places, including fallopian
tubes, bladder and intestines, uterine wall, and the lining of the
pelvis. Patients with endometriosis present with the dys Syndrome:
dysmenorrhea, dyspareunia (painful intercourse), dyschezia (painful
defecation) and dysuria.
Physical exam often reveals tender adnexal mass and firm nodularities
along the broad ligament, uterosacral ligament or cul de sac.
The chocolate cyst of endometriosis in the ovary is usually seen as a
heterogeneous adnexal mass.
Endometriosis can cause infertility; in fact 30 % of infertile couples
are diagnosed with this condition.
The gold standard for diagnosis of endometriosis remains Laparoscopy.

An infant is born. At one minute the heart rate is 120 per minute,
respiratory effort is a good strong cry, muscle tone is active, reflex
irritability is absent, colour is pink with blue extremities. What is
the one minute Apgar score?

a)
b)
c)
d)
e)

5
6
7
8
9

The correct answer is C


Explanation The following table shows how to calculate Apgar scores:
Sign
A

Activity
(Muscle tone)

Pulse
(heart rate)

0Point

Grimace
(response to smell or foot slap)

1Point

2Point

Limp

limbs flexed

active movement

absent

< 100 /min

> 100 /min

absent

grimace

cough or sneeze (nose)


cry and withdrawal of foot (foot slap)

Appearance
(color)

blue

body pink

pink all over


extremities blue

Respiration
(breathing)

absent

irregular

good strong cry


weak crying

This baby would have an APGAR score of 2+2+0+1+2 = 7.


A 25-year-old female at 36 weeks gestation presents for a routine
prenatal visit. Her blood pressure is 118/78 mm Hg and her urine has no
signs of protein or glucose. Her fundal height shows appropriate fetal
size and she says that she feels well. On palpitation of her legs, you
note 2 + pitting edema bilaterally.
Which one of the following is true regarding this patients condition?
a) You should order a 24-hr urine for protein
b) A workup for possible cardiac abnormalities is necessary
c) Her leg swelling requires no further evaluation
d) She most likely has preeclampsia
e) She most likely has deep venous thrombosis
The correct answer is C
Explanation
Lower-extremity edema is common in the last trimester of normal
pregnancies and can be treated symptomatically with compression
stockings. Edema has been associated with preeclampsia, but the majority
of women who have lower-extremity edema with no signs of elevated blood
pressure will not develop preeclampsia or eclampsia. For this reason,
edema has recently been removed from the diagnostic criteria for
preeclampsia. Disproportionate swelling in one leg versus another,
especially associated with leg pain, should prompt a workup for deep
venous thrombosis but is unlikely given this patients presentation, as
are cardiac or renal conditions.
A 24 year old gravida 3 para 1 is admitted to the hospital at 29 weeks
gestation with a high fever, flank pain ,and an abnormal urinalysis. You
order blood and urine cultures, a CBC, electrolyte levels, and a serum
creatinine level. You also start her on intravenous ampicillin and
gentamicin (Garamycin). After 24 hours of antibiotic treatment she is
clinically improved but continues to have fever spikes. Appropriate
management at this time would be to
a) Continue current management
b) Change her antibiotics, as her infection is likely due to a
resistant organism
c) Order a plain abdominal radiograph to rule out a renal stone
d) Order modified intravenous pyelography to rule out urinary tract

obstruction
e) Order renal ultrasonography to rule out a perinephric abscess
The correct answer is A
Explanation
Renal infection is the most common serious medical problem that
complicates pregnancy. Infection is more common after midpregnancy, and
is usually caused by bacteria ascending from the lower tract. Escheria
coli is the offending bacteria in approximately 75% of cases. About 15%
of women with acute pyelonephritis are bacteremic. A common finding is
thermoregulatory instability, with very high spiking fevers sometimes
followed by hypothermia. Almost 95% of women will be afebrile by 72
hours. However, it is common to see continued fever spikes up until that
time. Thus, further evaluation is not indicated unless clinical
improvement is not obvious at 48-71 hours. If this is the case, the
patient should be evaluated for urinary tract obstruction, urinary
calculi and an intrarenal or perinephric abcess. Ultrasonography, plain
radiography, and modified intravenous pyelography are all acceptable
methods, depending on the clinical setting.
A 30 year old woman presents with a long standing history of irregular
menstrual periods which are often associated with severe pelvic pain and
menorrhagia She has also been experiencing dyspareunia and pain during
defecation. Which one of the following is most likely to assist you in
diagnosing this patient's condition?
a) Clinical history
b) Laparoscopy
c) Pelvic examination
d) Hysterosalpingogram (HSG)
e) Culdocentesis during menses
The correct answer is B
Explanation
Endometriosis is a noncancerous disorder in which functioning
endometrial tissue is implanted outside the uterine cavity. Symptoms
depend on location of the implants and may include dysmenorrhea,
dyspareunia, infertility, dysuria, and pain during defecation.
Pelvic pain, pelvic mass, alteration of menses, and infertility are
typical. Some women with extensive endometriosis are asymptomatic; some
with minimal disease have incapacitating pain. Dyspareunia and midline
pelvic pain before or during menses may develop. Such dysmenorrhea is an
important diagnostic clue, particularly if it begins after several years
of pain-free menses.
Diagnosis is suspected based on typical symptoms but must be confirmed
by biopsy, usually via pelvic laparoscopy.
In the investigation of infertility, a normal semen analysis must contain
a) A volume of 0.5-1 mL
b) Morphology > 85% normal forms
c) WBC < 1 per high power field
d) Motility > 75%
e) Sperm count > 20 million sperm/mL
The correct answer is E
Explanation
The following are normal semen analysis:
Volume >2.0 mL
pH 7.2-7.8
Concentration >20x10^6/mL (20 million/mL)
Motility >50%
Morphology >30% normal
WBC < 1x10^6/mL
40 year old woman mother of four children and whose last child had
problems at birth with hemolytic disease of the newborn has just been
rushed to the emergency room following a motor vehicle accident. She has

lost a lot of blood and needs immediate transfusion but there is no time
to cross and type her blood. She should receive blood that is

a) O, Rh positive
b) AB, Rh positive
c) O, Rh negative
d) AB, Rh negative
e) B, Rh positive
The correct answer is C
Explanation
Blood is classified by type. A person's blood type is determined by the
presence or absence of certain proteins (Rh factor and blood group
antigens A and B) on the surface of red blood cells.
The four main blood types are A, B, AB, and O, and for each type, the
blood is either Rh-positive or Rh-negative. For example, a person with
O-negative blood has red blood cells that lack both A and B antigens and
the Rh factor. A person with AB-positive blood has red blood cells that
have A and B antigens and the Rh factor. Some blood types are far more
common than others.
A blood transfusion is safest when the blood type of the transfused
blood precisely matches the recipient's blood type. Therefore, before a
transfusion, blood banks perform a test called a ?type and cross-match?
on the donor's and the recipient's blood. This test minimizes the chance
of a dangerous or possibly fatal reaction.
However, in an emergency, anyone can receive type O red blood cells.
Thus, people with type O blood are known as universal donors. People
with type AB blood can receive red blood cells from any blood type and
are thus known as universal recipients. Recipients whose blood is
Rh-negative must receive blood from Rh-negative donors, but recipients
whose blood is Rh-positive may receive Rh-positive or Rh-negative blood.
Hemolytic disease of the newborn occurs in women who are Rh-negative.
A 41 year old woman presents with obesity, hirsutism and oligomenorrhea.
Which of the following tests will give the *least* useful information?
a)
b)
c)
d)
e)

Estradiol
LH
Testosterone
TSH
DHEA
The correct answer is B

Explanation
Polycystic ovary syndrome is characterized by mild obesity, irregular
menses or amenorrhea, and signs of androgen excess (hirsutism, acne).
Typically, the ovaries contain multiple cysts. Diagnosis is by pregnancy
testing, hormone level measurement, and imaging to exclude a virilizing
tumor.
Testing includes pregnancy testing and measurement of serum estradiol,
prolactin, and thyroid-stimulating hormone. Diagnosis is confirmed by
ultrasonography showing > 10 follicles per ovary; follicles usually
occur in the periphery and resemble a string of pearls.
If ovarian follicles or hirsutism is present, serum testosterone and
dehydroepiandrosterone sulfate (DHEAS) levels are measured.
LH/FSH ratio is more important than LH by itself. The ratio is normally
about 1:1 in premenopausal women, but with PCOS a ratio of greater than
2:1 or 3:1 may be considered diagnostic.
Vaginal discharge which is fishy in odor and associated with >20% clue
cells on microscopy will not be associated with which of the following

a) Be due to gardnerella vaginalis overgrowth

b) Can be treated with metronidazole or clindamycin


c) With the addition of KOH may see hyphae or spores
d) Must be treated in all pregnant women, including asymptomatic
e) Is rarely associated with inflamed or itchy vulva
The correct answer is C
Explanation
When a woman reports an unusual vaginal discharge, the doctor will ask
her a series of routine questions to help distinguish mild from more
serious conditions. Additional issues that might indicate the presence
of a more serious condition include fever, pelvic pain, new or multiple
sexual partners (especially with unprotected intercourse), and a history
of sexually-transmitted infections.
Examining the vaginal discharge under the microscope can help
distinguish bacterial vaginosis from yeast vaginitis (candidiasis) and
trichomonas (a type of sexually transmitted infection). A sign of
bacterial vaginosis under the microscope is an unusual vaginal cell
called a clue cell. Clue cells are believed to be the most reliable
diagnostic sign of bacterial vaginosis. In addition to clue cells, women
with bacterial vaginosis have fewer of the normal vaginal bacteria,
called lactobacilli. A vaginal pH greater than 4.5 is also suggestive of
bacterial vaginosis.
Hyphae or spores are seen in fungal infections such as candida vaginitis.
A female patient is a newcomer to the country. She is single and in
labor. She has had no prenatal care. You decide to give her anti-Rh Ig
(RhoGam) to prevent which one of the following, in subsequent pregnancies
a)
b)
c)
d)
e)

Immunodeficiency
Neonatal lupus
Hemolytic disease of the newborn
T cell immune deficiency syndrome
Transient hypogammaglobulinemia
The correct answer is C

Explanation
Hemolytic disease of the newborn (also called erythroblastosis fetalis)
is a condition in which red blood cells are broken down or destroyed
more rapidly than is normal. The newborn's red blood cells are destroyed
by antibodies that were produced by the mother and crossed the placenta
from the mother's circulation into the fetal circulation before
delivery. A mother who is Rh-negative may have produced antibodies
against Rh-positive blood cells after she was exposed to red blood cells
of a previous fetus that was Rh-positive. Such exposure may occur during
pregnancy or labor, but may also occur if the mother had been
accidentally transfused with Rh-positive blood at any time earlier in life.
The mother's body responds to the ?incompatible blood? by producing
antibodies to destroy the ?foreign? Rh-positive cells. These antibodies
cross the placenta during a subsequent pregnancy. If the fetus she is
carrying is Rh-negative, there is no consequence. However, if the fetus
has Rh-positive red blood cells, the mother's antibodies attach to, and
start to destroy, the fetal red blood cells, leading to anemia of
varying degrees. This anemia begins in the fetus and continues after
delivery.
A 28-year-old female presents 2 weeks post partum complaining of
palpitations, diarrhea, weight loss, and being ?jumpy.? Her examination
is normal except for a slightly enlarged and tender thyroid gland. Her
TSH level is 0.02 U/mL (N 0.5?5.5), with a higher than normal level of
free T_3 .
Which one of the following would be the most appropriate treatment?
a)
b)
c)
d)
e)

Levothyroxine (Synthroid)
Prednisone
Propranolol (Inderal)
Propylthiouracil
Radioactive iodine
The correct answer is C

Explanation
This patient presents with signs, symptoms, and laboratory evidence of
postpartum thyroiditis. This is an autoimmune attack of the thyroid
gland that occurs in 5%?10% of all mothers within a year of delivery.
The transient increase of thyroid hormone that results is often
unnoticed but can cause clinical hyperthyroidism. A -blocker is
recommended to reduce heart irregularities and other symptoms related to
high levels of circulating thyroid hormone. Propylthiouracil prevents
the production of new thyroid hormone and is not indicated because this
condition results only in a release of thyroid hormone that has already
been created. Up to one-third of women with this condition will become
chronically hypothyroid and will require regular thyroid replacement.
This patient is not currently hypothyroid, so she would not benefit from
replacement with levothyroxine.
A 60 year old female has been on conjugated equine
estrogens/medroxyprogesterone (Prempro) since she went through menopause
at age 52. She still has her uterus and ovaries. She is having no side
effects that she is aware of and is experiencing no vaginal bleeding.
She is worried about the health effects of her hormone replacement
therapy and asks your advice about risks versus benefits. Which one of
the following would be accurate advice regarding these risks and benefits

a)
b)
c)
d)
e)

The incidence of stroke is decreased


The incidence of myocardial infarction is decreased
The incidence of pulmonary embolus is decreased
The incidence of breast cancer is increased
The incidence of colorectal cancer is increased

The correct answer is D


Explanation
The Womens Health Initiative Randomized Controlled Trial concluded that
the health risks of hormone replacement therapy with combined estrogen
plus progestin exceeded the benefits. Absolute risk reductions per
10,000 person-years attributable to estrogen plus progestin were 6 fewer
colorectal cancers and 5 fewer hip fractures. However, absolute excess
risks per 10,000 person-years included 7 more coronary heart disease
events, 8 more strokes, 8 more pulmonary emboli, and 8 more invasive
breast cancers.
A woman who is 28 weeks pregnant is seen on ultrasound as having
placenta previa. All of the following are common complications of this
condition, *except*

a) Lower incidence of fetal malformations


b) Placental abruption
c) Hemorrhage
d) Premature delivery
e) Abnormal fetal presentation
The correct answer is A
Explanation
Placenta previa involves implantation of the placenta over the internal
cervical os. Variants include complete implantation over the os
(complete placenta previa), a placental edge partially covering the os
(partial placenta previa) or the placenta approaching the border of the
os (marginal placenta previa). A low-lying placenta implants in the
caudad one half to one third of the uterus or within 2-3 cm from the os.
Complications:
Hemorrhage (Hemorrhage is expected secondary to the poor contractibility
of the lower uterine segment. Planning delivery and control of
hemorrhage is critical in cases of placenta previa as well as placenta
accreta, increta, and percreta.)
Preterm delivery
Congenital malformations
Abnormal fetal presentation
Placental abruption
Higher rates of blood transfusion1
Increased incidence of postpartum endometritis1

Hemostasis
Prognosis:
Fifty percent of women with placenta previa have preterm delivery.
Those cases complicated with vaginal bleeding and extreme prematurity
are at an increased risk of perinatal death.
A greater incidence of fetal malformations and growth restriction is
noted with placenta previa.
An increased risk of neurodevelopmental delay and sudden infant death
syndrome (SIDS) is associated with placenta previa.
Neonates are also more likely to have low birth weight (<2500 g),
respiratory distress syndrome, jaundice, NICU admissions, and longer
hospital stay.
A 34 year old white female at 32 weeks gestation develops a venous
thromboembolism. Following 5 days of intravenous heparin in the
hospital, which one of the following regimens would be most appropriate?

a)
b)
c)
d)
The correct answer is B

Warfarin (Coumadin) throughout the remainder of her pregnancy


Subcutaneous heparin every 12 hours until delivery
Outpatient intravenous heparin every 6 hours until delivery
Aspirin twice a day until delivery

Explanation
Heparin does not cross the placenta and is safe for the fetus, whereas
coumarin derivatives can cause fetal bleeding and are teratogenic during
weeks 6-12. Therefore, pregnant women with venous thromboembolism should
receive intravenous heparin for 5 days, followed by adjusted-dose
subcutaneous heparin every 12 hours until delivery. Increasingly,
low-molecular-weight heparins are being used instead of unfractioned
heparin because of ease of administration and the reduced need for
coagulation monitoring. Intravenous heparin is not necessary after the
patient leaves the hospital, and aspirin has not been shown to be
beneficial.
A patient at 17 weeks' gestation is diagnosed as having an intrauterine
fetal demise. She returns to your office 5 weeks later and has not had a
miscarriage, although she has had some occasional spotting. This patient
is at increased risk for which of the following?

a)
b)
c)
d)
e)

Septic abortion
Recurrent abortions
Consumptive coagulopathy with hypofibrinogenemia
Future infertility
Ectopic pregnancies

The correct answer is C


Explanation
Disseminated intravascular coagulation (DIC) begins with excessive
clotting. The excessive clotting is usually stimulated by a substance
that enters the blood as part of a disease (such as an infection or
certain cancers) or as a complication of childbirth, retention of a dead
fetus, or surgery. As the clotting factors and platelets are depleted,
excessive bleeding occurs.
DIC may appear to develop suddenly and usually causes bleeding, which
may be very severe. If the condition follows surgery or childbirth,
bleeding may be uncontrollable. Bleeding may occur at the site of an
intravenous injection or in the brain, digestive tract, skin, muscles,
or cavities of the body. If DIC develops more slowly, as in people with
cancer, then clots in veins are more common than bleeding.
Blood tests may show that the number of platelets in a blood sample has
dropped and that the blood is taking a long time to clot. The diagnosis
of DIC is confirmed if test results show diminished amounts of clotting
factors and large quantities of proteins that are produced when clots
are broken up by the body (fibrin degradation products).

An 18-year-old female college freshman, who says she has never been
sexually active, has just begun a serious relationship with a
19-year-old male. Although they have not yet engaged in sexual
intercourse, she wants to begin oral contraception. Her periods are
regular. She began her current menses 2 days ago and would prefer to
delay a pelvic examination until her period has ended. Her blood
pressure is normal.
Which one of the following is the most appropriate plan for this visit?
a) Prescribe an oral contraceptive and have her return in 2 weeks for
a pelvic examination
b) Delay prescribing an oral contraceptive until after you can
complete a pelvic examination with STD screening
c) Delay prescribing an oral contraceptive until the result of a
Papnicolaou smear is known
d) Delay prescribing an oral contraceptive until a pregnancy test is
negative 2 weeks after the onset of her menses
The correct answer is A
Explanation
A history, pregnancy test (if indicated), and blood pressure reading
constitute an adequate evaluation before beginning hormonal
contraception. The pelvic examination may be deferred to a later visit.
It is now recommended that cervical cancer screening be delayed until 3
years after the onset of vaginal intercourse, or no later than 21 years
of age, neither of which have occurred in this case. However, sexually
active women under the age of 25 should be screened for chlamydial
infection. If a pregnancy test were necessary it could be done at the
initial visit, but waiting another 2 weeks delays the initiation of
birth control, increasing the risk for pregnancy.

A 45-year-old otherwise healthy woman complains of urine loss with


coughing, laughing and sneezing. This has become noticeable since the
vaginal delivery of her third child. Due to the urine loss, she finds it
necessary to wear a pad when exercising or walking. On exam, you note a
large midline bulge in the anterior vaginal wall that descends to the
introitus on valsalva. The most likely reason for urine loss is:
a) Stress incontinence
b) Urge incontinence
c) Vesicovaginal fistula
d) Detrussor dyssynergia
e) Neurogenic bladder
The correct answer is A
Explanation
Based on the exam findings in this multiparous woman, one would suspect
that an anatomic stress incontinence picture is the most likely. If she
had a fistula, the leaking would occur even at rest. As she is otherwise
healthy, the likelihood of a neurogenic bladder (seen in diabetes,
neuromuscular disorders, spinal cord injury) is small.
B. Urge incontinence, usually due to detrussor dyssynergia, has a
hypercontractile bladder. The patient will sense the urge t void and
often will not make it to the bathroom in time, as the spontaneous
bladder contraction will cause urine loss. Anticholinergics are often
used in this condition.
C. This condition will lead to leaking at all times, even at rest, since
the defect is above the urethral sphincter.
D. See answer to B.
E. A neurogenic bladder will have overdistension and incomplete filling.
With increased abdominal pressure, leaking can occur. Conditions such as
chronic diabetes and spinal cord injury can cause this. Since she is
otherwise healthy, this is unlikely.
A 24-year-old female presents to your office with lower abdominal pain,
dyspareunia, and a vaginal discharge. She has a history of multiple sex
partners. Examination shows that the cervix is tender to manipulation
and the uterus is tender and enlarged to the size expected at 6?8 weeks

gestation. No adnexal masses are noted. She has no rebound tenderness on


abdominal examination.
Which one of the following indicates that the patient should be
hospitalized for parenteral therapy?
a)
b)
c)
d)
e)

No improvement with 24 hours of outpatient antibiotics


A previous history of pelvic inflammatory disease
An elevated erythrocyte sedimentation rate and WBC count
Laboratory confirmation of gonorrhea or chlamydial infection
Pregnancy

The correct answer is E


Explanation
The criteria for hospitalizing patients with pelvic inflammatory disease
include failure to improve after 3 days of oral therapy, inability to
tolerate antibiotics, suspicion that the patient will not comply with
therapy, tubo-ovarian abscess, severe illness with high fever, vomiting,
pain, pregnancy, and the underlying possibility of a surgical problem
such as appendicitis.
A 45-year-old woman is referred to your office for a Pap smear that is
suspicious for malignancy. The cervix appears grossly normal on speculum
exam. The next most appropriate procedure is:
a)
b)
c)
d)
e)

Radical hysterectomy
Simple hysterectomy
Cervical cone biopsy
Cryotherapy of the cervix
Colposcopic directed biopsy

The correct answer is E


Explanation
The Pap smear is a screening test. Although the cervix appears normal to
the naked eye, once acetic acid is applied and the magnification of the
colposcope added, vascular abnormalities will often be seen. The most
concerning colposcopic finding is the presence of abnormal vessels.
These suggest carcinoma in situ or invasive carcinoma. If the
colposcopic exam is negative, a cone biopsy is performed to find the
source of the suspicious cells.
A. This is the therapy for early stage cervical carcinoma. The diagnosis
of cancer has not yet been made. A biopsy is necessary.
B. Simple hysterectomy is not the proper cure for invasive cervical
carcinoma. Also, the diagnosis of cancer has not yet been made. A biopsy
is necessary.
C. It is still likely that a cone biopsy will be necessary if a
colposcopic exam is unsatisfactory. The first step, however, in the
workup, is a colposcopy with directed biopsy. If invasive cancer is then
found, appropriate therapy can be started.
D. Cryotherapy is only indicated for low-grade dysplastic lesions of the
cervix, after adequate colposcopic exam to rule out malignancy.
A 26 year old woman comes to the office for a gynecologic examination.
On physical examination her cervix appears friable with a slight area of
ulceration. There are several perineal and vaginal lesions that appear
as small "cauliflower-like" projections. The results of the Pap smear,
which return in 1 week, show mild dysplasia (LGSIL). Which of the
following factors in this patient's history most closely correlates with
the abnormal finding on Pap smear?

a)
b)
c)
d)
e)

Condylomata acuminata
Condylomata lata
Early age at menarche
History of chlamydia
Nulliparity

The correct answer is A


Explanation
Condyloma acuminata is the physical manifestation of HPV (human

papilloma virus), a sexually transmitted disease. There are several


serotypes of HPV, with subtypes 6 and 11 causing benign
cauliflower-like projections and low grade squamous intraepithelial
lesions, and subtypes 16 and 18 most commonly associated with high-grade
squamous intraepithelial lesions and cervical cancer.
Condyloma lata is a gray-white highly infectious lesion associated with
secondary syphilis; early age at menarche has no correlation with
abnormal Pap smear. Early age of menarche and nulliparity has no
correlation with abnormal Pap smear. History of chlamydia is a
reasonable answer, since chlamydia is the most common sexually
transmitted cervicitis, but it has not been associated with precancerous
dysplasia.
A 29 year old gravida 0 para 0 presents to your office with a chief
complaint of irregular infrequent menses. Over the past few years, she
has noted increasing dark hair growth on her chin and above her upper
lip. On examination, she is normotensive and moderately overweight.
Examination of the skin reveals acne and abdominal striae. The remainder
of the examination is normal.
Laboratory Findings:
TSH 2.1 U/mL (N 0.3-5.0)
FSH 8 U/L (N 1-10)
LH 38 U/L (N 1-20)
Free testosterone 50 ng/dL (N 0.3-1.9)
Prolactin 28 ng/mL (N 0-23)
17 hydroxyprogesterone 4 ng/dL (N 0-8)
DHEA-S (dehydroepiandrosterone 2.0 ?g/dL (N 0-3.0)
sulfate)
In addition, a dexamethasone suppression test. Which one of the
following is the most likely diagnosis for this patient?

a) Cushings syndrome
b) Polycystic ovarian syndrome
c) Virilizing adrenal tumor
d) Prolactinoma
e) Adult-onset congenital adrenal hyperplasia
The correct answer is B
Explanation
Polycystic ovarian syndrome (PCOS) is a very heterogenrous syndrome, but
is often characterized by signs and symptoms of androgen excess and
chronic anovulation (menstrual irregularity, oligo- or amenorrhea).
Hirsutism and acne are common presenting signs of hyperandrogenism.
Seventy percent of patients are obese, and hyperinsulinemia and insulin
resistance are common. LH is usually elevated, but an LH:FSH ratio of
3:1 or greater is a better indicator of PCOS. The FSH is usually normal
or mildly low. Serum testosterone and prolactin are often mildly to
moderately elevated. DHEA-S levels are normal in PCOS. Fasting insulin
levels may be high and these patients should be screened for overt
diabetes mellitus or glucose intolerance, and hyperlipidemia.
Patients with Cushings syndrome and adult-onset congenital adrenal
hyperplasia (CAH) may also present with hirsutism, acne, and menstrual
abnormalities. However, in this patient a normal dexamethasone
suppression test makes Cushings syndrome an unlikely diagnosis. 17
hydroxyprogesterone (17 OHP) is a screening test for adult-onset CAH.
CAH patients may also have an elevated LH. In contrast, 17 OHP levels
are normal in women with PCOS.
A virilizing adrenal tumor is very unlikely unless mean testosterone
measurements are markedly elevated (>150-200 ng/dL). In addition, DHEA-S
level s are usually very high (> 700 g/dL). The clinical syndrome
usually presents with rapidly progressive hirsutism.
Lastly, serum prolactin levels are usually very high in women with
prolactinomas ( > 200 ng/mL). These patients may present with signs of
androgen excess and menstrual irregularity, as well as headache, visual

disturbance, and galactorrhea.


A 55-year-old female consults you about her urinary incontinence. She
denies dysuria or hematuria. She has a frequent urge to urinate, up to
10 times per day, including at night. On a few occasions, she has been
unable to get to the toilet in time and notes the loss of urine to be
?more than a dribble?. She has researched the problem on the Internet
and wants you to prescribe the best treatment.
Which one of the following would be the treatment of choice for this
patients incontinence?

a) Oxybutynin (Ditropan)
b) Pseudeoephedrine
c) Topical estrogen (Premarin)
d) Tolterodine (Detrol)
e) Kegel exercises
The correct answer is E
Explanation
Urge incontinence is characterized by urinary urgency and frequency,
often resulting in uncontrollable bladder contractions with large-volume
urine loss. Stress incontinence is brought on by a sudden increase in
intra-abdominal pressure associated with coughing, sneezing, or
laughing. Urine volume is small, as urine loss occurs only during the
strain.
Although the selective anticholinergic medicine tolterodine and the
older nonselective oxybutynin are often prescribed as initial therapy
for urge incontinence, behavioral therapy is the treatment of choice,
including bladder training and pelvic floor muscle (Kegel) exercises.
Kegel exercises are particularly useful, and randomized, controlled
trials have consistently shown them to be more effective than
anticholinergic medication in the treatment of urge incontinence.
Topical estrogens and alfa-adrenergic agonists have been used with
inconsistent results in the treatment of stress incontinence.

Which one of the following is associated with the use of epidural


anesthesia during labor and delivery?
a)
b)
c)
d)

A shorter first stage of labor


A longer second stage of labor
An increased rate of cesarean delivery
An increased likelihood of postpartum urinary incontinence

The correct answer is B


Explanation
Studies have shown that epidural analgesia increases the length of both
the first and second stage of labor. Although there is an increase in
the rate of instrument-assisted delivery and fourth degree laceration,
an increase in the rate of cesarean sections has not been shown. An
increase in the rate of urinary incontinence also has not been shown.
A 23-year-old sexually active female presents to your office with a
2-week history of vaginal discharge and mild coital discomfort. On
physical examination, you note the presence of a mucopurulent vaginal
discharge and cervical friability. She is afebrile and there are no
other positive physical findings. No trichomonads or yeast is seen on
vaginal preparations. Material for Chlamydial trachomatis-specific DNA
testing is submitted and results will be available in 2 days.
Which one of the following is true regarding appropriate management?

a) A 7-day course of doxycycline is superior to a 1-g dose of


azithromycin (Zithromax)

b) A 2-g dose of metronidazole (Flagyl) given now will clear her


discharge
c) No treatment is indicated until laboratory results are known
d) The patient should be instructed to refrain from sexual
intercourse until 7 days after initiating therapy
The correct answer is D
Explanation
Patients with suspected genital chlamydial infections and their partners
should be instructed to refrain from sexual intercourse until therapy is
completed (specifically, until 7 days after a single-dose regimen or
until the completion of a 7-day regimen). Suspicion of chlamydial
infection warrants treatment, with or without positive laboratory
findings. Metronidazole is not useful for treatment and there is no
difference in random clinical trials with regard to the efficacy of a
7-day course of doxycycline compared to that of a single 1-g dose of
azithromycin.

A 21-year-old patient comes to you for a pelvic examination and


contraceptive counseling. She tells you she has never had intercourse
and fears her vagina may be too small, even though she has used tampons
for 3 years. You reassure her that her examination is normal. Which of
the following approaches is the next step?

a) Gently dilate her hymen


b) Demonstrate perineal sensory reflex
c) Explain the normal physiology of the female sexual response
d) Explain surgical procedures to enlarge the hymenal opening
e) Prescribe dilators of progressive size
The correct answer is C
Explanation
During intercourse, the vagina can significantly expand and elongate.
After surgical reconstruction of the vagina or with creation of a
neovagina, intercourse is the best technique to maintain normal caliber
and size of the vagina when compared with dilators.
A. Dilation of the hymen is unnecessary.
B. With the normal exam, explanation of the normal physiology of the
female sexual response is the appropriate next step.
D. No surgical procedures are indicated.
E. See above explanation.
A 24 year old female presents for her annual examination. She is single
and has had several male sexual partners during the past year. You
include screening for chlamydial infection in your evaluation, and the
test is reported as positive. She is asymptomatic. Which one of the
following is true concerning this situation?

a) Failure to treat this patient would place her at higher risk of


later infertility
b) Only sexual partners with whom she has been active during the last
2 weeks need to be treated
c) She should avoid sexual intercourse for 1 month of treatment
d) Use of barrier methods of contraception increases her risk for
repeat infection
The correct answer is A
Explanation
It is recommended that sexually active women under the age of 25 years
be screened routinely for Chlamydia trachomatis. Treatment of
asymptomatic infections in women reduces their risk of developing pelvic
inflammatory disease, tubal infertility, ectopic pregnancy, and chronic
pelvic pain. A 1-gram dose of oral azithromycin is an appropriate
treatment, including during pregnancy. Sexual contacts during the
preceding 60 days should be either treated empirically or tested for
infection and treated if positive. The patient should avoid sexual
intercourse for 7 days after initiation of treatment. Consistent use of

barrier methods for contraception reduces the risk of C. trachomatis


genital infection.
All of the following are risk factors for developing preeclampsia, except
a)
b)
c)
d)

Nullipara
Diabetes
History of preeclampsia
Multiparity

The correct answer is D


Explanation
Preeclampsia is pregnancy-induced hypertension plus proteinuria.
Preeclampsia affects 3 to 7% of pregnant women, usually primigravidas
and women with preexisting hypertension or vascular disorders (eg, renal
disorders, diabetic vasculopathy).
Other risk factors may include maternal age < 20, a family history of
preeclampsia, preeclampsia or poor outcome in previous pregnancies,
multifetal pregnancy, obesity, and thrombotic disorders (eg,
antiphospholipid antibody syndrome).
The most common manifestation of uterine rupture during labor is
a)
b)
c)
d)
e)

Fetal distress
Sudden, tearing uterine pain
Vaginal hemorrhage
Cessation of uterine contractions
Regression of the fetus

The correct answer is A


Explanation
Fetal distress with prolonged, variable, or late decelerations and
bradycardia is the most common, and often only, sign of uterine rupture.
The other signs listed are unreliable and often absent.
A 60 year old female presents with 5 x 5 cm adnexal mass. After a workup
she is diagnosed with ovarian cancer. What is the most appropriate
management?

a) Surgery
b) Chemotherapy
c) Radiotherapy
d) Hormone treatment
The correct answer is A
Explanation
Ovarian cancer is often fatal because it is usually advanced when
diagnosed. Symptoms are usually absent in early stage and nonspecific in
advanced stage. Evaluation usually includes ultrasonography, CT or MRI,
and measurement of tumor markers (eg, cancer antigen 125). Diagnosis is
by histologic analysis. Staging is surgical.
Treatment requires hysterectomy, bilateral salpingo-oophorectomy,
excision of as much involved tissue as possible, and, unless cancer is
localized, chemotherapy.
18 year old white nulliparous student comes to your office. She had
unprotected intercourse within the past 24 hours and does not want to
become pregnant. You explain all options, including nonintervention. She
decides she does not want to be pregnant and requests emergency
contraception. Of the following, which one would be the most
appropriate, effective, and best tolerated for this patient?

a) Diethylstillbestrol, 50 mg twice a day for 5 days


b) High-dose combination oral contraceptive, 2 pills now and 2 pills
in 12 hours
c) Immediate insertion of an intrauterine device
d) Endometrial aspiration

The correct answer is B


Explanation
Emergency postcoital contraception, commonly called ?morning after
protection? (MAP) is intended to be one-time, ?emergency? protection,
not regular contraception. Combination estrogen-progesterone pills can
be given in a form that contains 50 mg ethinyl estradiol and 0.5 mg
norgestrel. Two pills are given immediately and two more 12 hours later.
This method is considered effective if given within 72 hours of
unprotected intercourse.
Levonorgestrel in a dose of 0.75 mg (also called Plan B) is given in two
doses 12 hours apart, also within 72 hours. This dose is equivalent to
10 pills of the levonorgestrel progestin-only minipill. Plan B may be
better tolerated, but is not as readily available in family physicians
offices.
Endometrial aspiration is not 100% effective, and if it were used for
contraception many unnecessary procedures would be done.
Diethylstilbestrol was the choice for MAP for a few years but is rarely
used now; in addition, the correct dosage is 25-50 mg/day for 5 days.
Nausea and menstrual irregularity were common side effects, and both
physicians and patients were uneasy about using a potential teratogen.
Intrauterine devices are not generally recommended in nulliparous women
and should be used only in women who wish to use the device on a
continual basis and who are not at high risk for sexually transmitted
diseases.
A 33-year-old female requests combined oral contraceptive pills (OCPs)
for birth control. Which one of the following would be a
contraindication to prescribing OCPs for this patient?

a) A history of controlled hypertension


b) A family history of ovarian cancer
c) A history of thromboembolic disease
d) A current history of smoking
e) A history of hepatitis C infection with no liver disease
The correct answer is C
Explanation
Contraindications to the use of oral contraceptive pills (OCPs) include
previous thrombosis, preexisting vascular disease, coronary artery
disease, congestive heart failure, active liver disease,
estrogen-dependent cancers, breast cancer, and untreated hypertension.
Patients whose hypertension is well controlled can take OCPs. Current
smoking would be a contraindication in a patient over 35 years of age.
There is some evidence that OCPs may protect against ovarian cancer.

The likelihood of postpartum depression is increased by which one of the


following?
a) Low educational level
b) Bottle feeding
c) Cesarean delivery
d) Unplanned pregnancy
e) A prior history of depression
The correct answer is E
Explanation
Predictors of major depression in women, such as prior depression,
family history of mood disorders, and stressful life situations, are
also predictors of postpartum depression.
A 25 year old female complains of severe dysmenorrhea and dyspareunia.
She states that she, with her husband, has been trying for almost a year
now to get pregnant and has been unsuccessful. What is an appropriate
next step in management?
a) Pregnancy test
b) Laparoscopy

c) Dilation and curettage (D&C)


d) Prescribe oral contraception
The correct answer is B
Explanation
Endometriosis is a noncancerous disorder in which functioning
endometrial tissue is implanted outside the uterine cavity. Symptoms
depend on location of the implants and may include dysmenorrhea,
dyspareunia, infertility, dysuria, and pain during defecation. Diagnosis
is by biopsy, usually via laparoscopy.
Diagnostic laparoscopy is a surgical procedure used to evaluate
intra-abdominal or pelvic pathology (eg, tumor, endometriosis) in
patients with acute or chronic abdominal pain and operability in
patients with cancer. It is also used for lymphoma staging and liver biopsy.
You diagnose Trichomonas vaginitis in a 25 year old white female, and
treat her and her partner with metronidazole (Flagyl), 2 g in a single
dose. She returns 1 week later and is still symptomatic, and a saline
wet prep again shows Trichomonas. Which one of the following is the most
appropriate treatment at this time?

a) Metronidazole gel 0.75% (MetroGel) intravaginally for 5 days


b) Metronidazole, 2 g orally, plus metronidazole gel 0.75%
intravaginally for 5 days
c) Metronidazole, 500 mg orally twice a day for 7 days
d) Clindamycin cream (Cleocin) 2% intravaginally for 7 days
e) Sulfadiazine (Microsulfon), 4 g orally in a single dose, plus
pyrimethamine (Daraprim), 200 mg orally in a single dose
The correct answer is C
Explanation
The preferred treatment for Trichomonas vaginitis is metronidazole, 2 g
given in a single oral dose. Certain strains of Trichomonas vaginalis,
however, have diminished sensitivity to metronidazole. Patients who fail
initial treatment with metronidazole should be retreated with 500 mg
orally twice a day for 7 days. If treatment fails again, the patient
should be treated with 2 g daily for 3-5 days. Metronidazole gel and
clindamycin cream are useful for treating bacterial vaginosis, but are
not effective in the treatment of Trichomonas vaginitis. Sulfadiazine
and pyrimethamine are used to treat toxoplasmosis.
You have provided care for a 27-year-old married, monogamous female for
several years. One year ago, she had abnormal cervical cytology that was
interpreted as ?atypical squamous cells of undetermined significance?
(ASC-US). She had repeat cervical cytologic examinations 6 months ago
and again last week, both reported as negative.
Which one of the following would be the most appropriate next step?

a) Repeat cervical cytology again in 4-6 months


b) Screening for human papillomavirus
c) Colposcopic examination with a biopsy and endocervical curettage
d) Resuming a routine screening protocol
e) Cervical culture for herpesvirus
The correct answer is D
Explanation
The cervical cytology category of atypical squamous cells of
undetermined significance (ASC-US) is one that is poorly reproducible,
having been shown to be frequently downgraded to negative or upgraded to
a low- or high-grade squamous intraepithelial lesion on review.
Recommended management strategies women with ASC-US include repeat
cytology at 4-6 months, immediate colposcopy, and reflex DNA testing for
oncogenic HPV types. Should two repeat cytologic examinations at 4- to
6-month intervals prove negative, the patient can safely return to
routine cytologic screenings. Should any repeat examination detect

ASC-US or more significant cytology, colposcopy is indicated.


A 28 year old mother has delivered a full-term healthy newborn baby.
Mother has tested positive for hepatitis HBe antigen and HBs antigen two
weeks before delivery. Which one of the following would you recommend
for managing the newborn?
a) Check mother's serum for anti-HBsAg before giving the infant any
immunizations
b) Advise not to breast-feed to prevent hepatitis B virus
transmission to the infant
c) Hepatitis B vaccine and intravenous IgG
d) Hepatitis B vaccine and hepatitis B immune globulin
e) Hepatitis B hyper immune globulin in the first 12 hours after
delivery
The correct answer is D
Explanation
Neonates whose mothers are HBsAg-positive should be given 1 dose of HBIG
0.5mL IM within 12 h of birth. Recombinant hepatitis B virus vaccine
should be given IM in a series of 3 doses. (Note: Doses vary among
proprietary vaccines.) The 1st dose is given concurrently with HBIG but
at a different site. The 2nd dose is given at 1 to 2 mo, and the 3rd
dose is given at 6 mo after the first.
When repairing a perineal laceration after a vaginal delivery, which one
of the following suture materials decreases both wound dehiscence and
postpartum perineal pain?

a) Polyglactin 910 (Vicryl)


b) Plain catgut
c) Chromic catgut
d) Silk
e) Polypropylene
The correct answer is A
Explanation
Repair of an obstetric perineal laceration requires use of proper
surgical material compared with repair using plain catgut or chromic
catgut, use of 3-0 polyglactin 910 suture material results in decreased
wound dehiscence and less postpartum perineal pain. Nonabsorbable
sutures are not indicated for the repair of obstetric perineal lacerations.

A 21-year-old nulligravid comes to your office for a routine physical


exam. She is sexually active and has never had a Pap smear performed.
She asks you what benefit the pap smear has for her. You can tell her
that having an annual Pap test screening is estimated to reduce a
womans chance of dying of cervical cancer by approximately:
a) 10%
b) 30%
c) 50%
d) 70%
e) 90%
The correct answer is E
Explanation
The use of the routine Pap smear as a screening tool for cervical cancer
has helped to significantly decrease the number of women dying from this
disease over the last 50+ years. The most common cancer caused death in
women is now lung followed by breast, colorectal, ovary, and pancreas.
In the world, cervical cancer remains the leading gynecologic cancer
killer of women.
Which one of the following is safest for use in pregnancy?
a) Trimethoprim/sulfamethoxazole (Bactrim, Septra)
b) Ciprofloxacin (Cipro)
c) Nitrofurantoin (Macrodantin)

d) Gentamicin
The correct answer is C
Explanation
The FDA has established a fetal risk summary dividing drugs into
categories. Category A drugs have been shown in controlled studies to
pose no risk. At present there are no category A antibiotics. Most fall
into categories B and C, with category B drugs thought to be relatively
safe in pregnancy. When possible, a category B antibiotic should be
chosen for treatment of a pregnant patient. Category C drugs have
unknown fetal risk with no adequate human studies, and the possibility
of risks and benefits must be considered before prescribing them for
pregnant women. Category D drugs show some evidence for fetal risk;
although there may be times when use of these drugs is necessary, they
should not be used unless there is a very serious or life-threatening
situation. Category X drugs have proven fetal risk and are
contraindicated in pregnancy.
Of the drugs listed, only nitrofurantoin is in category B. The others
are all category C drugs.
The FDA is currently in the process of revising their classification and
labeling for drugs in pregnancy and lactation.

Which one of the following represents an advantage of injectable


medroxyprogesterone acetate (Depo-Provera)?
a) There is a general lack of menstrual irregularities
b) It is not causally linked with thromboembolic events
c) There is no weight gain
d) There is no measurable effect on blood lipid levels
e) The cost is half that of generic combination oral contraceptives
The correct answer is B
Explanation
Injectable medroxyprogesterone acetate is not causally linked with
thromboembolic events. The most common side effect is menstrual
irregularities; weight gain is also a bothersome side effect. There may
be a decrease in HDL-cholesterol and an increase in LDL. The cost is
similar to that of combination oral contraceptives.
Risk factors for shoulder dystocia include all the following, except

a)
b)
c)
d)
e)

Maternal obesity
Macrosomia
Maternal diabetes
Prolonged second stage of labour
Outlet forceps delivery

The correct answer is E


Explanation
In this shoulder dystocia, presentation is vertex, but the anterior
fetal shoulder is lodged behind the symphysis pubis, preventing vaginal
delivery. Risk factors include a large fetus and maternal obesity or
diabetes mellitus. Shoulder dystocia is identified if the head, after
passing through the cervix, appears to be pulled back tightly against
the vulva (turtle sign). Asphyxiation can occur because the vaginal
canal compresses the chest so that the fetus cannot breathe. Hypoxic
injury can begin within 4 to 5 minutes.
A 28-year-old secundigravida at 40 weeks gestation is undergoing a
trial of labor. Her prior delivery was complicated by cephalopelvic
disproportion and required a low transverse cesarean section for
delivery. For the last 4 hours, she has been on pitocin for augmentation
for her contractions. About 30 minutes ago she started complaining of
persistent lower abdominal pain. Repetitive variable decelerations began
at the same time. Your exam of the patient notes that she is 3 cm
dilated, has moderate vaginal bleeding, and the presenting part is no
longer palpable. What is the most likely diagnosis

a)
b)
c)
d)
e)

Placental abruption
Placental previa
Uterine hyperstimulation
Uterine rupture
Cord prolapse

The correct answer is D


Explanation
Given the prior cesarean section, the sudden onset of repetitive
variable decelerations along with the sudden loss of fetal station is
highly suspicious for uterine rupture. This occurs in about 0.5-1% of
all women undergoing a trial of labor with a prior low transverse
segment cesarean section. Classical uterine incisions have a much higher
risk of uterine rupture, and labor is contraindicated in these women.
Since uterine rupture can be catastrophic for both the mother and fetus,
women who wish a trial of labor should deliver at a facility where
emergency cesarean deliveries can be performed in a prompt and timely
fashion.
A. Abruption is often associated with uterine pain and bleeding, but
should not be accompanied by a loss of fetal station.
B. Previa is more often associated with painless vaginal bleeding. Also,
the placenta would have been palpated during your pelvic exam.
C. Uterine hyperstimulation can be associated with late decelerations
and uteroplacental insufficiency. No loss of fetal station should occur.
E. Cord prolapse would be palpable on pelvic exam. Loss of fetal station
will increase this risk of this complication. Prolapse is not associated
with bleeding and persistent pelvic pain.
Fetal tricuspid valve abnormalities are associated with maternal use of
which of the following
a)
b)
c)
d)
e)

Lithium
Warfarin
Chloramphenicol
Estrogen
Heroin

The correct answer is A


Explanation
Ebstein's anomaly, also called Ebstein's malformation, is a heart defect
in which the tricuspid valve is abnormally formed. The tricuspid valve
normally has three "flaps" or leaflets. In Ebstein's anomaly, one or two
of the three leaflets are stuck to the wall of the heart and don't move
normally. Often there's also a hole in the wall between the atria, the
heart's two upper chambers. This hole is called an atrial septal defect
or ASD. Because the tricuspid valve is malformed in Ebstein's anomaly,
it often doesn't work properly and may leak. If the valve leaks, some of
the blood pumped by the right ventricle goes backwards through the valve
with each heartbeat.
Studies suggest that lithium might be involved as a teratogen increasing
the incidence of Ebstein's anomaly in the offspring of female patients
with manic-depressive psychosis and lithium-administered during pregnancy.
A 65-year-old woman comes to your office for routine well-woman exam.
Her last menstrual period was 15 years ago. She has not been on estrogen
replacement therapy and now desires to start due to concerns about
osteoporosis. On routine pelvic exam, you palpate a small uterus and
cervix along with palpable ovaries bilaterally. Of the following, your
next step in the management of this patient should be:
a) Start cyclic hormone replacement therapy: premarin 0.625 mg, 1-25
days; provera 10 mg, 16-25 days
b) Start continuous hormone replacement therapy, premarin 0.625 mg
and provera 2.5 mg qday
c) Pelvic ultrasound
d) Dual photon densitometry for evaluation of bone density
e) Exploratory laparotomy

The correct answer is C


Explanation
In a postmenopausal woman, the ovaries should not be palpable; if they
are, it should raise the concern that an ovarian malignancy is present.
Before one would perform an operative evaluation, radiologic assessment
should be done.
A. Although this is an accepted regimen for estrogen replacement
therapy, the palpable ovaries need to be evaluated to rule out malignancy.
D. Dual photon densitometry will give a reliable measure of bone
density. Again, however, the palpable ovary is the first thing that
needs to be worked up.
E. Although surgical exploration may be warranted, initial workup of the
adnexal mass should include an ultrasound along with tumor markers. A CT
scan may also be warranted.
A 20-year-old female long-distance runner presents with a 3-month
history of amenorrhea. A pregnancy test is negative, and other blood
work is normal. She has no other medical problems and takes no
medications. With respect to her amenorrhea, you advise her
a) to increase her caloric intake
b) that this is a normal response to training
c) to begin an estrogen-containing oral contraceptive
d) to stop running
The correct answer is A
Explanation
Amenorrhea is an indicator of inadequate calorie intake, which may be
related to either reduced food consumption or increased energy use. This
is not a normal response to training, and may be the first indication of
a potential developing problem. Young athletes may develop a combination
of conditions, including eating disorders, amenorrhea, and osteoporosis
(the female athlete triad). Amenorrhea usually responds to increased
calorie intake or a decrease in exercise intensity. It is not necessary
for patients such as this one to stop running entirely, however.
You see a 17 year old white female who has recently become sexually
active. She requests oral contraceptives and you perform a brief
evaluation, including blood pressure measurement. A pregnancy test is
negative. She is resistant to further evaluation unless it is
necessary. In addition to appropriate counseling, which one of the
following should be done before prescribing oral contraceptives

a)
b)
c)
d)

No further evaluation at this visit unless indicated by history


A pelvic examination and Papanicolaou test
Screening for sexually transmitted diseases
A breast examination

The correct answer is A


Explanation
Policy statements from major organizations based on reviews of relevant
medical literature support the practice of prescribing initial hormonal
contraception after performing only a careful review of the medical
history plus measurement of blood pressure. Requiring that patients
undergo pelvic and breast examinations lends many young women to avoid
this most reliable method of contraception, resulting in much higher
rate of unwanted pregnancy, Follow-up blood pressure measurements are
important. Often, a younger woman will be willing to undergo
Papanicolaou (pap) tests and STD screening later, and periodic follow-up
must be scheduled. Sexually active adolescents should have annual
screening for cervical cancer and sexually transmitted diseases, but
these are not necessary before prescribing oral contraceptives. The
longest period of time a prescription should be given without a Pap test
is 1 year, but this restriction is under study. Obviously, any history
indicative of high risk would modify this approach.
Which one of the following is consistent with current recommendations

a) Cervical cancer screening with Papanicolaou (Pap) smears should


begin within 1 year after the onset of vaginal intercourse
b) Cervical cancer screening should begin no later than 18 years of
age in all women
c) Screening with vaginal cytology is not indicated in women who have
had a total hysterectomy for benign gynecologic disease
d) Cervical cytologic screening should be done yearly prior to the
age of 30, with either a conventional Pap smear or liquid-based cytology
e) Cervical cytologic screening should routinely cease at age 80
The correct answer is C
Explanation
There are published new guidelines for cervical cancer screening.
According to the new guidelines, cervical cytologic screening should
begin approximately 3 years after the onset of vaginal intercourse, and
no later than 21 years of age. After initiation of screening, cervical
screening should be done annually with conventional Papanicolaou (Pap)
smears, or every 2 years using liquid-based cytology. At or after age
30, women are at conventional risk who have had 3 consecutive negative
results and no abnormal results within the past 10 years. Screening with
vaginal cytology following total hysterectomy for benign gynecologic
disease is not indicated. Women who have had a subtotal hysterectomy and
still have a cervix should continue routine screening.
A 22-year-old gravida 2 para 1 presents to your office with a 1-day
history of vaginal bleeding and abdominal pain. Her last menstrual
period was 10 weeks ago, and she had a positive home pregnancy test 6
weeks ago. She denies any passage of clots. On pelvic examination, you
note blood in the vaginal vault. The internal cervical os is open.
Which one of the following best describes the patients current condition?

a)
b)
c)
d)
e)

Inevitable abortion
Completed abortion
Threatened abortion
Incomplete abortion
Missed abortion

The correct answer is A


Explanation
Inevitable abortion is defined by bleeding, an open os, and no passage
of products of conception (POCs). Bleeding also occurs with completed
abortion, but the os is closed and there is complete passage of POCs.
Threatened abortion is also characterized by bleeding and a closed os,
but there is no passage of POCs. With incomplete abortion there is
bleeding and an open os, but POCs are visualized in the os or vaginal
vault. There are no symptoms with missed abortion, but there is no
embryo or fetus on ultrasonography.
A 19-year-old white female presents for an initial family planning
evaluation. Specifically, she is interested in oral contraception. She
is not presently sexually active, but has a steady boyfriend. She has no
contraindications to oral contraceptive use. She has mild acne vulgaris.
You discuss possible side effects and benefits of combined oral
contraceptives, including improvement of her acne. Which one of the
following is also associated with oral contraceptive use?
a)
b)
c)
d)

Increased risk of ovarian cancer


Decreased risk of ovarian cysts
Increased risk for ectopic pregnancy
Increased incidence of dysmenorrhea

The correct answer is B


Explanation
Women who take combination oral contraceptives have a reduced risk of
both ovarian and endometrial cancer. This benefit is detectable within a
year of use and appears to be persistent for years after
discontinuation. Other benefits include a reduction in dysfunctional
uterine bleeding and dysmenorrhea; a lower incidence of ovarian cysts,
ectopic pregnancy and benign breast

disease; and an increase in hemoglobin concentration. Many woman also


benefit from the convenience of menstrual regularity. All combination
oral contraceptives raise sex hormone-binding globulin and decrease free
testosterone concentrations, which can lead to improvement in acne.
Due to an abnormal Pap smear suggestive of dysplasia, a woman is
referred for evaluation. You colposcopic exam is unsatisfactory since
the entire transformation zone cannot be seen. The endocervical
curettage result is negative for dysplasia, and the biopsy sample
reveals CIN III. Your next step is to:
a) Repeat Pap smear
b) Repeat endocervical curettage
c) Repeat colposcopy and biopsy
d) Perform conization of cervix
e) Perform total abdominal hysterectomy
The correct answer is D
Explanation
Conization of the cervix should be performed because the transformation
zone or the lesion cannot be visualized entirely (unsatisfactory
colposcopy). The procedure is done to rule out occult invasive disease.
A. A biopsy is positive for CIN III. Excision of the transformation zone
is required (cone biopsy).
B. Even if the repeat ECC is negative, there is still the indication for
cervical conization with the CIN biopsy result.
C. See answer to A.
E. The purpose of the cervical conization is to rule out an occult
malignancy. If a hysterectomy is done, and an invasive carcinoma is
found, the the wrong procedure may have been done, and the patients
final prognosis may be worse.
Causes of secondary (not primary) amenorrhea include
a)
b)
c)
d)
e)

Turner syndrome
Anorexia
Androgen insensitivity syndrome
Gonadal dysgenesis
Imperforate hymen

The correct answer is B


Explanation
Secondary amenorrhea is the absence of menstrual periods for 6 months in
a woman who had previously been regular, or for 12 months in a woman who
had irregular periods. This problem is seen in about 1% of women of
reproductive age.
Primary amenorrhea is when the woman has never had a period in her life.
This page will not discuss primary amenorrhea which is rare. The causes
of primary amenorrhea are also quite different from secondary amenorrhea.
Secondary amenorrhea can be caused by hyperprolactinemia., certain
medications such as phenothiazines (used for psychiatric disorders),
anorexia nervosa, cushing's disease, hypothyroidism, sheehan's syndrome
and asherman's syndrome.
A 26-year-old primigravida is at 20 weeks gestation by dates but her
fundal height is consistent with a 26-week gestation. She has had
episodes of vomiting during the pregnancy that were more severe than the
physiologic vomiting typically seen in pregnancy. A sonogram performed
at about 5 weeks gestation for vaginal bleeding was normal and showed a
single fetus.
Which one of the following would be most appropriate at this point?

a)
b)
c)
d)

Serum HCG
Ultrasound
AFP
CT abdomen

e) MRI abdomen
The correct answer is B
Explanation
Ultrasonography is the initial test of choice for evaluating the
possibility of multiple gestation. It should be done if uterine size is
larger than expected, or if pregnancy-associated symptoms are excessive.
It should also be done in women who received fertility treatment. An
initial sonogram that shows a single pregnancy does not rule out
multiple gestation. In one study, 30 of 220 twin pregnancies had an
original sonogram which showed a single pregnancy. Serum HCG and MRI
would not be indicated at this stage of the evaluation.
A 17 year old white female visits you for a physical examination prior
to entering college. During the review of systems her only complaint is
cyclic lower abdominal cramps around the onset of menstruation. She
reports that pain has been present to some degree with most of her
periods since about 6 months after menarche. The pain is often severe
enough for her to miss school. Each episode lasts 24-48 hours and is
somewhat relieved by rest and acetaminophen. Her menstrual history is
otherwise normal. She denies ever being sexually active and tells you
that she has received little empathy from her mother, who has similar
symptoms as an adolescent that improved after her first pregnancy.
Pelvic and rectal examinations are within normal limits. Which one of
the following management choices would be appropriate at this time?
a)
b)
c)
d)
e)

Referral for hysterosalpingography


Referral for psychological counseling
Danazol (Danocrine)
Acetaminophen/hydrocodone (Vicodin HP)
Naproxen sodium (Anaprox)

The correct answer is E


Explanation
The patients history is typical of primary dysmenorrheal, defined as
severe cramping pain in the lower abdomen that occurs during menses; it
may also occur prior to the onset of menses in the absence of associated
pelvic pathology. Although many women complain of pain beginning with
the first cycle, symptoms usually begin at the onset of ovulation around
6-12 months after menarche. Symptoms typically last 48 hours of less,
but sometimes may last up to 72 hours. It is common to find daughters
with dysmenorrheal whose mothers had the same symptoms.
In this patient, who has no history suggesting an emotional disorder,
there is no need for psychological counseling at this time. Further
evaluation could include ultrasonography to rule out causes of
dysmenorrheal such as uterine leiomyomata, adnexal masses, and
endometrial polyps. However, a trial of symptomatic therapy is most
reasonable before other invasive studies, such as a laparoscopic
examination or a hysterosaalpingogram, are ordered. It is not reasonable
to begin danazol without a diagnosis of endometriosis, which is by
definition seconday dysmenorrheal. Since neither inhibits prostaglandin
synthetase, acetaminophen (which she had already tried without complete
relief) combined with a narcotic is not an appropriate management
strategy. Multiple placebo-controlled studies have shown that NSAIDS
such as naproxen, at the onset of symptoms, provide significant relief
of primary dysmenorrheal compared to placebo.
A 21 year old primigravida at 39 weeks gestation presents to the
hospital complaining of labor. Membranes are intact. The cervix is
dilated to 2 cm, 90% effaced and the vertex at 0 station. Contractions
occur every 5-15 minutes and last about 15-25 seconds each. Two hours
later, both her cervical exam and her contraction pattern remain
unchanged. The fetal heart tracing is reassuring. The most appropriate
next step is:
a)
b)
c)
d)
e)

Cesarean section
Augmentation of labor with oxytocin
Radiographic pelvimetry
Reexamine in 1 hour
Amniotomy an placement of internal monitors

The correct answer is D


Explanation
This is a patient in the latent phase of the 1st stage of labor. For a
primigravid, this can last up to 20 hours before it is considered
prolonged. At this point in time, there is no need for any intervention,
since everything is normal. Often, these patients are even sent home and
told to return when more active labor occurs.
A. The patient is in the latent phase of labor and the fetal condition
is fine. No indication for cesarean exists at this point in time.
B. The patient is in the latent phase of labor and the fetal condition
is fine. No indication for augmentation exists at this point in time.
C. The patient is in the latent phase of labor and fetal condition is
fine. There is no concern about the adequacy of the maternal pelvis.
E. The patient is in the latent phase of labor and the fetal condition
is fine. No indication for amniotomy nor intervention exists at this
point in time.
A 33-year-old female presents with 3 months of irregular vaginal
bleeding. Prior to this her menstrual periods were normal.
Which one of the following is the most appropriate initial laboratory
test for this patient?
a) Hemoglobin and hematocrit
b) TSH
c) LH and FSH
d) Estradiol
e) hCG
The correct answer is E
Explanation
In women of childbearing age, the most likely cause of abnormal vaginal
bleeding is pregnancy; thus, the most appropriate initial test would be
an hCG level. Once pregnancy has been excluded, patient history would
guide further testing. Iatrogenic causes, usually resulting from certain
medicines or supplements, are the next most common cause in this age
group, followed by systemic disorders. Hemoglobin and hematocrit would
be appropriate only if the patient seemed acutely anemic due to the
abnormal bleeding.
A 20 year old college student presents with lower abdominal pain and
fever. Physical exam shows bilateral lower abdominal tenderness. Her
vaginal exam shows tenderness with cervical mobilization. Her pregnant
test is negative. What is the most likely diagnosis

a) Acute salpingitis
b) Ectopic pregnancy
c) UTI
d) Trichomonas
The correct answer is A
Explanation
Pelvic inflammatory disease is infection of the upper female genital
tract: the cervix, uterus, fallopian tubes, and ovaries; abscesses may
occur. Common symptoms and signs include lower abdominal pain, cervical
discharge, and irregular vaginal bleeding. Long-term complications
include infertility, chronic pelvic pain, and ectopic pregnancy.
Pelvic inflammatory disease (PID) results from microorganisms ascending
from the vagina and cervix into the endometrium and fallopian tubes.
Infection of the cervix (cervicitis) causes mucopurulent discharge.
Infection of the fallopian tubes (salpingitis) and uterus (endometritis)
tend to occur together. If severe, infection can spread to the ovaries
(oophoritis) and then the peritoneum (peritonitis). These infections are
called salpingitis even though they involve other structures.
Diagnosis includes PCR of cervical specimens for Neisseria gonorrhoeae
and chlamydiae, microscopic examination of cervical discharge (usually),
and ultrasonography or laparoscopy (occasionally). Treatment is with

antibiotics.
A 30 year old white gravida 2 para 2 presents to your office complaining
of mild to moderate tenderness in the left groin. She and her husband
use condoms and contraceptive foam for birth control. An examination
fails to elicit any significant abdominal pain, but on pelvic
examination a tender, 4-5 cm. freely movable left adnexal mass is felt.
A qualitative serum hCG is negative. The most likely diagnosis is

a)
b)
c)
d)
e)

Dysgerminoma
Corpus luteum cyst
Serous cystadenoma
Dermoid cyst
Ovarian fibroma

The correct answer is B


Explanation
Ovarian granulose cells become luteinized after ovulation to form a
corpus luteum. At times blood may enter this body of cells and form a
corpus hemorrhagicum, with later resolution by cyst formation. Focal
pain and tenderness are prominent features. Following menarche, adnexal
masses are most likely to be follicular and corpus luteum cysts of the
ovary.
While dysgerminomas are the most frequent ovarian malignancy in young
women, they are not as frequent as corpus luteum and follicular cysts.
Fibromas and thecomas account for up to 4% of ovarian tumors. They are
most commonly seen in patients aged 40-60.
Estrogen replacement therapy in postmenopausal women with no history of
heart disease has been shown to do all of the following EXCEPT:
a)
b)
c)
d)
e)

Reduce the risk of osteoporosis


Reduce the risk of coronary heart disease
Reduce the risk of ovarian cancer
Reduce the risk of bowel cancer
Prevent genitourinary atrophy

The correct answer is C


Explanation
Postmenopausal estrogen therapy has been shown to do all of the
following except decrease the risk of ovarian cancer. Although 5 years
use of oral contraceptives has been, shown to reduce the risk of ovarian
cancer, no such association has been shown with hormone replacement.
A. ERT clearly decreases the rate of bone loss in the postmenopausal woman.
B. In women without preexisting coronary heart disease, there is a
decreased rate of coronary death in women taking ERT. Some recent data
suggest, however, that women with preexisting heart disease may be at
greater risk for the first few years of therapy.
D. Studies have shown that the rate of bowel cancer is less in women on ERT.
E. The tissues of the pelvic floor and perineum are sensitive to
estrogen. With the drop in circulating estrogens due to menopause,
atrophy will occur.
A 24 year old G1P0 is seen in the Emergency Department. Her LMP was 8
weeks ago. She is experiencing lower abdominal cramping and heavy
vaginal bleeding with clots. Examination reveals a soft abdomen with
mild lower abdominal tenderness. On pelvic exam, the vagina is filled
with blood and clots. The cervical os is opened and tissue is
protruding. The uterus is enlarged to a 6 week size. Which of the
following is the most likely diagnosis
a)
b)
c)
d)
e)

Ectopic pregnancy
Threatened abortion
Degenerating fibroid
Placenta previa
Incomplete abortion

Correct Answer:* e)
Explanation
An incomplete abortion occurs when the woman's body expels only a

portion of the pregnancy tissue. During an incomplete abortion, portions


of the fetus, amniotic sac or placenta may be retained. Symptoms include
an open cervix, cramping, and the discharging of blood and fetal matter.
During your yearly exam of a 13-year-old girl, you evalute her pubertal
development. Breast development started about 2 years ago, followed more
recently by pubic and axillary hair growth. When you examine her from
the side, you see the following breast contour (Figure 20D). This is an

example of Tanner stage:


a)
b)
c)
d)
e)

One
Two
Three
Four
Five

The correct answer is D


Explanation
Tanner stage 1 is defined as the prepubertal breast. The presence of a
breast bud becomes stage 2. As the breasts develop and become conical,
they are Tanner stage 3. When the areola elevates off of the breast
mound, then stage 4 is reached. Tanner stage 5 is the mature adult
breast contour.
The most common retinal change associated with pregnancy-induced
hypertension is
a) Hemorrhage
b) Papilledema
c) Spasm of arterioles
d) Detachment
e) Exudates
The correct answer is C
Explanation
Pregnancy-induced hypertension (PIH), also known as gestational
hypertension, is a potentially life-threatening disorder that usually
develops late in the second trimester or in the third trimester.
Preeclampsia, the nonconvulsive form of PIH, may be mild or severe.
Eclampsia is the convulsive form of PIH.
Mild preeclampsia generally produces the following clinical effects:
hypertension, proteinuria (less than 5 g/24 hours), generalized edema,
and sudden weight gain of more than 3 lb (1.4 kg) per week during the
second trimester or more than 1 lb (0.5 kg) a week during the third
trimester.

Severe preeclampsia is marked by increased hypertension and proteinuria,


eventually leading to the development of oliguria. Hemolysis, elevated
liver enzymes, and low platelets (the HELLP syndrome) is a severe
variant. Other symptoms that may indicate worsening preeclampsia include
blurred vision due to retinal arteriolar spasms, epigastric pain or
heartburn, and severe frontal headache.
You are evaluating a 28-year-old primigravida for an abnormal
Papanicolaou (Pap) test. Which one of the following procedures would be
contraindicated?
a)
b)
c)
d)
e)
The correct answer is B

Colposcopy
Endocervical curettage
Human papillomavirus (HPV) testing
Cervical staining
A cervical biopsy

Explanation
Of the choices listed, only endocervical curettage is contraindicated in
pregnancy. Colposcopy, cervical biopsy, cervical staining, and HPV
testing can all be safely performed during pregnancy.

A 25 year old primigravida presents with sharp, stabbing, left-sided


pelvic pain that started yesterday, 45 days after her last menstrual
period. Her past history is not remarkable, and a physical examination
is normal except for moderate tenderness in the left adnexa on pelvic
examination. A urinalysis is normal, as is a CBC. Her Beta-hCG level is
1500 mIU/mL. Assuming no adnexal mass is seen, which one of the
following transvaginal pelvic ultrasonography findings would be
consistent with the highest likelihood of an ectopic pregnancy?

a) Empty uterus: empty endometrial cavity with or without a thickened


endometrium
b) Abnormal gestational sac: anechoic intrauterine fluid collection
either >10 mm in mean sac diameter or with a grossly irregular border
c) Nonspecific fluid: anechoic intrauterine fluid collection <10 mm
in mean sac diameter without an echogenic border
d) Echogenic material: echogenic material within the endometrial
cavity without a defined sac, or multiple discrete anechoic collections
of various sizes divided by echogenic septations
Correct Answer:* a)
Explanation
At this time in the patients pregnancy, a gestational sac should be
visible on ultrasonography. An empty uterus presents the highest risk
(14%) for ectopic pregnancy, while nonspecific fluid and echogenic
material are associated with a 5% and 4% risk, respectively. An abnormal
or normal sac is associated with no risk, with the rare exception of
multiple pregnancies with one being heterotopic.
Which of the following is not an absolute contraindication for the
placement of an Intra-uterine device (IUD) in a woman for contraception
purposes

a) Intramural fibroid
b) Didelphys uterus
c) Previous cervical gonoccocal infection
d) Pregnancy
e) Vasovagal reactivity
The correct answer is E
Explanation
Intrauterine devices (IUDs) are used widely by women as a means of
contraception because they are highly effective, have no systemic
effects and can last up to 5 years.

IUDs induce endometrial inflammation, this inflammation attracts


neutrophils, which are toxic to sperm and prevent fertilization of the ovum.
Absolute contraindications to IUD insertion are: confirmed pregnancy,
severe distortion of the uterine cavity (such as by fibroids or anatomic
anomalies), acute, recent (within 3 months) or recurrent uterine
infection (includes sexually transmitted, postpartum and postabortal
infections).
Relative contraindications or warnings for IUD insertion include: risk
factors for STDs (non-monogamous relationship, history of STDs), history
of a previous IUD problem (perforation, expulsion, significant pain,
menorrhagia), unresolved abnormal uterine bleeding, known
immunocompromise, past history of severe vasovagal reactivity.
When a woman less than 50 years of age develops vulvar cancer, which one
of the following associated conditions is most frequently present?
a) Human papillomavirus
b) Lichen sclerosus
c) Diabetes mellitus
d) Syphilis
e) Lymphogranuloma venereum
The correct answer is A
Explanation
There has been an increase in vulvar cancer in women 35-65 years of age
over the last decade. This increase is associated with human
papillomavirus infection, particularly involving subtypes 16 and 18.
Lichen schlerosus is associated with vulvar cancer in older women.
Hypertension, diabetes mellitus, and obesity may coexist, but are not
felt to be independent risk factors. Syphilis and other granulomatous
diseases have been associated with vulvar cancer in the past; they are
not currently considered to be significant risk factors, but are
considered markers for sexual behavior associated with increased risk.
Contraindications to vaginal delivery include each of the following, except

a) Previous classical cesarean section


b) Squamous cell carcinoma in situ of the cervix
c) Total placenta previa
d) Transverse lie with ruptured membranes
e) Primigravida breech with cephalo-pelvic disproportion
The correct answer is B
Explanation
Contraindications to vaginal delivery include footling breech, large
fetus (>3800 grams), hyperextended fetal head, inadequate pelvic size,
medical/obstetric contraindications to labour and vaginal delivery,
abnormal fetus, previous classic cesarean, total placenta previa and
transverse lie presentation.
A 26 year old woman presents in labor at 39 weeks of gestation. Upon
artificial membrane rupture, there are several milliliters of blood in
addition to the amniotic fluid. Within minutes, fetal tachycardia
develops. There is no change in the contraction pattern or in maternal
discomfort. Which one of the following is the most likely diagnosis

a)
b)
c)
d)
e)

Placenta previa
Abruptio placentae
Vasa previa
Marginal sinus bleeding
Rupture of cervical varix

The correct answer is C


Explanation
Vasa previa is an uncommon condition in which fetal blood vessels
traverse the lower uterine segment in advance of the presenting part.
Neither the umbilical cord nor the placenta supports the vessels. Vasa
previa presents with painless vaginal bleeding at the time of

spontaneous rupture of membranes or amniotomy (AROM). Fetal shock or


demise can occur rapidly. Fetal mortality for cases not recognized
before the onset of labor is reported to range between 22.5% and 100%.
A multipara has just delivered a 10-pound infant after a precipitous
labor. When the placenta delivers a short while later, it appears to be
attached to a large round firm mass that fills the vagina. A large
amount of active bright red vaginal bleeding is noted. The patient
becomes pale and her blood pressure is noted to be 70/30 mm Hg. What is
the most likely diagnosis
a) Intraabdominal hemorrhage
b) Uterine inversion
c) Retroperitoneal hemorrhage
d) Uterine atony
e) Uterine rupture
The correct answer is B
Explanation
Complete uterine inversion after delivery is typically due to excessive
cord traction with a fundal placenta. Accreta and the use of magnesium
may increase the risk of this complication. Life threatening hemorrhage
and profound hypotension may rapidly occur. Prompt treatment is
necessary to prevent a possible fatal outcome. Immediate replacement of
the uterus should be attempted. While attempting this maneuver, large
bore intravenous lines must be placed and adequate fluid resuscitation
started. If successful, then uterotonic agents should be given. If
unsuccessful, then various agents to relax the uterus may be necessary.

A. Intraabdominal hemorrhage will not be visible from the vagina.


C. Retroperitoneal hemorrhage will not be visible from the vagina.
D. The uterus will be soft and boggy, not firm as in this case.
E. Uterine rupture can have significant bleeding, but a mass will not be
seen.
Pelvic inflammatory disease presents with all of the following, except
a)
b)
c)
d)
e)

Adnexal pain
Cervical motion tenderness
Lower abdominal pain
Pelvic mass
Fever

The correct answer is D


Explanation
Pelvic inflammatory disease is an infection of the upper female genital
tract: the cervix, uterus, fallopian tubes, and ovaries.
The main clinical criteria for the diagnoses of pelvic inflammatory
disease (PID) is: lower abdominal tenderness, uterine/adnexal tenderness
and cervical motion tenderness. Fever, cervical discharge, and irregular
vaginal bleeding may also occur.
Long-term complications include infertility, chronic pelvic pain, and
ectopic pregnancy. Diagnosis includes testing cervical specimens for
Neisseria gonorrhoeae and chlamydiae, microscopic examination of
cervical discharge (usually), and ultrasonography or laparoscopy

(occasionally). Treatment is with antibiotics.


A 27 year old woman comes to the emergency department because of
increasingly severe lower quadrant pain and nausea for the past 2 days.
She is sexually active and does not use any contraception. Her last
menstrual period started last week and ended 4 days ago. Temperature is
38.3C (101.0F), orally. On physical examination, there is bilateral
lower quadrant tenderness with rebound and guarding. Pelvic examination
shows leukorrhea at the cervical os and the uterus is tender to
palpation. The adnexal areas are tender but no masses are palpable.
Which of the following is the most appropriate diagnostic study?

a)
b)
c)
d)
e)

Cervical culture
Culdocentesis
Laparoscopy
Serum beta-HCG concentration
Ultrasonography of the pelvis

The correct answer is A


Explanation
The most likely diagnosis in this sexually active young woman who does
not use contraception and has had a recent period, is PID (pelvic
inflammatory disease). The three major criteria are abdominal pain,
cervical motion tenderness, and adnexal tenderness. Minor criteria are
fever, elevated white blood cell count, purulent cervical discharge, and
elevated ESR (erythrocyte sedimentation rate). The diagnosis is
confirmed by cultures of the cervix for Chlamydia trachomatis (more
common) and Neisseria gonorrhea (less common).
Culdocentesis, transvaginal sampling of the cul-de-sac, can be performed
to confirm the diagnosis of an ectopic pregnancy; the presence of
nonclotting blood is highly suggestive. Laparoscopy can be performed if
there is suspicion of Tubo-ovarian abscess; in this case, however, no
masses are palpated. Since the patient has recently had a menstrual
period, measurement of serum hCG will not provide additional
information. Ultrasonography is also employed to better characterize a
physical finding such as a pelvic mass.
Which one of the following is the most reliable clinical symptom of
uterine rupture?
a) Sudden, tearing uterine pain
b) Vaginal bleeding
c) Loss of uterine tone
d) Fetal distress
The correct answer is D
Explanation
Fetal distress has proven to be the most reliable clinical symptom of
uterine rupture. The ?classic? signs of uterine rupture such as sudden,
tearing uterine pain, vaginal hemorrhage, and loss of uterine tone of
cessation of uterine are not reliable and are often absent. Pain and
bleeding occur in as few as 10% of cases. Even ruptures monitored with
an intrauterine pressure catheter fail to show loss of uterine tone.
Signs of fetal distress are often the only manifestation of uterine rupture.
A 36-year-old gravida 4 para 4 presents to your office due to irregular
vaginal bleeding. Her last delivery was 2 years ago and uncomplicated.
Since then, she has had two normal periods, but only intermittent
spotting and bleeding for the last 7 months. Pelvic exam demonstrates a
normal sized uterus and adnexa. You perform an endometrial biopsy to
rule out the possibility of a malignancy. The biopsy finding is most
likely to show:
a) Endometrial adenocarcinoma
b) Adenomatous hyperplasia with atypia
c) Adenomatous hyperplasia without atypia
d) Proliferative endometrium
e) Arias Stella reaction
The correct answer is D

Explanation
The most common cause of secondary amenorrhea is anovulation. Since she
had a vaginal delivery 2 years ago, she has not had a prolonged exposure
to unopposed estrogen, so the likelihood of an endometrial malignancy is
very low. The most common finding is proliferative endometrium in this
case. An Arias Stella reaction is found in pregnancy; it is the
hypersecretory gland appearance seen on histopathology.
A. There have been only 2 years since her last delivery. It is unlikely
that she has developed an estrogen induced neoplasia in that time frame.
B. See answer to A.
C. See answer to A.
E. This is the hypersecretory reaction to pregnancy.
A 31 year old woman who is 18 weeks pregnant with her third child comes
to the office for a routine prenatal visit. She has used crack cocaine
on and off throughout this pregnancy. You have encouraged her to seek
help, however, she has made no attempt to abstain from using cocaine and
she refuses to commit herself to another drug treatment program. In your
attempt to persuade this patient to stop using cocaine, you advise her
that if she continues to use cocaine during the pregnancy, she increases
her risk for which of the following?

a) Chorioamnionitis
b) Gestational diabetes
c) Placental abruption
d) Placenta previa
e) Preeclampsia
The correct answer is C
Explanation
Cocaine toxicity results from its intensive activation of the
sympathetic nervous system, producing vasoconstriction and hypertension.
Vasoconstriction produces decreased placental perfusion and placental
insufficiency. Hypertension may result in placental abruption. There may
be coexisting factors contributing to placental abruption in cocaine
addicts. In addition to an increased risk of abruption, there is an
increased incidence of congenital anomalies, particularly of the
gastrointestinal and urinary tracts, when cocaine has been abused during
pregnancy. This fact may also be used in a persuasive manner to convince
the patient to discontinue her abuse of cocaine.
Other potential risks to the cocaine-abusing gravida include premature
labor, premature delivery, and intrauterine growth retardation. If a
patient refuses treatment, continued visits and inquiries about
substance abuse are appropriate. Any positive efforts by the patient to
discontinue use should be encouraged. Documentation of referral efforts
and the fact that the patient was informed about the effects of illicit
drugs on the pregnancy should be made. There is no effective chemical
detoxification or replacement therapy for cocaine addicts. Treatment
emphasizes abstinence and psychosocial counseling.
A 28 year old white female presents with painful genital ulcers. She has
not had any previous episodes of similar outbreaks. She is single, but
has had several heterosexual relationships. She has been with her
current partner for 3 years. A culture confirms a herpes simplex virus
(HSV) infection. Which one of the following is true regarding her situation?
a) Suppressive therapy can reduce the risk of transmission to her
partner
b) In the genital area, HSV type 1 infection can be differentiated
clinically from HSV type 2 infection
c) This outbreak is conclusive evidence of infidelity in her partner
d) An HSV vaccine is available for her partner to reduce his risk of
infection
The correct answer is A
Explanation
Suppressive therapy with acyclovir, valacyclovir, or famiciclovir
reduces, but does not eliminate, the risk of transmission of HSV to

sexual partners. HSV type 1 and HSV type 2 infections in the genital
area are clinically identical. Psychological issues, including anger,
guilt, low self-esteem, anxiety, and depression are common after first
receiving a diagnosis of genital HSV infection. Initial clinical
outbreaks of genital HSV infections are often recurrences of previous
infection. Either of the partners may have had an asymptomatic infection
acquired in a previous relationship. An experimental HSV type 2 vaccine
has been developed, but it is ineffective in men.
A 21-year-old married Hispanic female who is using no method of
contraception presents to your office for evaluation of vaginal spotting
6 weeks after her last menstrual period. Her periods have previously
been regular. She has had one previous episode of pelvic inflammatory
disease. A home pregnancy test is positive.
Which one of the following is true in this situation?

a) Serum hCG levels should double every 2 ? 3 days if the pregnancy


is viable
b) Painless bleeding excludes the diagnosis of ectopic pregnancy
c) Laparoscopy should be performed to exclude ectopic pregnancy
d) A serum progesterone level > 25 ng/mL indicates that ectopic
pregnancy is likely
The correct answer is A
Explanation
Early diagnosis of ectopic pregnancy requires a high index of suspicion.
Risk factors include previous ectopic pregnancy, tubal sterilization,
pelvic inflammatory disease, IUD use, and in utero exposure to
diethylstilbestrol. The classic triad of missed menses, pain, and
bleeding may not also be present. In early pregnancies of less than 5
weeks gestation, serial hCG levels are helpful. Serum hCG levels double
every 1.4 ? 2 days. In a healthy pregnancy the level is expected to
increase by at least 66% in 48 hours. Combining serial hCG levels with
transvaginal ultrasonography is the best combination for evaluation of
first-trimester problems.
Serum hCG levels correlate well with sonographic landmarks. At 5 weeks
gestation in a normal pregnancy, serum hCG is > 1000 mIU/mL and a
gestational sac can be visualized in the uterus. Serum hCG is > 2500
mIU/mL at 6 weeks and a yolk sac can be seen within the gestational sac.
An hCG level of 5000 mIU/mL is compatible with visualization of a fetal
pole. When the level is 17,000 mIU/mL, cardiac activity is detectable.
Progesterone levels are also predictive of fetal outcome. A single level
of 25 ng/mL or higher indicates a healthy pregnancy and excludes ectopic
pregnancy with a sensitivity of 98%. If the level is < 5 ng/mL, the
pregnancy is nonviable. Assessment of fetal well-being is difficult if
levels are in the intermediate range of 5 ? 25 ng/mL.
The most important factor in improving perinatal outcome in diabetic
pregnancies is
a)
b)
c)
d)
e)

The use of nonstress tests


Serial biparietal diameter estimations
Amniocentesis for fetal lung maturity
Control of maternal glucose levels
Intrapartum fetal monitoring

The correct answer is D


Explanation
Poor control of preexisting or gestational diabetes that occurs during
organogenesis (up to about 10 weeks gestation) increases risk of major
congenital malformations. Gestational diabetes can result in fetal
macrosomia (fetal weight > 4500 g at birth) even if plasma glucose is
kept nearly normal.
Preconception counseling and optimal control of diabetes before, during,
and after pregnancy minimize maternal and fetal risks, including
congenital malformations. Because malformations may develop before

pregnancy is diagnosed, the need for constant, strict control of glucose


levels is stressed to women who have diabetes and who are considering
pregnancy (or not using contraception).
You advise a woman who is pregnant to take folic acid during her
pregnancy. She ask why, you tell her it is to prevent the development of

a) Fetal lung immaturity


b) Premature delivery
c) Trisomy (Eg Down syndrome)
d) Neural tube defects (eg anencephaly)
The correct answer is D
Explanation
Pregnant women should take folic acid prior to getting pregnant. Neural
tube defects usually develop in the first 28 days of pregnancy, often
before a woman even knows that she is pregnant.
The absence of folic acid increases the possibility of a neural tube
defect, which are defects in the development of the spinal cord.
Spina bifida is a condition in which the spinal cord is exposed. If the
vertebrae (bones of the spinal column) surrounding the spinal cord do
not close properly during the first 28 days after fertilization, the
cord or spinal fluid bulge through, usually in the lower back.
Anencephaly is the severe underdevelopment of the brain.
Late decelerations on fetal monitoring are thought to indicate which one
of the following?
a) Fetal head compression
b) Umbilical cord compression
c) Fetal sleep
d) Uterine hypotonus
e) Uteroplacental insufficiency
The correct answer is E
Explanation
Late decelerations are thought to be associated with uteroplacental
insufficiency and fetal hypoxia due to decreased blood flow in the
placenta. This pattern is a warning sign and is associated with
increasing fetal compromise, worsening fetal acidosis, fetal central
nervous system depression, and/or direct myocardial hypoxia. Early
decelerations are thought to result from vagus nerve response to fetal
head compression, and are not associated with increased fetal mortality
or morbidity. Variable decelerations are thought to be due to acute,
intermittent compression of the umbilical cord between fetal parts and
the contracting uterus.
A 16 year old sexually active nulliparous white female complains of
pelvic pain and vaginal discharge. On examination she is found to have a
temperature of 39.8C (102.0F) pain with movement of the cervix, and
tenderness and a mass in the right adnexa. According to CDC guidelines,
which one of the following treatments would be appropriate?

a) Outpatient treatment with penicillin G procain (Wycillin)


intramuscularly; probenecid (Benemid) orally; plus doxycycline
(Vibramycin) orally for 14 days and reexamination in 3 days
b) Outpatient treatment with ceftriaxone (Rocephin) intramuscularly;
probenecid orally; plus doxycycline twice a day for 14 days and
reexamination in 1 week
c) Outpatient treatment with cefoxitin (Mefoxin) intramuscularly;
plus docycycline twice a day for 14 days and reexamination in 10 days
d) Hospitalization for treatment with cefoxitin intravenously and
doxycycline orally twice a day to complete 14 days of treatment
The correct answer is D
Explanation
Patients with PID and tubo-ovarian abscess and high fever should be

hospitalized and treated for at least 24 hours with intravenous


antibiotics. Amoxicillin and penicillin G procaine are no longer
recommended because of the increasing prevalence of
penicillinase-producing and chromosomally-mediated resistant Neisseria
gonorrhoeae. If cefoxitin is used intramuscularly for outpatient
treatment, it should be combined with probenecid. If ceftriaxone is used
for outpatient treatment, probenecid is not required. Re-examination
should be done within 3 days of initiation of therapy.
The most common cause of jaundice in pregnancy is

a) HELLP syndrome
b) Acute fatty liver of pregnancy
c) Acute cholelithiasis
d) Acute viral hepatitis
e) Intrahepatic cholestasis of pregnancy
The correct answer is D
Explanation
Viral hepatitis is the most common cause of jaundice in pregnancy,
accounting for 50% of all cases. This is important to recognize, as it
should be considered first in the differential diagnosis of jaundice and
elevated transaminase levels, in particular ALT. In this situation, the
diagnosis can be made or excluded by ordering serologic tests for
hepatitis. If these are negative, further evaluation for more infrequent
causes of jaundice in pregnancy should be pursued.
HELLP syndrome is associated with preeclampsia and is rare, occurring in
0.1% of all pregnancies. It presents with hemolysis, elevated liver
enzymes, and low platelets. Acute fatty liver of pregnancy presents as a
complication of preeclampsia and occurs in 1 in 13,000 pregnancies.
Acute cholelithiasis usually does not present with jaundice, but with
right upper quadrant pain. Jaundice does occur, however, if
choledocholithiasis is present. Intrahepatic cholestasis of pregnancy is
also rare, occurring in 0.01% of all pregnancies. This presents with
pruritis and/or jaundice.
A 17-year-old white female presents with new-onset left-sided lower
abdominal pain. Color flow Doppler ultrasonography, in addition to
pelvic ultrasonography, would be most useful for evaluating.

a)
b)
c)
d)

Adnexal torsion
Pelvic abscess
Pelvic inflammatory disease
Ruptured ovarian cyst

The correct answer is A


Explanation
Color Doppler flow studies are useful for evaluating blood flow to the
ovary in possible cases of adnexal or ovarian torsion. Adnexal torsion
is a surgical emergency.
Pelvic ultrasonography, preferably with a vaginal probe, can be
beneficial in the workup of ruptured ovarian cyst, pelvic abscess, and
pelvic inflammatory disease without abscess. The Doppler flow study is
not required with these conditions.
Which of the following medications is contraindicated in the last
trimester of pregnancy?
a)
b)
c)
d)

Sulfonamide
Ampicillin
Diazepam
Heparin

The correct answer is A


Explanation
Sulfonamides (the "sulfa" drugs) are safe early in pregnancy, but their
use in the last trimester might result in a jaundiced infant. The other
medications listed are safe in pregnancy.

A patient with gonorrhea was treated with ceftriaxone. Afterwards there


is recurrence of symptoms. What is the most likely reason and what is
the appropriate management?

a) Chlamydia, doxycycline
b) Syphilis, penicillin
c) Herpes, acyclovir
d) UTI, ciprofloxacin
The correct answer is A
Explanation
Uncomplicated gonococcal infection of the urethra, cervix, rectum, and
pharynx is treated with a single dose of ceftriaxone. Patients are also
empirically treated for chlamydia infection, which is often asymptomatic
or masked by symptoms of gonorrhea.
Concomitant infection with Chlamydia occurs in 15 to 25% of heterosexual
men and 35 to 50% of women.
A 21 year old woman returns to the office 2 months after having a
medroxyprogesterone injection for contraception. She is complaining of
nonstop bleeding since her menses 3 weeks ago. She is using eight pads a
day. She denies any sexual activity since she received the injection.
She realizes that spotting is a side effect; however, she is anxious
about the length of time and the amount of the bleeding. She tells you,
"I can't stand this, Doctor. I want the bleeding to stop now!" Repeat
pregnancy test is negative. Which of the following is the most
appropriate treatment option for her bleeding?

a)
b)
c)
d)
e)

Conjugated estrogen therapy for 2 weeks


Dilatation and curettage
Reassurance and counseling
A second injection of medroxyprogesterone
Triphasic oral contraceptive therapy for one cycle

The correct answer is A


Explanation
Progesterone implants for birth control are a popular form of
contraceptive device. The benefits of prolonged duration of action and
no requirement for daily pills are offset for many women by the side
effects of weight gain and heavy bleeding. After ruling out pregnancy
(the most common cause of abnormal uterine bleeding), the most
appropriate intervention to ?oppose? the progesterone with estrogens to
make the uterine lining for ?physiologic?. This intervention is fairly
effective at minimizing bleeding.
An ultrasound will diagnose all of the following at 20 weeks gestation,
except
a) Twin pregnancy
b) Polyhydramnios
c) Trisomy 21
d) Gender of fetus
The correct answer is C
Explanation
An ultrasound done at 20 weeks gestation can show many things including
multiple gestation, polyhydramnios, oligohydramnis and gender. To
diagnose a trisomy, further testing such as a triple screen and
amniocentesis would need to be done.

A pregnant woman at 20 weeks gestation has a uterine size of 25 cm. All


of the following are possibilities, except
a)
b)
c)
d)
The correct answer is D

Normal pregnancy
Wrong estimation of gestational age
Twin pregnancy
Bilateral renal agenesis

Explanation
Bilateral renal agenesis is associated with oligohydramnios, as the
fetus cannot produce urine and contribute to the volume of the amniotic
fluid. Therefore uterine size would be small for dates. The other answer
choices could have a large uterine size on physical exam.

A previously healthy 22-year-old female presents for her regular


prenatal checkup at 38 weeks gestation. She has a blood pressure of
145/95 mm Hg today and this is unchanged 1 hour later. Her blood
pressure was normal before pregnancy. She is otherwise feeling well. She
has moderate edema at the ankles and 3+ reflexes at the knees and
ankles. A urinalysis for protein is normal.
Given this presentation, which one of the following is the most likely
diagnosis

a)
b)
c)
d)
e)

Preeclampsia
Unmasked chronic hypertension
Essential hypertension
Gestational hypertension
Hemolysis, elevated liver enzymes, low platelets (HELLP) syndrome

The correct answer is D


Explanation
This patient has gestational hypertension. This condition is diagnosed
when elevated blood pressure without proteinuria develops after 20 weeks
gestation and blood pressure returns to normal within 12 weeks of delivery.
The new criteria for preeclampsia specify a new onset of hypertension
(systolic blood pressure > 140 mm Hg or diastolic pressure > 90 mm Hg)
after 20 weeks gestation, along with 300 mg protein in a 24-hour urine
specimen. A certain amount of increase in blood pressure is no longer a
criterion. Edema is also no longer a criterion. Elevated uric acid
levels are no longer considered necessary for the diagnosis of preeclampsia.
Chronic hypertension can only be diagnosed if present before pregnancy,
or if it does not resolve by 12 weeks post partum. Essential
hypertension is the most common cause of chronic hypertension. HELLP
syndrome is a serious, but relatively rare, form of pregnancy-related
hypertension associated with hemolysis, elevated liver enzymes, and low
platelets.
A 23 year old woman presents to your office with chief complaint of
smelly vaginal discharge. You decide to do a test to check for bacterial
vaginosis. She is not sexually active. What is the proper method to look
for clue cells See picture:

a)
b)
c)
d)
The correct answer is B
Explanation

Vaginal secretion with KOH and cover


Vaginal secretion with saline and cover
Cervical secretion with saline and cover
Cervical secretion with KOH and cover

In bacterial vaginosis (BV), a woman will complain of odorous vaginal


discharge. Physical exam may show the discharge upon speculum exam. The
following tests are done to diagnose common vaginal infections which are
bacterial vaginosis, trichomoniasis and yeast infections.
Wet prep (wet mount/wet smear): A sample of the vaginal discharge is
placed on a glass slide and mixed with a saline solution. The slide is
looked at under a microscope for bacteria, yeast cells, trichomoniasis
(trichomonads), white blood cells that show an infection or clue cells
that show bacterial vaginosis.
KOH slide. A sample of the vaginal discharge is placed on a slide and
mixed with a solution of potassium hydroxide (KOH). The KOH kills
bacteria and cells from the vagina, leaving only yeast for a yeast
infection.
Vaginal pH. The normal vaginal pH is 3.8 to 4.5. Bacterial vaginosis,
trichomoniasis, and atrophic vaginitis often cause a vaginal pH higher
than 4.5.
Treatment of BV is with metronidazole (Flagyl). Since this is an
infection caused by the overgrowth of the bacteria normally in the
vagina it is not an STD so any sexual partners would not need treatment.
A 23-year-old gravida 3 para 1 at 28 weeks gestation whose blood type is
O-negative is antibody positive (D antibody) on a routine 28-week
screen. Which one of the following best describes the clinical
significance of this finding?

a) The fetus HAS hemolytic disease and requires appropriate


monitoring and treatment
b) The fetus is AT RISK for hemolytic disease only if the biological
father is Rh-negative
c) The fetus is AT RISK for hemolytic disease only if the biological
father is Rh-positive
d) The current fetus is NOT at risk for hemolytic disease, but
subsequent pregnancies may be at risk
The correct answer is C
Explanation
When a person is Rh negative, this indicates that they do not have type
D antigen on their red blood cells. If a woman is exposed to Rh D
antigen-positive red blood cells, she can have an immune response of
variable strength. This may occur in the setting of pregnancy
(transplacental fetomaternal transfusion), or exposure outside of
pregnancy (e.g., transfusion with mismatched blood).
If a maternal antibody screen for D antigen is positive, this indicates
that the current fetus MAY be at risk for hemolytic disease. The level
of risk is detemined by the antibody titer. For example, an antibody
titer of 1:4 poses much less risk to the fetus than a titer of 1:64.
Determination of the blood type of the father is helpful if paternity is
certain. If the father is homozygous Rh negative, there is no risk of
alloimmunization to the fetus and the fetus is NOT at risk for hemolytic
disease. In this scenario, maternal sensitization occurred either from a
prior pregnancy with a different partner or from another source (e.g.
transfusion). If the father is heterozygous Rh positive, then the fetus
IS at risk. If paternity is uncertain, a polymerase chain reaction can
be performed on 2 mL of amniotic fluid or 5 mL of chorionic villi to
accurately determine the fetal Rh status.
Maternal parvovirus B19 poses which one of the following risks to the
fetus

a)
b)
c)
d)
e)

Hydropos
Laryngeal papillomas
Chronic active hepatitis
Seizures
Pneumonia

The correct answer is A


Explanation
Maternal parvovirus B19 infection poses several risks to the fetus and
newborn. The virus readily crosses the placenta, but transplacental
transmission is not known to cause congenital defects, even during early
pregnancy infections. Early pregnancy infection is associated with a
high rate of spontaneous abortion, usually in 1-10 weeks after maternal
infection. Infection later in pregnancy may destroy immature red blood
cells in the fetus, causing anemia, which may lead to non-immune hydrops
due to heart failure; it may also lead to fetal death. The degree of
fetal illness is unpredictable, and most women with prenatal infection
deliver healthy infants.
A pregnant patient is positive for hepatitis B surface antigen (HbsAg).
Which one of the following would be most appropriate for her infant?

a)
b)
c)
d)
e)

Hepatitis B immune globulin (HBIG) and hepatitis B vaccine at birth


Hepatitis B vaccine only, at birth
HBIG only, at birth
Testing for HbsAg before any immunization
No immunization until 1 year of age

The correct answer is A


Explanation
Infants born to hepatitis B-positive mothers should receive both immune
globulin and hepatitis B vaccine. They should receive the entire series
of the vaccine, with testing for seroconversion only after completion of
the vaccination series; the recommended age for testing is 9-12 months
of age.
During pregnancy, lactation is suppressed by the action of:

a) Insulin
b) Estrogen
c) Thyroid hormone (T4)
d) Human placental lactogen
e) Inhibin
The correct answer is B
Explanation
The high level of estrogen is inhibitory to the production of milk by
the breast despite the extremely high level of prolactin. After delivery
of the fetus and placenta, the level of estriol decreases until the
inhibitory effect is removed, at which time significant milk production
begins (2-3 days).
A. Insulin levels elevate in pregnancy. This hormone, however, does not
suppress lactation.
C. Total T4 levels rise in pregnancy due to an increase in thyroid
hormone binding globulin. Free T4 levels stay essentially unchanged.
D. HPL increases free fatty acids, allowing glucose and amino acids to
be conserved for use by the fetus.
E. Inhibin is produced by the placenta. Levels rise during pregnancy,
causing a suppression of maternal gonadotropins.
A 27 year old woman who is 15 weeks pregnant with her first child
presents to the office with exquisitely painful, blister-like lesions on
her labia. She had similar episodes before pregnancy. Her temperature is
normal. Which one of the following statements about her pregnancy is true?

a) She should undergo cesarean section to protect her infant from


infection
b)
c)
d)
e)
labour

Her fetus has an increased risk of congenital malformations


Transplacental transmission to her fetus is a significant concern
Breast-feeding of her infant is probably unsafe
Decisions regarding route of delivery are best made at onset of

The correct answer is E


Explanation
With genital herpes, if a woman does have a lesion or prodromal symptoms
at delivery, the safest practice is a cesarean delivery to prevent the
baby from coming into contact with active virus. Many women find that
their outbreaks tend to increase as the pregnancy progresses, probably
because of the immune suppression that takes place to prevent the
mother's body from rejecting the fetus. Between 10% and 14% of women
with genital herpes have an active lesion at delivery. The odds are
higher for women who acquire herpes during pregnancy, and lower for
women who have had herpes for more than six years.
If a woman doesn't have herpes lesions at the time of delivery, the
standard of care recommended is vaginal delivery. This does expose the
baby to a very small risk of infection from possible asymptomatic shedding.
A young couple undergo a postcoital test as part of an infertility
evaluation. Several hours after coitus, the cervical mucus is thick and
tenacious. No sperm are seen in the mucus, although they are present in
the vagina. Semen analysis is normal. Eight days later, the patient
menstruates. Her basal body temperature (BBT) record for that cycle
indicates ovulation and a normal luteal phase. The most appropriate
management of this patient is to:

a)
b)
c)
d)
e)

Perform antisperm antibody studies


Attempt intrauterine insemination with washed sperm
Prescribe low dose estrogen for days 7-14 of the cycle
Repeat the postcoital test 6-7 days earlier in the next cycle
Start clomiphene citrate therapy on days 5-9 of the cycle

The correct answer is D


Explanation
The luteal phase of the menstrual cycle is about 14 days in most women.
Differences in cycle length are due to differences in the proliferative
phase length. The reason that the cervical mucus is progestational is
that the test was done 6 days too late (A postcoital test is done at
ovulation, which occurs 14 days prior to the NEXT menstrual period). The
correct thing to do is to repeat the test 6 or 7 days earlier on the
next cycle.
A. Since the test was done at the wrong time of the cycle, no assessment
of male factor can be made at this time.
B. See answer to A.
C. Although estrogen may change the nature of the cervical mucus, the
test was done at the wrong time in the cycle.
E. The basal body temperature chart indicates ovulation (biphasic); so
there is no benefit at this time for clomiphene citrate therapy.
Which of the following statements regarding malignant cervical lesions
is true?

a) 95% are squamous cell carcinoma


b) CA-125 levels to monitor treatment effectiveness are indicated
c) The majority of lesions arise outside the transformation zone of
the cervix
d) They are not associated with HPV infection
e) Treatment for stage 4 disease radical hysterectomy with chemotherapy
The correct answer is A
Explanation
Cervical cancer is usually a squamous cell carcinoma that is caused by
human papillomavirus infection or an adenocarcinoma. Early cancer is
asymptomatic; the 1st symptom of later cancer is usually postcoital
vaginal bleeding. Diagnosis is by screening cervical Papanicolaou (Pap)
test and biopsy. Staging is clinical. Treatment usually includes
surgical resection, radiation therapy, and, unless cancer is localized,
chemotherapy; if cancer is widely metastasized, treatment is primarily
chemotherapy.

A 23 year old woman in her first trimester of pregnancy is diagnosed as


having deep venous thrombosis in her right lower extremity. Which one of
the following is the most appropriate therapy?

a)
b)
c)
d)
e)

Warfarin
Heparin
Clopidogrel
Dipyridamole
Streptokinase

The correct answer is B


Explanation
Deep venous thrombosis (DVT) is clotting of blood in a deep vein of an
extremity (usually calf or thigh) or the pelvis. DVT is the primary
cause of pulmonary embolism. DVT results from conditions that impair
venous return, lead to endothelial injury or dysfunction, or cause
hypercoagulability. DVT may be asymptomatic or cause pain and swelling
in an extremity.
Diagnosis is by history, physical examination, and duplex
ultrasonography, with d-dimer or other testing as necessary. Treatment
is with anticoagulants. In pregnancy, heparin is safe to give.
A 38 year old woman develops pruritus and an erythematous lesion on her
vulva with satellite lesions over the medial aspect of her thigh and
inguinal fold. She is diagnosed with candidiasis. What is the most
likely predisposing disease?

a)
b)
c)
d)

Diabetes Mellitus
Vulvar intraepithelial neoplasia
Lichen sclerosis
Pubic lice

The correct answer is A


Explanation
Candidal vaginitis is the most common form of mucosal candidiasis.
Vulvovaginal candidiasis is usually secondary to overgrowth of normal
flora Candida species in the vagina. Bacteria such as Lactobacillus
acidophilus balance Candida and prevent yeast overgrowth and pathogenic
infection.
Conditions that disrupt the balance of normal vaginal flora include
antibiotic use, oral contraceptives, contraceptive devices, high
estrogen levels, and immunocompromised states such as diabetes mellitus
and HIV.
For recurrent vulvovaginitis, patients should be screened for HIV
infection, diabetes mellitus, leukemia, or other immunologic dysfunction.
Which one of the following vaccines is safe to give during pregnancy?

a) Varicella
b) Influenza
c) Rubella
d) Pneumococcal vaccine
The correct answer is B
Explanation
Influenza vaccine should be given to all pregnant women who will be in
the second or third trimester during the influenza season. High-risk
patients should be vaccinated regardless of the trimester. The vaccine
is a killed virus preparation and its safety in pregnancy is
well-established. Rubella and varicella vaccines contain live attenuated
virus and are associated with adverse effects on the fetus. The safety
of the pneumococcal vaccine in pregnancy has not been evaluated.

Which one of the following fluoroquinolones should NOT be used in the


treatment of urinary tract infections

a) Ciprofloxacin (Cipro)
b) Gatifloxacin (Tequin)
c) Levofloxacin (Levaquin)
d) Moxifloxacin (Avelox)
e) Norfloxacin (Noroxin)
The correct answer is D
Explanation
When trimethoprim/sulfamethoxazole is contraindicated, a 3-day course of
ciprofloxacin, levofloxacin, norfloxacin, lomefloxacin, or gatifloxacin
is a reasonable alternative. Moxifloxacin attains inadequate urinary
concentrations and should not be used in the management of urinary tract
infections.
A 22 year old woman comes to the emergency department complaining of
sudden onset of severe cramping in the right lower quadrant. Her
temperature is 37.4?C (99.4?F), pulse is 90 bpm, and blood pressure is
100/70 mm Hg. The abdomen is tender to palpitation in the right lower
quadrant, and peritoneal signs are present. Pelvic examination reveals
an exquisitely tender 8-cm right adnexal mass. Urine pregnancy test is
negative. She continues to complain of unbearable pain. The most likely
diagnosis is:

a)
b)
c)
d)
e)

Appendicitis
Torsion of ovary
Ectopic pregnancy
Rupture of corpus luteum
Rupture of tuboovarian abscess

The correct answer is B


Explanation
Ovarian torsion may account for up to 3% of gynecologic emergencies
requiring operative intervention. This usually occurs in a reproductive
age woman who has an 8-12 cm benign mass of the ovary. Dermoids are the
tumors most associated when a torsion has occurred, but paratubal and
paraovarian cysts have the highest relative risk for torsion due to
their thin stalk. If it is a benign cystic mass, and the ovary is not
necrotic, the treatment of choice is a cystectomy with preservation of
the remaining ovarian tissue. Preoperative Doppler flow can be helpful
in determining the viability of the adnexa.
A. Although right lower quadrant pain is commonly due to appendicitis,
it is the sudden onset of cramping pain along with lack of a fever that
suggests ovarian torsion as the most likely etiology.
C. Urine pregnancy test being negative rules out ectopic.
D. Rupture of an ovarian cyst can cause peritoneal findings. With the
8-cm tender mass, a torsion is more likely.
E. It is unlikely that a woman would have been asymptomatic with a
tuboovarian abscess prior to its rupture.
Of the following, which is the most appropriate initial antibiotic
treatment for a tuboovarian abscess

a)
b)
c)
d)
e)

Clindamycin and ampicillin


Tetracycline and penicillin
Clindamycin and gentamicin
Ciprofloxacin and gentamicin
Ampicillin and gentamicin

The correct answer is C


Explanation
The CDC recommendations for the reatment of PID with intravenous
medications include (A) cefoxitin and doycycline, (B) clindamycin and
gentamicin. In treating an abscess, it is necessary to have anaerobic
coverage by agents such as clindamycin or metronidazole. Quinolones with
clindamycin can also be used. Aminoglycosides do not cover against

anerobes since their transport into the bacterial cell is coupled with
oxidative phosphorylation.
A. With this regimen, Gram-negative coverage may not be appropriate.
B. With this regimen, anaerobic coverage may not be appropriate.
D. With this regimen, anaerobic coverage may not be appropriate.
E. With this regimen, anaerobic coverage may not be appropriate.
A male infant weighing 3000 g (6 lb 10 oz) is born at 36 weeks
gestation, with normal Apgar scores and an unremarkable initial
examination. At 48 hours of age he is noted to have dusky episodes while
feeding, and does not feed well.
On repeat examination the child is tachypneic, with subcostal
retractions. Lung sounds are clear and there is no heart murmur. Pulse
oximetry on room air is 82%. Arterial blood gases on 100% oxygen show a
pCO2 of 26 mm Hg (N 27-40), a pO2 of 66 mm Hg (N 83-108), a blood pH of
7.50 mg/dL (N 7.35 - 7.45), and a base excess of 2 mmol/L (N 10 to 2).
Laboratory Findings
Hemoglobin---------------------22.0 g/dL (N 13.0-20.0)
Hematocrit---------------------66% (N 42-66)
WBCs---------------------19,000 mm3 (N 9000-30,000)
Chest radiograph---------------------increased vascular marking; large thymus
Blood culture results are pending.
Which one of the following is the most likely diagnosis

a)
b)
c)
d)
e)

Transient tachypnea of the newborn


Congenital heart disease
Hyaline membrane disease
Neonatal sepsis
Hyperviscosity syndrome

The correct answer is B


Explanation
Cyanotic congenital heart disease can appear at the time of ductus
closure. A heart murmur is not usually audible, and murmurs heard this
early are usually not due to heart disease. The failure to correct
hypoxemia with 100% oxygen is diagnostic for abnormal mixing of blood
from the right and left circulations.
Transient tachypnea presents earlier, and the hypoxia corrects with
supplemental oxygen. Hyaline membrane disease can occur at 36 weeks, but
would cause problems in the first hours of life. It can make oxygenation
difficult, but would cause extreme distress with CO2 retention in such
cases. This patient has the energy to hyperventilate and has slight
respiratory alkalosis as a result.
Neonatal sepsis can cause V/Q mismatching and hypoxia, and can have a
delayed presentation. Concern would be high enough in this case that the
patient would probably receive broad-spectrum antibiotics while awaiting
culture results. On the other hand, the clinician would not want to be
distracted from the evidence for congenital heart disease.
The baby is polycythemic from poor intake in the first 2 days of life.
The hyperviscosity syndrome can occur when the hematocrit is over 65%.
It can cause poor feeding, tachypnea, and sluggishness, but does not
cause hypoxia.
A 23 year old woman presents with a chief complaint of foul smelling
vaginal discharge. She is not sexually active. A wet mount and KOH prep
are performed demonstrating clue cells, What is the likely diagnosis

a) Candida
b) Bacterial vaginosis
c) Trichomonas

d) Chlamydia
The correct answer is B
Explanation
Bacterial vaginosis shows clue cells on a wet mount (which are
epithelial cells of the vagina that get their distinctive stippled
appearance by being covered with bacteria). KOH prep would release a
fishy odor.
Candida infection would present as thick, white, cottage cheese-like,
non-odorous discharge. Diagnostic exam would show the yeast (hyphae and
pseudo spores of Candida are visible) on the KOH prep.
Trichomonas is a sexually active disease. Presents with foamy, frothy
yellow-green discharge, fishy/foul odor. A wet mount will show the
Trichomonads, which are ovoid-shaped parasites, identified by their
mobility.
A postmenopausal woman comes to your office for advice because her best
friend has been diagnosed with endometrial cancer. The patient is
concerned that she too may develop the disease. You tell her that risk
factors associated with endometrial cancer include the following EXCEPT:

a) Nulliparity
b) Late menopause
c) DES exposure
d) Obesity
e) Polycystic ovarian disease
The correct answer is C
Explanation
DES exposure is associated with adenosis and clear cell adenocarcinoma
of the vagina and cervix. If a patient is nulliparous, obese, and
reaches menopause at age 52 or later, there appears to be a 5-fold
increase in the risk of endometrial cancer over the patient who does not
fulfill these criteria. Also upper body fat localization, which is
related to lower serum hormone-bound globulin and higher endogenous
production of nonprotein-bound estradiol, is a risk factor for
endometrial cancer.
A. Increased relative risk for developing endometrial adenocarcinoma.
B. Increased relative risk for developing endometrial adenocarcinoma.
D. Increased relative risk for developing endometrial adenocarcinoma due
to an increase in the circulating levels of estrogens (peripheral
conversion of androgens to estrogens by the adipose tissue).
E. Increased relative risk for developing endometrial adenocarcinoma due
to the long exposure to unopposed estrogen.
The probability of pregnancy after unprotected intercourse is the
highest at which one of the following times

a) 3 days before ovulation


b) 1 day before ovulation
c) The day of ovulation
d) 1 day after ovulation
e) 3 days after ovulation
The correct answer is B
Explanation
There is a 30% probability of pregnancy resulting from unprotected
intercourse 1 or 2 days before ovulation, 15% 3 days before, 12% the day
of ovulation, and essentially 0% 1-2 days after ovulation. Knowing the
time of ovulation therefore has implications not only for natural
family planning, but also for decisions regarding postcoital contraception.
A 72 year old woman complains of a lump protruding through the vagina
with local pressure symptoms. On examination, there is a visible uterine
prolapse. All of the following may be etiologic factors, except

a)
b)
c)
d)
e)

Multiparity
Chronic smoking
History of large babies
Stress urinary incontinence
Postmenopausal status

The correct answer is D


Explanation
Uterine prolapse is descent of the uterus toward or past the introitus.
Vaginal prolapse is descent of the vagina or vaginal cuff after
hysterectomy. Symptoms include vaginal pressure and fullness. Diagnosis
is clinical. Treatment includes reduction, pessaries, and surgery.
A prolapsed uterus is graded based on level of descent: to the upper
vagina (1st degree), to the introitus (2nd degree), or external to the
introitus (3rd degree or total, sometimes referred to as procidentia).
Vaginal prolapse may be 2nd or 3rd degree.
Pregnancy and trauma incurred during childbirth, particularly with large
babies or after a difficult labor and delivery, are the main causes of
muscle weakness leading to uterine prolapse. Loss of muscle tone
associated with aging and reduced amounts of circulating estrogen after
menopause also may contribute to uterine prolapse. In rare
circumstances, uterine prolapse may be caused by a tumor in the pelvic
cavity.
Smoking (which causes chronic cough) and genetics also may play a role.
Women of Northern European descent have a higher incidence of uterine
prolapse than do women of Asian and African descent.
Symptoms tend to be minimal with 1st-degree uterine prolapse. In 2nd- or
3rd-degree uterine prolapse, fullness, pressure, and a sensation of
organs falling out are common. Diagnosis is confirmed by speculum or
bimanual pelvic examination.
You are asked to consult on a young woman with a preexisting cardiac
defect. She wants to become pregnant in the near future and seeks advice
about what risks to her health that this will create. You tell that the
highest maternal mortality rates are associated with which of the
following cardiac defects:

a)
b)
c)
d)
e)

Aortic stenosis
Mitral stenosis
Ebstein anomaly
Atrial-septal defect
Eisenmenger syndrome

The correct answer is E


Explanation
Eisenmengers syndrome is one where there is communication between the
systemic and pulmonary system, along with increased pulmonary vascular
resistance, either to systemic level or above systemic level (right to
left shunt). A would-be mother must be informed that to become pregnant
would incur a 50% risk of dying. Even if she survives, fetal mortality
approaches 50% as well.
A. Severe symptomatic aortic stenosis has a mortality in pregnancy of
about 20%. Prevention of reduction in preload is necessary in all
obstructive cardiac lesions. Balloon valvuloplasty can be done in pregnancy.
B. Due to the increased blood volume and cardiac output in pregnancy,
mitral stenosis can lead to severe pulmonary edema. Balloon
valvuloplasty can be done in pregnancy.
C. Ebstein anomaly is a malformation of the tricuspid valve. It is
usually not associated with maternal mortality.
D. Atrial-septal defects rarely cause complications in pregnancy, labor,
or delivery.
A G1P0 woman comes to the labor and delivery department with regular
painful contractions. On exam, a footling presentation found. What is
the appropriate management?

a)
b)
c)
d)

Expectant vaginal delivery


Tocolysis
Cesarean section
Augmentation of labor with oxytocin

The correct answer is C


Explanation When a gravid woman goes into labor her baby can be in a
variety of positions. See picture:
Of these the most appropriate for delivery is the vertex face position.
Occasionally we see one of the other presentations. A footling
presentation is seen here:

This is a presentation with the feet entering the birth canal ahead of
any other part of the body. This may occur with two feet (double
footling) or a single foot (single footling). Most often one leg is
extended while the other is flexed at the knee. It is usually safer to
deliver this kind of baby by a Cesarean section early in labor or before
labor begins. If a footling breech is delivered vaginally, there is a
risk that the head may not easily pass through the birth canal.

A 44-year-old female is distressed because of incontinence. She reports


frequent episodes of an immediate need to urinate, which cannot always
be deferred. She admits to urinating more than 10 times a day, but
denies any urine leakage with coughing, laughing, or straining. Which
one of the following is the most appropriate initial treatment for this
patient?

a)
b)
c)
d)
e)

Solifenacin (Vesicare)
Oxybutynin (Ditropan XL)
Tamsulosin (Flomax)
Phenazopyridine (Pyridium)
Pelvic floor muscle training and bladder training

The correct answer is E


Explanation
Nonpharmacologic therapy is recommended for all patients with an
overactive bladder. Pelvic floor muscle training (e.g., Kegel exercises)
and bladder training are proven effective in urge incontinence or
overactive bladder, as well as in stress and mixed incontinence. In
motivated patients, training may be more effective than medications such
as oxybutynin and newer muscarinic receptor antagonists such as
solifenacin. Tamsulosin is used in benign prostatic hypertrophy and
phenazopyridine is a urinary tract anesthetic that has not been
recommended for treating overactive bladder.
Which one of the following is not an initial screening investigation for
infertility?

a)
b)
c)
d)
The correct answer is D

History and physical examination


Semen analysis
Basal body temperature charting
Hysteroscopy

Explanation
The process to diagnose the cause for infertility usually begins with
physical exams and health and sexual histories. If there are no obvious
problems, like poorly timed intercourse or absence of ovulation, tests
will be needed.
For a man, doctors usually begin by testing his semen. They look at the
number, shape, and movement of the sperm. Sometimes doctors also suggest
testing the level of a man's hormones.
For a woman, the first step in testing is to find out if she is
ovulating each month. There are several ways to do this. A woman can
track her ovulation at home by recording changes in her morning body
temperature (basal body temperature) for several months. Doctors can
also check if a woman is ovulating by doing blood tests and an
ultrasound of the ovaries. If the woman is ovulating normally, more
tests are needed.

Hysterosalpingography and laparoscopy can be done after the initial


tests if necessary.
Which one of the following is true concerning nausea and vomiting in
pregnancy?

a) Psychological factors play a causative role


b) Pharmacologic therapy, in general, is no more effective than
placebo in relieving symptoms and preventing hospitalization
c) Metoclopramide (Reglan) is contraindicated in the first trimester
but is safe and effective for nausea and vomiting in later trimesters
d) If nausea and vomiting began after 9 weeks gestation, secondary
causes are more likely to be presented
The correct answer is D
Explanation
While the exact etiology of nausea and vomiting in pregnancy remains
unclear, there are few data to support the theory that psychological
factors play a role. Although nausea is usually self-limited condition,
other causes must be ruled out. Secondary causes are more likely to be
present if the onset of symptoms occurs after 9 weeks gestation. Several
pharmacologic treatments are proven safe and are superior to placebo in
relieving symptoms and preventing hospitalization. Metoclopramide is
more effective than placebo and has not been associated with an
increased risk of adverse effects on the fetus.
A 24 year old woman with 6 weeks amenorrhea develops continuous lower
abdominal pain and minimal vaginal bleeding. The uterus is slightly
enlarged, the cervix is soft and tender on motion. There is adnexal
tenderness. The history and clinical findings are most suggestive of

a)
b)
c)
d)
e)

Salpingitis
Ruptured ovarian cyst
Ectopic pregnancy
Threatened abortion
Twisted ovarian cyst

The correct answer is C


Explanation
In ectopic pregnancy, implantation occurs in a site other than the
endometrial lining of the uterine cavity, for example in the fallopian
tube, uterine interstitium, cervix, ovary, or abdominal or pelvic
cavity. Ectopic pregnancies cannot be carried to term and eventually
rupture or involute.
Early symptoms and signs include pelvic pain, vaginal bleeding, and
cervical motion tenderness. Syncope or hemorrhagic shock can occur with
rupture.

Diagnosis is by B-human chorionic gonadotropin measurement and

ultrasonography. Treatment is with laparoscopic or open surgical


resection or with IM methotrexate.
A 20 year old patient has urinary frequency and dysuria. Pelvic
examination reveals a yellow discharge at the cervix and mild adnexal
tenderness. The best immediate test to aid your diagnosis is:

a) Gram stain of a cervical smear


b) Peripheral leukocyte count
c) Cervical smear for /Chlamydia trachomatis/
d) Cervical culture on Thayer Martin medium
e) Dipstick urinalysis of specimen obtained by bladder catheterization
The correct answer is A
Explanation
A mucopurulent cervicitis, along with urinary symptoms, makes one highly
suspicious for Neisseria gonorrhoeae. Although all of the above tests
would be helpful in determining the diagnosis, the only test that can
aid you immediately is a Gram stain. The Gram stain has a sensitivity of
only 60%, but a specificity of 95%. The key finding is a Gram-negative
intravellular diplococcus.
B. Often with a gonococcal cervicitis, the peripheral white blood cell
count is normal.
C. Chlamydial testing of the cervical smear is not an immediate test.
D. Cervical culture on Thayer Martin medium is confirmatory, but is not
an immediate test.
E. The urinary symptoms most likely are due to gonococcal urethritis as
well as a cervicitis. Routine dipstick urinalysis will not identify the
pathogen involved.
Which one of the following should be avoided in the treatment and
prophylaxis of migraine during early pregnancy?

a) Calcium channel blockers


b) Beta-Blockers
c) Triptans
d) NSAIDs
The correct answer is C
Explanation
Headaches, and migraines in particular, are very common in women of
childbearing age. Migraine sufferers usually have improvement of
symptoms in pregnancy and many have complete remission. Most medications
used for prophylaxis and abortive treatments of migraines in the
nonpregnant patient can also be used in pregnant patients. Most
Beta-blockers and calcium channel blockers are safe. Acetaminophen and
narcotics can be used for acute pain. Ibuprofen can also be used but
should be avoided late in pregnancy because it is associated with
premature closure of the ductus arteriosus and oligohydramnios.
Ergotamines should be avoided as they are uterotonic and have
abortifacient properties. They have also been associated with case
reports of fetal birth defects. Triptans have the potential to cause
vasoconstriction of the placental and uterine vessels and should be used
only if the benefit clearly outweighs the harm.
In normal pubertal development, which of the following is true?

a)
b)
c)
d)
e)

Pubarche usually precedes thelarche


Menarche usually precedes pubarche
Menarche usually precedes peak height velocity
Regular ovulatory cycles occur about 2 years after menarche
Ovulation must occur before menses can begin

The correct answer is D


Explanation
The normal stages of pubertal development in order are: thelarche
(breast budding), pubarche (sexual hair), peak height velocity, and
menarche.

The age of pubertal change has been getting earlier, with breast
development starting between the ages of 10 and 11 and menarche between
the ages of 12 and 13. The mean interval from thelarche to menarche is
2.3 years, with a standard deviation of 1 year. If no secondary sexual
characteristics occur by age 14, or no menarche by age 16.5, then the
diagnostic workup of primary amenorrhea is necessary.
A. Thelarche is before pubarche.
B. Menarche is the final event in the process.
C. Peak height velocity is about 1 year before the onset of menarche.
E. Often the first bleeding is not ovulatory.
A 36 year old white female presents with the chief complain of
infertility associated with a history of a menstrual period every 3-4
months since menarche at age 12. The physical examination is normal
except for moderate obesity, acne, and coarse facial hair. A urine hCG
is negative. Further evaluation to confirm your diagnosis of polycystic
ovary syndrome will most likely reveal

a)
b)
c)
d)

Elevated testosterone
Elevated TSH
Markedly elevated prolactin
Elevated 17-hydroxyprogesterone

The correct answer is A


Explanation
This patient presented with classic symptoms of polycystic ovary
syndrome (PCOS) oligomenorrhea, infertility, hirsutism, and
acne-reflecting hyperandrogenic anovluation. The underlying
pathophysiology of PCOS includes insulin resistance leading to increased
ovarian androgen production. LH and FSH levels are often elevated in
PCOS, with the LH:FSH ratio often being greater than 3:1. Prolactin is
usually normal, although mild elevations are possible. Marked elevations
should prompt investigation of other causes. Elevated
17-hydroxyprogesterone levels are seen in congenital adrenal
hyperplasia, and high TSH values would point to hypothyroidism as a
cause for this patients oligomenorrhea.
A woman at 39 weeks gestation suddenly develops vaginal bleeding and the
fetal rate drops to 80 beats/min, and is non-reassuring. What should
you do?

a)
b)
c)
d)

Amniotomy
Ultrasound for placenta location
C-section
Monitoring

The correct answer is C


Explanation
Vasa previa is an obstetric complication defined as "fetal vessels
crossing or running in close proximity to the inner cervical os. These
vessels course within the membranes (unsupported by the umbilical cord
or placental tissue) and are at risk of rupture when the supporting
membranes rupture."
Vasa previa is present when fetal vessels traverse the fetal membranes
over the internal cervical os. These vessels may be torn at the time of
labor, delivery or when the membranes rupture. It has a high fetal
mortality because of the bleeding that follows.
The classic triad are membrane rupture followed immediately by painless
vaginal bleeding and fetal bradycardia.
The diagnosis is usually confirmed after delivery on examination of the
placenta and fetal membranes. Treatment immediately with an emergency
cesarean delivery is usually indicated.
A couple are unable to conceive after 1 year. Of the following
statistics regarding infertility, which is correct

a)
b)
c)
d)
e)

10-15% of couples are infertile


Only 40% of couples achieve pregnancy within 6 months of trying
Only 20% achieve pregnancy within 1 year of trying
Male factors are responsible for over half of infertility causes
Infertility is labeled after failure to conceive after 1 year of

trying
The correct answer is E
Explanation
Infertility is the inability of a couple to achieve a pregnancy after
repeated intercourse without contraception for 1 year.
Infertility affects about one of five couples in the Canada. It is
becoming increasingly common because people are waiting longer to marry
and to have a child. Nevertheless, up to 60% of the couples who have not
conceived after a year of trying do conceive eventually, with or without
treatment. The goal of treatment is to reduce the time needed to
conceive or to provide couples who might not otherwise conceive the
opportunity to do so. Before treatment is begun, counseling that
provides information about the treatment process (including its
duration) and the chances of success is beneficial.
The cause of infertility may be due to problems in the man, the woman,
or both. Problems with sperm, ovulation, or the fallopian tubes each
account for almost one third of infertility cases. In a small percentage
of cases, infertility is caused by problems with mucus in the cervix or
by unidentified factors. Thus, the diagnosis of infertility problems
requires a thorough assessment of both partners.
A one year old female presents with vaginal bleeding. Vaginal inspection
reveals the presence of a multicystic grape-like lesion. The most likely
diagnosis is

a)
b)
c)
d)
e)

Sexual abuse
DES syndrome
Sarcoma botyroides
Clear cell adenocarcinoma
Exposure to exogenous estrogen

The correct answer is C


Explanation
Sarcoma botryoides, also known as embryonal rhabdomyosarcoma, is a rare
vaginal cancer. It occurs in the vagina of female infants and children,
typically younger than age 8. The name comes from the gross appearance
of "grape bunches". The most common clinical finding is vaginal bleeding
but vaginal bleeding is not specific for sarcoma botryoides: other
vaginal cancers are possible.
Which one of the following indicates that a patient has entered the
second stage of labor?

a)
b)
c)
d)
e)

A small amount of bloody, mucous discharge from the cervix (bloody show)
Braxton Hicks contractions
Spontaneous rupture of the chorioamnionic membranes
Complete dilation of the uterine cervix
Successful delivery of the placenta

The correct answer is D


Explanation
For many women, labor will be preceded by several hours, or even days,
by bloody show. So-called false labor, or Braxton Hicks
contractions, consists of weak, irregular, regional contractions that
usually occur for weeks before the onset of actual labor and abate with
time, analgesia and sedation. Spontaneous chorioamnionic membrane
rupture precedes the onset of labor in about 10% of pregnancies, and
amniotic fluid leaks through the cervix and out the vagina.
The second stage of labor is defined as the period from complete cervix

dilation to complete delivery of the baby. When the cervix is completely


dilated, the patient usually experiences the urge to push with
contractions. The third stage of labor begins with the delivery of the
baby and ends with the delivery of the placenta.
True statements concerning the diagnosis of an adnexal mass are the
following EXCEPT:

a) The diagnosis varies with the age of the patient


b) In patients in the reproductive age period, a cystic mass larger
than 5 cm should be explored immediately
c) In premenarchal patients, most neoplasms are germ cell in origin
and require surgical exploration
d) In postmenopausal women, enlargement of the ovary is abnormal and
should be considered malignant until proven otherwise
e) In patients in the reproductive age period, a solid mass larger
than 8 cm should be explored
The correct answer is B
Explanation
Adnexal masses can be found in all age groups, though there are
differences in the prevalence of certain conditions based on the
patients age. After menopause, the ovaries become nonpalpable, so any
enlargement should make one suspicious for cancer. In the reproductive
age, functional cysts are common. These can be up to 6 cm or more, so
any simple cyst less than 6 cm can be observed for several cycles. If
still present, a workup is indicated.
A. Various etiologies for adnexal masses have different incidences
depending on the patients age. Germ cell neoplasms are more likely to
be found in younger women. Ectopics will be found only in reproductive
age women. Epithelial malignancies are more common in postmenopausal women.
C. See answer to A.
D. Any palpable ovary in a postmenopausal woman needs to be evaluated to
rule out a malignancy.
E. Any solid mass of the ovary needs to be evaluated to rule out a
malignancy.
What is the most common management of carpal tunnel syndrome in pregnancy?

a)
b)
c)
d)

Diuretics
Splint
Avoid activities that precipitate it
Fasciotomy

The correct answer is B


Explanation
The carpal tunnel is a bony canal formed by the wrist bones on three
sides and a ligament that runs across the wrist on the other. The
swelling and fluid retention that's so common during pregnancy can
increase the pressure in this relatively narrow and inflexible space,
compressing the median nerve that runs through it.
The median nerve gives sensation to the thumb and the index, middle, and
half of the ring finger and is responsible for movement of a muscle at
the base of the thumb. Pressure on this nerve is what causes the symptoms.
To relieve the pain it is important to avoid any activity that requires
forceful, repetitive hand movements. Although these movements may not
have caused the carpal tunnel syndrome, they can make symptoms worse.
Accumulation of fluid is the likely etiology of carpal tunnel
syndrome in pregnant women. Symptoms may first be noted during any of
the three trimesters, but the diagnosis is most often made during the
final trimester and resolves with delivery.
Since its not due to repetitive movements and is more associated with
the increase of fluid, simply "avoiding repetitive or agrivating
movements" won't actually help much, and a better treatment would be
night splinting.

A 30-year-old gravida 3 para 2 at 28 weeks gestation is a restrained


passenger in a high-speed motor vehicle accident. After initial
stabilization in the field with supplemental oxygen and intravenous
fluids, she is brought into the emergency department on a backboard and
wearing a cervical collar.
Until you are able to rule out a spinal injury, in what position should
the patient be kept?

a) Supine
b) Supine, with the uterus manually deflected laterally
c) Prone
d) Trendelenburgs position
e) Left lateral decubitus
The correct answer is B
Explanation
In general, it is best to place a woman who is greater than 20 weeks
pregnant in the left lateral decubitis position because the uterus can
compress the great vessels, resulting in decreased systolic blood
pressure and uterine blood flow.
However, in the case of trauma where a spinal cord injury cannot be
ruled out, the woman needs to be kept supine on a backboard. The weight
of the uterus can be shifted off the great vessels by either manual
deflection laterally or by elevating the right hip 4-6 inches by placing
towels under the backboard. The Trendelenburg position does not relieve
the weight of the uterus on the great vessels. The prone position does
not provide adequate spinal cord protection, and would be extremely
awkward in a large pregnant woman.
A 30-year-old female presents with concerns about vaginal bleeding. She
states that her menstrual periods have occurred at regular intervals of
28-30 days for the past 15 years, but recently bleeding has also
occurred for a day or two in the middle of her cycle. This bleeding has
been heavy enough to require the use of multiple pads.
Which one of the following terms best describes her bleeding pattern?

a) Polymenorrhea
b) Mid-cycle spotting
c) Metrorrhagia
d) Menometrorrhagia
e) Acute emergent abnormal uterine bleeding
The correct answer is C

Explanation
This patient has metrorrhagia, or bleeding intermenstrual,
characterized by bleeding heavy enough to require the use of multiple
pads; the heavy bleeding occurs between normal menstrual bleeding. It is
important to evaluate metrorrhagia because potential causes include
cervical disease, problems with IUDs, endometritis, polyps, submucous
myomas, endometrial hyperplasia, and cancer. Mid-cycle spotting, as the
term implies, refers to light spotting, and is often caused by a decline
in estrogen levels. Polymenorrhea is bleeding occurring at intervals of
less than 21 days. Menometrorrhagia is heavy and/or prolonged bleeding
occurring at irregular, noncyclic intervals. Acute emergent abnormal
uterine bleeding is characterized by significant blood loss resulting in
hypovolemia.
A G5P5 post-delivery develops a postpartum hemorrhage. What is the most
likely cause?

a) Vaginal laceration
b) Chorioamnionitis
c) Uterine atony
d) Uterine inversion
The correct answer is C
Explanation
This patient having five deliveries most likely has uterine atony.
Postpartum hemorrhage commonly results from bleeding at the placental
implantation site. Risk factors for bleeding at this site include
uterine atony due to overdistention caused by multifetal pregnancy,
polyhydramnios, or an abnormally large fetus, prolonged or dysfunctional
labor, grand multiparity (delivery of ? 5 viable fetuses), relaxant
anesthetics, rapid labor, chorioamnionitis, and retention of placental
tissue (eg, due to placenta accreta).
Other possible causes of hemorrhage?lacerations of the genital tract,
extension of an episiotomy, or uterine rupture?must also be considered.
Uterine fibroids may contribute. Postpartum hemorrhage due to
subinvolution (incomplete involution) of the placental site usually
occurs early but may occur as late as 1 month after delivery.
A 23 year old asymptomatic woman is seen for routine examination. You
are able to palpate a 4 cm diameter right-sided cystic adnexal mass. The
appropriate management is

a)
b)
c)
d)
e)

Laparoscopy
Trans-vaginal aspiration
Pelvic ultrasound examination
Gonadotropin-releasing hormone (GnRH) agonist
Reassessment in 1 month

The correct answer is C


Explanation
Adnexal masses are frequently found in both symptomatic and asymptomatic
women. In premenopausal women, physiologic follicular cysts and corpus
luteum cysts are the most common adnexal masses, but the possibility of
ectopic pregnancy must always be considered. Other masses in this age
group include endometriomas, polycystic ovaries, tubo-ovarian abscesses
and benign neoplasms.
Malignant neoplasms are uncommon in younger women but become more
frequent with increasing age. In postmenopausal women with adnexal
masses, both primary and secondary neoplasms must be considered, along
with leiomyomas, ovarian fibromas and other lesions such as diverticular
abscesses.
Information from the history, physical examination, ultrasound
evaluation and selected laboratory tests will enable you to find the
most likely cause of an adnexal mass.
Which one of the following Papanicolaou (Pap) test results is most

likely to indicate a cancerous lesion?

a) Atypical squamous cells of undetermined significance (ASC-US)


b) Atypical squamous cells cannot exclude high-grade intraepitherlial
lesion (ASC-H)
c) Atypical glandular cells not otherwise specified (AGC-NOS)
d) Low-grade squamous intraepithelial lesion (LSIL)
e) High-grade squamous intraepithelial lesion (HSIL)
The correct answer is C
Explanation
Papanicolaou (Pap) tests are intended to screen for cervical cancer, but
abnormal Pap tests are associated with precancerous lesions or with no
abnormality. The category of atypical glandular cells not otherwise
specified (AGC-NOS) has a benign sound to it, although it is associated
with a 17% rate of cancer (8% carcinoma in situ, and 9% invasive
carcinoma). High-grade squamous intraepithelial lesion (HSIL), which
would seem worse intuitively, has only a 3% associated cancer rate.
AGC-NOS is associated with higher rates of cancer than the other choices
listed.
During the midluteal phase, progesterone secretion occurs immediately
after pulsatile secretion of:

a)
b)
c)
d)
e)

Follicle stimulating hormone (FSH)


Luteinizing hormone (LH)
Estradiol
Inhibin
Activin

The correct answer is B


Explanation
Beginning in the midluteal phase, progesterone is secreted in a
pulsatile fashion, occurring immediately following an LH pulse. Prior to
ovulation, progesterone levels are less than 1 ng/ml, but reach a
midluteal level of 10-20 ng/ml.
A. See explanation above. FSH stimulates the production of estrogen as
well as the production of FSH receptors.
C. See explanation above. When peak levels of estradiol are achieved,
the onset of the LH surge then occurs.
D. See explanation above. FSH stimulates secretion of inhibin from
granulose cells and, in turn, is suppressed by inhibin.
E. See explanation above. Activin is related to inhibin, but has an
opposite effect (stimulates FSH release and GnRH receptor number).
A 25-year-old female at 30 weeks gestation has a blood pressure of
140/90 mm Hg (baseline 135/85 mm Hg) and urine positive for 1+ protein.
This patient should be evaluated with a

a)
b)
c)
d)

Serum uric acid level


A 24-hour urine protein level
Calcium level
Magnesium level

The correct answer is B


Explanation
This patient may have preeclampsia. The current diagnostic criteria
require a blood pressure ?140/90 on repeated measurements and
proteinuria >300 mg/24 hr, which usually corresponds to a urine protein
of 1+ or greater on a urine dipstick test. A 24-hour urine collection is
the preferred means of making the diagnosis, along with serial blood
pressure measurements. Previous guidelines include edema in the
diagnostic criteria, and the elevated blood pressure criteria could be
met with an elevation of 30 mm Hg systolic or 15 mm Hg diastolic above
the baseline prepregnancy blood pressure. Elevated serum uric acid is
commonly seen in preeclampsia but lacks sensitivity and specificity in
making this diagnosis. Calcium levels and magnesium levels are not

diagnostic. Once the diagnosis of preeclampsia is made there are several


laboratory tests that are recommended weekly: hemoglobin, hematocrit,
platelets, serum creatinine, serum transaminases, serum lactic
dehydrogenase, serum albumin, coagulation profiles, and peripheral blood
smears. In addition, repeat 24-hour urine collections are advised, since
rising 24-hour urine protein levels are worrisome. Urine protein
excretion >5000 mg/24 hr is by itself diagnostic of severe preclampsia.
A 37 year old woman has severe pain during menses which radiates into
the anal region. On bimanual exam, she has a small, retroverted uterus
and tender nodules can be felt on palpation of the uterosacral
ligaments. Which of the following conditions does she most likely have

a) Chronic PID
b) Adenomyosis
c) Fibroids
d) Endometriosis
e) Uterine carcinoma
The correct answer is D
Explanation
Endometriosis is a condition in which bits of tissue from the lining of
the uterus (endometrium) grow outside the uterus. The endometrial
tissue, called an endometrial implant, usually adheres to the pelvic
organs, which include the ovaries, uterus, fallopian tubes, the cavity
behind the uterus, and the ligaments that support the uterus.
Endometrial implants may also adhere to the tubes leading from the
kidneys to the bladder, the bladder, the vagina, the outer surface of
the small and large intestine, or the lining of the chest cavity.
Endometriosis progresses slowly. So, a woman may have the disease but
not experience any symptoms for years. There is no link between the
severity of the symptoms and the severity of the disease. Some women
have great pain, but very few endometrial implants. The most common
symptoms include pain in the lower abdomen during the menstrual period,
spotting between periods, heavy or irregular periods, pain during
intercourse, pain during bowel movements or urination, and lower back
pain. Many women with endometriosis experience infertility because the
endometrial implants block the passage of the egg from the ovary to the
uterus.
Diagnoses is aided by a pelvic exam to feel for endometrial implants.
The best way to diagnosis this disease, however, is through a procedure
called laparoscopy.
A 16-year-old woman presents to the emergency department complaining of
severe left-sided pelvic pain and vaginal spotting. Her last menstrual
period was 6 weeks ago. A quantitative beta HCG is 9.000 mIU/ml. An
endovaginal ultrasound notes a complex left adnexal mass, moderate free
fluid, and no evidence of an intrauterine sac. The most likely site of
this pregnancy is:

a)
b)
c)
d)
e)

Cervix
Uterine cornua
Isthmus of the fallopian tube
Ampulla of the fallopian tube
Fimbria of the fallopian tube

The correct answer is D


Explanation
The most likely site of an ectopic is the ampulla. Cervical, ovarian,
and abdominal ectopics are very rare. Conual ectopics often present
later, and the rupture can be much more catastrophic due to the
vascularity of this portion of the uterus.

A. Cervical pregnancies can lead to massive bleeding. The cervix will


often feel very large and can be tender. Methotrexate therapy may
decrease the need for hysterectomy.
B. Cornual pregnancies often present later in gestation. When these
rupture, due to their size and location near branches of the uterine
arteries, the blood loss can be acute and massive.
C. Less common than ampullar ectopics.
E. See answer to C.
A 23-year-old female is at 8 weeks gestation with her first pregnancy.
She is planning to travel to Third World countries soon for job-related
reasons.
Which one of the following is contraindicated in this patient?

a)
b)
c)
d)
e)

Hepatitis A vaccine
Hepatitis B vaccine
Meningococcal vaccine
Rabies vaccine
Varicella vaccine

The correct answer is E


Explanation
Immunizations that are safe during pregnancy include tetanus and
diphtheria toxoids (Td), hepatitis B vaccine, influenza vaccine,
meningococcal vaccine, and rabies vaccine. Those that are
contraindicated, or whose safety has not yet been established, include
mumps, measles, rubella, and varicella vaccines.
Vaccines to which special recommendations pertain include anthrax,
hepatitis A, Japanese encephalitis, pneumococcal, polio (IPV), typhoid,
vaccinia, and yellow fever vaccines. In the case of hepatitis A, the
vaccine is produced from an inactivated virus and risk to the developing
fetus is low. The risk of vaccination should be weighed against the risk
of exposure to hepatitis A.
A 26 year old white female presents to you 5 weeks after her last
menstrual period with complaint of painless spotting for 2 days. She has
not been using contraception and is trying to conceive. She had regular
menses until her last menstrual period, and a home pregnancy test was
positive 5 days ago. Her only previous pregnancy, 4 years ago, was
electively terminated at 8 weeks gestation. Her past history is
significant for an episode of presumed salpingitis treated with
intravenous antibiotics 2 years ago.
Physical Findings

Abdomen----------------------nontender
Vagina----------------------small amount of dark blood in the posterior fornix
Cervix----------------------no active bleeding
Bimanual----------------------uterus slightly enlarged, adnexa nontender, no
masses
Laboratory Findings
Urine pregnancy test----------------------positive
Beta-hCG----------------------1400 mIU/mL
Vaginal ultrasound----------------------3-mm sac with no definite fetal contents,
adnexa negative
The most appropriate action at this point is to

a)
b)
c)
d)

Refer the patient for laparoscopy because of the possibility of ectopic pregnancy
Perform dilation and curettage
Instruct the patient to return in 4 weeks
Instruct the patient to call if she develops pain or increased bleeding; otherwise

repeat the Beta-hCG in 2-3 days


e) Administer progesterone, 100 mg intramuscularly, and repeat the Beta-Hcg in 2-3
days
The correct answer is D
Explanation
In a pregnancy complicated by bleeding at less than 8 weeks gestation,
it is often difficult to determine whether there is a viable
intrauterine pregnancy, a nonviable intrauterine pregnancy, or an ecopic
pregnancy. Vaginal ultrasonography can be used to detect intrauterine
pregnancy as early as 1 week after conception if the Beta-hCG level is >
1500 mIU/mL. An empty uterus with a Beta-hCG level > 1500 mIU/mL is
highly suggestive of ecoptic pregnancy. When the Beta-hCG is < 1500
mIU/mL and vaginal ultrasonography is nondiagnostic, the Beta-hCG should
be repeated in 2-3 days. Failure of the Beta-hCG to double in 2-3 days
suggests a blighted ovum or ectopic pregnancy. The efficacy of
progesterone in early pregnancy has not been proven, and there is some
potential risk (virilization of the female fetus). An ectopic pregnancy
is possible in this situation, but would be more like if the hCG were
higher in the presence of an empty uterus, or if there were an adnexal
mass on physical examination or ultrasonography.
On colposcopy, all of the following are true about the transformation
zone, except

a)
b)
c)
d)

It lies between the ?original? and ?new? squamocolumnar junctions


Columnar epithelium is replaced by squamous
Involves squamous metaplasia
Squamos epithelium is replaced by columnar

The correct answer is D


Explanation
The cervical transformation zone lies between the ?original? and ?new?
squamocolumnar junctions (SCJ). The SCJ is the visible border between
the squamous and columnar epithelia of the cervix and represents the new
squamocolumnar junction.

It is adjacent to the new SCJ that the dynamic process of squamous


metaplasia occurs throughout the reproductive years. This is a normal
process during which columnar epithelium is replaced by squamous. The

trigger for this process is thought to be the eversion of the columnar


epithelium under the influence of estrogen and its subsequent exposure
to the acidic vaginal pH. In response to the ?insult? of vaginal
acidity, the process of metaplasia replaces the more fragile columnar
epithelium with the more sturdy squamous type.
A 16-year-old female presents with a complaint of pelvic cramps with her
menses over the past 2 years. She describes her periods as heavy, and
says they occur once a month and last for 7 days, with no spotting in
between. She has never been sexually active and does not expect this to
change in the foreseeable future. An abdominal examination is normal.
Which one of the following would be the most appropriate next step?

a)
b)
c)
d)

A pelvic examination
Ultrasonography
A TSH level
Naproxen prior to and during menses

The correct answer is D


Explanation
This patient is experiencing primary dysmenorrhea, a common finding in
adolescents, with estimates of prevalence ranging from 20% to 90%.
Because symptoms started at a rather young age and she has pain only
during menses, endometriosis or other significant pelvic pathology is
unlikely. An infection is doubtful, considering that she is not sexually
active and that symptoms have been present for 2 years. In the absence
of red flags, a pelvic examination, laboratory evaluation, and pelvic
ultrasonography are not necessary at this time. However, they can be
ordered if she does not respond to simple treatment.
NSAIDs such as naproxen have a slight effect on platelet function, but
because they inhibit prostaglandin synthesis they actually decrease the
volume of menstrual flow and lessen the discomfort of pelvic cramping.
Acetaminophen would have no effect on prostaglandins.
A 24 year old with 6 weeks' amenorrhea develops continuous lower
abdominal pain and minimal vaginal bleeding. The uterus is slightly
enlarged, the cervix is soft and tender on motion. There is adnexal
tenderness. The history and clinical findings are most suggestive of

a)
b)
c)
d)
e)

Ectopic pregnancy
Salpingitis
Ruptured ovarian cyst
Threatened abortion
Twisted ovarian cyst

The correct answer is A


Explanation
In ectopic pregnancy, implantation occurs in a site other than the
endometrial lining of the uterine cavity, in the fallopian tube, uterine
interstitium, cervix, ovary, or abdominal or pelvic cavity. Ectopic
pregnancies cannot be carried to term and eventually rupture or
involute. Early symptoms and signs include pelvic pain, vaginal
bleeding, and cervical motion tenderness. Syncope or hemorrhagic shock
can occur with rupture.
Diagnosis is by ?-human chorionic gonadotropin measurement and
ultrasonography. Treatment is with laparoscopic or open surgical
resection or with IM methotrexate.
An 18 year old woman who has a height of 158 cm, and normal breast
development, presents with complaint of primary amenorrhea. Physical
exam does demonstrate a small uterus. All of the following should be
done to investigate, *except*

a)
b)
c)
d)

TSH and Prolactin levels


Progesterone test
FSH and LH levels
Laparoscopy

e) Karyotype
The correct answer is D
Explanation
Amenorrhea is absence of menstruation. The cause is usually endocrine
dysfunction resulting in anovulation, often with mild estrogen
deficiency and hyperandrogenism. Diagnosis is clinical and by pregnancy
testing, measurement of hormone levels, and a progesterone challenge.
Treatment aims to correct any underlying disorder and minimize excess
androgenic effects.
Routine testing includes a pregnancy test, a progesterone challenge, and
measurement of hormone levels (eg TSH, Prolactin, FSH, LH). If a genetic
defect is suspected (eg, in primary amenorrhea), karyotype is determined.
A 37 year old gravida 6 para 5 is given oxytocin (Pitocin) to induce
delivery at 41 weeks gestation. Her prenatal course is significant for
chronic hypertension. She delivers a 4020-g (8 lb 14 oz) baby. Soon
after delivery of the placenta, she begins to have excessive vaginal
bleeding. Which one of the following would you do initially?

a)
b)
c)
d)
e)

Perform manual uterine exploration and massage


Perform uterine curettage
Administer intramuscular methylergonovine (Methergine)
Administer subcutaneous terbutaline (Brethine, Bricanyl)
Administer intramuscular prostaglandin F2alpha (carboprost)

(Hemabate)
The correct answer is A
Explanation
The incidence of postpartum hemorrhage is 5%-8%. Causes include uterine
atony, lacerations, retained placental products, and defects of
coagulation. Uterine atony is the most likely cause of hemorrhage in
this patient with multiple risk factors, including grand multiparity, a
large fetus (uterine distension), and oxytocin induction. The initial
step in management of postpartum hemorrhage should be manual uterine
exploration followed by bimanual massage and compression of the uterus.
This maneuver may need to be performed for upwards of 30 minutes.
Intravenous oxytocin should also be infused simultaneously. Uterine
curettage may be performed to attempt to remove retained placental
products. however, it carries a significant risk of uterine perforation
and should be delayed unless bleeding cannot be controlled by other
means. Methergine is useful for postpartum hemorrhage but is
contraindicated in this patient with hypertension. Terbutaline is a
tocolytic and is not used for treatment of hemorrhage. Prostaglandin
F2alpha is an effective treatment for postpartum hemorrhage, but should
be reserved for use when uterine massage fails.
A woman who is 8 weeks pregnant has severe intractable vomiting. She has
no vaginal bleeding. What investigation is not appropriate?

a)
b)
c)
d)
e)

TSH/FT4
BHCG
BMP
Ultrasound
CT abdomen

The correct answer is E


Explanation
Hyperemesis gravidarum is uncontrollable vomiting during pregnancy that
results in dehydration and ketosis. The cause appears to be rapidly
increasing levels of estrogens or the ? subunit of human chorionic
gonadotropin (?-hCG). Psychologic factors (eg, ambivalence, anxiety) may
trigger hyperemesis gravidarum.
If hyperemesis gravidarum is suspected, urine ketones,
thyroid-stimulating hormone, serum electrolytes, AST, ALT, BUN, serum
creatinine, Mg, phosphorus, and sometimes body weight are measured.
Obstetric ultrasonography should be done to rule out hydatidiform mole

and multifetal pregnancy.


IV fluid hydration is given. Vomiting that persists after initial fluid
and electrolyte replacement is treated with an antiemetic promethazine,
ondansetron or prochlorperazine.
There is no indication for a CT of the abdomen in this patient.
When magnesium sulfate is given to a woman in pregnancy all of the
following are monitored, except

a)
b)
c)
d)

Serum creatinine
Reflexes (eg. knee jerk)
Liver enzymes
Respiratory rate

The correct answer is C


Explanation
During magnesium sulfate administration to a woman, usually given for
eclampsia, it is important to keep in mind the potential clinical signs
of magnesium toxicity. These include loss of patellar reflexes, slurred
speech, somnolence, flushing, muscle paralysis, decreased renal function
and respiratory arrest. Therefore the respiratory rate, creatinine level
and knee (patellar) reflexes must be checked.
Which of the following statements regarding endometriosis is incorrect?

a) Theories proposed to explain the histogenesis of endometriosis


include retrograde menstruation and coelemic metaplasia
b) CA-125 is often used to diagnose endometriosis
c) Symptoms of endometriosis may include urinary frequency and diarrhea
d) The differential diagnosis of endometriomas include hemorrhagic
corpus luteum cysts as well as neoplasms
e) The risk of endometriosis is several fold greater if there is a
first degree relative with this condition
The correct answer is B
Explanation
CA-125, cancer antigen-125, is a protein that is found at levels in most
ovarian cancer cells that are elevated compared to normal cells. CA-125
is produced on the surface of cells and is released in the blood stream.
CA-125 is not related to endometriosis.
A 25 year old woman, gravida 2, para 1, with chronic hypertension, is at
38 weeks gestation. Ultrasound examination shows an amniotic fluid index
of 4 cm and an estimated fetal weight below the 10th percentile. A
nonstress test (NST) is nonreactive with absent variability, and a
subsequent contraction stress test (CST) is positive. Her Bishop score
is 4. Which of the following should be the next step in managing this
patient?

a)
b)
c)
d)
e)

Cordocentesis for fetal karyotype


Cordocentesis for fetal blood pH
Biophysical profile
Immediate delivery
Repeat contraction stress test in 1 week

The correct answer is D


Explanation
Late decelerations as a rule generally indicate uteroplacental
insufficiency. A positive CST indicates that late decelerations are
present on at least 50% of the contractions. This, along with the
absence of variability, as well as other measures consistent with
chronic growth restriction (oligohydramnios and weight <10th
percentile), is an indication for immediate delivery. Since the Bishop
score indicates an unripe cervix, this would be best done by performing
a cesarean section.
A. Fetal karyotype is important for the workup of a symmetrical IUGR

fetus. This is the picture of uteroplacental insufficiency.


B. Fetal blood pH will not aid in the management of this patient.
Immediate delivery is the answer.
C. A positive CST with an unripe cervix and IUGR at 38 weeks is managed
by immediate delivery. Further fetal testing is not warranted.
E. This fetus needs delivery this day. Further delay may lead to
stillbirth or other serious sequelae.
Which one of the following is true concerning breastfeeding?

a) The first feeding should take place a few hours after delivery
when the mother is well rested
b) Packets containing information about breastfeeding and samples of
infant formula should be provided when mothers go home from the hospital
c) Ointment containing vitamins A and D should be applied to prevent
cracked nipples
d) When lactation is becoming established, infants should be weighed
before and after a feeding to determine whether they are getting enough
milk
e) Breastfeeding mothers should be taught that milk supply depends on
the amount of suckling
The correct answer is E
Explanation
Controlled trials have shown that delaying the first breastfeeding
session until more than 2 hours after delivery, test weighing to
determine the amount of milk ingested, and provision of formula samples
to nursing mothers all substantially decreases the proportion of women
successfully nursing by the first postpartum visit. Test weighing has
also been shown to be inaccurate and unreliable. Ointment containing
vitamins A and D was worse than nothing in clinical trials; the main
effective preventive and remedial measure for sore nipples is correct
positioning of the babys mouth on the breast. New mothers should be
taught the breastfeeding basics, including the fact that milk supply
adjusts to the infants demands, i.e., the frequency, vigor, and
duration of suckling.
Compared to anesthesia using only parenteral opioids, the use of
epidural anesthesia in labor and delivery increases the rate of which
one of the following?

a)
b)
c)
d)

Cesarean section
Low Apgar scores (< 7)
Maternal low backache 3 months post delivery
Prolonged second stage of labor

The correct answer is D


Explanation
Multiple systemic reviews have been conducted to examine effects of
epidural anesthesia on maternal and neonatal outcomes. There are many
confounding variables in the studies and, as a result, only a few
effects of epidural anesthesia are consistently seen on a statistically
significant basis: an increased duration of the second stage of labor,
an increased rate of instrument-assisted vaginal deliveries, and an
increased likelihood of maternal fever. Overall, there is no
statistically significant difference in the duration of the first stage
of labor, the incidence of low Apgar scores, or the incidence of
maternal backache at 3 months or 12 months.
A 28 year old white female consults you with a complaint of irregular
heavy menstrual periods. Her general physical examination, pelvic
examination, and Papanicolaou test are normal and she has a negative
pregnancy test. A CBC and chemistry profile are also normal. The next
step in her workup should be

a) Endometrial aspiration
b) Dilatation and curettage

c) LH and FSH assays


d) Administration of estrogen
e) Cyclic administration of progesterone for 3 months
The correct answer is E
Explanation
Abnormal uterine bleeding is a relatively common disorder which may be
due to functional disorders of the hypothalamus, pituitary, or ovary, as
well as uterine lesions. However, the patient who is younger than 30
years of age will rarely be found to have a structural uterine defect.
Once pregnancy, hematologic disease, and renal impairment are excluded,
administration of intramuscular or oral progesterone will usually
produce definitive flow and control the bleeding. No further evaluation
should be necessary unless the bleeding recurs.
Endometrial aspiration, dilatation and curettage, and other diagnostic
procedures are appropriate for recurrent problem or for older women.
Estrogen would only increase the problem, which is usually due to
anovulation with prolonged estrogen secretion, producing a hypertrophic
endometrium.
A healthy 40-year-old woman requests oral contraceptives. As her sole
health care provider, appropriate evaluation includes all of the
following tests EXCEPT:

a)
b)
c)
d)
e)

Mammography
Pap test
Endometrial sampling
Blood lipid determination
Blood pressure determination

The correct answer is C


Explanation
Routine health care screening is still necessary in this patient. This
will include a yearly Pap smear, q5 year routine cholesterol screening,
baseline mammography, and routine vital signs. Endometrial sampling is
not required.
A. Routine initial mammography screening is indicated at age 40.
B. Yearly Pap tests are indicated. Should the woman be low risk and have
three consecutive normal yearly Pap smears, then increasing the interval
length up to q3 years may be appropriate.
D. Every 5 years cholesterol screening is indicated.
E. Routine vital signs are part of the necessary yearly health care
screening.
A pregnant woman at 20 weeks presents with vaginal bleeding. She is type
O, Rh negative. She doesn't know the fathers blood type. When will you
give Rhogam?

a)
b)
c)
d)

28 weeks gestation
2 weeks postpartum
Now
At delivery

The correct answer is C


Explanation
In a woman with Rh incompatibility, there is a substance in her baby's
red blood cells that is not in her blood cells. This substance is called
the Rho(D) factor. People who have the Rho(D) factor are Rh positive.
People who do not have it are Rh negative.
While pregnant or during delivering, some of the baby's red blood cells
may come in contact with the mothers blood. The mothers body may then
make antibodies to the Rho(D) factor. This reaction is called
sensitization. The antibodies may cross the placenta and destroy the red
blood cells in the baby or any Rh-positive babies you have later. This
destruction of red blood cells is called hemolytic disease.

Rh incompatibility happens only if you are Rh negative and your baby is


Rh positive.
RhoGAM contains antibodies to the Rho(D) factor. The antibodies in the
shot will destroy any red blood cells from the baby that are the
mothers blood. Then the mother will not make its own antibodies to the
Rho(D) factor. A shot at 28 weeks and after delivery, sensitization will
be prevented and Rh incompatibility should not be a problem during the
next pregnancy.
If a woman has a threatened miscarriage but does not actually miscarry,
she will need a RhoGAM shot.
Fetal exposure to lithium in the first trimester has been strongly
associated with

a)
b)
c)
d)
e)

Alopecia
Cleft lip and palate
Congenital heart defects
Limb defects
Mental retardation

The correct answer is C


Explanation
Ebstein's anomaly, also called Ebstein's malformation, is a heart defect
in which the tricuspid valve is abnormally formed. The tricuspid valve
normally has three "flaps" or leaflets. In Ebstein's anomaly, one or two
of the three leaflets are stuck to the wall of the heart and don't move
normally. Often there's also a hole in the wall between the atria, the
heart's two upper chambers. This hole is called an atrial septal defect
or ASD. Because the tricuspid valve is malformed in Ebstein's anomaly,
it often doesn't work properly and may leak. If the valve leaks, some of
the blood pumped by the right ventricle goes backwards through the valve
with each heartbeat.
Studies suggest that lithium might be involved as a teratogen increasing
the incidence of Ebstein's anomaly in the offspring of female patients
with manic-depressive psychosis and lithium-administered during pregnancy.
You are evaluating a 25 year old gravida 5 para 1 at 6 weeks estimated
gestation. She has a history of three consecutive spontaneous
miscarriages. Her workup has been negative except for a positive lupus
anticoagulant on two occasions, separated by 6 weeks. You make the
diagnosis of antiphospholipid antibody syndrome. She has no previous
history of venous or arterial thrombosis. The best medical management at
this time is

a)
b)
c)
d)
e)

Acetaminophen
Warfarin (Coumadin)
Prednisone
Aspirin and heparin combined
Progesterone

The correct answer is D


Explanation
Antiphospholipid antibody syndrome in pregnancy is associated with an
increased risk of thromboembolism, fetal loss, thrombocytopenia, and
poor pregnancy outcome.
Studies comparing aspirin alone versus aspirin and heparin suggest that
the combination of aspirin and heparin is most effective for decreasing
fetal loss. One study showed decreased fetal loss with a combination of
corticosteroids and aspirin, but the results have not been reproduced in
subsequent studies. In addition, the use of prednisone was associated
with an increased risk of premature rupture of membranes, preterm
delivery, fetal growth restriction, infection, preeclampsia, diabetes,
osteopenia, and avascular necrosis. Progesterone may be useful for
recurrent spontaneous abortion related to a luteal phase defect, but has
not been shown to be effective in preventing complications associated

with antiphospholipid antibody syndrome.


A 38 year old widow consults you 2 years after her husbands accidental
death. She is planning to remarry and asks about the possibility of
resuming the low-dose oral contraceptives she took before she was
widowed. Which one of the following may contraindicate resumption of
oral contraceptives

a)
b)
c)
d)
e)

Her 42-year-old sister has breast cancer


Her blood pressure is 135/88 mm Hg
She smokes a pack of cigarettes a day
She has a history of migraines resistant to triptans
Her LDL/HDL ratio is 2.8

The correct answer is C


Explanation
Oral contraceptives increase the risk of venous thromboembolic
phenomena. The combination of oral contraceptives and smoking
substantially increases the risk of cardiovascular disease. In general,
oral contraceptive use is considered absolutely contraindicated in women
older than 35 who are heavy smokers. Women who smoke fewer than 15
cigarettes a day and patients with mildly elevated blood pressure and
elevated lipid levels are not at increased risk for cardiovascular
disease when oral contraceptives are used.
You have just diagnosed and treated gonorrheal cervicitis in a
24-year-old female who is in her second trimester of pregnancy. The
patient has a friend who had a stillborn infant and she is concerned
that the gonorrhea may predispose her to stillbirth.
Which one of the following would be most appropriate in this situation?

a) Inform the patient that stillbirth related to gonorrhea is very


rare, and that special fetal and maternal monitoring is not needed
b) Perform fetal monitoring and serial vaginal cultures, starting at
32 weeks gestation
c) Perform monthly vaginal cultures for gonorrhea, starting at 24
weeks gestation
d) Administer prophylactic ciprofloxacin (Cipro) weekly until delivery
The correct answer is A
Explanation
Gonorrhea is a rare cause of stillbirth, probably because it is unusual
for the gonorrhea organism to ascend into the uterus. The organism lacks
affinity for fetal membranes. Since gonorrhea is easily and effectively
treated with ceftriaxone or cefixime, follow-up cultures and monitoring
are not needed unless symptoms persist. Quinolones (including
ciprofloxacin) and tetracycline, which are indicated in nonpregnant
patients, are contraindicated in pregnancy.
If the embryonic disc divides 14 days after fertilization, the result is a:

a)
b)
c)
d)
e)

Diamniotic, dichorionic placentation


Diamniotic, monochorionic placentation
Velamentous cord insertion
Conjoined twins
Acardia

The correct answer is D


Explanation
Division before day 4: diamniotic, dichorionic. Division during day 4-8:
diamniotic monochorionic. Division during day 8-12: monoamniotic,
monochorionic. Division after day 12: conjoined. Acardiac twins only
occur in monochorionic with delayed cardiac function in one twin, with
arterial anastomosis leading to reversed arterial perfusion and absence
of development of the heart tube. (TRAP: twin reversed arterial
perfusion syndrome).

A. Occurs before day 4.


B. Occurs between day 4 and 8.
C. This is where the cord inserts into the membrane away from the
placental disc. It is more common in twin gestation that in singleton.
E. Acardiac twins are monochorionic twins, where the heart of one twin
does not develop. This is a very rare condition.
A woman develops severe post partum hemorrhage. 6 weeks later she
presents with fatigue and an inability to breast feed. All of the
following hormones could be decreased, except

a)
b)
c)
d)
e)

Prolactin
TSH
LH
Aldosterone
FSH

The correct answer is D


Explanation
Sheehan's syndrome is a rare complication of childbirth. Sheehan's
syndrome typically develops because of excessive blood loss and shock
during childbirth, which results in partial destruction of the pituitary
gland. Symptoms include fatigue, loss of pubic and underarm hair, and
inability to produce breast milk.
The pituitary hormones will be affected and they include FSH, LH, ACTH,
TSH, prolactin and growth hormone. Remember the mnemonic ?FLAT PiG? for
the anterior pituitary hormones.
Aldosterone is produced by the adrenal gland. In particular the zona
glomerulosa part of the adrenal cortex.
In which one of the following would misoprostol (Cytotec) be appropriate?

a) A gravida 2 para 1 with a long, thick cervix at term who is


interested in vaginal birth after a cesarean section
b) A primigravida at term whose cervix is dilated to 7 cm and whose
frequency of contractions has slowed
c) A primigravida with a long, thick cervix at term who has a
previous history of myomectomy
d) A diabetic primigravida with a long, thick cervix at 38 weeks
gestation for whom induction is indicated
The correct answer is D
Explanation
Misoprostol is valuable for labor induction or cervical ripening. It
should not be used in a woman with a history of uterine surgery and is
not indicated during active labor. Diabetes mellitus is not a
contraindication.
A pregnant woman presents in labor at 39 weeks gestation. Her fetus is
at zero station and she is dilated 10 cm. She also is noticed to have a
bulging membrane. What is the most appropriate management?

a)
b)
c)
d)

Cesarean
Oxytocin
Tocolysis
Amniotomy

The correct answer is D


Explanation
The 1st stage, from onset of labor to full dilation of the cervix (about
10 cm), has 2 phases, latent and active. During the latent phase,
irregular contractions become progressively better coordinated,
discomfort is minimal, and the cervix effaces and dilates to 4 cm.
During the active phase, the cervix becomes fully dilated, and the

presenting part descends well into the midpelvis. On average, the active
phase lasts 5 to 7 hours in nulliparas and 2 to 4 hours in multiparas.
If the membranes have not spontaneously ruptured, some clinicians use
amniotomy (artificial rupture of membranes) routinely during the active
phase. As a result, labor may progress more rapidly, and
meconium-stained amniotic fluid may be detected earlier.
Amniotomy during this stage may be necessary for specific indications,
such as facilitating internal fetal monitoring to confirm fetal
well-being. Amniotomy should be avoided in women with HIV infection or
hepatitis B or C, so that the fetus is not exposed to these organisms.
In a patient whose blood type is O-Rh negative and whose husband has
O-Rh positive blood, Rh immune globulin (RhoGam) should be given in all
the following cases, *except*

a) Prior to 25 wk gestation.
b) Full term delivery of a baby with O-Rh positive blood type
c) Amniocentesis
d) Spontaneous abortion
e) Molar pregnancy
The correct answer is A
Explanation
If the father of the baby or donor is Rh positive or unknown, the RH
negative patient is a candidate for RhoGAM prophylaxis in the following
cases:
-between 28 and 32 wk gestation
-Threatened abortion at any stage with confirmed pregnancy.
-Abortion, ectopic, or molar pregnancy at or beyond 13 weeks.
-Genetic amniocentesis.
-Unexplained first, second or third trimester bleeding.
-Abdominal trauma 2nd or 3rd trimester.
-Third trimester amniocentesis.
It would be appropriate to empirically treat asymptomatic bacteriuria in
which one of the following patients

a)
b)
c)
d)
e)

A 4-year-old female
A 30-year-old gravida 2 para 1 in the first trimester
A 40-year-old paraplegic with a neurogenic bladder
A 95-year-old male
A 95-year-old female

The correct answer is B


Explanation
Current guidelines strongly recommends that all pregnant women be
screened for asymptomatic bacteriuria with a urine culture at 12-16
weeks gestation. (Recommendation Level A). The treatment of asymptomatic
bacteriuria in pregnancy reduces symptomatic urinary tract infections,
pyelonephritis, low birth weight, and preterm delivery.
These guidelines also recommend against the routine screening of men and
nonpregnant women for asymptomatic bateriuria. (Recommendation level D).
No benefit has been shown and there are potential harms associated with
the overuse of antibiotics to treat asymptomatic patients with positive
urine culture results.
Studies have not demonstrated that either age or underlying medical
condition (other than pregnancy) are indications for treating
asymptomatic bacteriuria.
When performed during pregnancy, which one of the following imaging
procedures would expose the fetus to the highest radiation dose?

a) Ventilation-perfusion lung scan


b) Radiographs of both knees

c) A chest radiograph, with two views


d) Fluorocopic barium enema
e) Sinus radiographs
The correct answer is D
Explanation
While no single diagnostic imaging procedure would result in a radiation
dose that would threaten the well-being of a developing embryo or fetus,
and fetal risk is considered negligible at 5 rad or less, certain
imaging procedures do carry a higher radiation dose than others.
Fluoroscopic barium enema-------3.986 rad
Intravenous pyelogram-------1.395 rad
Lumbrosacral spine films-------0.359 rad
V/Q lung scan-------0.215 rad
Chest film, two views-------0.00007 rad
A 27-year-old patient complains of 6 months of amenorrhea. A pregnancy
test is negative. Which of the following is the most likely cause of
secondary amenorrhea in this patient?

a)
b)
c)
d)
e)
The correct answer is E

Abnormal chromosomes
Asherman syndrome
Hypothyroidism
Prolactinoma
Anovulation

Explanation
After pregnancy, anovulation is the most common cause of secondary
amenorrhea in a reproductive age woman. Ashermans syndrome is usually
associated with a prior uterine curettage.
A. This is a rare cause of amenorrhea.
B. This is often secondary to intrauterine trauma (dilation and
curettage, infection, etc).
C. Although this can be a cause, it is uncommon.
D. Prolactinomas are rare causes of amenorrhea. They are often
associated with galactorrhea.
A 27 year old white female sees you for the first time for a routine
evaluation. A Papanicolaou test reveals atypical glandular cells of
undetermined significance (AGUS). Of the following, which one is most
commonly found in this situation?

a) Cervical intraepithelial neoplasia


b) Endometrial hyperplasia
c) An endocervical polyp
d) Endometrial cancer
e) Ectopic deciduas
The correct answer is A
Explanation
Clinical practice guidelines recommend that all patients with atypical
glandular cells of undetermined significance (AGUS) be evaluated by
colposcopy and endocervical cottage; endometrial sampling is recommended
in women 35 years of age or older, and in those with AGUS favouring
neoplasia or suggesting an endometrial source. Cervical intraepithelial
neoplasia is the most common histologic diagnosis found in patients
evaluated for AGUS.
The Centers for Disease Control and Prevention (CDC) recommends
antenatal screening for group B streptococcal disease by

a) Culturing the urine at 20 weeks gestation


b) Obtaining cultures from the rectum and vaginal introitus at 20
weeks gestation
c) Obtaining a culture from the cervix at 35-37 weeks gestation

d) Obtaining cultures from the cervix and rectum at 35-37 weeks


gestation
e) Obtaining cultures from the rectum and vaginal introitus at 35-37
weeks gestation
The correct answer is E
Explanation
The gastrointestinal tract is the most likely reservoir of group B
Streptococcus with secondary spread to the genital tract. Cultures from
the vaginal introitus and the rectum are the most sensitive for
detecting colonization. No speculum examination is necessary. The
closest time to delivery that cultures can be performed and allow time
for results to be available is 35-37 weeks gestation. Culture-positive
women are then treated during labor. Other criteria for the use of
chemoprophylaxis during delivery continue to apply.
Variable decelerations on a fetal heart strip are commonly seen in which
prenatal condition

a) Uteroplacental insufficiency
b) Cord compression
c) Head compression
d) Sleeping fetus
The correct answer is B
Explanation
Uteroplacental insufficiency is associated with late decelerations. Cord
compression is associated with variable decelerations. Head compression
is associated with early decelerations.
A 34-year-old female who delivered a healthy infant 18 months ago
complains of a milky discharge from both nipples. She reports that
normal periods have resumed since cessation of breastfeeding 6 months
ago. She takes ethinyl estradiol/norgestimate (Ortho Tri-Cyclen) for
birth control. A complete review of systems is otherwise negative. The
most likely cause of the discharge is

a)
b)
c)
d)

a medication side effect


breast cancer
a hypothalamic tumor
hypothyroidism

The correct answer is A


Explanation
This patient has galactorrhea, which is defined as a milk-like discharge
from the breast in the absence of pregnancy in a non-breastfeeding
patient who is more than 6 months post partum. It is more common in
women ages 20?35 and in women who are previously parous. It also can
occur in men. Medication side effect is the most common etiology.
The most common pharmacologic cause of galactorrhea is oral
contraceptives. Oral contraceptives that contain estrogen can both
suppress prolactin inhibitory factor and stimulate the pituitary
directly, both of which can cause galactorrhea. Other medications that
can cause galactorrhea include metoclopramide, cimetidine, risperidone,
methyldopa, codeine, morphine, verapamil, SSRIs, butyrophenones,
dopamine-receptor blockers, tricyclics, phenothiazines, and
thioxanthenes. Breast cancer is unlikely to present with a bilateral
milky discharge. The nipple discharge associated with cancer is usually
unilateral and bloody. Pituitary tumors are a pathologic cause of
galactorrhea due to the hyperprolactinemia that is caused by the
blockage of dopamine from the hypothalamus, or by the direct production
of prolactin. However, patients often have symptoms such as headache,
visual disturbances, temperature intolerance, seizures, disordered
appetite, polyuria, and polydipsia. Patients with prolactinomas often
have associated amenorrhea. These tumors are associated with marked
levels of serum prolactin, often >200 ng/mL. Hypothalamic lesions such
as craniopharyngioma, primary hypothalamic tumor, metastatic tumor,
histiocytosis X, tuberculosis, sarcoidosis, and empty sella syndrome are
significant but infrequent causes of galactorrhea, and generally cause

symptoms similar to those of pituitary tumors, particularly headache and


visual disturbances. It is rare for primary hypothyroidism to cause
galactorrhea in adults. Symptoms that would be a clue to this diagnosis
include fatigue, constipation, menstrual irregularity, weight changes,
and cold intolerance.
A 19-year-old woman with regular menses and a 28-day cycle now presents
with 8 weeks of amenorrhea. If her last menstrual period was April 17,
using Naegeles rule, you would anticipate that her estimated date of
confinement (EDC) is:

a)
b)
c)
d)
e)

January 17
January 24
July 24
February 17
January 10

The correct answer is B


Explanation
Naegeles rule is used to estimate when 40 completed weeks of pregnancy
have occurred. Add 7 days and subtract 3 months from the LMP will result
in the EDC. This rule requires the woman to have regular cycles of
normal length.
A. For this date to be her EDC, her LMP would have been April 10.
C. For this date to be her EDC, her LMP would have been October 17.
D. For this date to be her EDC, her LMP would have been May 10.
E. For this date to be her EDC, her LMP would have been April 3.
Which one of the following single-dose regimens is effective for
treatment of uncomplicated urethral or cervical infection caused by
Chlamydia trachomatis

a) Azithromycin (Zithromax), 1 g orally


b) Doxycycline (Vibramcyin), 200 mg orally
c) Amoxicillin, 2 g orally
d) Cefixime (Supramax), 400 mg orally
e) Metronidazole (Flagyl), 2 g orally
The correct answer is A
Explanation
Azithromycin, 1 g orally once, is effective for treatment of
uncomplicated urethral or cervical infection caused by Chlamydial
trachomatis. Doxycycline and amoxicillin are effective for this
condition, but must be taken for 7-10 days. Cefixime and metronidazole
are effective as single doses for uncomplicated gonorrhea and vaginal
trichomoniasis respectively, but neither is indicated for the treatment
of Chlamydia.
A 21 year old married gravida 1 para 1 has not used her oral
contraceptives for 6 months. She comes to your office for evaluation
because her menstrual period is 2 weeks late. Her menses had been
regular since discontinuing the oral contraceptives. A urine hCG is
negative. Which one of the following is true regarding this situation?

a) It is unlikely that she is pregnant


b) Pregnancy cannot be diagnosed before the first missed menstrual
period
c) A serum progesterone level should be drawn
d) Transvaginal ultrasonography should be performed
The correct answer is A
Explanation
With the high level of sensitivity and specificity of current tests to
measure hCG in serum and urine, pregnancy can now be diagnosed before
the time of the first missed menstrual period. For current serum hCG
assays, the low threshold for detection is 10-25 IU/L, while for urine
assays it is 25-50 IU/L, which corresponds to approximately the seventh

day after conception. Because the levels of hCG in the blood are urine
are very similar, the tests are equivalent. Urine testing may reveal a
positive result as early as 3-4 days after implantation. By the time of
the expected menstrual period, the test will be positive 98% of the
time. If a test is negative more than 1 week after the expected time of
the menstrual period, it is almost certain the patient is not pregnant.
To cover these rare instances where a woman has a low hCG and conceived
later than expected, the test should be repeated in 1 week for a
definitive result.
Since ectopic pregnancy is not a life-threatening problem for the mother
until 2 months after conception, a patient with a negative urine hCG
does not require ultrasonography to exclude ectopic pregnancy. Patients
with a suspected ectopic pregnancy and a negative urine hCG should be
followed closely, as early laparoscopic intervention can improve the
chances of future fertility.
A 31-year-old sexually active woman comes to your office requesting a
cervical cap for contraception. You advise her that the maximum number
of hours that the cervical cap should be left in place is:

a)
b)
c)
d)
e)

4 hours
12 hours
24 hours
36 hours
48 hours

The correct answer is E


Explanation
The cervical cap, more popular in Europe than in the North America, has
a similar failure rate to the diaphragm (2 year pregnancy rate of
15-20%). It is much more effective in nulliparous than in parous women.
About two-thirds of the failures are user related. It should only be
used on women with normal Pap smears and should not be left in place
more than 48 hours because of the possibility of ulceration, unpleasant
odor, and infection.

A 19-year-old nulligravid healthy woman comes to see you for her annual
Pap smear and routine health care maintenance. During your routine
pelvic exam, you note that she has a 5-cm cystic, nontender, mobile mass
in her left adnexa. Rectovaginal exam confirms this and does not note
any abnormalities in the cul-de-sac. Transvaginal ultrasonography
results are consistent with pelvic exam findings. Which of the following
is the most appropriate next step?

a) Laparotomy with ovarian cystectomy


b) Repeat ultrasound in 2 months
c) MRI scan of the pelvis
d) Serum CA-125
e) Laparoscopy with ovarian cystectomy
The correct answer is B
Explanation
The most common cause for a cystic enlargement of the ovary in a
reproductive age woman is a functional cyst (follicular or corpus
luteum). These are thin walled and usually resolve or rupture
spontaneously. Any cystic mass 6 cm or less can be followed for two
cycles. Some texts recommend using oral contraceptive pills to decrease
the gonadotropin stimulation of the ovary during this time, but there is
no literature that shows any improvement over simple observation.
-If the mass persists, further evaluation and possible surgical
intervention is indicated. Laparoscopic surgery is less invasive and
just as successful as laparotomy.
-As most cystic masses in the adnexa are functional cysts that resolve
spontaneously, no other workup is indicated at this time.
Regarding oral contraceptive pills (birth control), which of the
following is false?

a)
b)
c)
d)
e)

It is contraindicated in women over age 35 who are smokers


Its efficacy may be decreased when taking antibiotics
Reduces the risk of ovarian carcinoma
Is associated with only a 10% failure rate
Can be associated with breast changes and weight gain

The correct answer is D


Explanation
Oral contraceptive pills are widely used and are generally safe and
effective for many women. The choice of pill formulation is influenced
by clinical considerations. By choosing appropriately from the available
pill formulations, physicians can minimize negative side effects and
maximize noncontraceptive benefits for their patients. Additional
monitoring and follow-up are necessary in special populations, such as
women over 35 years of age, smokers, perimenopausal women and
adolescents. Third-generation progestins are additional options for
achieving noncontraceptive benefits, but their use has raised new
questions about thrombogenesis.
Whereas only one of 1,000 women who take oral contraceptive pills
"perfectly" becomes pregnant within a year, 50 of 1,000 women who take
the pills "typically" become pregnant within one year.
An asymptomatic 24-year-old white female comes to your office for a
refill of oral contraceptive pills. A speculum examination is normal
with the exception of a slightly friable, well-demarcated, 1.4-cm raised
lesion involving a portion of the cervix. All previous Papanicolaou
(Pap) tests have been normal and she has no history of abnormal bleeding
or leukorrhea. Which one of the following would be most appropriate at
this point?

a) A Pap test, including a scraping of the erosion, with routine


follow-up unless the patient becomes symptomatic
b) A Pap test with follow-up in 3 months if results are normal
c) A Pap test and a colposcopically directed biopsy
d) A cone biopsy
e) Topical antibiotic cream
The correct answer is C
Explanation
The finding of a red, raised, friable lesion on the cervix, or a
well-demarcated cervical lesion, mandates a biopsy to exclude cervical
carcinoma, and treatment for chronic cervicitis should not be started
until the biopsy results are available. A Papanicolaou test by itself is
insufficient if there is a grossly visible lesion, as false-negatives

occur in 10%?50% of tests.


A 32-year-old female presents with bilateral pretibial tender, mildly
red nodules 2 ? 4 cm in diameter. A nodule that appeared earlier
resolved, leaving a ?bruised? area. She had a similar problem once when
she was pregnant but it resolved spontaneously. Her medications include
lovastatin (Mevacor) for hyperlipidemia and a low-dose oral
contraceptive prescribed 5 months earlier. Her past history and a review
of systems are otherwise unremarkable.
The most appropriate next step would be to

a)
b)
c)
d)

Order a serum creatinine phosphokinase level


Obtain a cervical culture for gonorrhea
Discontinue her oral contraceptives
Discontinue lovastatin

The correct answer is C


Explanation
Erythema nodosum (EN) is a panniculitis most often appearing on the
shins. In 35% - 55% of cases, no cause is found. EN has been associated
with pregnancy and oral contraceptives. Other drugs including
sulfonamides, bromides, iodides, and omeprazole have been associated
with EN. Statins have not been associated with EN. Infectious agents
associated with EN include beta-hemolytic streptococci, Mycobacterium,
Yersinia, fungi, syphilis, Campylobacter, hepatitis C, and Epstein-Barr
virus. Inflammatory conditions associated with EN include inflammatory
bowel disease, sarcoidosis, Lofgrens syndrome and Behets syndrome.
All of the following are contraindications to epidural anesthesia, *except*

a) Hypovolemia
b) Bleeding diathesis
c) Infection at site of injection
d) Decreased blood pressure
e) Pre-eclampsia
The correct answer is E
Explanation
Contraindications for epidural anesthesia include: patient refusal for
surgery, uncooperative patients. abnormal bleeding or clotting
parameters, anti-coagulant therapy, skin infection at/near injection
site, uncorrected fluid loss (hypovolemia), low blood pressure, presence
of neurological disorders, cardiovascular disease, anatomical
abnormalities of the vertebral column.
It is indicated in difficult or high-risk labour, twin pregnancy,
pre-eclampsia, prolonged labor.
A 35-year-old white female presents with a 6-month history of irregular
menstrual bleeding. Before this problem began, her periods occurred
every 30 days and lasted 5 days. Now they occur every 20 days and last
for 10 days, and are heavier than they were previously. A physical
examination reveals no obvious anatomic source of bleeding, and a
Papanicolaou (Pap) test is normal. A pregnancy test is negative and a
blood workup for organic causes of irregular menses is also negative.
She takes no medications.
Which one of the following would be most appropriate at this point?

a) Pelvic ultrasonography
b) Increasing the dosage of the oral contraceptive
c) Changing to a progesterone-only contraceptive
d) Reassurance that the problem will resolve on its own
The correct answer is A
Explanation

Dysfunctional uterine bleeding is common, but the diagnosis is made by


excluding other pathologies. Changes in bleeding pattern may be due to
cervical pathology which will usually be seen on a Papanicolaou (Pap)
smear; anatomic problems such as polyps which can be detected on
examination; organic causes such as thyroid problems; prolactinemia,
coagulopathy, hepatic dysfunction, or adrenal dysfunction, which can be
detected by laboratory testing; and pregnancy. If these conditions are
ruled out, endometrial evaluation is important to exclude cancer,
especially when any risk factors are present. Risk factors include age
over 35, a history of anovulatory cycles, obesity, nulliparity, a
history of tamoxifen use, and diabetes mellitus.
In this patient an endometrial biopsy is not mandatory given her age and
lack of other risk factors. Ultrasonography is recommended to evaluate
the thickness at the endometrial lining and to look for other uterine
causes of bleeding such as polyps or fibroids.
A 34-year-old white female visits your office with a chief complaint of
pelvic pain that intensifies with her menstrual period. She has a
history of pain during intercourse, which started in her mid-twenties
and has gradually become worse. She reports recently missing some work
during her menstrual period due to the pain. She has had two uneventful
deliveries and the pain was absent during and after each pregnancy, but
gradually returned. She and her husband do not wish to have any more
children and her husband has had a vasectomy.
The patient denies vaginal discharge or fever and a review of systems is
negative. A complete physical examination is normal except for moderate
non-specific tenderness on pelvic examination. In addition, her uterus
is moderately retroverted and has decreased mobility.
Which one of the following would be the most appropriate initial step in
the management of this patient?

a) Conjugated estrogens (Premarin)


b) Combination oral contraceptives
c) Depot medroxyprogesterone acetate (Depo-Provera)
d) Danazol (Danocrine)
e) A complete hysterectomy and bilateral oophorectomy
The correct answer is B
Explanation
This patient has endometriosis. Combination oral contraceptives should
be first-line therapy for women with endometriosis who do not wish to
become pregnant. Conjugated estrogens is not a treatment for
endometriosis. Depot medroxyprogesterone acetate and danazol are
accepted treatments, but each has undesirable side effects. A complete
hysterectomy and bilateral oophorectomy is considered a radical surgical
approach, and is reserved for more difficult endometriosis cases.
When evaluating the results of a human chorionic gonadotropin level in
the maternal blood, the peak value occurs at which menstrual week of the
pregnancy?

a)
b)
c)
d)
e)

3-4
9-10
14-16
24-28
36-42

The correct answer is B


Explanation
Human chorionic gonadotropin doubles every 1.2-3 days in early
pregnancy, with its peak being reached at 7-9 weeks after fertilization.
It then declines to a plateau level for the remainder of the gestation.
A. See explanation.
C. See explanation.
D. See explanation.

E. See explanation.
Most individuals with polycystic ovary syndrome have

a)
b)
c)
d)
e)

Decreased estrogen levels


Decreased androgen levels
Elevated FSH levels
Elevated TSH levels
Insulin resistance
The correct answer is E

Explanation
Polycystic ovary syndrome, one of the most common endocrine disorders,
is associated with insulin resistance and a higher risk for development
of glucose intolerance and type 2 diabetes mellitus. Patients have
elevated androgen levels and elevated LH levels, but estrogen and FSH
levels remain normal. This condition is not associated with changes in
the TSH level or with thyroid abnormalities.
An absolute contraindication for vaginal delivery for a patient who had
a previous cesarean section is

a)
b)
c)
d)

Previous vertical uterine incision


Previous horizontal uterine incision
Twin gestation
Non cephalic presentation

The correct answer is A


Explanation
Vaginal birth after cesarean (VBAC) is done. But only if conditions are
favorable. Absolute contraindication to VBAC are include prior classical
(vertical) uterine incision
The most serious risk of a VBAC is that the previous C-section scar
could come open. This can be very serious for both the mother and the
baby. The risk that a scar will tear open is very low during VBAC when
you have just one low horizontal cesarean scar and your labor is not
started with medicine.
You are asked to see a young woman in the emergency department for a
miscarriage. On exam, she has passed the conceptus, and no products of
conception remain inside the uterus. The cervical os is closed and has
minimal bleeding. Vital signs are stable. The patient is inquiring as to
the reason for her miscarriage. Knowing that the most likely cause of
this spontaneous abortion is due to aneuploidy, you can tell her that
the most common chromosomal abnormality is:

a)
b)
c)
d)
e)

Tetraploidy
Triploidy
Autosomal trisomy
Haploid of paternal origin
Diploid of paternal origin

The correct answer is C


Explanation
Overall, trisomy as a group accounts for 50% of all first trimester
abortuses (of which trisomy 16 is the most common). Monosomy X is the
most frequent single anomaly found. Diploid of androgenetic origin is
associated with gestational trophoblastic neoplasias such as complete
and partial molar pregnancy.
A. An uncommon finding in abortuses.
B. An uncommon finding in abortuses.
D. Haploid of paternal origin will be a 23X or a 23Y.
E. Diploid of paternal origin is associated with complete molar pregnancy.
A 27-year-old sexually active woman presents to your office for
evaluation. She hasn?t had her period for the last 3 months. Prior to
that time, they were regular, every 28 days, with a light flow lasting
for 4 days. In your initial evaluation of her condition, which of the
following serum tests is most important?

a)
b)
c)
d)
e)

Prolactin
Luteinizing hormone (LH)
Estimated free thyroxine
Human chorionic gonadotropin (hCG)
Follicle stimulating hormone (FSH)

The correct answer is D


Explanation
The number one reason for absence of periods in a sexually active woman
is pregnancy. Although this is not yet by definition secondary
amenorrhea (>6 months of amenorrhea in a woman who has had prior
menses), ruled out pregnancy is the first step that should be taken.
A. This would be ordered in a patient with galactorrhea and
oligomenorrhea or amenorrhea.
B. This is not a usual test in the workup of amenorrhea.
C. Although abnormalities in thyroid function can lead to amenorrhea, in
a patient with normal cycles until 3 months ago, this diagnosis would be
unlikely.
E. This patient has no other symptoms of premature menopause (hot
flashes etc).
In a 34 year old primigravida at 35 weeks gestation, which one of the
following supports a diagnosis of mild preeclampsia rather than severe
preeclampsia?

a)
b)
c)
d)
e)

A blood pressure of 150/100 mm Hg


A 24-hr protein level of 6 g
A platelet count <100,000/mm3
Liver enzyme elevation with epigastric tenderness
Altered mental status

The correct answer is A


Explanation
The criteria for severe preeclampsia specify a blood pressure of 160/110
mm Hg or above on two occasions, 6 hours apart. Other criteria include
proteinuria above 5 g/24 hr, thrombocytopenia with a platelet count <
100,000/mm3, liver enzyme abnormalities, epigastric or right upper
quadrant pain, alteration of mental status.
A 34 year old woman presents to her doctor because she thinks she is
pregnant, but is frightened because she has had a lot of bleeding
yesterday with clumps that looked like ?grapes. She is very worried and
upset. Which of the following did she likely have?

a) Ovarian cyst
b) Ectopic pregnancy
c) Spontaneous abortion
d) Molar pregnancy
The correct answer is D
Explanation
A hydatidiform mole is growth of an abnormal fertilized egg or an
overgrowth of tissue from the placenta. Most often, a hydatidiform mole
is an abnormal fertilized egg. The abnormal egg develops into a
hydatidiform mole rather than a fetus (a condition called molar
pregnancy). However, a hydatidiform mole can develop from cells that
remain in the uterus after a miscarriage or a full-term pregnancy.
Women who have a hydatidiform mole feel as if they are pregnant. But
because hydatidiform moles grow much faster than a fetus, the abdomen
becomes larger much faster than it does in a normal pregnancy. Severe
nausea and vomiting are common, and vaginal bleeding may occur. These
symptoms indicate the need for prompt evaluation by a doctor.
Often, doctors can diagnose a hydatidiform mole shortly after
conception. No fetal movement and no fetal heartbeat are detected. As

parts of the mole decay, small amounts of tissue that resemble a bunch
of grapes may pass through the vagina. After examining this tissue under
a microscope, a pathologist can confirm the diagnosis.
A 36 week pregnant woman comes for routine prenatal visit. You do not
feel the presenting part. What is the most accurate method to determine
this

a)
b)
c)
d)

Send her to an obstetric specialist


Ultrasound
Re-Check in 2 weeks
Manual exam

The correct answer is B


Explanation
The figure above demonstrates the different types of presenting parts of
a baby. The method for determining this is with a manual exam, but in
this question since the doctor has already done that and was unable to
determine the presenting part, the next step would be to obtain an
ultrasound.

An 18-year-old single white female at 30 weeks gestation presents to the


hospital with uterine contractions 10 minutes apart. Her previous
pregnancy 18 months ago resulted in a preterm birth at 29 weeks gestation.
The most accurate test to determine whether this patient will need
hospitalization and tocolysis would be

a)
b)
c)
d)
e)

Serum corticotropin-releasing hormone


Maternal serum alpha-fetoprotein
Serum human chorionic-gonadotropin (hCG)
Salivary estriol concentration
Vaginal fetal fibronectin

The correct answer is E


Explanation
Of the biochemical markers listed, the most clinically useful test to
differentiate women who are at high risk for impending preterm delivery
from those who are not is the fetal fibronectin in cervical or vaginal
secretions. In symptomatic women, this is most accurate in predicting
spontaneous preterm delivery within 7 ? 10 days. It is less accurate in
those who are asymptomatic. If the fetal fibronectin is negative, it may

be possible to avoid interventions such as hospitalization, tocolysis,


and corticosteroid administration.
A 21 year old female is currently being treated for chlamydial
cervicitis with azithromycin (Zithromax), 1 g in a single dose. She had
a pelvic examination and a normal Papanicolaou (Pap) test 3 months ago.
When should she next have a test for Chlamydia?

a)
b)
c)
d)

3-4 weeks
1-2 weeks
9 months (at her next routine pelvic examination)
Never, unless she is symptomatic or has a suspected exposure to

Chlamydia at some point in the future


The correct answer is D
Explanation
A ?test for cure? for Chlamydia is no longer recommended because of the
efficacy of treatment and the persistence of a false-positive polymerase
chain reaction (PCR) test for Chlamydia in the cervix for as long as 3-4
weeks despite treatment.
Test of cure is required in pregnancy (cure rates lower in pregnant
patients), therefore pregnant women should be retested 3-4 weeks
post-initiation of therapy.
Regular screening for Chlamydia is generally recommended for sexually
active women age 25 and under. In fact, in 2000, annual Chlamydia
screening of sexually active women between the ages of 15 and 25 years
was added to the National Committee for Quality Assurance (NCQA) Health
Plan Employer Date and Information Set (HEDIS) quality measures.
A molar pregnancy is most readily diagnosed by

a)
b)
c)
d)
e)

Ultrasound
Magnetic resonance imaging
Computed tomography
Urinary human chorionic gonadotropin in excess of 50,000 IU/24 hour
Elevated beta sub-units of human chorionic gonadotropin

The correct answer is A


Explanation
A molar pregnancy is a mass of abnormal tissue (hydatidiform mole) that
comes from the placenta inside the uterus, which triggers symptoms of
pregnancy. About 1 out of 1,000 women with early pregnancy symptoms has
a molar pregnancy.
There are two types of molar pregnancy: complete and partial.
Complete molar pregnancy - in place of a normal placenta and embryo, the
hydatidiform mole is abnormal placental tissue that grows into a
grapelike cluster that can fill the uterus.
Partial molar pregnancy - the placenta grows abnormally into molar
tissue. Any fetal tissue that develops is likely to have severe defects.
If there are symptoms that suggest a molar pregnancy, one should do a
pelvic exam, an hCG level, and a pelvic ultrasound, which can confirm a
molar pregnancy. Molar pregnancy may also be found during a routine
ultrasound in early pregnancy.
A 32-year-old woman at 21 weeks gestation presents with acute shortness
of breath and pleuritic chest pain. Her medical history is significant
for antiphospholid antibody syndrome. Which of the following tests would
be most helpful in confirming the diagnosis

a)
b)
c)
d)
e)
The correct answer is C

Chest radiograph
Electrocardiogram
Ventilation perfusion scan
Lower extremity venous Doppler
Arterial blood gas

Explanation
With the hypercoagulable state of pregnancy combined with her history of
antiphospholipid antibody syndrome, this patient most likely has
developed a venous thrombosis leading to a pulmonary embolus.
Perfusion scanning alone is recommended initially, and the ventilation
scan is added when perfusion defects are noted. Pulmonary angiography
might be necessary if lung scan findings are of low probability or
indeterminate and clinical suspicion remains high.
A 58-year-old female sees you for a routine annual visit. She has had a
hysterectomy for endometriosis. She asks you if she needs a Papanicolaou
(Pap) test.
Which one of the following would be appropriate advice regarding Pap
tests for this patient?

a)
b)
c)
d)

Screening is not needed


Screening should be performed every year until she is 70
Screening should be performed every 3 years
Screening is recommended only if she has a positive family history

of cervical or ovarian cancer


The correct answer is A
Explanation
Current guidelines recommend against Papanicolaou (Pap) smear screening
in women who have had a hysterectomy for benign disease. An effort
should be made to confirm that the cervix was completely removed and
that the pathology from the hysterectomy was benign. In women who have a
past history of cervical intraepithelial neoplasia (CIN) 2/3, or in
cases where it cannot be confirmed that the hysterectomy was NOT
performed for CIN 2/3, screening should be performed until three
consecutive, adequately performed Pap tests are negative. Women who had
a subtotal hysterectomy and still have a cervix should continue to be
screened. In addition, women with a history or diethylstilbestrol
exposure in utero, or with a history of cervical cancer, should continue
to be screened.
A 24-year-old primiparous female at 8 weeks gestation presents to your
office for her initial prenatal visit. She has no identifiable risks for
the pregnancy. You counsel her regarding what lifestyle choices she
should, and should not, make to prevent risks to herself and her child.
Which one of the following would be appropriate advice?

a) Abstain from sexual intercourse in the last trimester


b) Avoid aspartame and sucralose in the first trimester
c) Avoid intercontinental air travel after the first trimester
d) Consume 1000-1300 mg of calcium daily
e) Consume 4-5 servings of fish such as swordfish or mackerel per week
The correct answer is D
Explanation
The recommended intake of calcium during pregnancy is 1000-1300 mg
daily. There are many concerns about toxic exposures during pregnancy,
some of them based on evidence and some not. Air travel is discouraged
in the 4 weeks prior to a patients due date, but no recommendations
restrict intercontinental flights for the entire pregnancy. Sexual
intercourse is deemed safe during pregnancy (assuming a normal
pregnancy). Aspartame and sucralose are also generally considered safe.
Exposure to high levels of mercury in fish can lead to neurologic
abnormalities in women and their infants. Therefore, pregnant women
should avoid shark, swordfish, king mackerel, and tuna steak, and limit
intake of other fish (including canned tuna) to 12 oz (2-3 meals) per week.
After fitting a 30 year old gravida 2 para 2 for a diaphragm, you advise
her not to leave the diaphragm in place for longer than 24 hours because

of the risk of which one of the following?

a)
b)
c)
d)
e)

Loss of contraceptive effectiveness


STD
Toxic shock syndrome
Human papillomavirus (HPV) infection
Adhesions

The correct answer is C


Explanation Much like with tampons, leaving diaphragms in place for
more than 24 hours is associated with toxic shock syndrome.
A 32 year old white female comes to your office complaining of dysuria.
She denies fever, back pain, and urinary frequency. She appears to be
well otherwise, and has a normal abdominal examination. A clean-catch
urinalysis shows 15-20 WBC/hpf and a dipstick test for leukocyte
esterase is positive. You send a urine sample for culture and start the
patient on nitrofurantoin (Macrodantin), as she is allergic to sulfa.
Three days later, the patient returns with persistent dysuria despite
taking the medication as prescribed. Her urine culture has returned with
no growth. A pelvic examination is normal, and the rest of the physical
examination is unchanged. A wet prep is normal and tests for sexually
transmitted diseases are pending. Which one of the following antibiotics
is most appropriate for this patient now?

a) Amoxicillin/clavulanate (Augmentin)
b) Cephalexin (Keflex)
c) Metronidazole (Flagyl)
d) Doxycycline
e) Ciprofloxacin (Cipro)
Correct Answer:* d)
Explanation
The urethral syndrome is characterized by dysuria and pyuria in the
presence of a negative culture for uropathogens. Frequency and urgency
are often absent. The infecting organism is typically Chlamydia
trachomatis although other organisms such as Ureaplasma urealyticum and
Mycoplasma species may be involved. Effective medication choices include
doxycycline, ofloxacin, levofloxacin, and macrolides such as
erythromycin and azithromycin.
A patient diagnosed as having a fetus with trisomy 18 asks that you do
not share this finding with her family. You may discuss her tests with:

a)
b)
c)
d)
e)

Her husband
Her mother
Her father
No one
Her son

The correct answer is D


Explanation
Again, this covers the ethical concept of confidentiality. Even though
her husband is the father of the affected child, the patient has the
right to request that you do not disclose this information to him.
A. See above discussion.
B. See above discussion.
C. See above discussion.
E. See above discussion.
A young woman with phenylketonuria (PKU) has not been following her diet
over the past several years. She comes to you in her first trimester of
pregnancy. Which one of the following congenital abnormalities would the
baby most likely develop?

a)
b)
c)
d)
e)

Neurological
Renal
Musculoskeletal
Respiratory
Visual

The correct answer is A


Explanation
Phenylketonuria (PKU) is an autosomal recessive genetic disorder
characterized by a deficiency in the enzyme phenylalanine hydroxylase.
This enzyme is necessary to metabolize the amino acid phenylalanine to
the amino acid tyrosine. When it is deficient, phenylalanine accumulates
and is converted into phenylketones, which are detected in the urine.
Left untreated, this condition can cause problems with brain
development, leading to progressive mental retardation and seizures.
However, PKU is one of the few genetic diseases that can be controlled
by diet. A diet low in phenylalanine and high in tyrosine can be a very
effective treatment. There is no cure. Damage done is irreversible so
early detection is crucial.
For women affected with PKU, it is essential for the health of their
child to maintain low phenylalanine levels before and during pregnancy.
Though the developing fetus may only be a carrier of the PKU gene, the
intrauterine environment can have very high levels of phenylalanine,
which can cross the placenta. The result is that the child may develop
congenital heart disease, growth retardation, microcephaly and mental
retardation.
A 68 year old woman with history of hysterectomy, comes to see you
because of problems with micturation. She has no digestive trouble, she
has a feeling of a painless mass intravaginally, she has to try twice in
order to void. What is the most likely diagnosis

a)
b)
c)
d)
e)

Rectocele
Cystocele
Prolapse (Procidentia)
Urethral sphincter spasm
Post-surgical stricture

The correct answer is B


Explanation
A cystocele occurs when the wall between a woman's bladder and vagina
weakens and stretches, allowing the bladder to bulge into the vagina. A
cystocele may also be called a prolapsed bladder. A cystocele may occur
from excessive straining, such as during childbirth, chronic
constipation or heavy lifting. It may also occur after menopause, when
estrogen decreases. Therefore, older women and those who have given
birth to several children are more likely to develop a cystocele.
For mild and moderate cystoceles, self-care measures or nonsurgical
treatments are often effective. In more severe cases of cystocele,
surgery may be necessary to keep the vagina and other pelvic organs in
their proper positions.
In mild cases, it's possible to not even notice a bulge. When cystocele
symptoms do present themselves, they may include a feeling of fullness
or pressure in your pelvis and vagina, especially when standing for long
periods of time. Increased discomfort when you strain, cough, bear down
or lift. A bulge of tissue that, in severe cases, protrudes through your
vaginal opening.
Which one of the following intravenous agents given to the mother during
labor is the drug of choice for prophylaxis of neonatal group B
streptococcal disease?

a) Clindamycin (Cleocin)
b) Erythromycin

c) Gentamicin (Garamycin)
d) Metronidazole (Flagyl)
e) Penicilllin G
The correct answer is E
Explanation
Intravenous penicillin G is the drug of choice for prophylaxis of
neonatal group B streptococcal disease, although shortages during 1999
required the interim use of broader-spectrum antibiotics. Group B
streptococci have remained sensitive to penicillin, but they may be
resistant to clindamycin and erythromycin, the drug recommended for
women allergic to penicillin.
Which one of the following is characteristic of the first stage of labor?

a) Uterine shape changes from discoid to globular, along with a rise


in fundal height
b) Regular uterine contractions are confirmed
c) In the absence of anesthesia, this stage of labor typically lasts
20-50 minutes
d) In the absence of anesthesia, each contraction stimulates the urge
to push
e) An episiotomy should be performed during stage of labor
The correct answer is B
Explanation
The first stage of labor begins with the onset of regular contractions
and ends with complete cervical dilation. The change in the shape of the
uterus from discoid to globular, along with a rise in fundal height, an
apparent lengthening of the umbilical cord, and a gush of vaginal
bleeding are signs of spontaneous separation that occurs in the third
stage of labor. It is not uncommon for the first stage of labor to last
around 18-20 hours in nulliparas and 14-16 hours in multiparas.
The second stage of labor (the period between the complete dilation of
the cervix to the delivery of the baby) usually lasts about 50 minutes
in nulliparas and 20 minutes in multiparas. This stage of labor can be
prolonged somewhat by the use of regional anesthesia. An episiotomy is
an incision of the perineum used to facilitate vaginal delivery; it is
useful for patients in whom the perineum does not readily stretch and/or
when delivery must be expedited. It is part of the management of the
second stage of labor. After the cervix becomes completely dilated (by
definition, the second stage of labor), patients usually experience an
urge to push with contractions. Regional anesthesia may partially or
completely blunt this urge.
The most frequent cause of dyspareunia is:

a) Vaginismus
b) Endometriosis
c) Retroverted uterus
d) Inadequate vaginal lubrication
e) Pelvic inflammatory disease
The correct answer is D
Explanation
All of these conditions are associated with dyspareunia. Inadequate
vaginal lubrication, however, is the most common cause of pain with
intercourse and can be due to a wide variety of causes.
A. Although a cause, it is not the most common. Psychosexual therapy may
be of benefit to this patient.
B. When the uterosacral ligaments and cul-de-sac are involved, or due to
pelvic ahesive disease, then intercourse can become painful. It is not
the most common cause.
C. Has been associated with dyspareuina. Not a common cause.
E. See answer to B.
Which one of the following is a risk factor for preterm birth?

a) Prior preterm delivery

b) High prepregnancy weight


c) First-trimester bleeding
d) Smoking
The correct answer is A
Explanation
Preterm delivery (16-36 weeks gestation) is three to four times more
likely in women who have had a prior preterm delivery. Multiple
gestation also makes preterm delivery more likely. Low prepregnancy
weight is associated with preterm delivery in Caucasian women. Secondor third-trimester bleeding is a risk factor as well. Smoking can result
in low-birthweight infants, but is not itself a risk factor for preterm
delivery.
A 37-year-old woman, who complains of heavy painful menses, requests
contraception. She smokes a pack of cigarettes a day. Of the following,
the best choice of contraceptive for this patient is:

a)
b)
c)
d)
e)

Copper t380a intrauterine device (IUD)


Low-dose, combined oral contraceptives
Contraceptive implant (Norplant)
Endometrial ablation
Hysterectomy

The correct answer is C


Explanation
Any smoker over the age of 35 has a significantly increased risk of
myocardial infarction, stroke, and death if they are on oral
contraceptive pills. Since she has heavy and painful menses, she is not
an ideal candidate for the IUD. Endometrial ablation may lessen her
menorrhagia, but it is not an accepted form of contraception. Norplant
has no estrogen component; so it will not increase her risk of
thromboembolic phenomena.
A. Due to her heavy, painful menses, not a good candidate for the IUD.
B. Smokers over 35 have an increased risk of MI and death when using
estrogen-containing oral contraceptives.
D. This is not an accepted method of contraception. It may reduce her
degree of menorrhagia.
E. Although this will successfully prevent pregnancy, contraception
alone is not an acceptable indication for hysterectomy.
A 23-year-old multipara has been in active labor for the last 8 hours.
Her cervix is dilated to 8 cm and the fetal vertex is at plus 2 station.
The fetus is of average size and she has had a prior uneventful 9-pound
fetus deliver vaginally. Recently, her contractions have been augmented
by an oxytocin intravenous infusion. Membranes are ruptured and the
amniotic fluid is clear. The patient is afebrile and normotensive. She
has not required any medication for pain control. Her nurse has notified
you of a recent change in the character of the fetal heart tracing. On
arrival at the patients bedside, you note the following fetal heart
tracing. Of the following, which is the most appropriate next step:

a)
b)
c)
d)

Immediate forceps delivery


Immediate cesarean delivery
Amnioinfusion of the fetus and supplemental oxygen to the mother
Discontinue oxytocin infusion and give supplemental oxygen to the

mother
e) Observation and reevaluation in 2 hours
The correct answer is D
Explanation
These are examples of repetitive late decelerations. Late decelerations
are felt to be consistent with uteroplacental insufficiency. This can be
due to a number of reasons, including the following: the maternal
circulation is not adequately perfusing the placental bed; maternal
hypoxia; inadequate exchange across the placental bed (Abruption,
infarct); and also inadequate fetal perfusion of the placenta. Since
this is a recent change in the prior character of the fetal heart
tracing and since the patient is on oxytocin infusion, allowing

intrauterine resuscitation would be the most optimal choice. Should no


improvement in the fetal condition occur, then the next step after this
would be delivery.

A. Instrumented vaginal delivery is never indicated before the cervix is


completely dilated.
B. Cesarean delivery would be indicated if measures to allow
intrauterine resuscitation are unsuccessful (left lateral position,
supplemental oxygen to the mother, discontinue contractions).
C. Amnioinfusion is appropriate for repetitive variable decelerations
must be evaluated and measures undertaken to improve the fetal heart
tracing. It is possible in the future, with the use of fetal pulse
oximetry, that in some conditions this heart tracing can be further
observed without intervention.
A 28-year-old G2P2 woman, noticed a painful mass in her left breast.
Examination disclosed a discrete 2 cm apparently cystic mass in the
upper outer quadrant. Supraclavicular and axillary areas were negative.
Needle aspiration was done but it did not eliminate the mass. The most
appropriate next step in management is:

a) Mammography
b) Excisional biopsy
c) Needle aspiration
d) Reexamination after onset of menses
e) Excisional biopsy with lymph nodes sampling
The correct answer is A
Explanation
Initially, you should try to differentiate solid from cystic lumps
because cysts are rarely cancerous. Typically, ultrasonography is done.
Lesions that appear cystic are sometimes aspirated, and solid lumps are
evaluated with mammography followed by imaging-guided biopsy. Sometimes
evaluation of all lumps with needle aspiration is done; if no fluid is
obtained or if aspiration does not eliminate the lump, mammography
followed by imaging-guided biopsy is done.
*Additional notes:*
Fibrocystic changes (previously, fibrocystic disease) is a catchall term
that refers to mastalgia, breast cysts, and nondescript lumpiness, which
may occur in isolation or together; breasts have a nodular and dense
texture and are frequently tender when palpated. Fibrocystic changes
cause the most commonly reported breast symptoms and have many causes.
Most causes are not associated with increased risk of cancer; they
include adenosis, ductal ectasia, simple fibroadenoma, fibrosis,
mastitis, mild hyperplasia, cysts, and apocrine or squamous metaplasia.
Other causes, particularly if fibrocystic changes require biopsy, may
slightly increase risk of breast cancer. Fibrocystic changes are more
common among women who had early menarche, who had their first live

birth at age > 30, or who are nulliparous.


Red flags: Certain findings are of particular concern:
-Lump fixed to the skin or chest wall
-Stony hard, irregular lump
-Skin dimpling
-Matted or fixed axillary lymph nodes
-Bloody nipple discharge
Interpretation of findings: Painful, tender, rubbery lumps in younger
women with a history of similar findings suggest fibrocystic changes.
On routine yearly exam of an otherwise healthy 45-year-old woman, you
note a 1-cm erosive ulceration on the lower portion of the ectocervix.
Which of the following is the most appropriate next step?

a)
b)
c)
d)
e)

Punch biopsy of the lesion


Viral culture of the lesion for herpes simplex virus
Pap smear of the cervix
Dark field microscopy of a scraping of the lesion
Cold knife conization of the cervix

The correct answer is A


Explanation
It is important to remember that a Pap smear is only a screening test
for cervical cancer. If a Pap smear is done of a visible cancer, it will
only show cancer about 50% of the time. With a visible lesion, you must
biopsy the lesion to diagnose; if it is invasive cancer then you would
proceed with the appropriate staging workup. If the biopsy came back as
microinvasive, a cone biopsy would be the next step.
B. Although this could be a herpetic lesion, any visible lesion of the
cervix must be biopsied to rule out malignancy.
C. A Pap smear is only a screen. In the face of a visible cancerous
lesion, the Pap smear will not be conclusive for cancer in a significant
percentage of cases. If a lesion is seen, a biopsy must be done.
D. Although this could be a primary syphilitic chancre, the most likely
diagnosis is a cervical neoplasia.
E. This would be done if the biopsy came back inconclusive for invasion
(i.e., microinvasive). If the punch biopsy came back conclusive for a
frankly invasive carcinoma, it would be at least a stage IB, requiring
radical hysterectomy or radiation therapy.
A woman whose mother had osteoporosis wants prophylaxis for this
condition. What is the most appropriate treatment?

a) Vitamin D and calcium


b) Swimming
c) Analgesics
d) Vitamins
e) Avoiding physical activities
The correct answer is A
Explanation
Osteoporosis is a condition in which a progressive decrease in the
density of bones weakens the bones, making fractures likely. Risk
factors for osteoporosis in women are: family members with osteoporosis,
insufficient calcium in the diet, sedentary lifestyle, white or asian
race and thin build.
The most useful diagnostic test is the dual-energy x-ray absorptiometry
(DEXA), which measures bone density at the sites at which major
fractures are likely to occur - the spine and hip. Prevention involves
maintaining or increasing bone density by consuming adequate amounts of
calcium and vitamin D and engaging in weight-bearing exercises.
Exercise of the right type, called ?weight-bearing? or ?load-bearing?
exercise,? helps keep bones strong by causing the muscles and tendons to
pull on the bones, which in turn stimulates bone cells to produce more bone.

All exercise benefits your general fitness. Weight-bearing exercise is


best for strengthening bones. Here are some examples: running and
jogging, gymnastics, eeight lifting -- dumbbells, barbells, machines,
body weight exercises; walking (but less effective than running or jogging)
The least effective exercises for bones are:
-Swimming or water aerobics
-Cycling
-Parachuting and base jumping
-Other minimal weight-bearing exercise activities
A 30 year old primigravida presents at 34 weeks gestational age with
blood pressure of 160/90 mmHg, headache, epigastric pain, visual
abnormalities and 3+ proteinuria. Biophysical profile of the fetus is
8/8. Which one of the following is the best course of action?

a)
b)
c)
d)
e)

Induce labour and attempt vaginal delivery


Start magnesium sulfate intravenously
Perform an emergency C-section
Give betaclomethasone to induce fetal lung maturity
Perform an amniocentesis to assess fetal lung maturity

The correct answer is B


Explanation
Preeclampsia is pregnancy-induced hypertension plus proteinuria.
Eclampsia is unexplained generalized seizures in patients with
preeclampsia. Preeclampsia and eclampsia develop between 20 weeks
gestation and the end of the 1st week postpartum.
Patients with severe preeclampsia require Magnesium sulfate as soon as
diagnosis is made. When delivery is delayed before about 32 to 34 weeks
in patients who are not clinically deteriorating, corticosteroids are
given for 48 h. Delivery is mandated at 34 weeks or when deterioration
of maternal or fetal status or documentation of fetal lung maturity occurs.
Eclampsia always requires delivery after seizures and severe
hypertension have been controlled. All hospitalized patients are checked
frequently for seizures, symptoms of severe preeclampsia, and vaginal
bleeding. BP, reflexes, and fetal heart rate are monitored continuously
or several times a day.
Patients with severe preeclampsia or with eclampsia are often admitted
to the ICU. These patients IV Mg sulfate to stop or prevent seizures and
reduce reflex reactivity and BP. Dose is adjusted based on the patient's
reflexes, blood pressure, and serum Mg levels.
All of the following are appropriate indications for immunization during
pregnancy EXCEPT:

a)
b)
c)
d)
e)

Chronic bronchial asthma during an anticipated influenza epidemic


Known exposure to hepatitis B
Puncture wound of the foot
Known exposure to mumps
Known exposure to hepatitis A

The correct answer is D


Explanation
Remember that live virus vaccines such as measles, mumps, and rubella
are contraindicated. Inactive virus vaccines such as influenza are given
only to those with serious underlying diseases. Tetanus is a toxoid
vaccination.
A. Influenza is a killed virus vaccine and is safe in pregnancy.
B. Hepatitis B is not a live virus vaccine and is safe in pregnancy.
C. Tetanus vaccination is a toxoid, not a virus.
E. The safety of hepatitis A vaccine in pregnancy has not been
determined. However, since the vaccine is made from inactivated virus,
the risk is thought to be very low.
A 28 year old female presents for evaluation of a persistent thin

discharge, with a fishy odor particularly noticeable after


intercourse. She has no dyspareunia or dysuria, is in a monogamous
relationship, and has used oral contraceptives for many years. Physical
examination reveals no vulvar, vaginal, or cervical erythema. There is a
homogenous white discharge that coats the vaginal walls. The vaginal pH
is 7.0 and on microscopy you note stippled epithelial cells but no
hyphae or trichomonads. Which one of the following is true regarding
this patient?

a) The treatment of choice may interact with alcohol


b) The patients partner needs to be treated simultaneously
c) The diagnosis should be confirmed with a culture
d) Oral contraceptives contribute to the risk for this condition
The correct answer is A
Explanation
The patient has the typical symptoms and signs of bacterial vaginosis.
There is no need for confirmatory testing. The treatment of choice is
oral metronidazole, which may cause a disulfiram-like interaction with
alcohol. Treatment of the partner has not been shown to improve the outcome.
Oligohydramnios is associated with which one of the following fetal
conditions

a) Renal dysplasia
b) Fetal erythroblastosis
c) Tracheo-esophageal fistula
d) Down syndrome
e) Anencephaly
The correct answer is A
Explanation
Potter syndrome is a term used to describe the typical physical
appearances of a fetus or neonate due to a dramatically decreased
amniotic fluid volume oligohydramnios, or absent amniotic fluid
anhydramnios, secondary to renal diseases such as bilateral renal
agenesis. Other causes of Potter syndrome can be obstruction of the
urinary tract, polycystic or multicystic kidney diseases, renal
hypoplasia and rupture of the amniotic sac.
The decreased volume of amniotic fluid causes the growing fetus to
become compressed by the mother's uterus. This compression can cause
many physical deformities of the fetus, most common of which is Potter
facies. Lower extremity anomalies are frequent in these cases, which
often presents with clubbed feet and/or bowing of the legs.
A woman develops swelling in her labia minora. The area is painful and
red. See illustration:

What is the most likely diagnosis

a)
b)
c)
d)

Hematoma
Bartholin cyst
Gartner duct cyst
Lipoma

The correct answer is B


Explanation
Bartholin's gland cysts are mucus-filled and occur on either side of the
vaginal opening. They are the most common large vulvar cysts.
Bartholin's cysts may form an abscess, which are painful and usually red.

Most cysts are asymptomatic, but large cysts can be irritating,


interfering with intercourse and walking. Most cysts are nontender,
unilateral, and palpable near the vaginal orifice.
Diagnosis is usually by physical examination. In women < 40,
asymptomatic cysts do not require treatment. Symptomatic cysts may
require surgery. Because cysts often recur after simple drainage,
surgery aims to produce a permanent opening from the duct to the
exterior. A small balloon-tipped catheter (called a ?word? catheter) may
be inserted, inflated, and left in the cyst for 4 to 6 weeks.
An otherwise healthy 21 year old primigravida comes to your office for a
routine visit at 16 weeks gestation. She has had a normal pregnancy to
date, and her only medication is a multivitamin with 0.4 mg folic acid.
You order a maternal serum alpha-fetoprotein level. Adjusted for
gestational age, maternal weight, and race, the results are
significantly elevated. Which one of the following would you now recommend?

a) No further testing
b) Amniocentesis
c) A maternal serum hCG level
d) Fetal ultrasonography
e) Chorionic villus sampling
The correct answer is D
Explanation
A 16-week visit is advised for all pregnant women to offer an
alpha-fetoprotein (AFP) screening for neural tube defects and Down
syndrome. An AFP level 2-5 times the median value for normal controls at
the same gestational age is considered elevated. Approximately 5%-10% of
patients who undergo AFP screening will have an elevated level, and most
of these women will have normal fetuses. Fetal ultrasonography should be
performed to detect multiple gestation, fetal demise, or fetal anomalies
(neural tube defects, ventral abdominal wall defects, and urinary tract
anomalies) as well as to confirm gestational age, as all of these
factors are associated with elevated AFP levels. Amniocentesis is
offered if the ultrasonography does not indicate the reason for the
elevated AFP. Chorionic villus sampling is offered in the evaluation of
suspected chromosomal anomalies as an adjunct to amniocentesis. Serum
hCG would be indicated in the workup of suspected Down syndrome, where
the AFP would be low, not elevated. The hCG level would be expected to
be over 2-5 multiples of the mean (MoM) with Down syndrome.
A woman who is 36 weeks pregnant presents with watery vaginal discharge,
there is no blood. Mother is afebrile and is not contracting. What is
the appropriate next step?

a)
b)
c)
d)

Do sterile speculum exam


Do an ultrasound
Nonstress test (NST)
Give antibiotics to mother

The correct answer is A


Explanation
Premature rupture of membranes (PROM) may occur at term or earlier
(called preterm PROM). Preterm PROM predisposes to preterm delivery.

PROM at any time increases risk of infection in the woman


(chorioamnionitis), neonate (sepsis), or both; prolapse of the cord; and
fetal complications, such as abnormal joint positioning and pulmonary
hypoplasia, which may occur with PROM at < 24 weeks. Group B
streptococci are the most common cause of infection.
Unless complications occur, the only symptom is leakage or a sudden gush
of fluid from the vagina. Fever, heavy vaginal discharge, abdominal
pain, and fetal tachycardia, particularly if out of proportion to
maternal temperature, strongly suggest infection.
Sterile speculum examination is done to verify PROM, estimate cervical
dilation, collect amniotic fluid for culture and fetal maturity tests,
and obtain cervical cultures. Digital pelvic examination, particularly
multiple examinations, increases risk of infection and is best avoided.
Diagnosis is assumed if amniotic fluid appears to be escaping from the
cervix or if the fetal vernix or meconium is visible. Other less
accurate indicators include vaginal fluid that ferns when dried on a
glass slide or turns Nitrazine paper blue (indicating alkalinity, and
hence amniotic fluid, normal vaginal fluid is acidic).
A pregnant woman has a placental abruption. She then develops ecchymosis
and bleeding. You strongly suspect disseminated intravascular
coagulation (DIC). What is the most specific test to confirm this

a)
b)
c)
d)

Increased fibrin degradation products


CBC
PT and INR
PTT
The correct answer is A

Explanation
Disseminated intravascular coagulation (DIC) involves abnormal,
excessive generation of thrombin and fibrin in the circulating blood.
During the process, increased platelet aggregation and coagulation
factor consumption occur. DIC that evolves slowly (over weeks or months)
causes primarily venous thrombotic and embolic manifestations; DIC that
evolves rapidly (over hours or days) causes primarily bleeding.
Severe, rapidly evolving DIC is diagnosed by demonstrating
thrombocytopenia, an elevated PTT and PT, increased levels of serum
fibrin degradation products, and a decreasing plasma fibrinogen level.
Treatment includes correction of the underlying cause and replacement of
platelets, coagulation factors (in fresh frozen plasma), and fibrinogen
(in cryoprecipitate) to control severe bleeding. Heparin is used as
therapy (or prophylaxis) in patients with slowly evolving DIC who have
(or are at risk for) venous thromboembolism.
Tocolysis is contraindicated in all of the following, except

a)
b)
c)
d)

Twin gestation pregnancy, in labor at 33 weeks


Cervix 4 cm
Chorioamnionitis
Single pregnancy in labor at 34 weeks

The correct answer is B


Explanation
Tocolysis is the delaying or inhibition of labor during the birth
process. It is a way to ?buy time? when a woman goes into pre-term
labor, but conditions are not favorable for delivery. Common medicine
used for tocolysis are Terbutaline, Magnesium Sulfate, Calcium Channel
Blockers and NSAID (Indomethacin).
Tocolytic therapy should only be considered between 23 and 33 completed
weeks gestational age. There is no good evidence that tocolytic therapy
is successful in the presence of multiple gestation, ruptured membranes,
or with cervical dilation >4cm.

But certain conditions are contraindications for tocolysis. Those are:


bleeding (placenta previa or abruption), maternal HTN, maternal DM,
maternal heart disease, eclampsia or pre-eclampsia, chorioamnionitis,
erythroblastosis fetalis, severe congenital anomalies, fetal
distress/demise and IUGR.
Alpha-fetoprotein (AFP) is increased on a triple screen test when the
fetus has

a)
b)
c)
d)

Meningomyelocele
Renal agenesis
Down syndrome
Esophageal atresia

The correct answer is A


Explanation
With maternal alpha-fetoprotein, elevated levels suggest neural tube
defects such as (open spina bifida, meningomyelocele, anencephaly)
increased risk of pregnancy complications (eg, intrauterine growth
restriction, abruptio placentae), or, occasionally, twins or other
multifetal pregnancy. Closed spina bifida is usually not detected.
A meningomyelocele is a defect that is large enough to allow meninges
and a portion of spinal cord to protrude through the defect. Such
defects can be suggested by an elevated maternal serum alpha-fetoprotein.
A 25-year-old female has unprotected intercourse and chooses to take
Plan B (two 0.75 mg tablets of levonorgestrel, taken 12 hours apart) as
a form of emergency contraception. Plan B has been shown to

a)
b)
c)
d)
e)

Prevent implantation, ovulation and fertilization


Protect against sexually transmitted diseases
Can be an effective routine contraceptive
Be teratogenic to an already established pregnancy
Be effective only if used within 24 hours of unprotected intercourse

The correct answer is A


Explanation
Emergency contraception with Plan B has been available for several
years. It has been shown to prevent implantation, ovulation, and
fertilization. It is not teratogenic to a developing fetus. It is not
recommended as routine contraception; failure rates tend to accumulate
over time. Plan B does not protect against sexually transmitted
diseases. It is effective when used up to 72 hours after unprotected
intercourse (other studies have demonstrated effectiveness at 5 days
also), but the longer the patient waits to take the first dose, the more
its effectiveness wanes.
In a complete hydatidiform mole, what are the origins of the chromosomes

a)
b)
c)
d)
e)
The correct answer is D

Maternal haploid
Paternal haploid
Maternal diploid
Paternal diploid
Maternal triploid

Explanation
Complete mole, paternal diploid, 95% XX, 5% XY. Incomplete mole:
triploid, with two sets of paternal origin, 69 XXZ, etc. A complete mole
does not have a fetus. Due to the high levels of HCG, it can be
associated with hyperthyroidism, theca lutein cysts, and hyperemesis
gravidarum. It can also be associated with preeclampsia in the first
trimester.
A 24-year-old white gravida 2 para 1 at 39 weeks gestation who is
currently not in active labor is seen in the labor and delivery suite
for evaluation of decreased fetal movement. Her blood pressure is 120/74
mm Hg and she has a temperature of 36.8 C (98.4 F). A representative
portion of her nonstress test is shown here. The best management at this
time is

a)
b)
c)
d)
e)

Immediate cesarean section


A biophysical profile
Urinary estriol determination
A repeat nonstress test in 2 days
Human placental lactogen determination

The correct answer is B


Explanation
A reactive (negative) nonstress test (NST) is characterized by the
presence of 2 or more accelerations of the fetal heart rate in 20
minutes or less. The accelerations must be at least 15 beats per minute
above the baseline. A nonreactive NST is characterized by fewer than 2
accelerations in a 40-minute interval. A nonreactive NST is a screening
test and biophysical profile (BPP) or a contraction stress test (CST)
must be urgently completed to further assess fetal status. NSTs are
commonly nonreactive and most CSTs and BPPs subsequently show normal

results; therefore, an immediate cesarean section is unwarranted.


Estriol and human placental lactogen levels take time to complete and
are difficult to interpret.
In performing preconceptual counseling to a young couple, you inform
them that the most susceptible time period for teratogen exposure to the
human embryo is at which embryonic week (week after conception)?

a)
b)
c)
d)
e)

2 weeks
6 weeks
10 weeks
14 weeks
18 weeks

The correct answer is B


Explanation
Organogenesis is the most susceptible time during pregnancy for the
fetus. Gestational age of maximal embryonic susceptibility to
teratogens, week 6.
A. See explanation.
C. See explanation.
D. See explanation.
In the first 100 days of pregnancy, human chorionic gonadotropin
(Beta-HCG) titer doubles

a)
b)
c)
d)
e)

Every day
Every week
Every 2 days
Every month
Every 2 weeks

The correct answer is C


Explanation
Concentration of beta HCG throughout normal pregnancy
Beta HCG "the pregnancy hormone" is produced by the cells of the
implanting egg and can be produced in the absence of an embryo. Beta hCG
can be detected in maternal plasma or urine by 8 to 9 days after
ovulation. The chart to the right illustrates the normal rise and fall
of hCG levels throughout pregnancy. hCG reaches a peak level at about 8
to 10 weeks and then declines for the remainder of the pregnancy.

Beta hCG levels should double approximately every 2 days for the first
four weeks of pregnancy. As pregnancy progresses the doubling time
increases. By 6 to 7 weeks gestation beta hCG levels may take as long as
3 1/2 days to double.
A 40-year-old multiparous patient presents with a 10-day history of
heavy vaginal bleeding and lower abdominal cramping that began at the
expected time of her menses. Pelvic examination reveals a 6-cm mass
judged to be a prolapsed submucosal myoma protruding from the cervix on
a 1.5 cm stalk. The uterus is enlarged to twice normal size and is

mobile. Active bleeding is present, and the patients hematocrit is 26%.


Which of the following is optimal management at this time?

a) Transfusion and vaginal hysterectomy


b) Transfusion and abdominal hysterectomy
c) Biopsy of the mass and transfusion is necessary
d) Transvaginal myomectomy and transfusion if necessary
e) High dose birth control pills
The correct answer is D
Explanation
The most likely diagnosis is a prolapsed submucous fibroid. If this
patient were 80 years old, presenting in the same fashion, one would be
suspicious of a stromal sarcoma. Although this could be a carcinoma, the
simplest and safest way to stop the bleeding is to remove the mass
vaginally (ligate the stalk and then excise). The mass can be sent for
pathologic evaluation with further therapy is indicated.
A. Although this would stop her bleeding, it carries a much higher risk
to the patient (blood loss, ureteral injury, bladder injury). Should the
mass turn out to be a malignancy, then the wrong procedure may have been
done.
B. See answer to A.
C. Biopsy of the mass alone will not solve the bleeding. Since the stalk
is visible and only 1.5 cm in diameter, then excision of the mass would
be the best therapy.
E. High dose birth control pills can be used for dysfunctional uterine
bleeding. Here, the bleeding is due to a prolapsed fibroid. OCPs will
not affect her bleeding and pain.
A 20-year-old multiparous woman underwent AFP testing at 16 weeks
gestation. The initial result was 0.3 multiple of the median (MoM)
(normal 0.5-2.0 MoM). The result of a repeat test performed 5 days later
was 0.2 multiple of the median. The abnormality most likely to be
associated with these results is:

a)
b)
c)
d)
e)

Trisomy 21
Triploidy
Twin pregnancy
Meningomyelocele
Trisomy 13

The correct answer is A


Explanation
Due to the large overlap between a normal fetus and a fetus with trisomy
21 or 18, low alpha fetoprotein values should not be repeated, as the
second value is more likely to be reported as normal (regression to the
mean). Most patients now receive a triple screen, which includes AFP,
HCG, and uE3. This triple screen test looks at the age-related risk for
aneuploidy for each of the markers and then predicts a composite risk
for both trisomy 21 and trisomy 18. With the triple screen, about 60-70%
of all trisomy 21 pregnancies can be detected prenatally. In trisomy 18,
all three values are low. Trisomy 13 cannot be screened by a low MSAFP.
B. Triploidy is not associated with a low MSAFP
C. Twin pregnancy is often associated with an elevated MSAFP when
compared to a singleton pregnancy
D. Open neural tube defects are associated with an elevated MSAFP
E. Trisomy 13 cannot be routinely detected by the MSAFP value, unlike
trisomies 18 and 21
Which one of the following is recommended for routine prenatal care?

a)
b)
c)
d)
e)
The correct answer is D

Hepatitis C antibody testing


Parvovirus antibody testing
Cystic fibrosis carrier testing
HIV screening
Examination of a vaginal smear for clue cells

Explanation
HIV screening is recommended as part of routine prenatal care, even in
low-risk pregnancies. Counseling about cystic fibrosis carrier testing
is recommended, but not routine testing. Hepatitis C and parvovirus
antibodies are not part of routine prenatal screening. Routine screening
for bacterial vaginosis with a vaginal smear for clue cells is not
recommended.
Of the following, the initial treatment of choice in the management of
severe hypertension during pregnancy is

a) Labetalol (Trandate, Normodyne) intravenously


b) Reserpine (Serpasil) intramuscularly
c) Nifedipine (Procardia, Adalat) sublingually
d) Enalapril (Vasotec) intravenously
The correct answer is A
Explanation
In pregnant women with severe hypertension, the primary objective of
treatment is to prevent cerebral complications such as encephalopathy
and hemorrhage. Intravenuous hydralazine, intravenous labetalol, or oral
nifedipine may be used. Sublingual nifedipine can cause severe
hypotension, and reserpine is not indicated. Nitroprusside can be used
for short intervals in patients with hypertensive encephalopathy, but
fetal cyanide toxicity is a risk with infusions lasting more than 4
hours. ACE inhibitors are never indicated for hypertensive therapy
during pregnancy.
A 28 year old gravida 2 para 1 presents to the emergency department at
16 weeks gestation. She has noted the sudden onset of dyspnea, pleuritic
chest pain, and mild hemoptysis. Both calves are mildly edematous and
somewhat tender. A lung scan shows a high probability of pulmonary
emboli. Which one of the following would be appropriate management at
this time?

a) Placement of an inferior venous umbrella filter


b) Intravenous heparin for 5-10 days followed by subcutaneous heparin
for the duration of the pregnancy
c) Intravenous heparin for 5-10 days followed by warfarin
anticoagulation
d) Warfarin therapy only, with the prothrombin time maintained at
18-20 seconds (INR 2.0-3.0)
e) Aspirin, 81 mg/day throughout the pregnancy
The correct answer is B
Explanation
The risk of pulmonary embolism is five times higher in pregnant women
than in nonpregnant women of similar age, and venous thromboembolism is
a leading cause of illness and death during pregnancy. Warfarin, which
readily crosses the placenta, should be avoided throughout pregnancy. It
is definitely teratogenic during the first trimester, and extensive
fetal abnormalities have been associated with exposure to warfarin in
any trimester.
Because heparin does not cross the placenta, it is considered the safest
anticoagulant to use during pregnancy. Initially, patients with venous
thromboembolism during pregnancy should be managed with heparin given
according to the recommendations for nonpregnant patients. These women
should receive intravenous heparin for 5-10 days followed by
subcutaneous heparin for the duration of the pregnancy. Warfarin can be
given after delivery, since it is not present in breast milk.
The indications for placement of an inferior vena cava filter are not
changed by pregnancy, and include any contraindication to anticoagulant
therapy, the occurrence of heparin-induced thrombocytopenia, and
recurrence of pulmonary embolism in a patient receiving adequate
anticoagulant therapy.
There are no data to support the use of aspirin for treatment or
prophylaxis of pulmonary embolism either during or after pregnancy.

A 28-year-old white female at 28 weeks gestation with an uncomplicated


pregnancy presents with pain in the right wrist. She thinks it may be
related to hand-washing several articles of clothing in a creek and
hanging them on a line to dry while on a camping trip. The pain in the
wrist radiates down the thumb and the radial side of the wrist. There is
no history of trauma or penetrating injury. On examination there is
tenderness when the right radial styloid is compressed. The pain is
exacerbated by ulnar rotation of the wrist while the patients fingers
are folded over the thumb.
The most likely diagnosis is

a) Osteoarthritis of the first carpometacarpal joint


b) Occult navicular fracture
c) Radial sensory nerve entrapment
d) Carpal tunnel syndrome
e) De Quervains tenosynovitis
The correct answer is E
Explanation
De Quervains tenosynovitis is a cumulative movement disorder due to
chronic overuse of the wrist and hand, involving the abductor pollicis
longus and extensor pollicis brevis tendons, which share a common tendon
sheath. Repeated forceful gripping, grasping, and wringing movements
lead to thickening of the tendon sheath and subsequent inflammation and
stenosis of the sheath as it passes over the distal radius. This can
progress to fibrosis and loss of flexibility in the thumb in flexion.
The condition is more common in pregnancy, after trauma, and with
rheumatoid arthritis.
The diagnosis is made by physical examination. There is tenderness over
the radial styloid, and resisting thumb extension and abduction will
elicit pain. Finkelsteins test, folding the fingers over the thumb and
gently passively deviating the thumb in an ulnar direction, will often
cause such pain that the patient will lift the shoulder to prevent the
examiner from stretching the tendon.
Other diagnostic considerations include osteoarthritis of the first
carpometacarpal joint, which may also cause pain over the radial
styloid, and a positive Finkelsteins test. If injecting the site with
local anesthetic gives relief, an arthritic etiology can be excluded.
Navicular (scaphoid) fracture is caused by a fall or a direct blow to
the wrist. The symptoms are pain on the thumb side of the wrist,
swelling in that area, and difficulty gripping objects.
Individuals with radial sensory nerve entrapment will have paresthesias
over the dorsum of the hand and a positive Tinels sign over the nerve
in the mid-forearm, just distal to the belly of the brachioradialis
muscle. Hyperpronation of the forearm reproduces these symptoms.
Carpal tunnel syndrome involves entrapment of the median nerve, causing
pain in the thumb, index finger, long finger, and radial half of the
ring finger.
A 52-year-old recently menopausal female has developed hot flashes that
she describes as severe. She currently takes no prescribed or
over-the-counter medications and declines estrogen replacement therapy.
Which one of the following would be most effective for relieving this
patients symptoms

a)
b)
c)
d)
e)
The correct answer is A
Explanation

Fluoxetine (Prozac)
Black cohosh
Red clover
Soy protein
Vitamin E

Hot flashes have been shown to be most effectively treated with hormone
replacement therapy (HRT). However, given the concerns regarding HRT,
alternative strategies for managing hot flashes have been evaluated.
Placebo-controlled trials have shown that vitamin E, soy protein, red
clover, and black cohosh are not better or only slightly better than
placebo in reducing hot flashes. SSRIs have been shown to be
significantly more effective than placebo in relieving this symptom.
Methyldopa has shown some benefit compared to placebo, but causes
frequent side effects.

A 49-year-old female complains of disabling hot flashes. Her last


menstrual period was 6 months ago.
Which one of the following nonhormonal treatments has been shown to be
helpful in alleviating menopausal hot flashes

a)
b)
c)
d)
e)

Paroxetine (Paxil)
Black cohosh
Regular exercise
Phytoestrogens
Clonidine (Catapres)

The correct answer is A


Explanation
The rate of self-reported improvement with placebo treatment of hot
flash symptoms after menopause is significant; thus randomized,
controlled trials are important when determining efficacy of treatment.
There is no evidence that regular exercise, dietary soy, various
phytoestrogen preparations, black cohosh, or other alternative therapies
are effective in improving menopausal hot flashes. Studies have shown
little or no benefit from clonidine. Paroxetine has been shown to have
modest value in the treatment of hot flashes.
A 28 year old gravida3 para2 develops painless vaginal bleeding at 30
weeks gestation. Which one of the following is most accurate regarding
diagnostic evaluation by ultrasonography?

a) Transvaginal ultrasonography is absolutely contraindicated for the


evaluation of this patient
b) The bladder must be filled before diagnostic ultrasonography is
performed in this patient
c) Transvaginal ultrasonography has a higher sensitivity and
specificity for placenta previa than transabdominal ultrasonography
d) Ultrasonography is more useful for the diagnosis of bleeding from
placental abruption than for bleeding from placenta previa
e) Sonographic evaluation of vaginal bleeding during pregnancy is
useful only in the first trimester
The correct answer is C
Explanation
Transvaginal ultrasonography has been shown to have higher sensitivity
and specificity than transabdominal ultrasonography, and has been
demonstrated to be a safe procedure. The angle between the cervix and
the vaginal probe is sufficient to prevent probe entry into the cervix.
A distended bladder is a common cause of false-positive ultrasound
evaluation for placenta previa. The sensitivity of ultrasonography is
lower for abruptio placenta than for placenta previa. This is true even
for the abdominal approach.
A 27 year old woman has just given birth to a 3200-g (7-lb 1-oz) boy
with an Apgar score of 9 at both 1 and 5 minutes. mother is HIV positive
and received no prenatal care. She acquired HIV infection from her
husband who is also HIV positive. When you visit the mother to discuss
the medical care of her baby and to obtain more history, you find her in
tears. She asks you what the chances are of her baby being infected with
HIV. It is most appropriate to tell her which of the following?

a) Since she has no symptoms of AIDS, there is a strong likelihood


that her baby will not be infected
b) Having untreated syphilis at delivery increases her baby's risk
for being infected by 50%
c) Her baby has at least a 50% chance of not being infected and will
need to be followed closely since it is difficult to make the diagnosis
at birth
d) If the results of ELISA and Western blot testing of the baby are
positive, the baby is definitely infected
e) Because she acquired her infection through a heterosexual
transmission, there is only a 20% risk that her baby is infected
The correct answer is C
Explanation
The risk of vertical transmission of HIV (from mother to child) has been
revived a number of times since the initial clinical description of HIV
and AIDS in the 1980s. As it currently stands, the consensus opinion on
risk is approximately 50% in women with children that have not received
perinatal therapy with AZT. The medication protocol reduces the risk to
less than 25%. Because newborn children have antibody complements that
are from their mothers, it is crucial to follow them over the first year
of life as their own immune system develops.
Symptoms of AIDS are not related to risk of transmission of HIV. AIDS is
defined as a CD4 count of less than 200 with an HIV-associated disease
present (such as PCP pneumonia or Kaposi sarcoma). The risk of infection
instead relates to the viral burden in the bloodstream of the mother.
This burden is quantified using HIV RNA copy number. The HIV RNA copy
number relates to symptomatic AIDS in a complex manner but low copy
number patients can be symptomatic and vice versa.
A 32-year-old gravida 3 para 2 is in labor at term following an
uncomplicated prenantal course. As you deliver the fetal head it
retracts against the perineum. Downward traction fails to free the
anterior shoulder.
The most appropriate course of action would be to

a)
b)
c)
d)
e)

Apply increasingly strong downward traction to the fetal head


Have an assistant apply fundal pressure
Deliberately fracture the clavicle of the fetus
Begin an intravenous nitroglycerin drip
Place the mothers thighs on her abdomen

The correct answer is E


Explanation
While there are several risk factors for shoulder dystocia, most cases
occur in fetuses of normal birth weight and are not anticipated. Once it
does occur, excessive force should not be applied to the fetal head or
neck and fundal pressure should be avoided, as these maneuvers are
unlikely to free the fetus and can injure both mother and infant.
Up to 40% of shoulder dysotica cases can be successfully treated with
the McRoberts maneuver, in which the maternal hips are flexed and
abducted, placing the thighs up on the abdomen. Adding suprapubic
pressure can resolve about half of all shoulder dysoticas. Additional
maneuvers include internal rotation, removal of the posterior arm, and
rolling the patient over into the all-fours position. As a last resort,
one can deliberately fracture the fetal clavicle, perform a cesarean
section with the vertex being pushed back into the birth canal, or have
the surgeon rotate the infant transabdominally with vaginal extraction
performed by another physician. General anesthesia or nitroglycerin,
orally or intravenously, may be used to achieve musculoskeletal or
uterine relaxation. Intentional division of the cartilage of the
sympysis under local anesthesia has been used in developing countries,
but should be used only if all other maneuvers have failed and a
cesarean delivery is not feasible.

A 27 year old white female has a 10 year history of significant


premenstrual dysphoria. Her condition has significantly worsened in the
past 3 years, to the point that it is endangering her marriage of 5
year. Her symptoms are worse for the 10 days prior to her menstrual
period and are gone by day 2 of her period. She has tried several
measures without success, including birth control pills, various herbal
preparations, and counseling at a womans health center. You recommend

a)
b)
c)
d)
e)

Reduction of caffeine and refined sugar intake


Alprazolam (Xanax)
Bupropion (Wellbutrin)
Progesterone for 2 weeks starting at about the time of ovulation
Fluoxetine (Prozac, Serafem) for the last 2 weeks of the menstrual

cycle
The correct answer is E
Explanation
SSRIs are considered first-line treatment for premenstrual dysphoric
disorder. Several randomized trials have shown that they are superior to
placebo for this condition. Fluoxetine and sertraline have been studied
the most. There have been no controlled trials to support anecdotal
reports of benefit from the reduction of caffeine or refined sugar.
Studies using alprazolam have shown it to be effective for premenstrual
anxiety only. Progesterone has not been proven more effective than
placebo in clinical trials, and bupropion is less effective than agents
that primarily boost serotonergic activity. Treatment during the luteal
phase alone has been shown to be more effective than continuous
treatment for this condition.
You are performing the admission history and physical on an otherwise
healthy 17-year-old woman who presents to Labor & Delivery for induction
at 41 weeks gestation. Her past medical history is unremarkable, and in
reviewing her prenatal records, you note that no abnormalities were
found on her initial prenatal physical exam at 8 weeks gestation. On
your admission physical, you note subtle systolic ejection murmurs heard
best over the aortic valve region, and you rate it a 1 on a scale of 6.
This murmur is most likely due to:

a) Increased cardiac size


b) Increased stroke volume
c) Decreased peripheral resistance
d) Decreased cardiac valvular resistance
e) Increased hematocrit in pregnancy
The correct answer is B
Explanation
Maternal blood volume is up 40-50% by the start of the third trimester.
Maternal stroke volume and cardiac output are up by a similar fraction,
leading to an increase in turbulent flow with ventricular emptying. This
gives rise to a flow murmur at the aortic valve. The hematocrit in
pregnancy should decrease since the red cell mass only goes up 30%
(physiologic anemia of pregnancy).
A. Ejection murmurs are due to the increase in turbulent flow.
C. Although peripheral resistance decreases, reaching a nadir in the
second trimester, the murmur is still due to turbulent flow secondary to
the 50% increase in cardiac output.
D. See answer to A.
E. The hematocrit decreases in pregnancy due to a relative increase in
plasma volume relative to red cell mass.
A 36 year old female presents with a mucopurulent vaginal discharge.
Gram stain of a cervical swab shows gram negative diploccocci. Which of
the following is correct?

a) If untreated this condition will likely resolve spontaneously

b) The most likely diagnosis is Strep group B


c) Appropriate treatment is Metronidazole vaginal cream
d) Public Health will most likely need to be notified
The correct answer is D
Explanation
Gonorrhea is caused by the bacterium Neisseria gonorrhoeae. It typically
infects epithelia of the urethra, cervix, rectum, pharynx, or eyes,
causing irritation and purulent discharge. Dissemination to skin and
joints occurs infrequently. Diagnosis is by culture or genetic methods.
Several antibiotic regimens, oral or parenteral, can be used. Diagnosis
is by Gram stain, culture, or a number of commercially available genetic
techniques. Confirmed cases should be reported to the public health
system. Reporting is mandatory throughout Canada.
A mother is nearing the end of maternity leave. She asks for your advice
regarding breastfeeding her infant after she returns to work. Which one
of the following would be accurate advice?

a)
b)
c)
d)
e)

Refreezing breast milk destroys some proteins


Separation of breast milk indicates spoilage
Breast milk in a refrigerator at work is a health hazard
Breast milk should be thawed in boiling water
Breast milk should be stored in glass bottles

The correct answer is A


Explanation
Physicians commonly counsel women on how to continue breastfeeding after
maternity leave. Breastfeeding should be well established by 4 weeks
postpartum. Pumping may allow a woman to continue to provide breast milk
while not physically available. Refreezing breast milk destroys valuable
proteins and is not recommended. Separation of breast milk, in the
refrigerator or at room temperature, is normal; the milk just needs to
be shaken to re-emulsify the fat. The Occupational Safety and Health
Administration (OHSA) states that exposure to breast milk does not
constitute an occupational hazard. Breast milk should never be thawed in
boiling water or in the microwave; this destroys valuable proteins.
Breast milk may be stored in either glass or plastic bags or bottles.
A 28 year old gravida 2 para 1 successfully delivers a full-term infant
vaginally. Her first child was delivered by cesarean section. With this
delivery there are no signs of maternal pain, significant bleeding, or
hemodynamic compromise during the first two stages of labor. However, 30
minutes after delivery of the infant there is still no umbilical cord
elongation or contraction of the uterus, and the placenta is not at the
cervical os. Manual exploration of the uterus reveals that the placenta
is attached in a low anterior position and there is no easily
discernible plane of separation. During this procedure, uterine bleeding
becomes brisk and continues despite bimanual massage and administration
of oxytocin (Pitocin), 10 U intramuscularly.
Vital Signs
Temperature 37.0C (98.6F)
Pulse 120 beats/min
Blood pressure 88/46 mm Hg
Respirations 20/min
O2 Saturation 98% on room air
Which one of the following should be done next?

a) Increase traction on the umbilical cord


b) Insert a fist into the uterine fundus and use firm upward pressure
toward the patients head
c) Use a ring forceps to forcefully remove the placenta from the uterus
d) Prepare for urgent hysterectomy
e) Wait an additional 15-20 minutes for spontaneous placental separation
The correct answer is D

Explanation
This case demonstrates the clinical outcome of abnormal placental
attachment. In this situation, the uterus has either partially or
completely attached to the myometrium instead of the endometrium, which
normally sloughs after birth, thus resulting in detachment of the
placenta. The three forms of abnormal attachment (accreta, increta, and
percreta) are difficult to differentiate by examination and are treated
as the same condition, sometimes generally referred to as ?placenta
accreta?. No conservative mode of management will definitively treat
these conditions when a significant portion of the placenta is
abnormally attached. Therefore, if bleeding becomes profuse and the
patients condition is unstable, prompt hysterectomy is required.
This patient has low blood pressure and an elevated pulse, likely due to
significant blood loss. It is inappropriate to await spontaneous
delivery of the placenta, which may never occur. Trying further to
remove the placenta will only result in increased bleeding. Pulling on
the umbilical cord may result in uterine inversion. There is no
indication for any other manual maneuver.
A 47 year old gravida 3 para 3 is seen for a physical examination. She
has a total abdominal hysterectomy for benign uterine fibroids. Which
one of the following is the recommended interval for Papanicolaou (Pap)
screening in this patient?

a) Every 5 years
b) Every 3 years
c) Every 2 years
d) Annually
e) Routine screening is not necessary
The correct answer is E
Explanation
Most women who have undergone hysterectomy are not at risk of cervical
cancer, as they underwent the procedure for benign disease and no longer
have a cervix. The Preventive Services Task Force recommendations stated
that routine Papanicolaou (Pap) screening is unnecessary for these
women. Nevertheless, data from the Behavioral Risk Factor Surveillance
System (1992-2002) indicated that in the previous 3 years, some 69% of
women with a previous history of hysterectomy for benign causes had
undergone screening.
A 25 year old black female at 32 weeks gestation is found to have a
blood pressure of 170/100 mm Hg during a prenatal visit. Previously, her
blood pressure had been normal. Although her blood pressure remains
elevated during the remainder of her pregnancy, she denies any
headaches, visual symptoms, chest pain, dyspnea, or abdominal pain. She
develops edema, but does not have proteinuria. Her delivery is
uneventful, and her blood pressure at her first postpartum visit is
124/78 mm Hg.
This patient most likely had

a) Mild preelampsia
b) Severe preeclampsia
c) Gestational hypertension
d) Preeclampsia superimposed on prehypertension
e) Preeclampsia with HELLP syndrome
The correct answer is C
Explanation
Hypertension may be diagnosed for the first time during pregnancy.
Gestational hypertension is defined as systolic blood pressure (SBP) >
140 mm Hg or diastolic blood pressure (DBP) > 90 mm Hg on at least two
occasions during a pregnancy. In this condition, the blood pressure
returns to normal within 6 weeks after delivery. The presence of
hypertension and proteinuria (> 300 mg/24 hours) indicates preeclampsia.

Criteria for severe preeclampsia include an SBP ?160 mm Hg or a DBP ?110


mm Hg; proteinuria ?5 g/224 hrs; oliguria; cerebral or visual symptoms;
pulmonary edema; epigastric or right upper quadrant abdominal pain;
impaired liver function; thrombocytopenia; and intrauterine growth
retardation. The HELLP syndrome consists of hemolysis, elevated liver
enzymes, and thrombocytopenia. Eclampsia is the new occurrence of
seizures in women with preeclampsia. Edema is no longer part of the
diagnostic criteria for preeclampsia. Prehypertension is defined as an
SBP of 130-140 mm Hg or a DBP of 80-90 mm Hg.
All of the following may occur as a result of postpartum pituitary
necrosis (Sheehan syndrome), except

a) Pallor
b) Galactorrhea
c) Bradycardia
d) Amenorrhea
e) Asthenia
The correct answer is B
Explanation
Prolactin deficiency reduces or eliminates a woman's ability to produce
breast milk after childbirth. One cause of low prolactin levels and
deficiency of other pituitary hormones is Sheehan's syndrome, a rare
complication of childbirth. Sheehan's syndrome typically develops
because of excessive blood loss and shock during childbirth, which
results in partial destruction of the pituitary gland. Symptoms include
fatigue, loss of pubic and underarm hair, and inability to produce
breast milk. Prolactin deficiency has no known ill effects in men.
Oral contraceptive pills have been shown to raise the risk of which of
the following?

a) Ovarian cancer
b) Breast cancer
c) Endometrial cancer
d) Cervical cancer
e) Ovarian cysts
The correct answer is B
Explanation
Possible side effects of oral contraceptive pills (OCPs) include
vaginal discharge, changes in menstrual flow, breakthrough bleeding,
unusual build-up of the uterine lining, nausea, headaches, depression,
vaginitis, urinary tract infection, changes in the breasts, changes in
blood pressure, skin problems, skin improvements, and gum inflammation.
Cautions and contraindications for OCP use are: coagulation, increasing
the risk of deep venous thrombosis (DVT) and pulmonary embolism (PE),
stroke and heart attack.
Combined oral contraceptives are generally accepted to be
contraindicated in women with pre-existing cardiovascular disease, in
women who have a familial tendency to form blood clots (such as familial
factor V Leiden), women with severe obesity and/or hypercholesterolemia,
and in smokers over age 35.
There is a small increase in the relative risk of breast cancer while
taking OCPs.
Note: Oral contraceptive pills decrease chance of getting endometrial
cancer, ovarian cancer, and ovarian cysts.
A 45 year old white female presents for her yearly health maintenance
examination and Papanicolaou (Pap) test. She has been in good health and
has no family history of significant medical disorders. Her examination
is normal, and she asks about screening for breast cancer. Which one of
the following screening methods would be most appropriate?

a)
b)
c)
d)
e)

A dedicated breast CT scan


Thermography
MRI
Ultrasonography
Mammography

The correct answer is E


Explanation
Of all modalities that are used for screening, the only level A (strong
clinical evidence for effectiveness in screening) technology is
screen-film mammography. Ultrasonography and MRI have level B evidence,
while a dedicated CT scan has level C. There is little clinical evidence
that thermography is effective as a screening tool (evidence level D).
You have diagnosed cervical gonorrhea in a 23 year old female. Which one
of the following is considered first-line therapy for this infection?

a)
b)
c)
d)
e)

Intramuscular ceftriaxone (Rocephin) in a single dose


Erythromycin orally for 7 days
Metronidazole (Flagyl) orally in a single dose
Metronidazole orally for 14 days
Amoxicillin orally in a single dose

Correct Answer:* a)
Explanation
First-line treatment for gonorrhea includes intramuscular ceftriacone,
oral cefixime, ciprofloxacin, ofloxacin, and levofloxacin. Oral
amoxicillin is no longer listed as first-line treatment for gonorrhea.
Erythromycin and metronidazole are useful in nongonococcal cervicitis,
but are not effective in the treatment of gonorrhea.
What is the most common cause of neonatal mortality in twin pregnancies

a)
b)
c)
d)

Respiratory distress syndrome


Congenital anomalies
Short gestation/low birth weight
Maternal complications of pregnancy

The correct answer is A


Explanation
The following table illustrates the mortality rate due to
cause in twin births:
Cause of neonatal death in a twin

Risk of neonatal mortality (per 1,000 livebirths)

Congenital anomalies

2.8

Maternal complications of pregnancy

1.0

Complications of placenta/cord/membranes

0.5

Short gestation/low birth weight

1.7

Intrauterine hypoxia and birth asphyxia

0.2

Respiratory distress syndrome

3.3

Infections

0.8

Sudden infant death syndrome

0.1

A concerned mother brings in her 16-year-old daughter because she hasn?t

ever had a menstrual period. On exam, the girl is 5 feet 8 inches tall
with mature adult breast development and scant to no pubic nor axillary
hair. Vaginal exam is difficult and you are unable to identify a cervix
nor palpate a uterus. The most likely diagnosis is:

a)
b)
c)
d)
e)

Androgenital syndrome
Imperforate hymen
Turner syndrome
Complete androgen insensitivity syndrome
Rokitansky Kuster Hauser syndrome

The correct answer is D


Explanation
Complete androgen insensitivity syndrome is due to a congenital lack of
androgen receptors. The patient never develops the Mullerian system
since the gonad produces anti-Mullerian hormone (AMH or MIF) form the
Sertoli cells during organogenesis. Although the patient has a male
level of testosterone and male levels of estrogen, since the androgens
are not recognized, the breasts develop due to the presence of
estrogens. Without androgens, these patients often have sparse to no
sexual hair. As the gonad is an XY gonad, it must be removed to prevent
the risk of malignant transformation; this is rare prior to puberty, so
it can be removed after normal pubertal development has occurred (most
common malignancy is a gonadoblastoma).
A. One would see the effects of excess androgen: hair growth,
virilization, etc.
B. The vagina would be behind the imperforate hymen and not visible. If
menses has begun, then there would be a bluish bulging mass (vagina full
of old menstrual blood).
C. A uterus is present in Turner syndrome.
E. Although a uterus is absent in this syndrome, sexual hair should be
present since there is no defect in either androgen production or in
androgen receptors.
A 50-year-old female presents with a history of small amounts of urine
leakage with coughing, sneezing, and exercising over the past few
months. She denies any nocturia, urinary urgency, or frequency. A
urinalysis is normal.
Of the following, the most effective treatment for this patients
problem would be

a)
b)
c)
d)

Kegel exercises
estrogen
oxybutynin (Ditropan)
tolterodine (Detrol)

The correct answer is A


Explanation
The symptoms presented describe stress incontinence. This condition is
commonly confused with urge incontinence, which would be highly unlikely
without accompanying urge, frequency, nocturia, and large amounts of
urine leakage. Urge incontinence may be treated with oxybutynin or
tolterodine, but these are not effective for treating stress
incontinence. Estrogen was at one time touted as a therapy for this
condition also, but now is not considered an effective treatment (level
B evidence). Of the options given, the only effective treatment for
stress incontinence is Kegel exercises. Other effective treatments
include surgery and a pessary.
The most common cause of perinatal mortality is

a) IUGR
b) Congenital abnormality
c) Infection
d) Prematurity
The correct answer is D

Explanation
The perinatal period is defined as the period that commences at 20
completed weeks (140 days) of gestation and ends 28 completed days after
birth. During this time the top causes of death in the fetus are (in
order): prematurity, congenital abdnormalities, infections and
Intra-uterine growth retardation (IUGR).
A 24 year old white female in her first trimester of pregnancy presents
with low-grade fever, myalgias, headache, and a rash consistent with
erythema migrans. Ten days ago she was hiking in an area where deer
ticks are present. She remembers being bitten by a tick which she
discovered and removed 2 days after her hike. Which one of the following
is the most appropriate treatment option?

a)
b)
c)
d)

Amoxicillin
Azithromycin (Zithromax)
Doxycycline
Erythromycin

The correct answer is A


Explanation
Amoxicillin is preferred for the treatment of Lyme disease in children,
as well as for pregnant or lactating women. Doxycycline is effective,
but should not be used in pregnant women. Macrolides are not considered
first-line agents because controlled trails of azithromycin or
erythromycin in patients with erthema migrans found a high rate of
clinical failure.
Which one of the following vaccinations is contraindicated in pregnancy?

a) Diphtheria
b) Hepatitis B
c) Influenza
d) Tetanus
e) Varicella
The correct answer is E
Explanation
The varicella vaccine contains live attenuated varicella-zoster virus.
Immunization during pregnancy is contraindicated because the effects on
the fetus are unknown. Diphtheria, hepatitis B, influenza, and tetanus
vaccines are not contraindicated during pregnancy.
A 24 year old white female contracts a primary genital herpes virus
infection during her 25th week of gestation. Which one of the following
statements is true?

a) She has a high likelihood of miscarriage


b) She should be treated with antiviral therapy for 7-14 days
c) She should be scheduled for cesarean section at term
d) She should be advised against breastfeeding her infant
The correct answer is B
Explanation
Most women who develop primary herpes virus (HSV) infection during
pregnancy will deliver healthy babies. Studies have not shown increased
rates of miscarriage, especially after the first trimester. Current
guidelines recommend treatment with antiviral agents for primary HSV
infection in pregnancy. Caesarean section is not routinely recommended
if a woman has had an infection during pregnancy unless active viral
lesions are present at the time of delivery. Breastfeeding is unlikely
to lead to neonatal infection.
A 25 year old woman comes to the office for her first prenatal visit.
Her last menstrual period was 16 weeks ago. This is her first pregnancy,
her family history is unremarkable. She has heard that people of
Mediterranean ancestry are at risk for carrying a gene for
beta-thalassemia. She asks to be tested for this. Which of the following
is the most appropriate diagnostic study?

a)
b)
c)
d)

Complete blood count with red cell indices


Hemoglobin electrophoresis
Red cell osmotic fragility test
Reticulocyte count

The correct answer is B


Explanation
Beta thalassemia, an autosomal recessive trait, has a carrier incidence
of approximately 1:33. For a patient to be affected significantly enough
to put her fetus at risk, she would have a mild microcytic hypochromic
anemia (Hct 28-32%, MCV < 80); a normal blood count and red cell index
rules out such a carrier status.
Hemoglobin electrophoresis can be used for confirmation of her carrier
status (Hgb A2 > 3.5%). The red cell osmotic fragility test is used to
determine the presence of hereditary spherocytosis, an uncommon
autosomal dominant disorder affecting the red cell membrane.
Reticulocyte count merely tells you that the patient is manufacturing
new red cells, and is depressed with anemias of chronic diseases and
aplastic anemia.
Which of the following sets of test results is most consistent with the
euthyroid state in pregnant women? (Values are expressed as increased or
decreased relative to those in nonpregnant patients; T4 = total
thyroxine, FT4I = free thyroxine index, T3RU = triiodothyronine resin
uptake).

a)
b)
c)
d)
e)

T4 increased, FT4I increased, T3RU increased


T4 increased, FT4I increased, T3RU normal
T4 increased, FT4I normal, T3RU decreased
T4 normal, FT4I increased, T3RU decreased
T4 normal, FT4I normal, T3RU normal

The correct answer is C


Explanation
Thyroid-binding globulin levels are increased, leading to an increase in
the amount of bound thyroid hormone. Free thyroid hormone stays in the
normal range, and total thyroid hormone levels are elevated. The T3
resin uptake is inversely proportional to the hormone-binding capacity,
so it decreases in normal pregnancy.
A. FT4I should be normal, T3RU should be decreased.
B. See answer to A.
D. T4 is increased, FT4I is normal.
E. T4 is increased, T3RU is decreased.
All of the following features have been described in women with androgen
insensitivity syndrome, except

a)
b)
c)
d)
e)

Mammary aplasia
Female phenotype
Absence of wolffian duct structures
Short vagina
Absence of mullerian duct structures

The correct answer is A


Explanation
Androgen insensitivity syndrome (AIS) is when a person who is
genetically male (has one X and one Y chromosome) is resistant to male
hormones called androgens. As a result, the person has some or all of
the physical characteristics of a woman, despite having the genetic
makeup of a man.
In its classic form (complete androgen resistance), the person appears
to be female but has no uterus, and has sparse armpit and pubic hair. At
puberty, female secondary sex characteristics (e.g., breasts) develop,
but menstruation and fertility do not.

Incomplete AIS, however, is often discovered during childhood because a


person may have both male and female physical characteristics. Many have
partial fusion of the outer vaginal lips, an enlarged clitoris, and a
short, blind-ending vagina.
The Mllerian ducts will develop to form the fallopian tubes, uterus,
and the upper portion of the vagina. The Wolffian duct develops into a
system of connected organs between the testis and the prostate, the
epididymis, the vas deferens, the seminal vesicle, and the prostate.
Both of these ducts will be absent in AIS.
Which of the following is not true about IUGR (Intrauterine Growth
Retardation)?

a)
b)
c)
d)

Absence of weight gain


Maternal short stature
Low fundal height
Maternal cigarette smoking

The correct answer is B


Explanation
Intrauterine growth restriction (IUGR) is a term used to describe a
condition in which the fetus is smaller than expected for the number of
weeks of pregnancy. Another term for IUGR is fetal growth restriction.
Newborn babies with IUGR are often described as small for gestational
age (SGA). A fetus with IUGR often has an estimated fetal weight less
than the 10th percentile.
Maternal factors associated with IUGR are high blood pressure, chronic
kidney disease, advanced diabetes, heart or respiratory disease,
malnutrition, anemia, infection, substance abuse (alcohol, drugs) and
cigarette smoking.
Factors involving the uterus and placenta are decreased blood flow in
the uterus and placenta, placental abruption (placenta detaches from the
uterus), placenta previa (placenta attaches low in the uterus) and
infection in the tissues around the fetus.
Factors related to the developing baby (fetus) are multiple gestation
(twins, triplets, etc.), infection, birth defects and chromosomal
abnormality.
A 25 year old woman who is pregnant with her third child comes to the
office for a regular prenatal visit. Medical history shows that she
developed deep vein thrombosis of the left calf in the 22nd week of her
last pregnancy 2 years ago. She is now 26 weeks pregnant, and she
complains of left calf tenderness during the examination. Deep vein
thrombosis is confirmed by Doppler ultrasonography. The most appropriate
management is to

a) Administer intravenous heparin initially, followed by warfarin


until delivery
b)
c)
d)
e)

Do venous ligation proximal to the point of obstruction


Prescribe heparin until delivery
Prescribe warfarin until delivery
Prescribe heparin and indomethacin therapy until delivery

The correct answer is C


Explanation
The occurrence of deep venous thrombosis during pregnancy is of concern
because 5-20% of patients will experience pulmonary embolism, a
potential fatal complication. In the early stages of thrombosis, the
clot may be loosely adherent to the vessel wall, and amenable to
pharmacologic intervention.
The preferred agent is heparin. It is a potent inhibitor of thrombin,
and thus prevents the conversion of fibrinogen to fibrin. Because of its
large molecular size and negative charge, it does not cross the
placenta, and does not appear in breast milk. The dosage of heparin is

monitored by measuring the partial prothrombin time, which should be two


to three times the control value. Intravenous administration of
protamine sulfate counteracts the effects of heparin quickly on a
milligram-for-milligram basis.
Which one of the following describes the McRoberts maneuver for managing
shoulder dystocia?

a)
b)
c)
d)
e)

Suprapubic pressure
Delivery of the posterior arm
Maximal flexion and abduction of the maternal hips
Rolling the mother to an all-fours position
Rotation of the fetal head

The correct answer is C


Explanation
When the just-delivered fetal head retracts firmly against the perineum,
shoulder dystocia is apparent. This is an obstetric emergency that
requires appropriate assistance and a calm but timely approach to ensure
a safe delivery. While all of the maneuvers described are steps in
managing shoulder dystocia, the McRoberts maneuver by itself (maximal
flexion and abduction of the maternal hips) relieves the impaction of
the anterior shoulder against the maternal symphysis in a large
percentage of cases, especially when combined with suprapubic pressure.
You see a 23 year old gravida 1 para 0 for her prenatal checkup at 38
weeks gestation. She complains of severe headaches and epigastric pain.
She has had an uneventful pregnancy to date and had a normal prenatal
examination 2 weeks ago. Her blood pressure is 140/100 mm Hg. A
urinalysis shows 2+ protein; she has gained 5 lb in the last week, and
has 2+ pitting edema of her legs. The most appropriate management at
this point would be

a) Strict bed rest at home and reexamination within 48 hours


b) Admitting the patient to the hospital for bed rest and frequent
monitoring of blood pressure, weight, and proteinuria
c) Admitting the patient to the hospital for bed rest and monitoring,
and beginning hydralazine (Apresoline) to maintain blood pressure below
140/90 mm Hg
d) Admitting the patient to the hospital, treating with parental
magnesium sulfate, and planning prompt delivery either vaginally or by
cesarean section
The correct answer is D
Explanation
This patient manifests a rapid onset of preeclampsia at term. The
symptoms of epigastric pain and headache categorize her preeclampsia as
severe. These symptoms indicate that the progress is well advanced and
that convulsions are imminent. Treatment should focus on rapid control
of symptoms and delivery of the infant.
A 73 year old white female presents to the office with her daughter. She
complains of a several-year history of intermittent involuntary loss of
urine which is gradually worsening. She frequently loses small amounts
of urine when she rises from a low chair, and greater amounts if she
coughs, sneezes, or laughs. She has taken to wearing absorbent
undergarments and admits to significant embarrassment and limitation of
activities as a result of her problem. Her daughter states that the
patient has gradually withdrawn from her usual social contacts.
Physical examination reveals a well-appearing elderly female. Her pelvic
examination shows mucosal atrophy and a mild cystocele. With a full
bladder, she loses a few mL of urine into a gauze pad when standing and
coughing. Her post-void residual is normal. Her physical examination is
otherwise normal for her age, and her urinalysis is normal. If
nonpharmacologic interventions are not completely successful in the
management of this patients problem, which one of the following
medications is most likely to help?

a)
b)
c)
d)
e)

Bethanechol (Urecholine)
Flavoxate (Urispas)
Hyoscyamine (Anaspaz NuLev)
Pseudoephedrine
Tolterodine (Detrol, Detrol LA)

The correct answer is D


Explanation
Stress incontinence is common in older women. It occurs when the
bladders internal sphincter is unable to keep the bladder neck closed
against the rise in intravesicular pressure resulting from normal
activities. While only small amounts of urine may be lost with each
episode, the cumulative effect can severely impair a patients quality
of life.
Nonpharmacologic therapy is important in the treatment of stress
incontinence. Adjusted voiding intervals and pelvic floor (Kegel)
exercises can be helpful, for example.
The tone of the internal sphincter of the bladder is modulated through
?-adrenergic receptors. Simulation of these receptors with agents such
as pseudoephedrine or imipramine can increase internal sphincter tone
and alleviate symptoms. Tolterodine, flavoxate, and hyoscyamne reduce
bladder contractions and are potentially helpful to patients with urge
incontinence from detrusor overactivity. Bethanechol is a bladder
stimulant and may be helpful in patients with transient urinary
retention and overflow incontinence due to decreased detrusor tone.
A 32-year-old Asian female presents to your office for a 6-month follow
up for hypothyroidism. She informs you that she is about 8 weeks
pregnant. She currently takes levothyroxine (Synthroid), 100 g daily.
Which one of the following changes should be made to her medication regimen?

a) Concurrent ferrous sulfate should be added to increase the level


of levothyroxine
b) The dosage of levothyroxine should be increased
c) The dosage of levothyroxine should be decreased
d) Levothyroxine should be discontinued until after delivery
The correct answer is B
Explanation
Women with hypothyroidism who become pregnant usually require an
increase of up to 40% in their thyroid maintenance dose. This adjustment
must be made early in pregnancy because normal fetal brain development
in the first 12 weeks of gestation depends upon maternal thyroxine as
its source of thyroid hormone. There is strong circumstantial evidence
that a deficiency of maternal T_4 in the first trimester is associated
with lower IQs in the offspring.
Concurrent iron supplementation will interfere with T_4 absorption and
should be given at separate times.
Which one of the following agents used for tocolysis has the unique
adverse effect of respiratory depression?

a)
b)
c)
d)
e)

Magnesium sulfate
Ritodrine (Yutopar)
Terbualine (Brethine, Bricanyl)
Indomethacin (Indocin)
Nifedipine (Adalat, Procardia)

The correct answer is A


Explanation
Magnesium sulfate infusions must be carefully monitored because
respiratory depression is a potential lethal side effect. Reflexes are
usually lost first.
Terbutaline and ritodrine have the potential to cause respiratory
distress in the form of pulmonary edema. They do not cause respiratory

depression. Indomethacin and nifedipine are rarely used tocolytics that


do not depress respiration.
A 36 year old Asian gravida 1 para 1 presents with pain and bleeding due
to a large uterine fibroid confirmed by examination and pelvic
ultrasonography. The patient requests treatment but wants to preserve
her fertility. Which one of the following is the best treatment?

a) Oral contraceptives
b) Uterine artery embolization
c) Metformin (Glucophage)
d) Myomectomy
The correct answer is D
Explanation
In the younger patient who wishes to retain fertility, myomectomy is the
surgical treatment of choice in significantly symptomatic uterine
fibroids. Uterine artery embolization, while effective, may induce
menopause of reduce fertility by decreasing ovarian reserve. It has also
been associated with adverse pregnancy outcomes including
malpresentation, preterm birth, and postpartum hemorrhage. Although oral
contraceptives are useful in dysmenorrhea and dysfunctional uterine
bleeding, they are ineffective in symptoms secondary to uterine myomas.
GnRH agonists, danazol, and mifepristone are effective medications for
symptomatic uterine fibroids. Metformin is useful in the treatment of
polycystic ovary syndrome, but not for fibroids.
The commonest type of bleeding encountered with uterine leiomyomata
(uterine fibroids) is

a)
b)
c)
d)
e)

Post-coital spotting
Mid-cycle bleeding
Hypermenorrhea
Oligomenorrhea
Post-coital staining

The correct answer is C


Explanation
Uterine fibroids are benign uterine tumors of smooth muscle origin.
Fibroids frequently cause abnormal vaginal bleeding (eg, menorrhagia,
menometrorrhagia), pelvic pain and pressure, urinary and intestinal
symptoms, and pregnancy complications. Diagnosis is by pelvic
examination and imaging.
Fibroids can cause hypermenorrhea, menorrhagia or menometrorrhagia. If
fibroids grow, degenerate, or hemorrhage, or if pedunculated fibroids
twist, severe acute or chronic pressure or pain can result. Urinary
symptoms (eg, urinary frequency or urgency) can result from bladder
compression, and intestinal symptoms (eg, constipation) can result from
intestinal compression.
All of the following increase the risk of developing ovarian cancer, except

a)
b)
c)
d)
e)

Family history
Oral contraceptive pills
Nulliparity
Late menopause
Caucasian

The correct answer is B


Explanation
Ovarian cancer affects mainly perimenopausal and postmenopausal women.
Nulliparity, delayed childbearing, and delayed menopause increase risk.
Oral contraceptive use decreases risk. A personal or family history of
endometrial, breast, or colon cancer increases risk. Probably 5 to 10%
of ovarian cancer cases are related to mutations in the autosomal
dominant BRCA gene.

Studies find that birth control pills can help reduce a woman's risk of
developing ovarian cancer.
Which one of the following is recommended for the treatment of
intravaginal genital warts in pregnant women?

a) Imiquimod 5% cream (Aldara)


b) Podofilox 0.5% solution (Condylox)
c) Podophyllin 10%-25% in tincture of benzoin (Podofin)
d) Cryotherapy with liquid nitrogen
e) Interferon
The correct answer is D
Explanation
Genital warts can proliferate and fragment during pregnancy, and many
specialists recommend that they be eliminated. Imiquimod, podophyllin,
and podofilox are not recommended for use during pregnancy. For the
treatment of vaginal warts, the Centers for Disease Control and
Prevention (CDC) recommends the use of cryotherapy. Liquid nitrogen,
rather than a cryoprobe, should be used to avoid possible vaginal
perforation and subsequent fistula formation. An alternative is the use
of trichloroacetic acid or bichloroacetic acid carefully applied to the
lesions to avoid damage to adjacent tissue. Interferon is no longer
recommended for routine use in treating genital warts, due to a high
frequency of systemic adverse effects.
A 43 year old woman has developed a pyometra, what is it *least* likely
to be related to?

a)
b)
c)
d)

Uterine fibroids
Post abortion
Endometrial cancer
Ovarian cancer

The correct answer is D


Explanation
A pyometra is a collection of pus within the uterine cavity. It occurs
principally when there is obstruction to the cervical os, and can be a
complication of fibroids, uterine or cervical malignancy, and scarring
after radiotherapy.
It can also occur secondary to intra-uterine infection. Recently there
has been an increase in genital tuberculosis, and this occasionally
presents with pyometra.

With each heat cycle, the uterine lining engorges in preparation for
pregnancy. Eventually, some tissue engorgement becomes excessive or
persistent. This lush glandular tissue is ripe for infection. Bacteria
ascend from the vagina and the uterus becomes infected and ultimately
pus filled. The usual treatment for pyometra is surgical removal of the
uterus and ovaries.
A female medical student at 10 weeks gestation is starting an
intravenous line in a patient who is a chronic active hepatitis B
carrier. After placing the line, the student inadvertently sticks
herself with the bloody needle. The student has never been immunized
against hepatitis B and is susceptible. Which of the following is the
best choice in managing her situation?

a)
b)
c)
d)
e)

Immune globulin only


Hepatitis B vaccine only
Hepatitis B immune globulin only
Hepatitis B vaccine and hepatitis B immune globulin
Observation only

The correct answer is D


Explanation
In a susceptible patient, even pregnant, the standard therapy is to give
the hepatitis B immune globulin to cover the needle stick exposure,
followed by the vaccination. This vaccination is not a live vaccine, so
it is not contraindicated in pregnancy.
A. Hepatitis-specific immune globulin and vaccinations are required.
B. Immune globulin to cover the immediate exposure is also needed.
C. Hepatitis vaccine is required as well.
E. The risk of infection is high and can be prevented by the recommended
therapy. Observation is not the standard care.
Engagement of the fetal head is defined by:

a)
b)
c)
d)
e)

The leading edge of the fetal head is at the ischial spines


The biparietal diameter is through the pelvic inlet
The occipitofrontal diameter is through the pelvic inlet
The lowest part of the head is at a plane between the ischial spines
The leading edge of the fetal head is at the vaginal introitus

The correct answer is B


Explanation
Engagement of the fetal head is defined by the biparietal diameter
entering the pelvic inlet. Although 0 station is often used to imply
engagement, this is only an estimation, since one cannot reach the
pelvic inlet on a normal pelvic exam. If the pelvis is long, or the head
very small, then the vertex could be engaged at a minus station.
Conversely, if the pelvis is short, or the head very big, then
engagement may not occur until a plus station has been achieved.
A. This is the definition of zero station, often used to imply engagement.
C. The biparietal diameter is the measurement used to define engagement.
D. See answer to A.
E. At this point, the fetal vertex is almost crowning (+4 station) and
has engaged long before.
A G3P2 woman at 8 weeks gestation is found to have an ovarian cyst 6cm
in size. What is the appropriate management at this time?

a)
b)
c)
d)

Laparotomy
Observation
Medications to shrink cyst
Aspiration

The correct answer is B


Explanation
Cysts (fluid-filled structures) can resolve on their own. Cysts are not
that uncommon during pregnancy, affecting about 1 in 1,000 pregnant
women. The vast majority of ovarian masses found during pregnancy are
benign. Ultrasound can be helpful in determining if a mass is benign or
malignant, but it cannot do so with 100 percent certainty. If ultrasound
shows that the mass is strictly fluid-filled, without septation or thick
walls, it is probably benign.
The problem with large, even benign, cysts during pregnancy is that they
may rupture or torse (twist on themselves). Either of these events leads
to significant pain for mom and the potential for miscarriage or preterm
labor and delivery for the baby. Large (more than 6-8cm) cysts are
usually removed surgically if they do not decrease in size spontaneously
over the course of a few weeks.
In pregnancy, the best time to operate is in the second trimester,
ideally around 14-16 weeks. Occasionally, a cyst may be dealt with via
laparoscopy, but very large cysts often require a large, open incision.
A 35-year-old female is planning a second pregnancy. Her last pregnancy
was complicated by placental abruption caused by a large fibroid tumor
of the uterus, which is still present. Which one of the following would
be the most appropriate treatment for the fibroid tumor?

a)
b)
c)
d)

Myomectomy
Myolysis with endometrial ablation
Uterine artery embolization
Observation

The correct answer is A


Explanation
There are numerous options for the treatment of uterine fibroids. When
pregnancy is desired, myomectomy offers the best chance for a successful
pregnancy when prior pregnancies have been marked by fibroid-related
complications. Endometrial ablation eliminates fertility, and there is a
lack of long-term data on fertility after uterine artery embolization.
Observation without treatment would not remove the risk for recurrent
complications during subsequent pregnancies.
Risk factors for cervical carcinoma includes all of the following, except

a)
b)
c)
d)
e)

Human papilloma virus type 16 and 18


Early age at first intercourse
Smoking
Nulliparity
Multiple sexual partners

The correct answer is D


Explanation
The cause of cervical cancer is not known, however, certain factors are
believed to increase ones risk of developing cervical cancer. Engaging
in sexual activity at a young age is one such factor. The cells lining
the cervix do not fully mature until the age of 18 and, therefore, are
more susceptible to cancer causing-agents and viruses.
More than 90% of women with cancer of the cervix are infected with the
human papilloma virus (HPV). HPV is the single most important factor for
cervical cancer. HPV usually causes warts in the genital area. The
viruses are passed from one person to another during unprotected sex.
Having multiple sexual partners increases ones risk of getting this
cancer, because the greater number of sexual partners, the greater is
the risk of acquiring HPV infection. Even if a woman has only one sexual
partner, but the man has several partners, he is considered a "high-risk
male" and can transmit HPV to the women.
Smoking is considered a risk factor, possibly because smoking causes
some abnormal changes in the cells and these cells have a higher
likelihood of becoming cancerous. There are also some statistical
studies that have suggested that long-term use of oral contraceptives
may put a woman at a slightly higher risk of developing this cancer.
A 28 year old woman has a cervical cytologic smear report which revealed
many severely dyskaryotic cells and a few frankly malignant cells. The
next step in this patient's management should be

a) Colposcopy and biopsy


b) Cone biopsy
c) Repeat smear
d) Hysterectomy
e) Dilation and curettage
The correct answer is A
Explanation
Cervical cytology testing (such as the Pap test) involves collecting a
sample of cells from the cervix and examining them under a microscope.
The sample is then sent to a laboratory, where it is examined under a
microscope for abnormal cells, which may indicate precancerous changes
or, rarely, cervical cancer.
Colposcopy is often done if results of a Papanicolaou (Pap) test are
abnormal. For colposcopy, a speculum is used to spread the walls of the

vagina and a binocular magnifying lens (similar to that of a microscope)


is used to inspect the cervix for signs of cancer. Often, a sample of
tissue is removed for examination under a microscope (biopsy).
A cervical biopsy is done when a condition likely to eventually lead to
cancer (precancerous condition) or cancer is suspected, usually because
a Pap test result was abnormal. A biopsy of the cervix or vagina is
usually done during colposcopy. During colposcopy, doctors can identify
the area that looks most abnormal and take tissue samples from it.
In the treatment of an ectopic pregnancy with methotrexate all of the
following are true, except

a)
b)
c)
d)
e)

The pregnancy must not have ruptured


It must be less than 5 cm in size
Patient should be clinically stable
Compliance and follow up are ensured
No fetal heart activity has been noted

The correct answer is B


Explanation
Criteria for methotrexate therapy include hemodynamic stability,
confirmation of ectopic pregnancy by ultrasound examination, significant
risk associated with general anesthesia, patient compliance, lack of
contraindications to methotrexate therapy, small size of ectopic mass
(an ectopic mass is less than 3.5 cm in greatest dimension) and lack of
fetal cardiac motion.
A 39-year-old female with no past history of abnormal Papnicolaou (Pap)
test receives a report indicating the presence of ?atypical glandular
cells. Appropriate management would include

a)
b)
c)
d)

HPV DNA testing, and if negative, a repeat Pap test in 4-6 months
Colposcopy with endocervical sampling and an endometrial biopsy
An endometrial biopsy
Referral for dilatation and curettage

The correct answer is B


Explanation
Atypical glandular cells (AGC) on a Papanicolaou (Pap) test refers to
tissue of endocervical or endometrial origin and not the typical
squamous cells of the ectocervix. A Pap test demonstrating AGC is an
indication for colposcopy with endometrial sampling. If the patient is
over 35 years of age, when the risk of endometrial adenocarcinoma
increases, an endometrial biopsy is also mandatory. AGC has not been
associated with HPV infection.
A 28 year old pregnant female presents with nocturnal numbness in her
hands and forearm. Which have woke her up from sleep for the past three
months. She also has difficulty grasping objects. What is the most
likely etiology?

a)
b)
c)
d)

Dermatomyositis
Multiple sclerosis
Carpal tunnel syndrome
Hyperventilation syndrome

The correct answer is C


Explanation
Carpal tunnel syndrome is compression of the median nerve as it passes
through the carpal tunnel in the wrist. Symptoms include pain and
paresthesias in the median nerve distribution. Diagnosis is suggested by
symptoms and signs and confirmed by nerve conduction velocity testing.
Treatments include ergonomic improvements, analgesia, splinting, and
sometimes corticosteroid injection or surgery.
Carpal tunnel syndrome is very common and most often occurs in women
aged 30 to 50. Risk factors include RA or other wrist arthritis

(sometimes the presenting manifestation), diabetes mellitus,


hypothyroidism, acromegaly, amyloidosis, and pregnancy-induced edema in
the carpal tunnel. Activities or jobs that require repetitive flexion
and extension of the wrist may contribute. Most cases are idiopathic.
A primigravid at term presents in labor. Her pregnancy is complicated by
the fact that she has a twin gestation. The most common presentation of
these twins at delivery is:

a)
b)
c)
d)
e)

Transverse/breech
Breech/transverse
Vertex/vertex
Vertex/breech
Breech/vertex

The correct answer is C


Explanation
The most common presentation of twins at delivery is vertex/vertex
followed next by vertex/breech.
A. Least common presentation.
B. Less common presentation.
D. Second most common presentation.
E. Third most common presentation.
If an abnormally high maternal serum alpha-fetoprotein level is found at
16-18 weeks gestation, the next step is

a) High-resolution ultrasonography
b) Amniocentesis for amniotic fluid alpha-fetoprotein,
acetylcholinesterase, and karyotyping
c) Recommending termination of the pregnancy
d) Repeating the measurement of maternal serum alpha-fetoprotein at
24 weeks gestation
The correct answer is A
Explanation
Screening maternal serum ?-fetoprotein levels should be offered to
patients at 16-18 weeks gestation to detect open neural tube defects.
Abnormally high levels must be confirmed by immediate repeat
measurement. The next step is level I ultrasonography to rule out other
conditions which can result in elevated levels, such as multiple
gestation, incorrect gestational age or fetal demise. More detailed
level II ultrasonography and amniocentesis are then performed if the
initial results are normal and confirm fetal age. An open neural tube
defect is found within only 2%-3% of positive maternal serum tests.
Therefore, recommendations for termination of the pregnancy should be
made only after confirmation of a problem by ultrasonography and
amniocentesis. The significance of ?-fetoprotein levels after 22 weeks
gestation is unknown. In addition, testing at 24 weeks would not allow
adequate time for termination of an abnormal pregnancy by those patients
who choose to do so.
Which one of the following hormones is responsible for the proliferation
of the milk ducts during pregnancy?

a)
b)
c)
d)
e)

Human chorionic gonadotropin


Progesterone
Prolactin
Estrogen
Human placental lactogen

The correct answer is D


Explanation
In some target tissues, the main effect of estrogen is to cause cells to
grow and divide, a process called cell proliferation.
In breast tissue, for example, estrogen triggers the proliferation of
cells lining the milk glands, thereby preparing the breast to produce
milk if the woman should become pregnant.

Estrogen also promotes proliferation of the cells that form the inner
lining, or endometrium, of the uterus, thereby preparing the uterus for
possible implantation of an embryo. During a normal menstrual cycle,
estrogen levels fall dramatically at the end of each cycle if pregnancy
does not occur. As a result, the endometrium disintegrates and is shed
from the uterus and vagina in a bleeding process called menstruation.
You see a 29 year old white gravida 2 para 1 at 18 weeks gestation. She
had routine prenatal laboratory tests 2 weeks ago, and her platelet
count was 100,000/mm3 (N 150,000-300,000). she is on no medications
except prenatal vitamins, her past medical history is negative, and she
denies drug use. On the review of systems, she denies easy bruising or
bleeding. Her previous pregnancy was normal.
You order test for antinuclear antibody, antiphospholipid antibody,
anticardiolipin antibody, lupus anticoagulant, and HIV. All results are
normal. A peripheral smear is normal except for a low platelet count.
Appropriate management of this condition would include which one of the
following?

a)
b)
c)
d)
e)

No intervention except periodic monitoring of maternal plateletcounts


Obtaining a fetal platelet count at 20 weeks gestation
Cesarean delivery
Prednisone to maintain a maternal platelet count above 150,000/mm3
Platelet transfusion if the maternal platelet count drops below75,000/mm3

The correct answer is A


Explanation
The most likely diagnosis is gestational thrombocytopenia, which
accounts for 75% of all cases of thrombocytopenia in pregnancy.
Characteristic findings include an asymptomatic patient with a platelet
count less than 150,000/mm3 but usually above 70,000/mm3, no prior
history of bleeding, no preconception history of thrombocytopenia,
normal first-trimester or preconception platelet counts, and a platelet
count that returns to normal 2-12 weeks post partum. no interventions
such as cesarean section or fetal platelet count monitoring are
necessary. Periodic monitoring of maternal platelet counts is
recommended. These patients are not at risk for fetal thrombocytopenia
or maternal bleeding complications.
At the time of physical examination, detection of a lower-abdominal
tumor in a 7-year-old girl is best accomplished by palpation of the
abdomen coupled with:

a)
b)
c)
d)

Rectal examination
Vaginal examination
Rectovaginal examination
Abdominal auscultation and percussion

The correct answer is A


Explanation
In a virginal 7-year-old, the best way to examine a lower abdominal
process is with a rectal examination, not with a vaginal exam. Rectal
exam will be able to detect most pelvic and lower abdominal masses.
B. Vaginal exam in a 7-year-old should be avoided.
C. See answer to B.
D. A lower abdominal may be missed by percussion, unless it is large
enough to have a significant portion within the abdominal cavity.
A 20-year-old female is seen for follow-up 6 weeks after delivery. Her
pregnancy was complicated by preeclampsia. Her examination is
unremarkable. This patient will be at increased risk for which one of
the following in midlife?

a) Breast cancer
b) Diabetes mellitus
c) Hypothyroidism

d) Kidney disease
e) Hypertension
The correct answer is E
Explanation
Preeclampsia affects as many as 5% of first pregnancies and is
manifested as hypertension, proteinuria, edema, and rapid weight gain
after 20 weeks gestation. Very young mothers and those over age 35 have
a higher risk. Patients who have had preeclampsia have a fourfold
increased risk of hypertension and a twofold increased risk of ischemic
heart disease, stroke, and venous thromboembolism. There does not appear
to be an association between preeclampsia and cancer, breast cancer in
particular.
A 27 year old woman in her first trimester of pregnancy develops a
urinary tract infection. Which one of the following antibiotics would be
best to prescribe in this situation?

a)
b)
c)
d)
e)

Nitrofurantoin
Ampicillin
Doxycycline
Trimethoprim-sulfamethoxazole
Ciprofloxacin

The correct answer is A


Explanation
The antibiotics recommended for UTI in pregnant woman are nitrofurantoin
and cephalosporins. These antibiotics are very effective in the
treatment of pregnant woman against urinary tract infection.
Pregnant woman with recurrent urinary tract infection can also be
treated with the antibiotic nitrofurantoin. Normally nitrofurantoin and
cephalexin are very effective in the treatment of urinary tract
infection in pregnant woman.
Certain antibiotics such as tetracycline, doxycycline and bactrim are
not safe to give during a pregnancy because of their teratogenic side
effects.
A 34-year-old female with menorrhagia is found to have iron deficiency
anemia. Which one of the following is true regarding the treatment of
this problem with oral iron?

a) An acidic environment enhances the absorption of iron from the


gastrointestinal tract
b) Iron is absorbed better if taken with food
c) Diarrhea is a common complication
d) Iron supplementation can be discontinued once the hemoglobin
reaches a normal level
e) Sustained-release formulations increase the total amount of iron
available for absorption
The correct answer is A
Explanation
Oral iron is absorbed better with an acidic gastric environment, which
can be accomplished with the concomitant administration of vitamin C.
Agents that raise gastric pH, such as antacids, proton pump inhibitors,
and H2 blockers, should be avoided if possible. Oral iron absorption is
improved if the iron is taken on an empty stomach, but this may not be
well tolerated because gastric irritation is a frequent side effect.
Constipation also is common with oral iron therapy. Iron therapy should
be continued for several months after the hemoglobin reaches a normal
level, in order to fully replenish iron stores. Sustained-release oral
iron products provide a decreased amount of iron for absorption.
Which one of the following is seen in patients receiving injectable
depot medroxyprogesterone acetate (DMPA)?

a) More regular and predictable menstrual periods


b) Improvement in acne

c) Fewer migraine headaches compared to patients using combination


oral contraceptives
d) Increased bone density
e) Minor weight loss
The correct answer is C
Explanation
Depot medroxyprogesterone acetate (DMPA) is useful in women with
contraindications to estrogen use (migraines, deep-vein thrombosis,
cigarette smoking in those over age 35). Amenorrhea and irregular
bleeding are the most common adverse effects of DMPA. Other side effects
include irritability, depression, weight gain, hair loss, and acne. Acne
and other skin problems are related to the drugs androgenicity. DMPA is
associated with an increase in bone resorption and a significant
reduction in bone mineral density, presumably due to the induction of
estrogen deficiency. In women of normal weight, DMPA has been shown to
cause no statistically significant change in weight; however, in obese
adolescents using DMPA there is an increased likelihood of weight gain
compared to oral contraceptives and nonhormonal contraception.
All of the following are normal during pregnancy, except

a)
b)
c)
d)

Heart rate of 90
Decreased blood pressure
Palmar erythema
White blood cell count of 3000

The correct answer is D


Explanation Below is a comparison of some of the labs in a normal
individual and a pregnant patient.
Value | Non-pregnant | Pregnant
Hematocrit (%) | 37-48 | 32-42
Hemoglobin (g/dL) | 12-16 | 10-14
WBC (cells/mm3) | 4300-10,800 | 5000-15,000
Platelets | 150,000-350,000 | 130,000-350,000
The white blood cell (WBC) count is increased during pregnancy.
Which one is the most worrisome finding in a post-dated pregnancy?

a)
b)
c)
d)

Decrease in fetal movement reported by mother


Non-reassuring NST
No increase in mother's weight for 2 weeks
Decrease in fundal height

The correct answer is B


Explanation
A nonstress test (NST) measures the fetal heart rate in response to the
fetus' movements. Generally, the heart rate of a healthy fetus increases
when the fetus moves. The NST is usually performed in the last trimester
of pregnancy.
A nonreactive/non-reassuring NST is when there is no change in the fetal
heart rate when the fetus moves. This may indicate a problem that
requires further testing.
A 32 year old woman presents to the outpatient clinic with a 7 month
history of amenorrhea and hot flushes. She denies any symptoms of
pregnancy and the uterus is of normal size. The most appropriate
diagnostic test to support a diagnosis of premature ovarian failure is

a)
b)
c)
d)
e)
The correct answer is D

Diagnostic laparoscopy
Serum estradiol
Karyotype
Serum luteinizing hormone and follicule-stimulating hormone
Medroxyprogesterone withdrawal test

Explanation
Premature menopause (premature ovarian failure) is the permanent end of
menstrual periods before age 40 because the ovaries become unable to
produce hormones because ovulation stops. You should suspect premature
menopause when women younger than 40 have menopausal symptoms. A
pregnancy test is done, and levels of estrogen and follicle-stimulating
hormone (which stimulates the ovaries to produce estrogen and
progesterone) are measured on multiple occasions to confirm the diagnosis.
Additional tests may be done to help you identify the cause of premature
menopause and thus evaluate health risks and recommend treatment. For
women younger than 35, a chromosome analysis may be done. If a
chromosomal abnormality is detected, additional procedures and treatment
may be required.
All of the following are cardiovascular adaptations to pregnancy, except

a) Cardiac output increases 33-45%


b) Stroke volume increases 10-30%
c) Heart rate decreases 12-18 beats per minute
d) Systolic blood pressure decreases 4-5 mmHg
e) Diastolic blood pressure decreases 8-15 mmHg
The correct answer is C
Explanation
The normal physiological changes in pregnancy are as follows:
Cardiac output rises 30 to 50 percent above baseline. It peaks by the
end of the second trimester, after which it reaches a plateau until
delivery. The change in cardiac output is mediated by 1) increased
preload due to the rise in blood volume, 2) reduced afterload due to a
fall in systemic vascular resistance, 3) a rise in the maternal heart
rate by 10 to 15 beats per minute.
Stroke volume increases during the first and second trimesters, but
declines in the third trimester due to caval compression by the gravid
uterus. The direct effect of pregnancy on cardiac contractility is
controversial.
Blood pressure typically falls, usually reaching a nadir of 10 mm Hg
below baseline by the end of the second trimester. The decline in blood
pressure is mediated by a fall in systemic vascular resistance induced
by hormonal changes and by the addition of a low-resistance circuit
through the uteroplacental bed.
A male infant is delivered at 41 weeks gestation by spontaneous vaginal
delivery. The amniotic fluid is meconium stained. Apgar scores are 7 at
1 minute and 7 at 5 minutes. The baby is noted to have respiratory
distress from birth and is hypoxic by pulse oximetry. Respiration
improves with supplemental oxygen, as does the hypoxia, but does not
return to normal. Which one of the following would most likely be seen
on a chest radiograph?

a)
b)
c)
d)

A normal heart and lungs


Fluid in the pulmonary fissures
Homogeneous opaque infiltrates with air bronchograms
Patchy atelectasis

The correct answer is D


Explanation
The chest radiograph of a child with meconium aspiration syndrome will
show patchy atelectasis or consolidation. If the child has a normal
chest film and respiratory distress, a noncardiopulmonary source should
be considered (i.e., a neurologic or metabolic etiology). The chest film
of a child with transient tachypnea of the newborn will show a wet
silhouette around the heart, diffuse parenchymal infiltrates, or
intralobar fluid accumulation. Homogeneous opaque infiltrates with air
bronchograms on a chest radiograph are seen with hyaline membrane disease.
During a routine office visit, a 19-year-old patient requests
contraception. After discussing options, you prescribe an oral
contraceptive. Several days later her father calls, asking if you

prescribed oral contraceptives for his daughter. Your response is best


guided by which of the following principles

a) Honesty
b) Beneficence
c) Confidentiality
d) Informed consent
e) Justice
The correct answer is C
Explanation
Beneficence is the obligation to promote the well being of others.
Justice is the right of individuals to claim what is due them based on
certain personal properties or characteristics. Informed consent can be
defined as the willing acceptance of a medical intervention after
adequate disclosure by the physician. Honesty means that the patient is
given complete and truthful information about her condition.
Confidentiality means that a patient has the right to make decisions
about her own care and to decide to whom these decisions and her medical
information are communicated.
A. Honesty means that the patient is given complete and truthful
information about her condition.
B. Beneficience is the obligation to promote the well being of others.
D. Informed consent can be defined as the willing acceptance of a
medical intervention after adequate disclosure by the physician.
E. Justice is the right of individuals to claim what is due to them
based on certain personal properties or characteristics.
A 53-year-old female presents with the complaint of another bladder
Infection. She describes suprapubic and urethral pain, urinary
frequency, and dysuria. A pelvic examination is within normal limits.
Her urinalysis is negative for nitrate and leukocyte esterase, with the
microscopic examination revealing 2 - 4 WBCs and 0 - 3 RBCs. Her records
show six visits for similar complaints and findings over the past 2
years. A urine culture performed at one of these visits was negative.
Ciprofloxacin (Cipro) has been prescribed for each episode, with
inconsistent improvement.
Which one of the following would be most appropriate?

a) A course of nitrofurantoin (Macrodantin, Macrobid)


b) Intravesical instillation of a potassium chloride solution to
improve her pain
c) Cystoscopy
d) Laparoscopy
The correct answer is C
Explanation
Interstitial cystits (IC) presents as a combination of urinary symptoms
and pain, including frequency, urgency, nocturia, dyspareunia, and
pelvic pain. The patient has typically been treated unsuccessfully with
repeated courses of antibiotics.
Basic criteria for IC include a negative urine culture, the absence of
genital infection or neoplastic disease, and no history of radiation,
tuberculosis, or chemical cystitis. Diagnostic tests may include the
potassium sensitivity test (PST), anesthetic bladder challenge, and
cystoscopy. Patients with IC have increased pain with the PST, which
involves intravesical instillation of a potassium chloride solution.
Laparoscopy is generally not indicated. Treatments include pentosan,
hydroxyzine, and amitriptyline.
Which factor brings the most increased risk for ectopic pregnancy?

a)
b)
c)
d)

History of tubal surgery and conception after tubal ligation


Intrauterine device (IUD) in place
Use of fertility drugs or assisted reproductive technology
3 Successive episodes of PID

e) History of prior normal pregnancy and OCP use


The correct answer is D
Explanation
Risk factors for ectopic pregnancy include previous ectopic pregnancy,
history of a sexually transmitted disease or pelvic inflammatory
disease, current use of an intrauterine device, prior pelvic
(particularly tubal) surgery, and smoking. (In theory, anything that
hampers the migration of the embryo to the endometrial cavity could
predispose women to ectopic gestation.)
Some organisms causing PID, such as Neisseria gonorrhoeae, increase the
risk of ectopic pregnancy. A history of salpingitis increases the risk
of ectopic pregnancy 4-fold. The incidence of tubal damage increases
after successive episodes of PID (ie, 13% after 1 episode, 35% after 2
episodes, 75% after 3 episodes).
After one ectopic pregnancy, a patient incurs a 7- to 13-fold increase
in the likelihood of another ectopic pregnancy. Overall, a patient with
prior ectopic pregnancy has a 50-80% chance of having a subsequent
intrauterine gestation, and a 10-25% chance of a future tubal pregnancy.
Oligohydramnios is seen in what perinatal condition?

a) Renal agenesis
b) Trisomy 21
c) Tracheal agenesis
d) Twin gestation
The correct answer is A
Explanation
Oligohydramnios is the term used to describe when the level of amniotic
fluid surrounding the baby is too low. In renal agenesis (absence of the
kidneys) the fetus cannot produce urine to contribute to the volume of
the amniotic fluid, therefore a relative oligohydramnios results.
The other answer choices are causes of polyhydramnios. For example in
esophageal atresia, the fetus cannot swallow the amniotic fluid, so the
amount of amniotic fluid that remains is increased.
A 29 year old woman who is breastfeeding her 2 week old infant develops
fever and severe myalgia. On examination, fissures are noted on her
breasts. The most likely organism causing her symptoms is

a)
b)
c)
d)
e)

Escherichia coli
Staphylococcus aureus
Streptococcus fecalis
Staphylococcus epidermidis
Bacteroides species

The correct answer is B


Explanation
A breast infection (mastitis) is rare, except around the time of
childbirth after an injury or surgery. The most common symptom is a
swollen, red area that feels warm and tender. An uncommon type of breast
cancer called inflammatory breast cancer can produce similar symptoms. A
breast infection is treated with antibiotics.
Staphylococcal breast infections (mastitis) and abscesses typically
develop 1 to 4 weeks after delivery. The infected area is red and
painful. Breast abscesses often release large numbers of bacteria into
the mother's milk, and these milk-borne bacteria may infect the nursing
infant.
A 27 year old multiparous woman is admitted to the case room at 31
weeks' gestation with a 1-hour history of vaginal bleeding. She has
painful uterine contractions, with poor uterine relaxation. The most
probable diagnosis is

a)
b)
c)
d)
e)

Blood-stained show
Vasa previa
Placenta previa
Abruptio placentae
Uterine rupture

The correct answer is D


Explanation
Abruptio placentae is premature separation of a normally implanted
placenta from the uterus during late pregnancy. Manifestations include
vaginal bleeding, uterine pain and tenderness, hemorrhagic shock, and
disseminated intravascular coagulation. Diagnosis is clinical and
sometimes by ultrasonography. Treatment is bed rest for mild symptoms
and prompt delivery for severe or persistent symptoms.
Compared to forceps, which one of the following is a potential advantage
of the vacuum extractor in an assisted vaginal delivery?

a)
b)
c)
d)

It can be used in patients with incomplete cervical dilation


The incidence of cephalohematoma is lower
It causes less maternal genital trauma
There is a higher rate of successful vaginal delivery

The correct answer is C


Explanation
The vacuum extractor has a probable advantage compared with forceps in a
number of areas. These induce easier application, lower maternal
anesthesia requirements, and less risk of maternal soft-tissue and fetal
facial injury. There is an increased incidence of cephalohematoma.
Neonatal outcomes as measured by Apgar scores and umbilical artery blood
gases, have not been shown to be significantly different between forceps
and vacuum deliveries. Forceps have been associated with higher rates of
successful delivery in some studies, as they may represent an option for
delivery when vacuum extraction has failed. Incomplete cervical
dilatation is a relative contraindication to use of the vacuum extractor.
Alpha-fetoprotein measurement on a triple screen done during a pregnancy
is increased in

a) Edward syndrome
b) Down syndrome
c) Myelomeningocele
d) Patau syndrome
Correct Answer:* c)
Explanation
Neural tube defects such as anencephaly, spina bifida and
myelomeningocele will show an increased alpha-fetoprotein on a triple
screen.
In trisomies such as Edward syndrome (Trisomy 18), Down syndrome
(Trisomy 21) and Patau syndrome (Trisomy 13) the alpha-fetoprotein value
would be low.
A 20 year old primigravid comes to you for her first prenatal visit at
12 weeks. She works in a daycare facility and developed a maculopapular
rash at 11 weeks gestation. It disappears after 3 days and she feels fine.

a) You should reassure her since the symptoms were mild


b) Offer termination of the pregnancy
c) Obtain her rubella IgG titer
d) Obtain a throat culture and treat with penicillin for 10 days
e) Obtain a toxoplasmosis IgG titer
The correct answer is C
Explanation
If a woman is nonimmune to rubella, then the risk of congenital rubella
syndrome is 20% for a primary infection in the first trimester.

Cataracts, patent ducuts arteriosus, and deafness are the most common
findings. In this case, she is coming to you within a few days of having
an exanthem; if the patients rubella IgG shows immunity, then the rash
was not due to rubella. If she is rubella IgG negative, then obtain an
IgM titer.
A. Rubella infection in an adult can be a mild viral exanthem. This
finding should never be ignored in a pregnant female.
B. No diagnosis of the condition has been made at this time.
D. Streptococcal pharyngitis is usually associated with a fever,
lyphadenopathy, and paryngeal symptoms.
E. Toxoplasmosis is not associated with a maculopapular rash.
The most common complication of epidural anesthesia in a pregnant woman is

a)
b)
c)
d)

Fetal hypoxia
Maternal hypotension
Decrease in contractions
Arrest of labor

The correct answer is B


Explanation
Significant low blood pressure is a complication of epidural anesthesia.
The ways that epiduralized patients must lie accentuate this. Their
position is limited since they are essentially paralyzed people for the
duration of the epidural. Hypotension occurs among almost one-third of
patients with serious hypotension occurring about 12% of the time.
Maternal hypotension is a major risk for the baby. The epidural blocks
the nerves which regulate blood pressure. It causes the blood in the
body to pool, keeping it from being pumped around the body in the proper
manner. The arteries dilate and relax their usual, necessary level of
tension, making it difficult for the heart to pump blood to the baby.
These changes lead to a decrease in the output of the mothers heart.
Less blood per unit time can reach the placenta and therefore the baby.
The baby is completely dependent on the mothers heart to pump blood to
the placenta to satisfy its needs. All of its oxygen comes across from
the placenta. All of the food for its brain and other organs comes
across the placenta. Brains cannot live without a relatively constant
supply of oxygen and glucose. Without this they become damaged.
A 48-year-old female who recently relocated to your area presents as a
new patient. A review of her medical records reveals that she underwent
a right mastectomy 1 year ago for a 1.5 cm invasive ductal carcinoma.
The procedure included axillary lymph node dissection and all nodes
studied were reported as negative for tumor. She received a course of
radiation therapy and chemotherapy and has been advised to continue a
tamoxifen (Nolvadex) regimen for a total of 5 years. She had a negative
mammogram, physical examination, and Papanicolaou (Pap) test just prior
to moving last month, and has been faithfully performing monthly
self-examination of her surgical site and left breast.
According to current recommendations, which one of the following
surveillance methods would be most appropriate in addition to
mammography for this patient?

a)
b)
c)
d)
e)
The correct answer is E

Annual bone scans


Blood tests for tumor markers
Liver function tests every 4 months
A chest radiograph every 6 months
A history and physical examination every 6 months

Explanation
Breast cancer survivors have an increased chance of developing a second
malignancy, compounding their risk of recurrence and metastatic spread
of the treated malignancy. Additionally, treatment with some medications

may increase risks of future problems, as is the case with tamoxifen


treatment, which may increase the risk of endometrial cancer. There has
been no documented improvement in either survival or quality of life
when intensive laboratory and radiographic studies are obtained
routinely. Currently accepted recommendations for routine surveillance
after breast cancer surgery include monthly breast self-examination,
annual mammography, and a careful history and physical examination every
6 months for the first 5 years and annually thereafter.
A patient, who is 8 weeks pregnant, has been using a copper-containing
intrauterine device. On vaginal examination, the string is seen. Which
one of the following would be the most appropriate management of this
patient?

a) Cut the string high in the endocervix, allowing the device to


retract as pregnancy develops
b) Perform laparoscopy to rule out ectopic pregnancy
c) Recommend therapeutic abortion
d) Remove the device if signs of infection develop
e) Remove the device immediately
The correct answer is E
Explanation
Following insertion of either device, a follow-up appointment should be
planned after the next menses to address any concerns or adverse
effects, ensure the absence of infection, and check the presence of the
strings.
The most common adverse effects of IUDs are cramping, abnormal uterine
bleeding, and expulsion. Adverse effects related specifically to the
hormone-releasing IUD include amenorrhea, acne, depression, weight gain,
decreased libido, and headache.
If the IUD threads are ever not present, a pregnancy test should be
performed. When the results are negative, a cytobrush can be inserted
gently into the cervical canal to locate the threads. If this method is
unsuccessful, radiography or ultrasonography may be used to locate the IUD.
When the results of the pregnancy test are positive, an ectopic
implantation must be ruled out. If the strings are visible and the
pregnancy is early, the IUD can be removed but with a risk of pregnancy
loss. If the strings are not visible, ultrasonography should be
performed to identify the IUD for removal.
The following statements are true about uterine leiomyomas EXCEPT:

a) They are 5 times more common in African-American women when


compared to Caucasian women
b) They can be diagnosed by ultrasound
c) They have been associated with infertility
d) They can be suppressed by estrogen therapy
e) They are asymptomatic in over 50% of those who have them
The correct answer is D
Explanation
Leiomyomas are generally found only in the reproductive age group. Since
they tend to regress after menopause and often grow during pregnancy,
estrogen is felt to be stimulatory to their growth. GnRH analogs can be
used to create a pseudomenopausal state and suppress their growth.
A. Leiomyomas are much more common in women of African-American descent.
B. Ultrasound is the most common diagnostic test done. The classic
whorl-shaped lesions can be easily seen by this modality along with
their location in the uterus.
C. Most women with fibroids are not infertile, but there is an
association with fibroids and infertility.
E. Most women with fibroids do not have any problems with menorrhagia,
pain, infertility, or other conditions.
Routine blood work during the first antenatal visit include all of the
following, except

a)
b)
c)
d)
e)

CBC
HIV
Rubella titer
Blood group and type
Rh antibodies

The correct answer is B


Explanation
Routine blood work during the first trimester includes CBC, Rubella
titer, ABO and RH, antibody screen, UA, urine culture, HBsAg, RPR.
Another test that can be done if the patient gives consent is HIV.
In patients with breast cancer, the most reliable predictor of survival is

a)
b)
c)
d)
e)

estrogen receptor status


cancer stage at the time of diagnosis
tumor grade
histologic type
lymphatic or blood vessel involvement

The correct answer is B


Explanation
The most reliable predictor of survival in breast cancer is the stage at
the time of diagnosis. Tumor size and lymph node involvement are the
main factors to take into account. Other prognostic parameters (tumor
grade, histologic type, and lymphatic or blood vessel involvement) have
been proposed as important variables, but most microscopic findings
other than lymph node involvement correlate poorly with prognosis.
Estrogen receptor (ER) status may also predict survival, with
ER-positive tumors appearing to be less aggressive than ER-negative tumors.
A full-term newborn, born 72 hours ago, is noted to be jaundiced. The
pregnancy was uneventful and the delivery uncomplicated. The mother has
type A-positive blood and the child has type O-positive. The child is
breastfed and has lost 9 ounces from a birth weight of 8 lb. He is
feeding for 20 minutes every 4 hours, and except for being icteric, has
a normal examination.
Laboratory evaluation reveals a total serum bilirubin level of 16 mg/dL
(N 1.4-8.7), with a conjugated bilirubin level of 1.0 mg/dL. His
hemoglobin level is 17.8 g/dL (N 13.4-19.8), his hematocrit is 55% (N
41-65), and his reticulocyte count is 3% (N 3-7). Appropriate management
would include

a)
b)
c)
d)
e)

Phototherapy
Exchange transfer
Blood cultures and antibiotic therapy
Dextrose and water supplementation
A recommendation to increase feedings to 10 times a day

The correct answer is E


Explanation
Hyperbilirubinemia can occur in up to 60% of term newborns during the
first week of life. Early guidelines on management of elevated bilirubin
were based on studies of bilirubin toxicity in infants who had hemolytic
disease. Current recommendations now support the use of less intensive
therapy in term newborns with jaundice who are otherwise healthy.
Phototherapy should be initiated when the bilirubin level is above 15
mg/dL for infants at age 29-48 hours old, at 18 mg/dL for infants 49-72,
and at 20 mg/dL in infants older than 72 hours. Generally, this problem
is not considered pathologic unless it presents during the first hours
after birth and the total serum bilirubin rises by more than 5 mg/dL/day
or is higher than 17 mg/dL, or if the infant has signs or symptoms
suggestive of a serious underlying illness such as sepsis. Fortunately,
very few term newborns with jaundice have serious underlying pathology.
Physiologic jaundice follows a pattern, with the bilirubin level peaking
on the third or fourth day of life and then declining over the first
week after birth. Infants with multiple risk factors may develop an
exaggerated form of physiologic jaundice, with the total bilirubin level
rising as high as 17 mg/dL. Breastfed infants are at an increased risk
for exaggerated physiologic jaundice because of relative caloric
deprivation in the first few days of life. Compared with formula-fed
infants, those who are breastfed are six times more likely to experience
moderate jaundice, with the bilirubin rising above 12 mg/dL.
For breastfed newborns who have an early onset of hyperbilirubinemia,
the frequency of feeding should be increased to more than 10 times per
day. If the newborn has a decrease in weight gain, delayed stooling, and
continued poor intake, then formula supplementation may be necessary.
Breastfeeding should be continued to maintain breast milk production.
Supplemental water or dextrose and water should not be given, as this
can decrease breast milk production and may place the infant at risk for
iatrogenic hyponatremia.
A 55 year old woman comes to the office to discuss menopause. You tell
her all of the following are a part of this stage, except

a)
b)
c)
d)
e)

Hot flushes
Vaginal dryness
Mood swings
Insomnia
Breast tissue hyperplasia

The correct answer is E


Explanation
The symptoms of menopause include vasomotor symptoms such as hot
flashes. Neuropsychiatric changes such as poor concentration, memory
loss, depression, anxiety. Genital symptoms such as vaginal and vulvar
dryness and thinning, which may result in inflammation of the vaginal
mucosa (atrophic vaginitis). Also there is risk of osteoporosis
increases because estrogen is decreased, increasing bone resorption by
osteoclasts.
Regression (not increase) of breast size during and after menopause is
psychologically distressing to some women. For those who have been
bothered by cyclic symptoms of breast pain and cyst formation, the
disappearance of these symptoms postmenopausally is a great relief.
A 14 year old girl is brought in by her mother for monthly cyclic pelvic
pain. She has never had a menstrual period. At age 10 she began
developing breasts followed several months later by pubic hair. She is 5

feet 7 inches tall. Currently she has Tanner stage 4 breasts and Tanner
stage 4 pubic hair. On perineal exam you see a suburethral bluish bulge.
Rectal exam notes a midline fullness. The most likely diagnosis is:

a) Transverse vaginal septum


b) Imperforate hymen
c) Gonadal dysgenesis
d) Complete androgen insensitivity syndrome
e) Congenital adrenal hyperplasia
The correct answer is B
Explanation
This is a young woman with normal height and normal breast and sexual
hair development. The lack of a menstrual period is due either to a lack
of the uterus or a blockage in the outlet of the menstrual efflux. The
large suburethral bluish bulge, which can be palpated rectally as a
midline mass, is a vagina full of menstrual blood (hematocolpos). THe
treatment is to incise the imperforate hymen and allow for normal
menstrual flow. These young women are at higher risk for endometriosis.
A transverse vaginal septum is higher in the vagina and is due to a
failure of canalization of the sinovaginal bulb in utero.
A. A transverse vaginal septum is due to failure of the sinovaginal bulb
to canalize during embryogenesis. This would be father back in the
vaginal vault.
C. No secondary sexual characteristics should be present.
D. Absence of androgen receptors would mean that there should not be any
sexual hair.
E. This will lead to ambiguous genitalia at birth and heterosexual
precocious puberty.
A newborn has a small head, small palpebral fissure, small smooth
philtrium and small eyes. What is the likely diagnosis

a) Fetal alcohol syndrome


b) Cocaine
c) Intrauterine infection
d) Trisomy 21
The correct answer is A
Explanation
Fetal alcohol syndrome (FAS) is a condition that results from prenatal
alcohol exposure. Signs of fetal alcohol syndrome may include:
distinctive facial features, including small eyes, an exceptionally thin
upper lip, a short, upturned nose and a smooth skin surface between the
nose and upper lip. See picture:

Trisomy 21 (Down syndrome) have features of: flattened facial features,


protruding tongue, small head, upward slanting eyes (see picture),
unusual for the child's ethnic group and unusually shaped ears.
Which one of the following is a contraindication to androgen therapy in
a woman who seeks treatment because of decreased libido?

a)
b)
c)
d)
e)

Hypertension
Hypothyroidism
Osteoporosis
Pregnancy
Rheumatoid arthritis

Correct Answer:* d)
Explanation
Clear contraindications to androgen therapy include pregnancy,
lactation, signs of hyperandrogenemia, and the presence of
androgen-dependent tumors.
You diagnose varicella in the two preschool children of a 28 year old
patient. She has no history of having had the disease, and serology done
in your office confirms her lack of immunity. A pregnancy test is also
negative and you tell her to return for a varicella vaccination, which
she does the following week.
She returns after 5 weeks because a home pregnancy test is now positive,
and a pregnancy test in your office is also positive. She is concerned
about the effect of varicella vaccine on the fetus. Which one of the
following would be the most accurate advice?

a) Since the varicella vaccine is an attenuated virus vaccine, it


should not be given in pregnancy.
b) She should receive varicella zoster immune globulin (VZIG) to
protect the fetus
c) She has a high likelihood of having a spontaneous abortion
d) Women receiving varicella vaccine during the first trimester are
likely to have children with congenital varicella
e) Her risk of problems would be must higher in the second of third
trimester
The correct answer is A
Explanation
In general, live and/or live-attenuated virus vaccines are
contraindicated during pregnancy, as there is a primarily theoretical
risk to the fetus. However, it is important to mention that, to date,
there is no evidence to demonstrate a teratogenic risk from any
currently available vaccines (e.g., mumps, measles, rubella varicella).
Immunity to varicella should be reviewed in the context of maternal
health care, and vaccination should be recommended as soon as
appropriate. Since the varicella vaccine is an attenuated virus vaccine
(two preparations are avail- able in Canada and both are live), it

should not be given in pregnancy. A program of administration to


susceptible post partum women should be developed. A second dose is recommended and should be administered approximately four weeks after the
first.
A patient has episodes of pain associated with menstrual periods.
Following appropriate investigations, you diagnose endometriosis. Which
one of the following is the most common pelvic site?

a)
b)
c)
d)
e)

Posterior cul-de-sac
Broad ligament of the uterus
Bladder
Ovary
Rectosigmoid colon

The correct answer is D


Explanation
Endometriosis is a noncancerous disorder in which functioning
endometrial tissue is implanted outside the uterine cavity. Symptoms
depend on location of the implants and may include dysmenorrhea,
dyspareunia, infertility, dysuria, and pain during defecation.
Endometriosis is commonly found in the cul-de-sac and in other
peritoneal structures such as bladder and ovaries resulting in pelvic
pain and infertility.
If we accept the theory that retrograde menstruation is in large part
responsible for the initiation of endometriosis in those women
susceptible to the implantation of the endometrial cells, then the
number one reason is the position of the ovary. The ovaries are adjacent
to the opening of the tube in the pelvic area and that location alone
will make the ovaries more prone to be contaminated with the
regurgitated menstrual flow.
Diagnosis is by biopsy, usually via laparoscopy. Treatments include
anti-inflammatory drugs, drugs to suppress ovarian function and
endometrial tissue growth, surgical ablation and excision of
endometriotic implants, and, if disease is severe and no childbearing is
planned, hysterectomy plus oophorectomy.
A 24-year-old worker at a local day-care center has not had a menstrual
period in over 2 months. A home pregnancy test is positive and is
confirmed in your office. She is concerned because she was recently
exposed to a child who was diagnosed with rubella. A serum rubella IgG
antibody test is negative.

a)
b)
c)
d)
e)

Tell the patient she is immune to rubella and is not at risk


Repeat the serologic testing in 2-3 weeks
Administer rubella immune globulin
Administer live-virus rubella vaccine
Order level III ultrasonography to screen for congenital anomalies

The correct answer is B


Explanation
Rubella infection during pregnancy can result in miscarriage, fetal
death, or congenital rubella syndrome. If a pregnant woman is exposed to
rubella, testing for rubella antibody should be performed as soon as
possible. Positive rubella IgG antibody means that the patient is immune
and not at risk. If serum IgG is not detectable, a second blood specimen
should be obtained in 2-3 weeks; evidence of seroconversion indicates
recent infection.
The routine use of immune globulin for postexposure prophylaxis is not
recommended, and it should be used only if pregnancy termination is not
an option. Live-virus rubella vaccine may cross the placenta and infect
the fetus, and therefore should not be given to pregnant women.
Congenital rubella syndrome is most commonly associated with the
following types of anomalies: ophthalmologic (cataracts, retinopathy,

congenital glaucoma), cardiac (patent duct arteriosus, peripheral


pulmonary artery stenosis), auditory (sensorineural hearing impairment),
and neurologic (behavioral disorders, meningoencephalitis, and mental
retardation). The nature of these disorders makes an
ultrasound diagnosis extremely difficult.
A woman presents with the following vaginal lesion. See picture:

What is the most likely diagnosis

a)
b)
c)
d)

Vaginal herpetic lesion


Trichomonas
Moniliasis
Chlamydia

The correct answer is C


Explanation
Candidiasis (moniliasis) is skin infection most commonly with Candida
albicans. Infections can occur anywhere and are most common in skinfolds
and web spaces, on the genital area and around fingernails.
Intertriginous infections manifest as pruritic, well-demarcated,
erythematous patches of varying size and shape.
Diagnosis is by clinical appearance and by findings of yeast and
pseudohyphae in potassium hydroxide wet mounts of scrapings from a
lesion. Intertriginous infection is treated with drying agents as needed
(eg, Burow's solution) and topical antifungals such as clotrimazole and
fuconazole.
A 25-year-old white female with heavy menstrual periods is noted to have
a hemoglobin level of 9.8 g/dL (N 12.0 ? 16.0). The red cell
distribution width is 16.0% (N 11.5 ? 14.5) and the mean corpuscular
volume is 75 ?m3 (N 78 ? 102).
The appropriate treatment for this condition can be enhanced by the use of

a)
b)
c)
d)
e)

Antacids
Soy milk
Iced tea
Bran
Ascorbic acid

The correct answer is E


Explanation
This patient has iron deficiency anemia. There are several substances
that decrease the absorption of iron, including antacids, soy protein,
calcium, tannin (which is in tea), and phytate (which is found in bran).
Since an acidic environment increases iron absorption, ascorbic acid
(vitamin C) can enhance absorption of an iron supplement.
Which one of the following is true regarding the use of a diaphragm for
contraception?

a) It must be refitted if the patient gains more than 15 lb


b) Use of nonoxynol-9 will prevent HIV

c) Diaphragms are made only of latex


d) Diaphragms are recommended for women with a history of toxic shock
syndrome
e) The diaphragm should be removed immediately after intercourse
The correct answer is A
Explanation
The diaphragm is an effective method of contraception if used correctly.
A weight change of more than 15 lb, pregnancy, or pelvic surgery may
necessitate refitting. If used with nonoxynol-9, a diaphragm may
actually increase the risk of HIV transmission. Diaphragms are made of
latex, but a wide seal rim model made of silicon is available for those
who are latex sensitive. Diaphragm use is contraindicated in women with
a history of toxic shock syndrome. The diaphragm should remain in place
for 6-24 hours after intercourse.
In utero, the highest concentration of oxygen is found in which of the
following fetal vessels

a) Abdominal aorta
b) Umbilical vein
c) Umbilical artery
d) Pulmonary artery
e) Femoral artery
The correct answer is B
Explanation
The blood returning to the fetus from the placenta has the highest
oxygen concentration. Blood in the umbilical vein travels through the
ductus venosus into the inferior vena cava and into the right atrium.
Most of this flow tends to go through foramen ovale into the left atrium
and eventually out the ascending aorta. Blood returning from the heart
via the superior vena cava tends to go through the tricuspid valve and
out the pulmonary trunk, then through the ductus arteriosus into the
aorta. Thus, higher oxygenated blood is found in the proximal aorta,
which is supplying the head and neck.
A. Abdominal aorta is a mixture of blood from the placenta and blood
returning from the body, since this artery is postductal.
C. This artery is taking blood to the placenta from the fetus.
D. The pulmonary artery is taking blood from the right ventricle. Due to
the direction of flow, most of this blood is coming from the superior
vena cava and is less oxygenated than the aortic arch.
E. This will have the same oxygen level as the abdominal aorta and the
umbilical artery.
A 40 year old man and his 34 year old wife are being evaluated for
infertility. The woman's gynecologist reports that the woman had a
normal anatomic and physiologic evaluation and suggests the need for
assessment of potential male factors. On examination, the man is 188 cm
(6 ft 2 in) tall with fair skin and little facial hair. His testicles
are small and firm, and he has mild gynecomastia. No sperm are seen on
analysis of his semen. Which of the following tests is most likely to
establish the underlying cause of the infertility?

a) Karyotype from peripheral leukocytes


b) Serum estrogen and testosterone concentrations
c) Follicle-stimulating hormone and luteinizing hormone
d) Serum prolactin concentration
e) Testicular ultrasonography
The correct answer is A
Explanation
This history is typical of Klinefelter syndrome. In Klinefelter, boys
have an extra X chromosome. Despite this fact, they are normal in
appearance before puberty. After puberty, they have gynecomastia, small
and firm testes, and disproportionately long legs and arms. The test
most likely to establish the chromosome pattern is karyotype from
peripheral leukocytes.

With respect to androgen insensitivity syndrome, which of the following


is true?

a)
b)
c)
d)
e)

Genotype is XY
Breast development and uterus are absent
Serum testosterone is below normal male range
Secondary sex characteristics are male
The condition is autosomal recessive

The correct answer is A


Explanation
Androgen insensitivity syndrome (AIS) is when a person who is
genetically male (has one X and one Y chromosome) is resistant to male
hormones called androgens. As a result, the person has some or all of
the physical characteristics of a woman, despite having the genetic
makeup of a man.
In its classic form (complete androgen resistance), the person appears
to be female but has no uterus, and has sparse armpit and pubic hair. At
puberty, female secondary sex characteristics (e.g., breasts) develop,
but menstruation and fertility do not.
Incomplete AIS, however, is often discovered during childhood because a
person may have both male and female physical characteristics. Many have
partial fusion of the outer vaginal lips, an enlarged clitoris, and a
short, blind-ending vagina.
The individual is often diagnosed because of ambiguous genitalia.
Sometimes, though, the person has primarily male characteristics and the
only symptom is a low sperm count as an adult, perhaps with breast
enlargement.
At a routine 6 week postpartum visit, a tearful, despondent-appearing
patient reports depressed mood, poor appetite, decreased sexual drive,
fatigue, and loss of interest in her usual activities. She denies
suicidal ideation. Which one of the following should you do now?

a) Reassure the patient that her ?postpartum blues should resolve in


the next few weeks
b) Prescribe a 10-day per month regimen of medroxyprogesterone (Provera)
c) Prescribe daily sertraline (Zoloft)
d) Prescribe no medications and see the patient back in the office in
2 weeks
e) Hospitalize the patient
The correct answer is C
Explanation
Postpartum depression is a highly prevalent disorder with consequences
that can be profound. The ?postpartum blues affect up to 85% of women
and typically resolve by the tenth postpartum day, whereas the onset of
postpartum depression may not occur until 6 months following delivery.
Evidence regarding the benefit of hormonal therapy for patients with
postpartum depression is lacking. Generally, postpartum depression can
be managed on an outpatient basis unless the illness is severe. SSRIs
are ideal first-line agents and should be used for similar periods of
time and in dosages comparable to those prescribed to patients who
suffer from nonpuerperal illness.
You see a 24 year old obstetric patient late in her third trimester for
mild dysuria and urinary urgency. Microscopic examination of the urinary
sediment is notable for bacteria and you make a presumptive diagnosis of
cystitis. Which one of the following antibiotics would be contraindicated?

a)
b)
c)
d)

Nitrofurantoin (Macrodantin)
Trimethoprim/sulfamethoxazole (Bactrim, Septra)
Amoxicillin/clavulnate (Augmentin)
Cephalexin (Keflex)

The correct answer is B


Explanation
Trimethoprim/sulfamethoxazole is a pregnancy category C antibiotic which
is contraindicated in the third trimester of pregnancy because its
sulfonamide component displaces bilirubin from albumin binding sites and
thus can increase the free bilirubin concentration in the newborns
blood. This theoretically could increase the risk of the infant
developing kernicterus, particularly preterm infants. The remaining
antibiotics listed are pregnancy category B agents, which can be used
safely throughout pregnancy.
A 39-year-old female presents for emergency contraceptive advice after
having unprotected intercourse last night. She has had two previous
normal deliveries and 6 months had a lower leg deep venous thrombosis.
Which one of the following would be appropriate advice for this patient?

a) If hormonal contraceptives are prescribed there is an increased


risk of adverse fetal development
b) High-dose estrogen-only (ethinyl estradiol) contraceptives given
for 2 days are as effective as estrogen/progestin combinations
c) A copper IUD inserted within 5 days of intercourse would be
comparable to hormonal methods in terms of pregnancy prevention
d) Previous venous thrombosis is an absolute contraindication to the
use of emergency hormonal contraception
e) Contraception is unlikely to be effective at this point
The correct answer is C
Explanation
Postcoital insertion of IUDs is highly effective and one of the most
effective methods of emergency contraception if placed within 5 days
after unprotected intercourse. Hormonal emergency contraception has no
known medical contraindication. Likewise, no risk to a fetus or any
adverse pregnancy outcome has ever been shown to result from the
inadvertent use of postcoital contraceptives. High-dose estrogens for 5
days was one of the earliest methods of emergency contraception, but
because of severe side effects is seldom used anymore. Emergency
contraception is beneficial if used within 72 hours of intercourse in
the case of hormonal therapy, and within 5 days in the case of the
copper IUD.
A 28 year old woman is currently 10 weeks pregnant. She is HBsAg
positive and is concerned about the risk of transmission to the baby.
Which one of the following is the most appropriate course to follow?

a)
b)
c)
d)
e)

Immediate termination of pregnancy


Avoid breast-feeding
Cesarean section at term
Administer Hepatitis B vaccine to the baby at birth
Administer hepatitis B immune globulin followed by hepatitis B

vaccine at birth
The correct answer is E
Explanation
Newborns who are exposed to hepatitis B have more than a 90% chance of
becoming chronically infected. This means the virus stays in their blood
and liver for possibly a lifetime. They can pass the virus on to others.
They will also live with a greater chance of developing liver failure or
liver cancer later in life.
It is most important that the newborn receive the first dose of the
hepatitis B vaccine in the delivery room. If possible, also give the
hepatitis B immune globulin (HBIG), which is another medication that
helps the vaccine to work even more successfully.
According to the Center for Disease Control and Prevention (CDC) and the
World Health Organization (WHO), it is safe for an infected woman to

breastfeed her child.


The most reliable diagnostic finding associated with chorioamnionitis is:

a) Maternal leukocytosis
b) Maternal tachycardia
c) Uterine tenderness
d) Maternal fever
e) Maternal bacteremia
The correct answer is D
Explanation
Diagnosis of intrauterine infection is usually based on maternal fever,
maternal or fetal tachycardia, uterine tenderness, foul odor of the
amniotic fluid, and leukocytosis. Bacteremia occurs in only 10% of
cases. Once membranes are ruptured, the presence of a fever 100.4?F. in
the absence of any other explanation for the elevated temperature should
strongly suggest chorioamnionitis. Amniocentesis can assist in the
diagnosis, when no other clinical signs are present besides a fever. The
presence of bacteria in the amniotic fluid, or a fluid glucose level of
less than 15 mg/dl, is presumptive evidence for infection. Interleukin 6
has the highest sensitivity of any diagnostic test.
A. Elevations in maternal leukocyte count can occur for a wide variety
of reasons in pregnancy, including labor. It is not a reliable
diagnostic finding.
B. Maternal tachycardia is a nonspecific finding and can also be
associated with a wide variety of conditions.
C. Uterine tenderness is not always present and may be a late finding in
the condition.
E. Maternal bacteremia occurs in only 10% of cases.
The most common cause of abnormal vaginal discharge in a sexually active
19-year-old female is

a)
b)
c)
d)
e)

Candida albicans
Trichomonas vaginalis
Staphylococcus
group B Streptococcus
mixed vaginal flora
The correct answer is E

Explanation
Bacterial vaginosis is the most common cause of acute vaginitis,
accounting for up to 50% of cases in some populations. It is usually
caused by a shift in normal vaginal flora. Mixed vaginal flora is
considerably more common as a cause of vaginal discharge than C.
albicans and T. vaginalis.
A 22 year old pregnant woman wants to know the chance of having a baby
with Down syndrome. She has a sister with a neural tube defect. Which of
the following will you do as an initial screen?

a)
b)
c)
d)

Chorionic villi sampling


Amniocentesis
Triple screen
Ultrasound
The correct answer is C

Explanation
The triple screen measures AFP, hCG and estriol. This test is the
initial test to screen for down syndrome and neural tube defects (NTD).
Testing will identify 60% of the babies with Down Syndrome and 80-90% of
the babies with neural tube defects.
The other answer choices may be considered if the triple screen comes
back positive for Downs syndrome or NTD.
A G3P2 at 23 weeks gestation develops pain in her flank, fever and

chills. Her urine is positive. What is the diagnosis

a) Pyelonephritis
b) UTI
c) Cystitis
d) Back pain
The correct answer is A
Explanation
Acute pyelonephritis is a leading cause of admission for pregnant women.
It is an ascending infection involving the kidney causing symptoms of
fever, nausea, vomiting and chills. Physical exam will show flank pain.
Initial labs should include urinalysis and urine culture. IV antibiotics
are the mainstay of treatment.
Which one of the following best describes vaginismus

a) Dyspareunia caused by a structurally small vagina


b) A voluntary contraction of the vaginal muscles to prevent penetration
c) An involuntary contraction of the vaginal and pelvic floor muscles
d) A condition associated with general sexual and orgasmic inhibition
The correct answer is C
Explanation
Vaginismus is a psychogenic phenomenon which produces spasms of the
vaginal muscles. These spasms may produce only a slight contraction at
the beginning of intercourse, but at the other extreme they can cause
severe pain and contractions so strong that even a fingertip cannot be
inserted into the vagina. Most women who suffer from vaginismus believe
that their vagina is simply too small, but it is actually normal. True
vaginal spasms cannot be produced voluntarily, and in fact many women
with vaginismums are sexually responsive, are orgasmic on clitoral
stimulation, and may enjoy sexual play without intercourse.
The only nonsexual behavior that is consistently and strongly correlated
with cervical dysplasia and cervical cancer is

a)
b)
c)
d)
e)

Alcohol consumption
Caffeine consumption
Cigarette smoking
Cocaine use
A high-fat diet
The correct answer is C

Explanation
Cigarette smoking is the only nonsexual behavior consistently and
strongly correlated with cervical dysplasia and cancer, independently
increasing the risk two- to fourfold.

Which one of the following FDA-approved regimens interrupts a pregnancy


once implantation has occurred?

a) Levonorgestrel (Plan B), two 0.75-mg tablets taken 12 hours apart


b) Ethinyl estradiol/levonorgestrel, two 100-g/0.75-mg doses taken
12 hours apart
c) Mifepristone (RU-486), taken as a single dose
d) Ethinyl estradiol/levonorgestrel, two 100-g/0.5-mg doses taken 12
hours apart (the
Yuzpe regimen)
e) No currently approved regimen
The correct answer is C
Explanation
Emergency contraception refers to the use of medications to prevent
pregnancy in the event of unprotected intercourse or failure of a method
of contraception. Many emergency contraception regimens have been
studied and utilized. The mechanism of action for these regimens is

uncertain and might include inhibiting ovulation, interfering with tubal


transport, preventing implantation, or stimulating regression of the
corpus luteum.
Several regimens are approved for emergency contraception but must be
used within 72 hours after unprotected intercourse to be effective.
These regimens are not effective following implantation.
Mifepristone (RU-486) is the only FDA-approved regimen that interrupts a
pregnancy after implantation has occurred. It is used for early
pregnancy termination, rather than emergency contraception.
While inserting an IUD (Intrauterine device) in a woman in your office
she suddenly becomes restless, pale and begins to lose consciousness.
The main immediate action you should do is

a) Remove the IUD


b) Inject subcutaneous epinephrine
c) Elevate her leg
d) Inject epinephrine intramuscularly
The correct answer is C
Explanation
Intrauterine device is a small T shaped flexible plastic device that is
inserted into the uterus and can prevent pregnancy for up to 5 years.

Fainting (syncope), slow heart rate (bradycardia) or vasovagal episode


during IUD insertion or removal can happen in a small number of cases.
At the earliest sign of fainting, stop the insertion and put a cool, wet
cloth to the client's forehead. Keep the client lying down with her head
lowered and legs elevated to ensure adequate blood flow.
A 30 year old black female is being evaluated because of absent menses
for the last 6 months. Menarche was at age 12; her menstrual periods
were frequently irregular and were accompanied only occasionally by
dysmenorrhea. She had her first child 4 years ago, but has not been able
to become pregnant since. Her physical examination and pelvic
examination are unremarkable. A serum pregnancy test is negative,
prolactin levels are normal, and LH and FSH levels are both three times
normal on two occasions. These findings are consistent with

a) Hypothalamic amenorrhea
b) Ovarian failure
c) Pituitary macroadenoma

d) Pituitary microadenoma
e) Polycystic ovarian syndrome
The correct answer is B
Explanation
The history and physical findings in this patient are consistent with
all of the conditions listed. However, the elevated FSH and LH indicate
an ovarian problem, and this case is consistent with ovarian failure or
premature menopause. Most pituitary tumors associated with amenorrhea
produce hyperprolactinemia. Polycystic ovarian syndrome usually results
in normal to slightly elevated LH levels and tonically low FSH levels.
Hypothalamic amenorrhea is a diagnosis of exclusion, and can be induced
by weight loss, excessive physical exercise (jogging, ballet), or
systematic illness, and is associated with tonically low levels of LH
and FSH.
Increased human chorionic gonadotropin (hCG) after pregnancy may
suggest the diagnosis of:

a) Carcinoma of the cervix


b) Uterine choriocarcinoma/hydatidiform mole
c) Carcinoma of the uterine corpus
d) Granulosa cell tumor of the ovary
e) Anencephalic pregnancy
The correct answer is B
Explanation A complete mole contains no fetal tissue. Ninety percent
are 46,XX, and 10% are 46,XY.1,2 Complete moles can be divided into 2 types:
-Androgenetic complete mole - homozygous (80% of complete moles) and
heterozygous (20% of complete moles).
-The biparental complete mole is rare.
With a partial mole, fetal tissue is often present. Fetal erythrocytes
and vessels in the villi are a common finding. The chromosomal
complement is 69,XXX or 69,XXY.
Gestational trophoblastic disease like Hydatidiform moles ("molar
pregnancy") or Choriocarcinoma may produce high levels of ?hCG (due to
the presence of syncytialtrophoblasts- part of the villi that make up
the placenta) despite the absence of an embryo. This, as well as several
other conditions (seminoma, germ cell tumors, teratoma with elements of
choriocarcinoma (this is rare), and islet cell tumor), can lead to
elevated hCG readings in the absence of pregnancy.
Which one of the following has proven most useful in preventing vertical
transmission of HIV infection from mother to neonate?

a) Zidovudine (Retrovir) for both the mother and neonate


b) Vitamin A supplementation for both the mother and neonate
c) Vaginal cleansing with chlorhexidine
d) Minimizing the frequency of cesarean delivery
e) Breastfeeding for at least the first month of life
The correct answer is A
Explanation
Antiretroviral treatment has been proven to reduce vertical transmission
of HIV from mother to child. Benefits have been shown for antenatal,
intrapartum, and postpartum treatments. Zidovudine and nevirapine have
the most well-established track record.
Maternal vitamin A therapy has not proven to reduce HIV transmission. It
was hoped that it would help maintain mucosal integrity. Chlorhexidine
vaginal rinses in early labor have also not proven to reduce HIV
transmission to the neonate. Cesarean section decreases the frequency of
neonatal transmission of HIV, but with current viral suppression therapy
the risk-to-benefit ratio of cesarean delivery is questionable. The
immunologic benefits of breastfeeding do not balance the increased risk
of HIV transmission to neonates who are breastfed by HIV-infected mothers.
The most common congenital complication of fetal alcohol syndrome is

a) Mental retardation
b) Microcephaly
c) Macrocephaly
d) Nephrotoxicity
e) Ototoxicity
The correct answer is A
Explanation
The relationship between maternal alcohol abuse during pregnancy and
developmental birth defects is well-documented in psychological and
medical literature. An accurate diagnosis of FAS or Fetal Alcohol
Effects (FAE), in which patients display partial effects of the syndrome
and evidence many of the same problems as full-blown FAS, must be made
by a doctor or geneticist.
Patients with FAS are of short stature, slight build, and have a small
head. Typically they are below the third to tenth percentile compared to
national norms. A pattern of dysmorphic facial features characterizes
these persons as well, and include 1) short eye openings; 2) a short,
upturned nose; 3) smooth area between the nose and mouth; and 4) a flat
midface and thin upper lip. The facial patterns made FAS patients
recognizable although not grossly malformed.
A considerable range of intellectual functioning is found among patients
with FAS. In a report of twenty cases of varying severity, there was a
range of IQ scores from 16 to 105 with a mean IQ of 65. Severity of the
syndrome was related to IQ, with the most severely affected children
having the lowest IQ scores.
Which of the following pairs of HPV types is most commonly associated
with the clinical picture seen here?

a) 6/11
b) 16/18
c) 31/33
d) 39/45
e) 1/3
The correct answer is A
Explanation
Over 60 types of HPV have been identified. Types 6/11 are associated
with genital condylomata and the minor CIN groups. When HPV 6/11 are
found with CIN, the regression rate is high. Types 16/18 have a high
association with cervical carcinoma and advanced cervical dysplasia.

B. These are the HPV serotypes that are commonly associated with
cervical neoplasia, with 16 having the greatest association with
invasive squamous cell carcinoma.
C. These have also been associated with cervical neoplasia.
D. Not associated.
E. Not associated with genital lesions.
A 26-year-old gravida 2 para 1 presents at 30 weeks gestation with a

complaint of severe itching. She has excoriations from scratching in


various areas. She says that she had the same problem during her last
pregnancy, and her medical records reveal a diagnosis of intrahepatic
cholestasis of pregnancy. Elevation of which one of the following
is most specific and sensitive marker of this disorder?

a)
b)
c)
d)
e)

Gamma - Glutamyltransferase (GGT)


Bile acids
Total bilirubin
Chenodeoxycholic acid
Alkaline phosphatase
The correct answer is B

Explanation
Intrahepatic cholestasis of pregnancy (ICP) classically presents as
severe pruritus in the third trimester. Characteristic findings include
the absence of primary skin lesions and elevation of serum levels of
total bile acids.
The most specific and sensitive marker of ICP is total serum bile acid
(BA) levels greater than 10 micromol/L. In addition to the elevation in
serum BA levels, the cholic acid level is significantly increased and
the chenodeoxycholic acid level is mildly increased, leading to
elevation in the cholic/chenodeoxycholic acid level ratio. The elevation
of aminotransferases associated with ICP varies from a mild increase to
a 10- to 25-fold increase.
Total bilirubin levels are also increased but usually the values are
less than 5 mg/dL. Alkaline phosphatase (AP) is elevated in ICP up to
4-fold, but this is not helpful for diagnosis of the disorder since AP
is elevated in pregnancy due to production by the placenta. Mild
elevation of gamma glutamyltransferase (GGT) is seen with ICP but occurs
in fewer than 30% of cases. However, if GGT is elevated in cases of ICP,
that patient is more likely to have a genetic component of the liver
disease.
A 55 year old postmenopausal female presents with a "period-like"
vaginal bleeding for 5 days that stopped spontaneously. Your approach to
the problem would be to

a) Reassure the patient and manage expectantly


b) Administer provera 10 mg for 15 days
c) Administer cyclic estrogen plus progesterone
d) Obtain endometrial tissue
e) Perform a pap smear plus endocervical curretage
The correct answer is D
Explanation
Any post menopausal woman who has vaginal bleeding should have an
endometrial biopsy. Symptoms include vaginal bleed after menopause,
bleeding after sexual intercourse, bleeding between periods in
pre-menopausal women.
If you suspect endometrial cancer, conduct a series of tests to confirm
the diagnosis. A complete personal and family medical history should be
taken. A physician examination should be done, which will include a
thorough pelvic examination.
Your should order an endometrial biopsy. This can be performed in the
physicians office. A very thin, flexible tube is inserted into the
uterus through the cervix. A small piece of endometrial tissue is
removed. If an adequate amount of tissue was not obtained by the
endometrial biopsy, or if the biopsy tissue looks abnormal and
confirmation is needed, your physician may perform a procedure known as
a D & C (dilatation and curettage).
A woman at 33 weeks gestation is brought to the emergency room
following an automobile accident. Her temperature is 36.7?C (98?F),
pulse 110, and BP 80/50. She is conscious, with no obvious head injury.

Her skin is cool and clammy. The lower portion of her abdomen is tense
and tender. Bowel sounds are decreased and the fetal heart tones are
absent. The most likely diagnosis is:

a) Ruptured spleen
b) Ruptured uterus
c) Perforated viscus
d) Abruptio placentae
e) Ruptured bladder
The correct answer is B
Explanation
In a car accident, the gravid uterus in the third trimester will have a
rapid deceleration within the abdominal cavity which can increase the
risk of a placental abruption, as well as uterine rupture. Most cases of
abruption occur within 24 hours of the accident. In this case - the
fetal heart tones are absent, her abdominal wall is tense and there is a
clinical evidence of hypovolemia - one should suspect the more serious
condition of uterine rupture. With her vital sign changes, evaluation
and correction of DIC as well as maternal blood volume replacement are
indicated. The blood loss will continue until the uterus is repaired,
therefore, Cesarean delivery of the fetus is indicated.
Ruptured spleen can definitely be associated with significant hemorrhage
and shock. But, the lower abdominal tenderness is more likely to be from
a pelvic origin.
Perforated viscus is uncommon without penetrating trauma.
Abruption is a common occurrence in abdominal trauma of this nature. Due
to the absent heart tones, tense abdomen, and signs of significant
hypovolemia, uterine rupture is more likely.
Ruptured bladder is not associated with absent fetal heart tones.
A 52 year old woman presents to your office complaining of vaginal
bleeding. Her last bleeding episode was 2 years ago. She is not on
hormone replacement therapy. Her hemoglobin is 13.4. A vaginal
ultrasound shows her uterus and adnexa to be of normal size and an
endometrial stripe of 11 mm. The next step in her evaluation should be:

a) Hysterectomy
b) Dilation and curettage
c) Endometrial biopsy
d) Endometrial ablation
e) Intermittent progestin therapy
The correct answer is C
Explanation
In any woman over the age of 35, with abnormal uterine bleeding, the
diagnosis of an endometrial malignancy must be entertained. With a
postmenopausal woman having an endometrial stripe over 4-5 mm, cancer
needs to be ruled out and tissue should be obtained. The simplest test
is to proceed with an endometrial biopsy.
A. This therapy would be indicated as therapy for adenocarcinoma of the
endometrium or for atypical endometrial hyperplasia. A diagnostic
sampling of the endometrium is the first necessary test.
B. Although this test would lead to a diagnosis, an endometrial biopsy
can be done more easily in the office with minimal discomfort.
D. This modality is used for the reproductive age female with severe
symptomatic uterine bleeding in the absence of endometrial pathology.
E. A diagnosis of the endometrium must be made before hormonal therapy
can be started in this case.
A 26 year old woman presents with vaginal discharge that has been smelly
and bothersome. You perform a wet mount and diagnose bacterial
vaginosis. All of the following would be present, except

a)
b)
c)
d)

Clue cells
Fishy odor with KOH test
pH > 4.5
Frothy discharge
The correct answer is D

Explanation
Bacterial vaginosis will present with a thin, white, gray or yellowish
cloudy discharge with a foul or fishy odor. Itching and irritation, grey
discharge, pH > 4.5, fishy odor and clue cells will be seen on exam and
testing.
A frothy discharge is more likely to be seen in a trichomonas infection.
A 32 year old G5P4 presents with an 8 week history of amenorrhea and
suggestive symptoms of pregnancy. Physical examination reveals an
irregular, enlarged uterus of 16 weeks size. Ultrasound confirms the
presence of an 8 week viable pregnancy and a multiple fibroid uterus.
The correct management for this patient is

a) Termination of the pregnancy with elective myomectomy 2 months later


b) Termination of the pregnancy with concomitant myomectomy
c) Prudent observation with elective C-section at term
d) Prudent observation anticipating probable vaginal delivery
e) Myomectomy and follow pregnancy in usual way
The correct answer is D
Explanation
Uterine fibroids are benign uterine tumors of smooth muscle origin.
Fibroids frequently cause abnormal vaginal bleeding (eg, menorrhagia,
menometrorrhagia), pelvic pain and pressure, urinary and intestinal
symptoms, and pregnancy complications. Diagnosis is by pelvic
examination and imaging. Treatment of symptomatic patients depends on
the patient's desire for fertility and desire to keep her uterus and may
include oral contraceptives, brief presurgical gonadotropin-releasing
hormone therapy to shrink fibroids, and more definitive surgical
procedures (eg, myomectomy, hysterectomy, endometrial ablation).
Some pregnant women will have fibroids, some of which occasionally
interfere with the normal progress of a pregnancy. Though fibroids tend
to grow in size during pregnancy, it is unlikely that they will cause
any symptoms. Some pregnant women do experience minor symptoms,
particularly pelvic pain and light spotting.
The majority of fibroids are of no significance and have no effect upon
a woman's fertility, her pregnancy or delivery. Some, however, impinge
upon or distort the actual cavity of the uterus and may cause
complications in pregnancy, as may a very large fibroid which by virtue
of its size distorts the uterus and the other pelvic organs.
Which of the following congenital birth defects occurs when a woman
using accutane for acne becomes pregnant and keeps the baby?

a) Microcephaly
b) Nephrotoxicity
c) Neurotoxicity
d) Ototoxicity
e) Spinal cord deformities
The correct answer is A
Explanation
Accutane or Isotretinoin is a powerful drug used to combat the most
serious and complicated cases of acne. One of the most serious
complications of accutane is its effect on pregnant women and their
unborn fetuses. Accutane is so dangerous to babies in the womb that no
amount of accutane should be taken by a pregnant woman. Some of the most
common birth defects caused by accutane include: Heart Defects, Mental

Retardation, Microcephaly (unusually small head), Hydrocephaly (enlarged


fluid filled spaces located in the brain), and deformities of the face
and ears.
In evaluating a reproductive age woman who presents with amenorrhea,
which of the following conditions will result in a positive (withdrawal)
progesterone challenge test?

a) Pregnancy
b) Ovarian failure
c) Pituitary failure
d) Mullerian agenesis
e) Polycystic ovary (PCO) syndrome
The correct answer is E
Explanation
In pregnancy, progesterone is produced by the corpus luteum followed by
the placenta. Exogenous progesterone will not lead to withdrawal
bleeding. In ovarian failure as well as pituitary failure, no estrogen
stimulation of the endometrium exists, and progesterone cannot cause
withdrawal bleeding. With Mullerian agenesis, there is no endometrium.
Polycystic ovarian syndrome has an abundance of circulating estrogen, so
the endometrium will proliferate.
A. Progesterone withdrawal will not occur since the corpus luteum is
producing progesterone. The placenta will take over, starting at 7
weeks, and will be the sole producer of progesterone by 12 weeks.
B. No estrogen will be produced; no proliferation of the endometrium
will occur.
C. Without gonadotropin stimulation, there will not be enough estrogen
to stimulate the endometrial lining.
D. There is no uterus, thus no bleeding.
A 26 year old gravida 2 para 1 at 10 weeks gestation presents to the
emergency department with abdominal pain and vaginal spotting.
Ultrasonography reveals an ectopic pregnancy. Her blood type is
A-negative, antibody-negative. Appropriate management with regard to her
Rh status includes

a) Administration of 50 micrograms of RHO immune globulin (RhoGAM)


b) Administration of 300 micrograms of RhoGAM
c) Administration of 50 micrograms of RhoGAM only if she requires
laparoscopic intervention
d) Administration of 300 micrograms of RhoGAM only if she requires
laparoscopic intervention
e) No RhoGAM, as it is not indicated in an Rh-negative woman with an
ectopic pregnancy
The correct answer is A
Explanation
Both ectopic pregnancy and spontaneous or therapeutic abortion pose a
significant risk for fetomaternal hemorrhage. Thus, administration of
RHO immune globulin (RhoGAM) is recommended in any Rh-negative patient
who is unsensitized (D antibody screen-negative prior to administration
of RhoGAM). If the estimated gestational age is 12 weeks or less, 50
micrograms of RhoGAM is recommended. If the estimated gestational age is
greater than 12 weeks, 300 micrograms of RhoGAM is recommended.
A nonpregnant 17 year old girl presents to your office for routine
examination. On pelvic exam, you note several raised fleshly lesions on
her vulva and vaginal wall. No vaginal nor cervical discharge is noted.
Her inguinal nodes are slightly tender and palpable bilaterally. She
appears to have a generalized maculopapular rash. On further
questioning, she recollected a painless labial ulcer that resolves about
2 months ago. The best treatment regimen for this patient is:

a) Laser ablation of the vulvar and vaginal lesions


b) Trichloroacetic acid application of the vulvar and vaginal lesions

c) Benzathine penicillin g 2.4 million units IM, one dose


d) Benzathine penicillin g 2.4 million units IM, q week times three
total doses
e) Acyclovir 400 mg po, 5 times per day for 14 days
The correct answer is C
Explanation
The fleshy lesions described are known as condyloma lata. Along with her
generalized rash (which does not spare the palms and soles), this
patient has the classic picture of secondary syphilis. Single dose
benzathine penicillin is the standard therapy for primary, secondary,
and early latent (latent of less than 1 year duration). Triple dose
therapy is necessary for late latent.
A. This therapy can be used for HPV lesions. This patient has secondary
syphilis.
B. See answer to A.
D. This therapy is used for late latent syphilis. This patient has
secondary syphilis and only needs a single dose of bezathine penicillin.
E. This therapy is used to decrease the duration of herpes simplex
infection.
A 34 year old woman presents to the Emergency Department with a 6 hour
history of intermittent severe crampy right lower quadrant pain. The
patient is vomiting and is afebrile. There is a right adnexal tender
mass on pelvic examination. The white blood cell count is 16 x 10^9/L
per high power field; urinalysis is within normal limits; and the beta
human chorionic gonadotropin (-HCG) is negative. The most probable
diagnosis is

a) Right pyelonephritis
b) Acute appendicitis
c) Right ovarian endometrioma
d) Acute salpingitis
e) Torsion of a right ovarian cyst
The correct answer is E
Explanation
Adnexal torsion is uncommon, occurring most often during reproductive
years. It usually indicates an ovarian abnormality. Risk factors include
pregnancy, induction of ovulation, and ovarian enlargement to > 4 cm
(eg, particularly by benign tumors or hyperstimulation with fertility
drugs). Torsion of normal adnexa, which is rare, is more common among
children than adults.
Torsion causes sudden severe pelvic pain and sometimes nausea and
vomiting. For days or occasionally weeks before the sudden pain,
intermittent, colicky pain may occur, presumably resulting from
intermittent torsion that spontaneously resolves. Cervical motion
tenderness, a unilateral tender adnexal mass, and peritoneal signs are
usually present.
Adnexal torsion is suspected based on typical symptoms and unexplained
peritoneal signs plus severe cervical motion tenderness or an adnexal
mass without evidence of pelvic inflammatory disease. Diagnosis is
usually confirmed by color Doppler transvaginal ultrasonography.
Immediate laparoscopy or laparotomy is done if torsion is suspected or
confirmed by ultrasonography. Treatment aims to salvage the ovary and
fallopian tube by untwisting them via laparoscopy or laparotomy.
Salpingo-oophorectomy is required for nonviable tissue, which may not be
necrotic yet.
A 25 year old woman who has a 4 year history of ulcerative colitis is
admitted to the hospital because of increasing abdominal pain, diarrhea
and hematochezia. Her disease is limited to the descending colon.
Current therapy includes sulfasalazine and corticosteroid enemas. While
she is receiving parenteral corticosteroid therapy, she is discovered to
be 8 weeks pregnant. The statement that should be emphasized in
discussing this pregnancy with her is

a) Abortion should be considered since ulcerative colitis increases


the incidence of premature births and congenital anomalies
b) Abortion should be considered because of the adverse effect of
pregnancy on ulcerative colitis
c) Abortion should be considered because of the teratogenic effects
of her colitis therapy
d) Occurrence of spontaneous abortions does not correlate with
disease activity
e) Sulfasalazine and corticosteroids are not known to be teratogenic
The correct answer is E
Explanation
Sulfasalazine and corticosteroids are not known to be teratogenic. The
general consensus is that ulcerative colitis does not have an adverse
effect on pregnancy outcome, and that pregnancy does not have an adverse
effect on disease progression. Furthermore, use of the medications
sulfasalazine and prednisone has not been associated with detrimental
effects to the mother or the fetus. Some believe that occurrence of
spontaneous abortions correlate with disease activity, therefore is
incorrect.
A 32-year-old gravida 3 para 2 presents for routine prenatal care. The
patient is at 14 weeks estimated gestational age by last menstrual
period, and ultrasonography at 8 weeks gestation was consistent with
these dates. Fetal heart tones are not heard by handheld Doppler.
Transvaginal ultrasonography reveals an intrauterine fetus without
evidence of fetal cardiac activity. The patient has not had any bleeding
or cramping, and otherwise feels fine. A pelvic examination reveals a
closed cervix without any signs of bleeding or products of conception.
Which one of the following is the most likely cause of this presentation?

a)
b)
c)
d)

Inaccurate dating
An inevitable abortion
A missed abortion
A blighted ovum
The correct answer is C

Explanation
In this case, the patient has a missed abortion, which is defined as a
dead fetus or embryo without passage of tissue and with a closed cervix.
This condition often presents with failure to detect fetal heart tones
or a lack of growth in uterine size. By 14 weeks estimated gestational
age, fetal heart tones should be detected by both handheld Doppler and
ultrasonography. An inevitable abortion presents with a dilated cervix,
but no passage of fetal tissue. A blighted ovum involves failure of the
embryo to develop, despite the presence of a gestational sac and
placental tissue.
You are asked to consult on a laboring 29 year old multipara in active
labor. The patient is concerned about the large size of her fetus. She
is concerned about the possibility of this fetus having a difficult
delivery. She tells you that her last delivery was complicated by a
shoulder dystocia. You tell her that shoulder dystocia has been
associated with all of the following EXCEPT:

a) Previous shoulder dystocia


b) Maternal obesity
c) Paternal diabetes
d) Prolonged second stage of labor
e) Fetal macrosomia
The correct answer is C
Explanation
Paternal diabetes has a genetic risk to the child, but does not pose any
risk during pregnancy and delivery compared to other normal infants. If

a patient is a gestational diabetic with a macrosomic fetus and a prior


shoulder dystocia, a cesarean delivery is indicated.
A. This is probably the greatest risk for another shoulder dystocia. If
her prior delivery resulted in a dystocia (especially if the child has
permanent sequelae), then most obstetricians would proceed with an
elective cesarean delivery.
B. Maternal obesity is associated with larger birth weights and an
increased risk of complicated delivery.
D. Prolonged second stage can be a warning feature of an impending
shoulder dystocia.
E. Fetal macrosomia increases the risk of complicated vaginal delivery,
especially when the fetal weight is over 4500 g.
At 32 weeks EGA, a 26-year-old multipara has been hospitalized for 10
days for PROM. She had a previous LTCS because of arrested dilation. For
2 hours she has had light vaginal bleeding and contractions every 15
minutes. Over the past 30 minutes the bleeding has increased slightly,
and she experiences lower abdominal pain between contractions. Her
temperature is 37.0?C (98.6?F). The uterus is tender and the FHR is 170.
Platelet count is 130K, leukocyte count is 14.3K, serum fibrinogen is
225 mg/dl, and the assay for fibrin split products is positive. Which of
the following is the most likely diagnosis

a)
b)
c)
d)
e)

Complete placenta previa


Chorioamnionitis
Abruptio placentae
Uterine scar dehiscence
HELLP syndrome
The correct answer is C

Explanation
Although this patient has had a prior cesarean section, the possibility
of a uterine scar separation is low. With the presence of ruptured
membranes, a complete previa is unlikely. Although the uterus is tender,
the patient is afebrile. There is literature to suggest that prolonged
preterm ROM is assoacited with an increased risk of abruptio placentae.
A. Ruptured membranes with a complete previa is very unlikely.
B. Chorioamnionitis can be a complication of prolonged preterm rupture
of membranes. It can be associated with contractions and uterine pain,
but is usually not associated with vaginal bleeding.
D. Uterine scar separation can occur with a prior cesarean, but usually
occurs in active labor. This patient is showing signs of early uterine
activity at 32 weeks gestation, making this diagnosis unlikely.
E. HELLP syndrome is hemolysis, elevated liver enzymes, and low platelets.
A 31 year old married white female complains of vaginal discharge, odor,
and itching. Speculum examination reveals a homogenous yellow discharge,
vulvar and vaginal erythema, and a strawberry? cervix. The most likely
diagnosis is

a) Candidal vaginitis
b) Bacterial vaginosis
c) Trichomonal vaginitis
d) Chlamydial infection
e) Herpes simplex type 2
The correct answer is C
Explanation
Trichomonal vaginitis usually causes a yellowish discharge which
sometimes has a frothy appearance. Colpitis macularis (strawberry
cervix) is often present. Monilial vaginitis classically causes a
cheesy, whitish exudates with associated vaginal itching and burning.
There may be a vaginal and vulvar erythema and edema, but colpitis
macularis is not a feature. Bacterial vaginosis is characterized by a
grayish discharge with few other physical signs or symptoms, if any.
Chlamydia may cause a yellowish cervical discharge and symptoms of
pelvic inflammatory disease, or alternatively, may be totally

asymptomatic. Herpes simplex type 2 causes ulcerations on the vulva and


vaginal mucosa which are exquisitely tender, often with marked
surrounding erythema and edema.
A 25 year old primigravida at 34 weeks gestation is thought to be small
for dates by her physician and is sent for a sonographic evaluation. The
ultrasound shows the biparietal diameter to be appropriate for 34 weeks
gestation. The abdominal circumference is appropriate for 30 weeks
gestation. The head:abdominal circumference ratio is < 1. The estimated
fetal weight is <10th percentile for 34 weeks gestation. The amniotic
fluid is decreased. Which of the following is the most likely diagnosis

a) Symmetrical IUGR
b) Asymmetrical IUGR
c) Congenital anomaly
d) Congenital infection
e) Unknown gestational age
The correct answer is B
Explanation
Intrauterine growth retardation (IUGR) is where a baby fails to grow as
normal in the womb. These babies are smaller than they should be for
their age and their weight is below what it should be for that stage of
the pregnancy (below the 10th weight percentile for the babys age).
Babies with IUGR are prone to a variety of problems before and after birth.
IUGR can be classified into symmetrical or asymmetrical. In symmetrical
IUGR, the baby's head and body are proportionately small. This usually
occurs when IUGR begins in early pregnancy. Head circumference, length
and weight are all decreased proportionately. These infants are more
likely to have significant intrinsic fetal problems than babies where
the IUGR begins later.
In asymmetrical IUGR, growth restriction is due to problems in late
pregnancy (such as maternal renal disease or pre-eclampsia). These
babies have a relatively normal head circumference, some reduction in
length, but a marked reduction in weight.
A 21 year old woman at 24 weeks gestation presents with dysuria,
vulvovaginal pruritus and vaginal discharge. A saline wet mount slide
shows the following on microscopic exam. The most likely diagnosis is:

a) Candida albicans
b) Bacterial vaginosis
c) Trichomoniasis
d) Chlamydia trachomatis
e) Human papilloma virus
The correct answer is A

Explanation
The most common form of vulvovaginitis is Candida. Only 20% of patients
will have a cottage cheese type of discharge. The image shows a direct
wet mount prepared from a white vaginal discharge showing pseudohyphae,
budding yeast, and human epithelial cells. Examination of a fresh
vaginal discharge provides the simplest and most rapid diagnostic test
in aiding presumptive identification of Candida species.

B. Bacterial vaginosis will show clue cells on a wet mount. These


epithelial cells demonstrate the classic findings of a clue cell: they
have a ground glass appearance with irregular borders due to the large
number of bacteria coating their surgance, and with the application of
KOH to the wet mount there is a characteristic fishy amine odor. The
discharge tends to be gray and clings to the vaginal wall.
C. This discharge is often frothy, and when severe the cervix will
appear strawberry red.
D. Often with chlamydia the patient is asymptomatic.
E. HPV can be associated with increased desquamation of the vaginal
walls and cytolysis, but usually this is not curd-like in nature. Most
HPV infections are not associated with a vaginal discharge.
Which one of the following is the current best management of a pregnant
woman with a past history of herpes simplex genital infection on the
inner labia minora?

a)
b)
c)
d)
e)

Daily application of topical acyclovir to site of previous lesions


Cesarean section only if herpes culture positive
Cesarean section if lesions are present at time of labor
Elective cesarean section at 38 weeks in all cases
Vaginal delivery in all cases
The correct answer is C

Explanation
Genital herpes can be transmitted to the neonate during delivery,
particularly if women have visible herpetic lesions or known infection
with prodromal symptoms or if herpes infection first occurs during the
late 3rd trimester (when the virus is likely to be excreted from the
cervix at delivery). In such cases, delivery by cesarean section is
preferred.
If visible lesions or prodrome is absent, even in women with recurrent
infections, risk is low, and vaginal delivery is possible. If women are
asymptomatic, serial antepartum cultures do not help identify those at
risk of transmission. If women have recurrent herpes infections during
pregnancy but no other risk factors for transmission, delivery can

sometimes be induced to occur between recurrences. When delivery is


vaginal, cervical and neonatal herpes virus cultures are done. Acyclovir
(oral and topical) appears to be safe during pregnancy.
A 23 year old Hispanic female at 18 weeks gestation presents with a 4
week history of a new facial rash. She has noticed worsening with sun
exposure. Her past medical history and review of systems is normal. On
examination, you note symmetric, hyperpigmented patches on her cheeks
and upper lip. The remainder of her examination is normal. The most
likely diagnosis is

a)
b)
c)
d)

Lupus erythematosus
Pemphigoid gestationis (herpes gestationis)
Melasma (chloasma)
Prurigo gestationis
The correct answer is C

Explanation
Melasma or chloasma is common in pregnancy, with approximately 70% of
pregnant women affected. It is an acquired hypermelanosis of the face,
with symmetric distribution usually on the cheeks, nose, eyebrows, chin,
and/or upper lip. The pathogenesis is not known. UV sunscreen is
important, as sun exposure worsens the condition. Melasma often resolves
or improves post partum. Persistent melasma can be treated with
hydroquinone cream, retinoic acid, and/or chemical peels performed post
partum by a dermatologist.
The facial rash of lupus is usually more erythematous, and lupus is
relatively rare. Pemphigoid gestationis is a rare autoimmune disease
with extremely pruritic, bullous skin lesions that usually spare the
face. Prurigo gestationis involves involves pruritic papules on the
extensor surfaces and is usually associated with significant excoriation
by the uncomfortable patient.
Of the following, which one has the greatest effect on the relative risk
of developing endometrial carcinoma?

a) Nulliparity
b) Infertility
c) Obesity
d) High socioeconomic status
e) Polycystic ovary disease
The correct answer is E
Explanation
The factor associated with the greatest relative risk for endometrial
carcinoma is polycystic ovary disease, which has a relative risk of 75.
The use of long-term high doses of postmenopausal estrogen carries an
estimated risk of 10?20. Living in North America or Europe also has an
estimated risk of 10?20.
A lower relative risk is associated with nulliparity, obesity,
infertility, late menopause, older age, and white race. The relative
risk associated with these factors falls into the range of 2?5. Early
menarche, higher education or income levels, menstrual irregularities,
and a history of diabetes, hypertension, gall bladder disease, or
thyroid disease have a relative risk of around 1.5?2.0.
The relative risk associated with cigarette smoking is 0.5, and for
long-term use of high-dose oral contraceptives it is 0.3?0.5.
A 52 year old menopausal female sees you because of vaginal bleeding for
3 days in the preceding month. Since developing hot flushes 12 months
ago, she has taken conjugated equine estrogens (Premarin), 0.9 mg/day.
You perform an endometrial biopsy and the pathologist reports a
histologic diagnosis of adenomatous hyperplasia with atypia. At this
point, which one of the following would be most appropriate?

a) Convert the patient to cyclic combined hormone replacement therapy

by adding medroxyprogesterone acetate (Provera), 10 mg/day on days 1


through 14 of each month, while continuing the daily estrogen
b) Convert the patient to continuous combined hormone replacement
therapy by adding medroxyprogesterone acetate 5 mg/day, while continuing
the estrogen
c) Discontinue the estrogen and observe for recurrence of bleeding
d) Discontinue the estrogen and prescribe medroxyprogesterone
acetate, 20 mg/day for the next 3 months, then repeat the endometrial
biopsy
e) Hysterectomy
The correct answer is E
Explanation
Treatment depends on the histology of the lesion and on the patients
age and her desire to preserve fertility. Simple hyperplasia can be
treated with induction of ovulation in premenopausal patients or with
long term progestin administration. Benewed evaluation of the
endometrium after about 4 months is recommended to rule out an
endometrial cancer that may have been missed the first time. The
alternative to endocrine therapy is hysterectomy.
Hysterectomy is the treatment choice for complex or atypical hyperplasia
(adenomatous hyperplasia with or without atypia). Bilateral
salpingo-oophorectomy is generally performed in patients with atypical
hyperplasia.
A 28 year old gravida 2 para 1 at 32 weeks gestation presents with
severe itching. She denies fever or vomiting. Her physical examination
is remarkable for jaundice, but is otherwise benign. Laboratory studies
reveal a normal CBC, normal platelets, normal glucose and serum
creatinine levels, normal transaminase levels, and a bilirubin level of
4.0 mg/dL (N 0.0-1.0)
Which one of the following is the most likely diagnosis

a)
b)
c)
d)
e)

Intrahepatic cholestasis of pregnancy


Acute viral hepatitis
Acute fatty liver of pregnancy
Pruritic urticarial papules and plaques of pregnancy (PUPPP)
Hemolysis, elevated liver enzymes, low platelets (HELLP) syndrome
The correct answer is A

Explanation
Intrahepatic cholestatis of pregnancy is rare, occurring in 0.01% of
pregnancies. It usually presents in the third trimester. Approximately
80% of patients present with pruritus alone, and another 20% with
jaundice and pruritis. Laboratory results usually reveal normal or
minimal elevation in transaminase levels, elevated bilirubin (usually <5
mg/dL), and occasional elevations in cholesterol and triglyceride
levels. It is important to recognize and diagnose this entity, as it is
associated with prematurity, fetal distress, and increased perinatal
mortality.
Acute viral hepatitis is a common cause of jaundice in pregnancy;
however, it usually does not present with severe pruritus, and
transaminase levels are markedly elevated. Acute fatty liver of
pregnancy is another rare condition occurring in the third trimester and
is usually associated with preeclampsia (50%-100% of cases). It presents
with nausea and vomiting, anorexia, jaundice, abdominal pain, headache,
and neurologic abnormalities. Transaminase levels are moderately
elevated, PT and PTT are prolonged, and profound hypoglycemia and renal
failure are usually present. Pruritic urticarial papules and plaques of
pregnancy (PUPPP) is more common in women that present with severe
pruritus. However, jaundice and liver function abnormalities are absent.
HELLP syndrome is an uncommon but serious condition which presents in
the third trimester with hemolysis, elevated transaminases, and low
platelet count.
A 26 year old G2P1 female comes to your office for prenatal checkup. She

recently immigrated to Canada and this is her first prenatal visit.


Based on the first day of her last menstrual period, she is at 18 weeks
of gestational age. Alpha-feto-protein testing returns abnormal with a
decreased ?-feto-protein level. Her past medical history is
unremarkable. She denies smoking or alcohol intake. What should you do next?

a)
b)
c)
d)
e)

Amniocentesis
Chorionic villus sampling
Induction of abortion
Ultrasonography
Umbilical cord blood sampling
The correct answer is D

Explanation
The most common cause of abnormal ?-feto-protein level is inaccuracy of
gestational age; thus, an invasive test is never appropriate before
performing an ultrasound. The ultrasound will let you make sure of the
gestational age and rule out fetal demise.
After gestational age error is ruled out, amniocentesis, chorionic
villus sampling or cordocentesis would be appropriate.
Abortion would be discussed as an option with the mother only after
definitive diagnosis of a chromosomal abnormalities or fetal demise.
A G1P0 woman at 35 weeks gestation presents with headache and a blood
pressure of 170/115. Protein is ++ on her urine. She has normal body
reflex, fetal heart 140 beats/min. What is the most appropriate next step?

a) C-section
b) Induce labour for delivery
c) Magnesium sulfate + Delivery
d) Give steroid
e) Labetalol
The correct answer is C
Explanation
Preeclampsia is pregnancy-induced hypertension plus proteinuria.
Eclampsia is unexplained generalized seizures in patients with
preeclampsia. Preeclampsia and eclampsia develop between 20 weeks
gestation and the end of the 1st week postpartum.
Diagnostic criteria for severe preeclampsia include at least one of the
following:
-Systolic BP greater than 160 mm Hg or diastolic BP greater than 110 mm
Hg on 2 occasions 6 hours apart with the patient at bed rest
-Proteinuria greater than 5000 mg in a 24-hour collection or more than
3+ on 2 random urine samples collected at least 4 hours apart
-Oliguria with less than 500 mL per 24 hours
-Persistent maternal headache or visual disturbance
-Pulmonary edema or cyanosis
-Concerning abdominal pain
-Impaired liver function test findings
-Thrombocytopenia
-Oligohydramnios, decreased fetal growth, or placental abruption
When severe preeclampsia is diagnosed after 34 weeks' gestation,
delivery is most appropriate. The mode of delivery should depend on
severity of the disease and the likelihood of a successful induction.
However, whenever possible, vaginal delivery should be attempted and
cesarean section should be reserved for routine obstetric indications.
In addition, women with severe preeclampsia who have nonreassuring fetal
status, ruptured membranes, labor, or maternal distress should be
delivered regardless of gestational age.
Magnesium sulfate is the drug of choice for seizure prophylaxis in women
with preeclampsia. Therapy is started at the beginning of labor or prior
to cesarean section and continued 24 hours postpartum in most cases. The
duration of postpartum therapy may be modified depending on the severity

of the disease.
Breastfeeding is contraindicated if the mother has

a) Acute EBV infection


b) Acute HepA infection
c) Chronic HepB infection
d) Asymptomatic HIV infection
The correct answer is D
Explanation
In developed countries, maternal infection with HIV is considered a
contraindication to breastfeeding, as is current alcohol and drug use
and/or dependence. Some medications (prescription and nonprescription)
are contraindicated or advised for use with caution and appropriate
clinical monitoring in lactating women.
A 16-year-old girl present to your office for gynecologic evaluation.
She has never had any vaginal bleeding. She does not recall ever having
started her breast development, nor has she had any growth of axillary
or pubic hair. Her height is 59 inches. On routine physical examination
you see cubitus valgus of the elbows, excess skin of the neck, and a
shield-shaped chest with wide-spaced nipples. What is the most
appropriate next step in her evaluation?

a) Hormone replacement therapy


b) Growth hormone therapy
c) Estradiol serum level
d) Pelvic ultrasonography
e) Gonadotropin levels
The correct answer is E
Explanation
This picture is a classic example of a phenotypic Turner syndrome
female. Ninety-nine percent of all monosomy X fetuses will spontaneously
abort. Congenital lymphademea in utero leads to the development of a
cystic hygroma along with many of the other visible external
manifestations. About 60% of Turner patients have total loss of one X
chromosome, the remainder have either a structural abnormality in one of
the X chromosomes or mosaicism with an abnormal X. Other phenotypic
findings include a high arched palate, renal abnormalities (horseshoe
kidney, partial or complete duplication, etc.), and a low posterior
hairline. One-third of these women will have cardiovascular
abnormalities (coarctation of the aorta, bicuspid aortic valve, etc).
Autoimmune disorders such as Hashimotos thyroiditis and Addisons
disease are common. The general recommendations are that gonadotropin
levels are the first test indicated when the clinical picture of a
classic Turner syndrome patient presents. A karyotype will confirm the
diagnosis and help determine further recommendations should any Y
chromosomal elements exist.
A. This patient will require hormone replacement therapy as she has no
ovaries. It is not, however, the next step in the evaluation.
B. Growth hormone therapy in childhood has been suggested to allow these
patients to approach near normal height in adulthood. It is
controversial at present.
C. As no secondary sexual characteristics due to estrogen have appeared,
an estradiol level will not be required.
D. Pelvic ultrasound may demonstrate tissue in the region of the adnexa.
The streak gonads measure about 0.5 by 2.0 cm.
A 26-year-old gravida 2 para 1 at 36 weeks gestation fell off her
bicycle 2 hours ago. She has that she has felt no fetal movement since
the fall. She has abrasions on the left forearm and left leg and
otherwise appears uninjured.
Which one of the following would be the most sensitive indicator of
fetal compromise in this patient?

a) Continuous electronic fetal monitoring


b) Uterine ultrasonography
c) Vaginal bleeding
d) Uterine tenderness
e) Frequent uterine contractions
Correct Answer:* a)
Explanation
Trauma complicates 6%-7% of pregnancies. Risk factors associated with
significant risk for fetal deaths include motorcycle and pedestrian
collisions, maternal tachycardia, abnormal fetal heart rate, lack of
restraint use, serious maternal injury, and vehicle ejections.
Electronic fetal monitoring is the most sensitive indicator of fetal
compromise. There is some controversy as to the length of monitoring
needed, with reports of delayed fetal death or placental abruption after
short monitoring periods (<6 hours), leading most experts to recommend
continuous monitoring for 24 hours. Clinical signs and symptoms such as
vaginal bleeding, uterine tenderness, and the presence of frequent
uterine contractions are not reliable indicators of fetal well being.
Likewise, uterine ultrasonography is not sensitive for predicting
placental abruption, and therefore, fetal compromise.
A 28-year-old nullgravid has been in active labor for the last 4 hours.
Her labor was induced with an oxytocin intravenous infusion. Her cervix
is dilated to 6 cm, and the fetal vertex is at plus 1 station. You
estimate the fetus to weigh approximately 7 pounds, and by clinical exam
you have determined that the maternal pelvis is adequate. Membranes have
ruptured and the amniotic fluid is clear. The patient is afebrile and
normotensive. She has not required any medication for pain control. Her
nurse has notified you of a recent change in the character of the fetal
heart tracing. On arrival at the patients bedside, you note the
following fetal heart tracing. Of the following, which is the most
appropriate next step?

a)
b)
c)
d)

Immediate forceps delivery


Immediate cesarean delivery
Amnioinfusion of the fetus and supplemental oxygen to the mother
Discontinue oxytocin infusion and give supplemental oxygen to the

mother
e) Observation and reevaluation in 2 hours
The correct answer is E
Explanation
These are examples of early decelerations and are felt to be due to head
compression. Increased intracranial pressure causes local changes in
cerebral artery blood flow, leading to a reflexive bradycardia mediated
by the vagal nerve. Although not all authors agree that this class of

decelerations is distinct from variable decelerations, there is good


evidence to suggest that early decelerations are not associated with
fetal asphyxia. Early decelerations are most often seen around 4-6 cm of
cervical dilation and should not have any associated tachycardia, loss
of variability, or other heart rate changes.
A. Instrumented vaginal delivery is never indicated before the cervix is
completely dilated.
B. Cesarean delivery would be indicated if evidence of uncorrectable
fetal hypoxia should develop.
C. Amnioinfusion is appropriate for repetitive variable decelerations
due to oligohydramnios. Its use for diluting thick meconium to decrease
the incidence of meconium aspiration syndrome is controversial.
D. There is no evidence of uteroplacental insufficiency. Thus, no
intervention is required.
A new born baby given Coumadin, develops non-severe bleeding from
umbilical cord after delivery. What is the appropriate next step?

a) Give Vit K
b) Apply gauze and bandage
c) Give Fresh Frozen Plasma
d) Give Protamine sulfate
The correct answer is A
Explanation
Warfarin inhibits the synthesis of biologically-active forms of the
vitamin K-dependent clotting factors II, VII, IX and X, as well as the
regulatory factors protein C, protein S, and protein Z. Warfarin can be
reversed with vitamin K, or for rapid reversal (e.g., in case of severe
bleeding), with fresh frozen plasma.
Protamine is given for heparin reversal.

Das könnte Ihnen auch gefallen